You are on page 1of 83

TOPNOTCH MEDICAL BOARD PREP PHARMACOLOGY SUPEREXAM

For inquiries visit www.topnotchboardprep.com.ph or email us at topnotchmedicalboardprep@gmail.com


DEAR TOPNOTCH FRIENDS:

PLEASE FOLLOW THESE INSTRUCTIONS:

1. These questions are previous diagnostic, midterm, and finals exams of Topnotch, almost all of them made by Topnotch Board Exam Topnotchers.
2. Answer this Topnotch Superexam seriously 100-items at a time. Cover the “Explanations” Column. Do not immediately look at the answers from the
answer key. That’s not the correct way of answering sample exams. You need to treat these MCQs as exercises and not as handouts.
3. Time yourself. 1.5 hours per 100-item block.
4. After answering each 100-item block, refer to the Topnotch Answer Key for the correct answers. Please be careful of “frameshift mutations” when
checking your answers – check every 10 items. (the format of the answer key was designed for you to practice against “frameshift mutations”)
5. The Topnotch Superexams are EXERCISES for the actual med boards. They will not appear verbatim in your future exams. More than knowing what’s
the correct answer, it’s more important for you to:
a. Know why the other choices are wrong
b. Know why the other choices were included in the first place
c. Know the explanation to the correct answer
6. Sharpen your mind by answering the Topnotch Superexams. Most of these questions based on past feedback are more difficult than the actual questions
in the med boards. In these exams made by Board Exam Topnotchers, if you’re getting a score of 60/100 , that’s already a good score. More than 80/100
is outstanding.

Item QUESTION EXPLANATION AUTHOR TOPNOTCH
# EXAM
1 A 6-year-old patient was recently diagnosed with Cat Know the doses of anti-TB drugs for BOTH children KRISTEL TANHUI DIAGNOSTIC
I PTB. DOTS Treatment is initiated for this patient. and adults. (TOP 3 - AUG EXAM -
The dosing should be: Adults: INH 5mg/kg + Rif 10mg/kg + Pyz 25mg/kg + 2015 MED MARCH 2016
A. INH 5mg/kg + Rif 10mg/kg + Pyz 25mg/kg + Eth Eth 15mg/kg BOARDS;
15mg/kg TOPNOTCH MD
B. INH 10mg/kg + Rif 15mg/kg + Pyz 35mg/kg + Eth Generally just make sure you are familiar with TB! It FROM LA SALLE)
20mg/kg can come out over and over in different subjects
C. INH 15mg/kg + Rif 10mg/kg + Pyz 35mg/kg + Eth from pharma, micro, pedia, im, patho, even gyne.
15mg/kg
D. INH 25mg/kg + Rif 10mg/kg + Pyz 25mg/kg + Eth Source: National TB Control Program Manual of
11mg/kg Procedures
E. INH 5mg/kg + Rif 20mg/kg + Pyz 15mg/kg + Eth
10mg/kg
2 Which of the following anti-TB drug is matched Anti-TB drug most associated with skin rashes is KRISTEL TANHUI DIAGNOSTIC
incorrectly with its adverse effects? streptomycin. (TOP 3 - AUG EXAM -
A. Most hepatotoxic – Pyrazinamide 2015 MED MARCH 2016
B. Psychosis – Isoniazid Source: National TB Control Program Manual of BOARDS;
C. Most ototoxic – Streptomycin Procedures TOPNOTCH MD
D. Most associated with skin rashes due to FROM LA SALLE)
hypersensitivity – Rifampicin
E. Most associated with visual impairment -
Ethambutol
3 A 49 year old made came to the ER with a 3 hour Other than Aspirin, NSAIDs are absolutely KRISTEL TANHUI DIAGNOSTIC
history of anginal chest pain. Past medical history is contraindicated to patients with acute MI. (TOP 3 - AUG EXAM -
significant only for uncontrolled hypertension. 2015 MED MARCH 2016
Cardiac markers and ECG both confirm the diagnosis Drugs which are administered post MI include beta- BOARDS;
of acute myocardial infarction probably of the blockers (unless with contraindications), ACE TOPNOTCH MD
anterior wall. Currently the patient has stable vital inhibitors (beneficial in preventing cardiac FROM LA SALLE)
signs within normal range with persistence of angina. remodeling), Statins (stabilizes the atheromatous
The following should be given except: plaque), antiplatelet therapy like aspirin or
A. Lisinopril clopidogrel, aldosterone antagonist (epleronone
B. Simvastatin only), nitrates (increase O2 supply).
C. Ibuprofen
D. Metoprolol
E. None of the above. All should be administered.
4 Which of the following antibiotic agents is Generally, cell wall inhibitors are –cidal, drugs which KRISTEL TANHUI DIAGNOSTIC
bacteriostatic? interfere with DNA are –cidal except sulfonamides (TOP 3 - AUG EXAM -
A. Piperacillin which are only static unless they are given with 2015 MED MARCH 2016
B. Vancomycin trimethoprim/pyrimethamine (in this case BOARDS;
C. Gentamycin synergism results to cidal activity) TOPNOTCH MD
D. Doxycycline FROM LA SALLE)
E. Co-trimoxazole Protein synthesis inhibitors are –static except
aminoglycosides because they cause frameshift
mutations by misreading of the genetic code.

Source: Topnotch handout on Pharmacology
5 Which of the following is not an indication for SSRIs? SIMILAR TO PREVIOUS BOARD EXAM KRISTEL TANHUI DIAGNOSTIC
A. Major depressive disorder CONCEPT/PRINCIPLE. SSRIs are the first line drugs (TOP 3 - AUG EXAM -
B. OCD for MDD. 2015 MED MARCH 2016
C. Anxiety and panic attacks OCD – SSRI! BOARDS;
D. Phobias TOPNOTCH MD
E. None of the above Source: Topnotch handout on Pharmacology FROM LA SALLE)

6 Propylthiouracil and Methimazole are both used in The halflife of PTU is 1.5hrs and methimazole is 6- KRISTEL TANHUI DIAGNOSTIC
the treatment of hyperthyroidism. They are 8hrs. PTU is generally given every 6-8hrs while (TOP 3 - AUG EXAM -
thioamides which inhibits the enzyme thyroid methimazole is given once a day. Thus methimazole 2015 MED MARCH 2016
peroxidase necessary for the synthesis of thyroid is more convenient for patients. Methimazole is also BOARDS;
hormone. Which of the following is false about 10 times more potent than PTU. TOPNOTCH MD
propylthiouracil? FROM LA SALLE)
A. PTU is more highly protein bound than Source: Katzung Basic and Clinical Pharmacology
methimazole 11th ed p671
B. PTU has a longer halflife and requires less frequent
dosing than methimazole
C. PTU is less potent than methimazole.
D. PTU has the added activity of preventing
TOPNOTCH MEDICAL BOARD PREP PHARMACOLOGY SUPEREXAM Page 1 of 83
For inquiries visit www.topnotchboardprep.com.ph or email us at topnotchmedicalboardprep@gmail.com
TOPNOTCH MEDICAL BOARD PREP PHARMACOLOGY SUPEREXAM
For inquiries visit www.topnotchboardprep.com.ph or email us at topnotchmedicalboardprep@gmail.com
Item QUESTION EXPLANATION AUTHOR TOPNOTCH
# EXAM
peripheral conversion of T4 into T3.
E. None of the above

7 Which of the following has the most potent Contraindication to thrombolysis KRISTEL TANHUI DIAGNOSTIC
mineralocorticosteroid activity? - History of CVS hemorrhage at ANY time (TOP 3 - AUG EXAM -
A. Hydrocortisone - History of CVS infarct within the past year 2015 MED MARCH 2016
B. Triamcinolone - Marked hypertension (>180/110) BOARDS;
C. Betamethasone - Suspicion of aortic dissection TOPNOTCH MD
D. Fludrocortisone - Active internal bleeding (excluding menstruation) FROM LA SALLE)
E. Dexamethasone Source: Topnotch handout on Pharmacology

8 Which of the following is not true regarding Competitive antagonism: ED50 increases, potency KRISTEL TANHUI DIAGNOSTIC
Noncompetitive antagonism? decreases, No effect of maximum efficacy (TOP 3 - AUG EXAM -
A. No effect on Emax 2015 MED MARCH 2016
B. No effect on ED50 Noncompetitive antagonism: No effect ED50, No BOARDS;
C. No effect on potency effect on potency, decreased Emax TOPNOTCH MD
D. A and B FROM LA SALLE)
E. All of the above Source: Topnotch handout on Pharmacology

9 Which of the following is incorrectly matched? Menotropin – FSH and LH analog KRISTEL TANHUI DIAGNOSTIC
A. Menotropin – FSH and LH analog Ocreotide – Somatostatin analog (TOP 3 - AUG EXAM -
B. Ocreotide – Somatostatin analog Desmopressin – ADH analog 2015 MED MARCH 2016
C. Desmopressin – ADH analog Ganirelix – GnRH antagonist BOARDS;
D. Ganirelix – GnRH analog Levothyroxine – T4 TOPNOTCH MD
E. Levothyroxine – T4 FROM LA SALLE)
Source: Topnotch handout on Pharmacology

10 A 45 year old male with history of polyuria, The first and second generation sulfonylureas are KRISTEL TANHUI DIAGNOSTIC
polyphagia and weight loss was recently started on associated with weight gain. (TOP 3 - AUG EXAM -
antidiabetic medications after workup confirmed DM 2015 MED MARCH 2016
type II. 2 months later, he notes increasing weight. Source: Katzung Basic and Clinical Pharmacology BOARDS;
Which of the following antidiabetic medications is 11th ed p748 TOPNOTCH MD
associated with this? FROM LA SALLE)
A. Chlorpropramide
B. Glimepiride
C. Exenetide
D. A and B
E. All of the above
11 Which of the following is not associated with All of the following are associated with pulmonary KRISTEL TANHUI DIAGNOSTIC
pulmonary fibrosis? fibrosis. (TOP 3 - AUG EXAM -
A. Bleomycin 2015 MED MARCH 2016
B. Busulfan Source: Topnotch handout on Pharmacology BOARDS;
C. Amiodarone TOPNOTCH MD
D. Methotrexate FROM LA SALLE)
E. None of the above

12 What is the treatment for malignant hyperthermia? Source: Topnotch handout on Pharmacology KRISTEL TANHUI DIAGNOSTIC
A. Dantrolene (TOP 3 - AUG EXAM -
B. Nitroglycerin 2015 MED MARCH 2016
C. Paracetamol BOARDS;
D. Halothane TOPNOTCH MD
E. All of the above FROM LA SALLE)

13 Which of the following is not a 3rd generation 1st gen – Cefazolin, Cephalotin, Cephalexin, KRISTEL TANHUI DIAGNOSTIC
cephalosporin? Cephradine, Cephapirin (TOP 3 - AUG EXAM -
A. Ceftazidime 2015 MED MARCH 2016
B. Ceftriaxone 2nd gen – Cefaclor, Cefuroxime, Cefoxitin, Cefotetan, BOARDS;
C. Ceftizoxime Cefamandole, Cefonicid, Cefprozil, Cefometazole TOPNOTCH MD
D. Cefixime FROM LA SALLE)
E. Cefepime 3rd gen – Ceftriaxone, Cefoperazone, Cefotaxime,
Cefixime, Ceftazidime, Ceftizoxime, Cefpodoxime
proxetil, Cefdinir, Cefditoren, Ceftibuten

4th gen – Cefepime

Source: Katzung Basic and Clinical Pharmacology
11th ed

TOPNOTCH MEDICAL BOARD PREP PHARMACOLOGY SUPEREXAM Page 2 of 83


For inquiries visit www.topnotchboardprep.com.ph or email us at topnotchmedicalboardprep@gmail.com
TOPNOTCH MEDICAL BOARD PREP PHARMACOLOGY SUPEREXAM
For inquiries visit www.topnotchboardprep.com.ph or email us at topnotchmedicalboardprep@gmail.com
Item QUESTION EXPLANATION AUTHOR TOPNOTCH
# EXAM
14 Which of the following vasodilators can worsen The mechanism of action of diazoxide is by opening KRISTEL TANHUI DIAGNOSTIC
Diabetes Mellitus? K channels causing hyperpolarization and relaxation (TOP 3 - AUG EXAM -
A. Hydralazine of vascular smooth muscles. 2015 MED MARCH 2016
B. Diazoxide BOARDS;
C. Verapamil However, recall the mechanism of release of insulin TOPNOTCH MD
D. Nifedipine in the pancreatic beta cells. Glucose enters cells FROM LA SALLE)
E. Nitroprusside resulting to increased ATP production. This closes
ATP dependent K channels resulting to
depolarization, entry of Calcium and release of
insulin. Diazoxide causes opening of K channels
resulting to hyperpolarization and inhibition of
insulin release. Diazoxide is actually used in
treatment of insulinomas.

Source: Katzung Basic and Clinical Pharmacology
11th ed p 180
15 What enzyme do statins inhibit? Statins inhibit the rate-limiting step of cholesterol KRISTEL TANHUI DIAGNOSTIC
A. HMG-CoA synthase synthesis. (TOP 3 - AUG EXAM -
B. HMG-CoA reductase 2015 MED MARCH 2016
C. HMG-CoA lyase Don’t get confused with HMG CoA synthase which is BOARDS;
D. HMG-CoA transferase the rate limiting step of ketogenesis. TOPNOTCH MD
A. All of the above FROM LA SALLE)
Source: Katzung Basic and Clinical Pharmacology
11th ed p 612

16 A 67-year-old male with ischemic heart disease, This is a case of acute renal failure secondary to KRISTEL TANHUI DIAGNOSTIC
compliant with his medications is admitted for acute statin induced rhabdomyolysis. (TOP 3 - AUG EXAM -
renal failure. Labs reveal hyperkalemia, a rapidly 2015 MED MARCH 2016
elevating creatinine, and markely increased CK and Statins inhibit cholesterol synthesis. One of the BOARDS;
myoglobin levels. Which of the following medications intermediates of cholesterol synthesis is farnesyl. TOPNOTCH MD
he was taking could have precipitated the condition? Farnesyl is needed for the synthesis of Coenzyme Q, FROM LA SALLE)
A. Aspirin which is important for ATP production in
B. Carvedilol metabolically active cells like the muscles. Depletion
C. Losartan of this can result to rhabdomyolysis. To prevent this,
D. Captopril one can administer Coenzyme Q10 with statins.
E. Rosuvastatin
Source: Katzung Basic and Clinical Pharmacology
11th ed p 1122
17 A 67-year-old male with ischemic heart disease, This is a case of acute renal failure secondary to KRISTEL TANHUI DIAGNOSTIC
compliant with his medications is admitted for acute statin induced rhabdomyolysis. (TOP 3 - AUG EXAM -
renal failure. Labs reveal hyperkalemia, a rapidly 2015 MED MARCH 2016
elevating creatinine, and markely increased CK and Statins inhibit cholesterol synthesis. One of the BOARDS;
myoglobin levels. Which of the following medications intermediates of cholesterol synthesis is farnesyl. TOPNOTCH MD
he was taking could have precipitated the condition? Farnesyl is needed for the synthesis of Coenzyme Q, FROM LA SALLE)
A. Aspirin which is important for ATP production in
B. Carvedilol metabolically active cells like the muscles. Depletion
C. Losartan of this can result to rhabdomyolysis. To prevent this,
D. Captopril one can administer Coenzyme Q10 with statins.
E. Rosuvastatin
Source: Katzung Basic and Clinical Pharmacology
11th ed p 1122
18 Which of the following drugs are beneficial in SIMILAR TO PREVIOUS BOARD EXAM KRISTEL TANHUI DIAGNOSTIC
delaying organ remodeling such as in cardiac CONCEPT/PRINCIPLE. (TOP 3 - AUG EXAM -
remodeling in heart failure and reduces progression 2015 MED MARCH 2016
of DM nephropathy? Source: Topnotch Handout on Pharmacology BOARDS;
A. Captopril TOPNOTCH MD
B. Losartan FROM LA SALLE)
C. Eplerenone
D. A and B only
E. All of the above

19 For the treatment of brain tumors, it is important for Procarbazine, Dacarbazine, Nitrosureas (Lomustine, KRISTEL TANHUI DIAGNOSTIC
a drug to be able to penetrate the blood brain barrier. Carmustine, Streptozocin) are chemotherapeutic (TOP 3 - AUG EXAM -
Which of the following antineoplastic agents cannot agents which are able to penetrate the blood brain 2015 MED MARCH 2016
cross the blood brain barrier? barrier. BOARDS;
A. Procarbazine TOPNOTCH MD
B. Dacarbazine Source: Topnotch Handout on Pharmacology FROM LA SALLE)
C. Streptozocin
D. Carmustine
E. 5-FU

20 Which of the following is not considered a controller Terbutaline is a bronchodilator used for acute relief KRISTEL TANHUI DIAGNOSTIC
drug for asthma? of asthma exacerbations. (TOP 3 - AUG EXAM -
A. Budesonide 2015 MED MARCH 2016
B. Terbutaline Source: Topnotch handout on Pharmacology BOARDS;
C. Montelukast TOPNOTCH MD
D. Nedocromil FROM LA SALLE)
E. Omalizumab

TOPNOTCH MEDICAL BOARD PREP PHARMACOLOGY SUPEREXAM Page 3 of 83


For inquiries visit www.topnotchboardprep.com.ph or email us at topnotchmedicalboardprep@gmail.com
TOPNOTCH MEDICAL BOARD PREP PHARMACOLOGY SUPEREXAM
For inquiries visit www.topnotchboardprep.com.ph or email us at topnotchmedicalboardprep@gmail.com
Item QUESTION EXPLANATION AUTHOR TOPNOTCH
# EXAM
21 A 45-year-old man with a history of medication- Hydrochlorothiazide, a thiazide diuretic, can LESTER BRYAN MIDTERM 1
controlled hypertension presents to your office with precipitate a gouty attack in predisposed CO (TOP 10 - AUG EXAM -
complaints of a painful, swollen big toe on the left individuals. This is because these agents increase 2015 MED MARCH 2016
foot. You suspect gout and check his uric acid levels, serum uric acid as a result of competition for the BOARDS;
which are elevated. From looking at the list of the organic acid carrier. Loop diuretics can have this TOPNOTCH MD
medications the patient is taking, you realize that one effect too. Acetazolamide is a carbonic anhydrase FROM UST)
of the medications may be the cause of his current inhibitor; this agent does not have a significant
symptoms. Which medication might that be?
impact on the levels of uric acid. Amiloride and
A. Acetazolamide spironolactone are potassium-sparing diuretics, and
B. Amiloride they do not have a significant impact on the levels of
C. Spironolactone uric acid either. The same is true for mannitol, an
D. Hydrochlorothiazide osmotic diuretic.
E. Mannitol
22 A 24-year-old student has been taking over-the- Diphenhydramine blocks H1-receptors in the brain, LESTER BRYAN MIDTERM 1
counter diphenhydramine for her allergy symptoms thereby producing sedation. The release of CO (TOP 10 - AUG EXAM -
most of her life. Lately, however, she has had more dopamine and serotonin is modulated via H3- 2015 MED MARCH 2016
frequent symptoms, so he increased the dose of the receptors. Diphenhydramine readily crosses the BOARDS;
medication. She now asks her friend, who is a medical blood–brain barrier. This agent has TOPNOTCH MD
student, to explain to her how exactly this agent muscariniccholinergic agonist properties. It is not FROM UST)
makes her more sleepy lately. What is the most likely known to contain tryptophan.
answer regarding diphenhydramine?
A. It blocks H1-receptors in the brain
B. It modulates the release of dopamine and serotonin
C. It acts peripherally, since it does not cross the
blood–brain barrier
D. It exerts its effects via muscarinic-cholinergic
agonist activity
E. It contains tryptophan, which produces sedation
23 Somatostatin interacts with a Somatostatin (growth-inhibiting factor) binds to Gi- LESTER BRYAN MIDTERM 1
A. Gq-protein–coupled receptor coupled protein receptor, initiating exchange of GTP CO (TOP 10 - AUG EXAM -
B. Gi-protein–coupled receptor for GDP, which inhibits AC and leads to reduced 2015 MED MARCH 2016
C. Ligand-activated ion channel cAMP production. BOARDS;
D. Receptor-activated tyrosine kinase TOPNOTCH MD
E. Intracellular nuclear receptor FROM UST)

24 A 50-year man with mild hypertension complains of SIMILAR TO PREVIOUS BOARD EXAM LESTER BRYAN MIDTERM 1
discomfort in his chest. He has slightly enlarged fat CONCEPT/PRINCIPLE. Spironolactone antagonizes CO (TOP 10 - AUG EXAM -
deposits in his breasts with prominent nipples. Which the action of the mineralocorticoid, progesterone, 2015 MED MARCH 2016
of the following medications might be causing this and androgen receptors. Inhibition of androgen BOARDS;
adverse effect? receptors can lead to gynecomastia and breast TOPNOTCH MD
A. Amiloride tenderness, most often in men. FROM UST)
B. Spironolactone
C. Metolazone
D. Hydrochlorothiazide
E. Acetazolamide
25 Zileuton is useful in the treatment of asthma because By inhibiting 5-lipoxygenase, zileuton reduces LESTER BRYAN MIDTERM 1
it leukotriene biosynthesis; it does not inhibit (and in CO (TOP 10 - AUG EXAM -
A. Inhibits prostaglandin biosynthesis fact it might increase) prostaglandin synthesis. 2015 MED MARCH 2016
B. Inhibits leukotriene synthesis BOARDS;
C. Inhibits leukotriene receptors TOPNOTCH MD
D. Inhibits 12-lipoxygenase FROM UST)
E. None of the above.

26 A 59-year-old female nurse who has been diagnosed Any of the sulfonylureas can cause hypoglycemia LESTER BRYAN MIDTERM 1
with type 2 diabetes is admitted to the emergency which can produce shock-like symptoms. Metformin CO (TOP 10 - AUG EXAM -
room. She is tachycardic, tachypneic, and appears and the α-glycosidase inhibitors such as acarbose 2015 MED MARCH 2016
very disoriented; she does not remember the day of rarely cause ypoglycemia. Glucagon would raise BOARDS;
the week or her address or any emergency contact plasma glucose. TOPNOTCH MD
numbers. She vaguely remembers taking her “sugar FROM UST)
medicine” earlier in the day. Which of the following
drugs is most likely responsible for her condition?
A. Metformin
B. Acarbose
C. Glipizide
D. Glucagon
E. Pioglitazone
27 A 53-year-old woman with breast cancer undergoes a Paclitaxel is often used in the treatment of breast as LESTER BRYAN MIDTERM 1
breast-conserving lumpectomy and lymph node well as ovarian and lung cancer. Its main toxicities CO (TOP 10 - AUG EXAM -
biopsy. The pathology report returns with mention of are myelosuppression and peripheral neuropathy 2015 MED MARCH 2016
cancer cells in two of eight lymph nodes removed. that usually manifest as numbness and tingling in BOARDS;
Following radiation therapy, chemotherapy is started the distal extremities. Blood in the urine can indicate TOPNOTCH MD
that includes the use of paclitaxel. Which side effect is hemorrhagic cystitis, a complication of FROM UST)
the patient likely to complain of? cyclophosphamide use. Easy bruising can result
A. Blood in the urine from mechlorethamine use. Hot flashes are a
B. Easy bruising common complaint in patients using tamoxifen.
C. Hot flashes Shortness of breath can result from pulmonary
D. Shortness of breath fibrosis secondary to busulfan or bleomycin use.
E. Numbness and tingling

TOPNOTCH MEDICAL BOARD PREP PHARMACOLOGY SUPEREXAM Page 4 of 83


For inquiries visit www.topnotchboardprep.com.ph or email us at topnotchmedicalboardprep@gmail.com
TOPNOTCH MEDICAL BOARD PREP PHARMACOLOGY SUPEREXAM
For inquiries visit www.topnotchboardprep.com.ph or email us at topnotchmedicalboardprep@gmail.com
Item QUESTION EXPLANATION AUTHOR TOPNOTCH
# EXAM
28 A 42-year-old MDS patient presents to the emergency Toxoplasmosis is treated with a combination of LESTER BRYAN MIDTERM 1
room with mental status changes and a headache. A pyrimethamine and sulfadiazine. Ivermectin is used CO (TOP 10 - AUG EXAM -
computed tomography scan is ordered and to treat filariasis, whereas praziquantel is used to 2015 MED MARCH 2016
demonstrates a ring-enhancing lesion. You decide to treat schistosomiasis. Niclosamide can be used to BOARDS;
treat him empirically due to the possibility of treat tapeworm infections, and pyrantel pamoate is TOPNOTCH MD
Toxoplasmosis gondii abscess. Which agent should be used to treat many helminth infections. FROM UST)
included in his treatment?
A. Ivermectin
B. Praziquantel
C. Sulfadiazine
D. Niclosamide
E. Pyrantel pamoate
29 A 74-year-old man with a 100-pack/year history of Etoposide is used in the treatment of small-cell lung LESTER BRYAN MIDTERM 1
smoking is evaluated for hemoptysis. A computed carcinomas as well as testicular tumors. Its CO (TOP 10 - AUG EXAM -
tomography (CT) scan of the chest shows numerous mechanism of action is related to its ability to inhibit 2015 MED MARCH 2016
pulmonary nodules. A nodule on the pleural surface is topoisomerase II. Methotrexate inhibits BOARDS;
selected for CT-guided biopsy by the interventional dihydrofolate reductase. Alkylating agents include TOPNOTCH MD
radiologist. The biopsy report is small-cell carcinoma mechlorethamine, cyclophosphamide, and FROM UST)
of the lung, and chemotherapy containing etoposide ifosfamide. Paclitaxel and docetaxel stabilize
is started. This drug works by microtubules and thereby disrupt mitosis.
A. Inhibiting topoisomerase II Bleomycin causes DNA chain scission and
B. Inhibiting dihydrofolate reductase fragmentation.
C. Alkylating double-stranded DNA
D. Stabilizing microtubules, with resultant mitotic
arrest
E. Causing DNA chain scission and fragmentation
30 A young couple present to their primary care Pregnancy and lactation are the states of increased LESTER BRYAN MIDTERM 1
physician stating that they are trying to conceive. iron demands. While increased bleeding tendency, CO (TOP 10 - AUG EXAM -
They would like to know if the future mom-to-be dietary deficiency, and malabsorption are all true 2015 MED MARCH 2016
needs to be on any supplements. Along with causes of iron deficiency anemia, they are not the BOARDS;
recommending a multivitamin with folic acid, the culprits during pregnancy. Iron storage is regulated TOPNOTCH MD
doctor also suggests an iron supplement. Pregnant at the level of absorption, and very little of it is lost FROM UST)
women develop iron deficiency anemia because of from the body.
A. Increased bleeding tendency
B. Increased dietary deficiency
C. Malabsorption
D. Increased iron demands
E. Increased excretion
31 Ephedrine can cause increased blood pressure by Ephedrine acts indirectly to release norepinephrine LESTER BRYAN MIDTERM 1
A. Indirect action on cholinergic receptors from nerve terminals, causing effects similar to CO (TOP 10 - AUG EXAM -
B. Blockade of adrenergic receptors those of catecholamines, including elevated blood 2015 MED MARCH 2016
C. Stimulation of release of epinephrine pressure. This potentially dangerous agent has been BOARDS;
D. Inhibition of reuptake of catecholamines removed from the OTC market because of an TOPNOTCH MD
E. Direct action on dopamine receptors increasing number of deaths being reported as FROM UST)
caused by this agent. An example of an indirect-
acting cholinergic agonist is edrophonium, which is
used for diagnosis of myasthenia gravis. Some
adrenoceptor blockers, such as atenolol, are used for
the treatment of hypertension. Catecholamine
reuptake inhibition is a property of some
antidepressant medications. Dopamine receptor
agonists are used in the treatment of Parkinson
disease.
32 Which of the following is a potential side effect of Agranulocytosis occurs more frequently with LESTER BRYAN MIDTERM 1
clozapine? clozapine than with other agents, requiring routine CO (TOP 10 - AUG EXAM -
A. Cholestatic jaundice blood tests. It is the only agent that improves the 2015 MED MARCH 2016
B. QT prolongation negative symptoms of schizophrenia. Cholestatic BOARDS;
C. Agranulocytosis jaundice and photosensitivity are common with TOPNOTCH MD
D. Photosensitivity chlorpromazine. Galactorrhea is a side effect of older FROM UST)
E. Galactorrhea high-potency agents that block dopamine. QT
prolongation is a complication of agents such as
thioridazine and ziprasidone.
33 Which of the following is a good choice to treat newly Buspirone is a partial serotonin 5-HT1A-receptor LESTER BRYAN MIDTERM 1
diagnosed generalized anxiety disorder in a truck agonist that has efficacy comparable to that of CO (TOP 10 - AUG EXAM -
driver? benzodiazepines for the treatment of anxiety, but is 2015 MED MARCH 2016
A. Alprazolam significantly less sedating. Alprazolam is an BOARDS;
B. Triazolam intermediate-acting benzodiazepine used in the TOPNOTCH MD
C. Buspirone treatment of generalized anxiety disorder (GAD) but FROM UST)
D. Trazodone still has some sedation, which would be undesirable
E. Thiopental in this situation. Triazolam is a short-acting
benzodiazepine, and trazodone is a heterocyclic
antidepressant, both used to induce sleep.
Thiopental is a barbiturate sometimes used to
induce anesthesia.
34 A 17-year-old girl sees her physician for swollen ABVD is a treatment regimen used for Hodgkin LESTER BRYAN MIDTERM 1
lymph nodes in the supraclavicular region. A core disease and includes adriamycin, bleomycin, CO (TOP 10 - AUG EXAM -
biopsy demonstrates Reed–Sternberg cells and vinblastine, and dacarbazine. R-CHOP is used for 2015 MED MARCH 2016
fibrotic bands, a finding characteristic of nodular treating non-Hodgkin’s disease. CMF, or BOARDS;
sclerosis Hodgkin disease. Which of the following cyclophosphamide, methotrexate, and fluorouracil, TOPNOTCH MD
combined regimens might be used in this patient? is used for breast cancer. FOLFOX, a regimen that FROM UST)
A. R-CHOP uses 5-fluorouracil, oxaliplatin, and leucovorin, is
B. CMF used in the treatment of colon cancer. BEP
C. FOLFOX (bleomycin, etoposide, and platinum [cisplatin]) is
D. BEP used in the management of metastatic testicular
E. ABVD neoplasms.

TOPNOTCH MEDICAL BOARD PREP PHARMACOLOGY SUPEREXAM Page 5 of 83


For inquiries visit www.topnotchboardprep.com.ph or email us at topnotchmedicalboardprep@gmail.com
TOPNOTCH MEDICAL BOARD PREP PHARMACOLOGY SUPEREXAM
For inquiries visit www.topnotchboardprep.com.ph or email us at topnotchmedicalboardprep@gmail.com
Item QUESTION EXPLANATION AUTHOR TOPNOTCH
# EXAM
35 Which of the following statements regarding the The metabolism of theophylline depends on age; the LESTER BRYAN MIDTERM 1
pharmacokinetics of theophylline is correct? half-life of the drug in children is much shorter than CO (TOP 10 - AUG EXAM -
A. It is primarily metabolized by the kidney in adults. The methylxanthines are all well absorbed 2015 MED MARCH 2016
B. Its metabolism depends on age and are metabolized in the liver. The BOARDS;
C. It is poorly absorbed after oral administration PHARMACODYNAMIC porperties of theophylline TOPNOTCH MD
D. It has a wide therapeutic index
include its adenosine-receptor antagonist activity FROM UST)
E. It stimulates phosphodiesterase and the inhibition of phosphodiesterase.

36 A 74-year-old woman who is undergoing Oprelvekin has been shown to reduce the need for LESTER BRYAN MIDTERM 1
chemotherapy for advanced lung cancer presents to platelet transfusions following myelosuppressive CO (TOP 10 - AUG EXAM -
the infusion center for her next treatment. Before chemotherapy. Erythropoietin is used for anemia. 2015 MED MARCH 2016
each treatment her white count, emoglobin, and Filgrastim and sargramostim are used for BOARDS;
platelet counts are checked to make sure she is not neutropenia. Leucovorin is used in patients TOPNOTCH MD
experiencing chemotherapy-related cytotoxicity. Her undergoing treatment with methotrexate, to prevent FROM UST)
blood sample is run in the analyzer, and her platelet some of its side effects.
count is reported to be at a dangerously low level.
Which medication is her oncologist likely to prescribe
in this situation, along with a platelet transfusion?
A. Erythropoietin
B. Oprelvekin
C. Filgrastim
D. Sargramostim
E. Leucovorin
37 A 34-year-old carpenter presents to the ER after an Epinephrine is contraindicated as an anesthetic LESTER BRYAN MIDTERM 1
accident in which he inadvertently chopped off the tip adjuvant for surgeries involving most facial CO (TOP 10 - AUG EXAM -
of his index finger. He is taken to the OR for structures, digits, and the penis, because of the risk 2015 MED MARCH 2016
reattachment of the digit, and after sedation, a local of vascular compromise. This agent causes BOARDS;
anesthetic is administered around the site of the decreased blood loss for most other surgeries TOPNOTCH MD
injury. The local anesthetic used in the procedure did because of vasoconstriction. Although local FROM UST)
not contain any epinephrine, as it usually does for anesthetic agents such as lidocaine or xylocaine can
most surgical procedures. The reason for this is:
cause mild local tissue swelling, epinephrine does
A. Epinephrine causes increased blood loss during not; either way, it is not a contraindication for hand
delicate surgery surgery. Epinephrine causes elevated blood
B. Epinephrine causes swelling of the tissues, making pressure when administered systemically; however,
surgery more challenging it has no systemic side effects when administered
C. Epinephrine is contraindicated in emergency locally.
surgery
D. Epinephrine causes vasoconstriction, which can
lead to vascular ischemia in digits
E. Epinephrine can cause hypotension when
administered with sedative agents
38 A 30-year-old woman has suffered from cyclical Sumatriptan is a 5-HT1D agonist. An example of an LESTER BRYAN MIDTERM 1
migraines for many years. She now presents to her agent known as a 5-HT1A agonist would be CO (TOP 10 - AUG EXAM -
physician asking for a medication designated buspirone, an antianxiety agent. Fluoxetine is an 2015 MED MARCH 2016
specifically for migraines, not just a general pain example of a serotonin-reuptake inhibitor. BOARDS;
reliever. Her physician decided to prescribe Ondansetron, an antinausea medication, is a 5-HT3 TOPNOTCH MD
sumatriptan as a trial medication. The patient, who is antagonist. The antipsychotic medication Risperdal FROM UST)
a biochemist, would like to know how this medication is an example of a 5-HT2A antagonist.
works.
A. It is a 5-HT1A agonist
B. It is a 5-HT1D agonist
C. It blocks reuptake of serotonin
D. It is a 5-HT3 antagonist
E. It is a 5-HT2A antagonist
39 A 12-year-old boy presents with a rash on the palms Doxycycline, a tetracycline, is the antibiotic of choice LESTER BRYAN MIDTERM 1
and soles of his feet as well as fever and headache. He to treat Rocky Mountain spotted fever, a rickettsial CO (TOP 10 - AUG EXAM -
was camping last weekend and admits to being bitten disease. Streptomycin can be used to treat plague 2015 MED MARCH 2016
by a tick. His Weil-Felix test result is positive, and brucellosis. Bacitracin is only used topically. BOARDS;
suggesting Rocky Mountain spotted fever. What Ciprofloxacin can be used to treat anthrax, and TOPNOTCH MD
antibiotic should be given? erythromycin is the most effective drug for the FROM UST)
A. Streptomycin treatment of Legionnaires disease.
B. Bacitracin
C. Ciprofloxacin
D. Doxycycline
E. Erythromycin
40 A 55-year-old woman is admitted to the surgical Milrinone reduces left ventricular filling pressure LESTER BRYAN MIDTERM 1
intensive care unit after having a coronary artery and thus enhances cardiac output. It is related to the CO (TOP 10 - AUG EXAM -
bypass grafting of four of her coronary vessels. anticholinergic agent biperiden. Milrinone inhibits 2015 MED MARCH 2016
Overnight she develops hypotension, and her cardiac cardiac phosphodiesterase type 3. It increases cAMP, BOARDS;
output, as measured by the Swan-Ganz catheter, is and therefore intracellular calcium. TOPNOTCH MD
significantly lower than it had been postsurgery. You FROM UST)
decide to give her a dose of milrinone. This results in
an increase in her cardiac output. How does
milrinone work?
A. It is a cholinergic agonist
B. It reduces left ventricular filling pressure
C. It potentiates cardiac phosphodiesterase type 3
D. It decreases cyclic AMP
E. It decreases intracellular calcium
41 Which of the following is true? Choramphenicol is one of the few antibiotics with GEORGE MIDTERM 2
A. Azithromycin has lesser bioavailability when higher oral bioavailability than parenteral form. MICHAEL EXAM -
taken with food SOSUAN (TOP 5 - MARCH 2016
B. Metronidazole has the same oral and parenteral AUG 2015 MED
bioavailability. BOARDS;
C. The oral bioavailabity of chloramphenicol is TOPNOTCH MD
higher than parenteral form. FROM UST)

TOPNOTCH MEDICAL BOARD PREP PHARMACOLOGY SUPEREXAM Page 6 of 83


For inquiries visit www.topnotchboardprep.com.ph or email us at topnotchmedicalboardprep@gmail.com
TOPNOTCH MEDICAL BOARD PREP PHARMACOLOGY SUPEREXAM
For inquiries visit www.topnotchboardprep.com.ph or email us at topnotchmedicalboardprep@gmail.com
Item QUESTION EXPLANATION AUTHOR TOPNOTCH
# EXAM
D. Both B and C
E. AOTA

42 The following has increased bioavailabity when taken All of the choices has increased bioavailability when GEORGE MIDTERM 2
with food, except taken with food. MICHAEL EXAM -
A. Cefuroxime SOSUAN (TOP 5 - MARCH 2016
B. Fusic acid AUG 2015 MED
C. Nitrofuratoin BOARDS;
D. Griseofulvin TOPNOTCH MD
E. NOTA FROM UST)

43 The following drugs have concentration dependent Concentration dependent or dose-dependent killing GEORGE MIDTERM 2
killing, except means that the higher the concentration, the greater MICHAEL EXAM -
A. Aminoglycosides the bactericidal effect. Examples are SOSUAN (TOP 5 - MARCH 2016
B. Fluoroquinolones aminoglycosides, fluoroquinolines, daptomycin, AUG 2015 MED
C. Penicillin metronidazole, streptogramins. Penicillin employs BOARDS;
D. Metronidazole time-dependent killing TOPNOTCH MD
E. NOTA FROM UST)

44 Strongest inhibitor of CYP3A4 amongst the protease Ritonavir - Strongest GEORGE MIDTERM 2
inhibitor MICHAEL EXAM -
A. Ritonavir SOSUAN (TOP 5 - MARCH 2016
B. Saquinavir AUG 2015 MED
C. Lopinavir BOARDS;
D. Amprenavir TOPNOTCH MD
E. Atazanavir FROM UST)

45 High dose statins are given during ACS, the MOA of GEORGE MIDTERM 2
which in these cases is: MICHAEL EXAM -
A. Inhibit HMG-CoA reductase SOSUAN (TOP 5 - MARCH 2016
B. Stabilize plaque AUG 2015 MED
C. Reduce TAG synthesis BOARDS;
D. Increase HDL TOPNOTCH MD
E. AOTA FROM UST)

46 Single dose drug used in eradication of Drugs used to eradicate meningococcal carrier state: GEORGE MIDTERM 2
meningococcal carrer state 1. rifampin 2. ciprofloxacin (single dose) 3. MICHAEL EXAM -
A. RIfampicin ceftriaxone (single dose) SOSUAN (TOP 5 - MARCH 2016
B. Ciprofloxacin AUG 2015 MED
C. TMP/SMX BOARDS;
D. Chloramphenicol TOPNOTCH MD
E. NOTA FROM UST)

47 Single dose oral antibacterial agent used to eradicate Ceftriaxone - single dose IM s GEORGE MIDTERM 2
uncomplicated gonorrhoea MICHAEL EXAM -
A. Ceftriaxone SOSUAN (TOP 5 - MARCH 2016
B. Cefixime AUG 2015 MED
C. Azithromycin BOARDS;
D. Both B and C TOPNOTCH MD
E. AOTA FROM UST)

48 Disruption of cell membrane is not seen with Fluconazole inhibits ergosterol syntehsis GEORGE MIDTERM 2
A. Amphotericin B MICHAEL EXAM -
B. Fluconazole SOSUAN (TOP 5 - MARCH 2016
C. Nystatin AUG 2015 MED
D. Polymxin B BOARDS;
E. Griseofulvin TOPNOTCH MD
FROM UST)

49 Highly leukogenic anti-neoplastic drugs Toposiomerase II inhibitors and alkylating agents GEORGE MIDTERM 2
A. Topoisomerase II inhibitor are the most leukogenic anti-neoplastic agents. MICHAEL EXAM -
B. Alkylating agents SOSUAN (TOP 5 - MARCH 2016
C. Anthrcycline antibiotics AUG 2015 MED
D. Both A and B BOARDS;
E. AOTA TOPNOTCH MD
FROM UST)

50 The prokinetic MOA of metoclopramide Metoclopramide inhibits the D2 receptors in the gut; GEORGE MIDTERM 2
A. Inhibition of dopamine receptors in the gut erythromycin stimulates the molitin receptors in the MICHAEL EXAM -
B. Inhibition of motilin receptors in the gut gut. SOSUAN (TOP 5 - MARCH 2016
C. Inhibition of serotonin receptors in the gut AUG 2015 MED
D. AOTA BOARDS;
E. Both A and B TOPNOTCH MD
FROM UST)

51 The following anti-convulsant is not sedating Phenytoin is the oldest non-sedating anti-seizure GEORGE MIDTERM 2
A. Phenobarbital drug MICHAEL EXAM -
B. Phenytoin SOSUAN (TOP 5 - MARCH 2016
C. Clonazepam AUG 2015 MED
D. Gabapentin BOARDS;
E. NOTA TOPNOTCH MD
FROM UST)

TOPNOTCH MEDICAL BOARD PREP PHARMACOLOGY SUPEREXAM Page 7 of 83


For inquiries visit www.topnotchboardprep.com.ph or email us at topnotchmedicalboardprep@gmail.com
TOPNOTCH MEDICAL BOARD PREP PHARMACOLOGY SUPEREXAM
For inquiries visit www.topnotchboardprep.com.ph or email us at topnotchmedicalboardprep@gmail.com
Item QUESTION EXPLANATION AUTHOR TOPNOTCH
# EXAM
52 IOP can be decreased by increasing aqueous outflow PG analogues decrease IOP by increasing aqueous GEORGE MIDTERM 2
with the following drugs outflow. CA inhibitors and Beta-blocker decrease MICHAEL EXAM -
A. Timolol aqeuous production. Alpha2 agonist inreases SOSUAN (TOP 5 - MARCH 2016
B. Latanoprost outflow and decreases production. AUG 2015 MED
C. Mannitol BOARDS;
D. Dorzolamide TOPNOTCH MD
E. NOTA FROM UST)

53 Ampicillin is eliminated by first-order kinetics. Which "First-order" means that the elimination rate is GEORGE MIDTERM 2
of the following statements best describes the proportional to the concentration perfusing the MICHAEL EXAM -
process by which the plasma concentration of this organ of elimination. The half-life is a constant. The SOSUAN (TOP 5 - MARCH 2016
drug declines? rate of elimination is proportional to the rate of AUG 2015 MED
A. There is only 1 metabolic path for drug elimination administration only at steady state. The order of BOARDS;
B. The half-life is the same regardless of the plasma elimination is independent of the number of TOPNOTCH MD
concentration compartments into which a drug distributes. FROM UST)
C. The drug is largely metabolized in the liver after
oral administration and has low bioavailability
D. The rate of elimination is proportional to the rate
of administration at all times
E. The drug is distributed to only 1 compartment
outside the vascular system
54 Which helminthic infection does not respond to In hydatid disease, praziquantel has marginal GEORGE MIDTERM 2
treatment with praziquantel? efficacy because it does not affect the inner germinal MICHAEL EXAM -
A. Hydatid disease membrane of Echinococcus granulosus present in SOSUAN (TOP 5 - MARCH 2016
B. Opisthorchiasis hydatid cysts. AUG 2015 MED
C. Paragonimiasis BOARDS;
D. Pork tapeworm infection TOPNOTCH MD
E. Schistosomiasis FROM UST)

55 This drug that produces a permanent reduction in Radioactive iodine is the only medical therapy that GEORGE MIDTERM 2
thyroid activity is produces a permanent reduction of thyroid activity. MICHAEL EXAM -
A. 131I SOSUAN (TOP 5 - MARCH 2016
B. Amiodarone AUG 2015 MED
C. Propranolol BOARDS;
D. Propylthiouracil TOPNOTCH MD
E. Triiodothyronine FROM UST)

56 Tolerance is seen in which of the following drugs: Tolerance develop with nitrates when used for more GEORGE MIDTERM 2
A. Nitrates than 8 hours without interruption MICHAEL EXAM -
B. Dopamine SOSUAN (TOP 5 - MARCH 2016
C. Epinephine AUG 2015 MED
D. Aspirin BOARDS;
E. AOTA TOPNOTCH MD
FROM UST)

57 The only beta-lactam antibiotcs that does not cross The monobactams have activity against aerobic GEORGE MIDTERM 2
react with penicillins Gram negative bacilli and dose not cross react with MICHAEL EXAM -
A. Ceftriaxone penicillin. SOSUAN (TOP 5 - MARCH 2016
B. Aztreonam AUG 2015 MED
C. Meropenem BOARDS;
D. Aminoglycosides TOPNOTCH MD
E. Vancomycin FROM UST)

58 Which of the following can't be used topically? Topical clindamycin and erythromycin are used GEORGE MIDTERM 2
A. Clindamycin topically againts acne. Topical fusidic acid and MICHAEL EXAM -
B. Erythromycin mupirocin are used topically againts other SOSUAN (TOP 5 - MARCH 2016
C. Amoxicllin superficial bacterial infection. AUG 2015 MED
D. Fusidic acid BOARDS;
E. Mupirocin TOPNOTCH MD
FROM UST)

59 Fifith Generation cephalosporin with activity against Both A and B are fifth generation but only B has GEORGE MIDTERM 2
Pseudomonas aeruginosa activity against Pseudomonas. MICHAEL EXAM -
A. Ceftraline SOSUAN (TOP 5 - MARCH 2016
B. Ceftabiprole AUG 2015 MED
C. Ceftazidime BOARDS;
D. Cefepime TOPNOTCH MD
E. Cefixime FROM UST)

60 Blocks the final common pathway of platelet ASA - inhibits COX; Abciximab - GPIIBIIIA inhibitor; GEORGE MIDTERM 2
aggregation Clopidogrel - ADP receptor inhibitor MICHAEL EXAM -
A. ASA SOSUAN (TOP 5 - MARCH 2016
B. Abciximab AUG 2015 MED
C. Clopidogrel BOARDS;
D. Both A and B TOPNOTCH MD
E. AOTA FROM UST)

61 Which of the ff statements regarding drug elimination Elimination of parent molecules does not terminate JAN CHRISTIAN MIDTERM 3
is correct? the drug's action for drugs with active metabolites. FELICIANO (TOP EXAM -
A. Elimination of parent molecules terminats the First order kinetics exhibits the characteristic half 2 - AUG 2015 MARCH 2016
drug's action for drugs with active metabolites life of most drugs. Zero order kinetics occurs when MED BOARDS;
B. First order eliminination occurs when the drugs have saturated their elimination mechanisms. TOPNOTCH MD
concentration of the drug decreases exponentially Phenobarbital does not display zero roder kinetics FROM UST)
over time
C. Zero order kinetics exhibits the characteristic half
life of most drugs
D. First order kinetics occurs when drugs have
saturated their elimination mechanisms

TOPNOTCH MEDICAL BOARD PREP PHARMACOLOGY SUPEREXAM Page 8 of 83


For inquiries visit www.topnotchboardprep.com.ph or email us at topnotchmedicalboardprep@gmail.com
TOPNOTCH MEDICAL BOARD PREP PHARMACOLOGY SUPEREXAM
For inquiries visit www.topnotchboardprep.com.ph or email us at topnotchmedicalboardprep@gmail.com
Item QUESTION EXPLANATION AUTHOR TOPNOTCH
# EXAM
E. Phenobarbital is an example of a drug that exhibits
zero order kinetics

62 27-year old female presents to the ER approximately The physiological antagonism is related to the fact JAN CHRISTIAN MIDTERM 3
one hour after swallowing 70 propranolol tablets that glucagon activates cAMP through non- FELICIANO (TOP EXAM -
with suicidal intent. At the time of assessment she is adrenergic pathways. Because the adrenergic 2 - AUG 2015 MARCH 2016
drowsy (GCS 13) with a heart rate of 46 bpm and BP pathways are blocked by propanol, glucagon MED BOARDS;
100/60. What drug/class can be given by virtue of essentially uses a back door to enhance myocardial TOPNOTCH MD
physiologic antagonism? activity. FROM UST)
A. Beta agonists
B. Alpha agonists
C. Glucagon
D. Diazepam
E. Atropine
63 A patient was put on standrard dose of warfarin INH is also an inhibitor. Rifampicin is an inducer.. JAN CHRISTIAN MIDTERM 3
therapy, aftter 1 week he suddenly experienced The patient probably is taking a CYP450 inbibitor FELICIANO (TOP EXAM -
hematochezia. Which of the ff drugs he is also such as valproic acid that decreased the metabolism 2 - AUG 2015 MARCH 2016
currently taking could have caused this? of warafarin leading to its toxic side effectds. All the MED BOARDS;
A. Valproic acid rest of the drugs are inducers TOPNOTCH MD
B. Ethanol FROM UST)
C. Carbamazepine
D. Isoniazid
E. Griseofulvin
64 Which of the ff teratogenic drugs causes atrialization Ebstein's anomaly from lithium is a congenital heart JAN CHRISTIAN MIDTERM 3
of the right ventricle? defect in which the septal and posterior leaflets of FELICIANO (TOP EXAM -
A. Misoprostol the tricuspid valve are displaced towards the apex of 2 - AUG 2015 MARCH 2016
B. Lithium the right ventricle of the heart. There is subsequent MED BOARDS;
C. Ethanol 'atrialization' of a portion of the morphologic right TOPNOTCH MD
D. Penicillamine ventricle FROM UST)
E. Phenytoin

65 A patient came to the ER with diarrhea, tremors and The patinet is exhibting symptoms of JAN CHRISTIAN MIDTERM 3
excessive sweating. HR is 50, BP is 120/70, RR is 22. organophosphate poisoning with increased FELICIANO (TOP EXAM -
Pupils are 1-2 mm. What drug can be given to the secretion of all body fluids plus bradycardia, miosis 2 - AUG 2015 MARCH 2016
patient? and skeletal muscle exictation. Antidote is Atropine MED BOARDS;
A. Naloxone and Pralidoxime TOPNOTCH MD
B. Neostigmine FROM UST)
C. Diazepam
D. Atropine
E. EDTA
66 On a patient with signs of shock, At a dose of 5 Dopamine has different dose dependent actions. At JAN CHRISTIAN MIDTERM 3
mcg/kg/min, dopamine will predominanty have what 1-2 mcg/kg/min, it preferentially activates D1 FELICIANO (TOP EXAM -
effect? receptor. At 2-10 mcg, it activates B1 receptor and at 2 - AUG 2015 MARCH 2016
A. Increased peripheral vascular resistance >10 mcg, it activates A1 receptors MED BOARDS;
B. Activation of the RAAS TOPNOTCH MD
C. Inotropic effect FROM UST)
D. Renal vasodilation
E. Arrythmias

67 Which of the ff drugs acts to improve aqueous humor Pilocarpine is a cholinomimeti agent that contracts JAN CHRISTIAN MIDTERM 3
outflow in patients with glaucoma? the ciliary muscle opening the trabecular meshwork. FELICIANO (TOP EXAM -
A. Pilocarpine All of the other drugs decreases secretion of aqueous 2 - AUG 2015 MARCH 2016
B. Mannitol humor MED BOARDS;
C. Apraclonidine TOPNOTCH MD
D. Timolol FROM UST)
E. Acetazlamide

68 All fo the ff are side effects of lithium therapy Lithium decreases thyroid function in most patients JAN CHRISTIAN MIDTERM 3
EXCEPT? exposed to the drug and some show symptoms of FELICIANO (TOP EXAM -
A. Tremors hypothyroidism. Tremors is the most common 2 - AUG 2015 MARCH 2016
B. Hyperthyroidism adverse effect and occurs even on therapeutic doses. MED BOARDS;
C. Nephrogenic diabetic insipidus TOPNOTCH MD
D. Edema FROM UST)
E. Sinus node depression

69 This beta blocker has a unique mechanism of action Nebivolol is a β1 receptor blocker with nitric oxide- JAN CHRISTIAN MIDTERM 3
owing to its it has a nitric oxide (NO)-potentiating potentiating vasodilatory effect used in treatment of FELICIANO (TOP EXAM -
vasodilatory effect in addition to beta blockade. hypertension 2 - AUG 2015 MARCH 2016
A. Carvedilol MED BOARDS;
B. Pindolol TOPNOTCH MD
C. Nadolol FROM UST)
D. Esmolol
E. Nebivolol

TOPNOTCH MEDICAL BOARD PREP PHARMACOLOGY SUPEREXAM Page 9 of 83


For inquiries visit www.topnotchboardprep.com.ph or email us at topnotchmedicalboardprep@gmail.com
TOPNOTCH MEDICAL BOARD PREP PHARMACOLOGY SUPEREXAM
For inquiries visit www.topnotchboardprep.com.ph or email us at topnotchmedicalboardprep@gmail.com
Item QUESTION EXPLANATION AUTHOR TOPNOTCH
# EXAM
70 What is the anti-hypertensive drug of choice for ACE inhibitors are renal efferent arteriole dilators JAN CHRISTIAN MIDTERM 3
patients with diabetic nephropathy? decreasing albumin excretion and slowing FELICIANO (TOP EXAM -
A. Nifedipine progression of CKD. 2 - AUG 2015 MARCH 2016
B. Clonidine MED BOARDS;
C. Enalapril TOPNOTCH MD
D. Atenolol FROM UST)
E. Thiazides

71 Which of the ff electrolyte abnormality will increase Digitalis toxicity is increased in patients with JAN CHRISTIAN MIDTERM 3
digitalis toxicity? hypokalemia, hypomagnesemia and hypercalcemia. FELICIANO (TOP EXAM -
A. Hyponatremia Potassium and digitalis interact by inhibiting each 2 - AUG 2015 MARCH 2016
B. Hypokalemia other’s binding to Na+/K+-ATPase; therefore, MED BOARDS;
C. Hypocalcemia hyperkalemia reduces the enzyme-inhibiting actions TOPNOTCH MD
D. Hypermagnesemia of cardiac glycosides, whereas hypokalemia FROM UST)
E. Hyperphosphatemia facilitates these actions.

72 What is the drug of choice for the rapid termination of Adenosine is the drug of choice for paroxysmal SVT. JAN CHRISTIAN MIDTERM 3
paroxysmal supraventricular tachycardia? Magnesium is the drug of choice for Torsades de FELICIANO (TOP EXAM -
A. Adenosine pointes. Lidocaine is the drug of choice for post MI 2 - AUG 2015 MARCH 2016
B. Magnesium arrythmias and digoxin induced arrythmias. MED BOARDS;
C. Lidocaine TOPNOTCH MD
D. Amiodarone FROM UST)
E. Verapamil

73 A child was brought to you with severe itching due to SIMILAR TO PREVIOUS BOARD EXAM JAN CHRISTIAN MIDTERM 3
multiple insect bites. The mother requested for a non CONCEPT/PRINCIPLE. Among the choices only FELICIANO (TOP EXAM -
sedating anti histaminic agent. What drug can you fexofenadine is a 2nd generation antihistamine. 2 - AUG 2015 MARCH 2016
give? Montelukast is a leukotriene receptor antagonist. All MED BOARDS;
A. Chlorpheniramine the others arer 1st gen antihistamines TOPNOTCH MD
B. Montelukast FROM UST)
C. Meclizine
D. Fexofenadine
E. Hydroxyzine
74 A 15 year old post partum girl in Fabella hospital was SIMILAR TO PREVIOUS BOARD EXAM JAN CHRISTIAN MIDTERM 3
bleeding profusely after giving birth despite oxytocin CONCEPT/PRINCIPLE. Methylergonovine can be FELICIANO (TOP EXAM -
administration, what class of drug can you give as given which is a 5-HT2 receptor antagonist selective 2 - AUG 2015 MARCH 2016
second line treatment? to the uterus. MED BOARDS;
A. 5-HT1 agonist TOPNOTCH MD
B. 5-HT2 antagonist FROM UST)
C. 5-HT3 antagonist
D. 5-HT4 antagonist
E. Alpha 1 agonist
75 This drug class used in asthma exerts its effect by SIMILAR TO PREVIOUS BOARD EXAM JAN CHRISTIAN MIDTERM 3
contraction of engorged vessels in the bronchial CONCEPT/PRINCIPLE. Corticosteroids do not relax FELICIANO (TOP EXAM -
mucosa and reduces bronchial hyperreactivity? airway smooth muscle directly but reduce bronchial 2 - AUG 2015 MARCH 2016
A. Beta 2 agonists reactivity and reduce the frequency of asthma MED BOARDS;
B. Inhaled corticosteroids exacerbations if taken regularly. Their effect on TOPNOTCH MD
C. Methylxanthines airway obstruction may be due in part to their FROM UST)
D. Mast cell stabilizers contraction of engorged vessels in the bronchial
E. Antimuscarinic agent mucosa and their potentiation of the effects of β-
receptor agonists, but their most important action is
inhibition of the infiltration of asthmatic airways by
lymphocytes, eosinophils, and mast cells.
76 Which of the ff anticonvulsant agents do not exerts its Ethosuxime blocks the T type Calcium current in the JAN CHRISTIAN MIDTERM 3
effect through Na channel blocakade? thalamus and is the drug of choice for absence FELICIANO (TOP EXAM -
A. Phenytoin seizures. All the other drugs block Na channels. 2 - AUG 2015 MARCH 2016
B. Carbamazepine MED BOARDS;
C. Topiramate TOPNOTCH MD
D. Valproic acid FROM UST)
E. Ethosuximide

77 Which of the inhaled anesthetics forms a toxic Well known side effects: Halothane-- Hepatitis. JAN CHRISTIAN MIDTERM 3
metabolite, Compound A, that is nephrotoxic at high Isoflurane- Coronary steal syndrome. Enflurane- FELICIANO (TOP EXAM -
doses? Seizures. Sevoflurane- Renal insufficiency. 2 - AUG 2015 MARCH 2016
A. Halothane Desflurane- Pulmonary irritant MED BOARDS;
B. Isoflurane TOPNOTCH MD
C. Enflurane FROM UST)
D. Sevoflurane
E. Desflurane

78 MRSA, a multi drug resistant organism is kept at bay Structural change in target PBP is mechanism of JAN CHRISTIAN MIDTERM 3
through administration of vancomycin. Currently, resistance of MRSA not VRSA. Resistance to FELICIANO (TOP EXAM -
reports of MRSA resistant to vancomycin (VRSA) are vancomycin in vancomycin-resistant S aureus 2 - AUG 2015 MARCH 2016
being reported. What is the mechanism of resistance strains is due to modification of the D-Ala-D-Ala MED BOARDS;
to vancomycin? binding site of the peptidoglycan building block in TOPNOTCH MD
A. Structural change in target PBP which the terminal D-Ala is replaced by D-lactate. FROM UST)
B. Formation of extended spectrum beta lactamases This results in the loss of a critical hydrogen bond
C. D ala-ala is modified to D ala-lactate that facilitates high- affinity binding of vancomycin
D. Changes in porin structure of outer cell wall to its target and loss of activity.
E. Extrusion of the drug through P-glycoprotein
efflux pump

TOPNOTCH MEDICAL BOARD PREP PHARMACOLOGY SUPEREXAM Page 10 of 83


For inquiries visit www.topnotchboardprep.com.ph or email us at topnotchmedicalboardprep@gmail.com
TOPNOTCH MEDICAL BOARD PREP PHARMACOLOGY SUPEREXAM
For inquiries visit www.topnotchboardprep.com.ph or email us at topnotchmedicalboardprep@gmail.com
Item QUESTION EXPLANATION AUTHOR TOPNOTCH
# EXAM
79 This antidiabetic drug because of its unique Acarbose and miglitol are competitive inhibitors of JAN CHRISTIAN MIDTERM 3
mechanism of action should be taken just before the intestinal α-glucosidases and reduce postmeal FELICIANO (TOP EXAM -
ingesting the first portion of each meal? glucose excursions by delaying the digestion and 2 - AUG 2015 MARCH 2016
A. Exenatide absorption of starch and disaccharide. Both MED BOARDS;
B. Miglitol acarbose and miglitol are taken in doses of 25–100 TOPNOTCH MD
C. Pramlintide mg just before ingesting the first portion of each FROM UST)
D. Pioglitazone meal
E. Metformin

80 What anti-TB medication readily penetrates into As stated verbatim in Katzung, rifampicin readily JAN CHRISTIAN MIDTERM 3
phagocytic cells and can kill organisms that are penetrates most tissues and penetrates into FELICIANO (TOP EXAM -
poorly accessible to many other drugs, such as phagocytic cells. It can kill organisms that are poorly 2 - AUG 2015 MARCH 2016
intracellular organisms and those sequestered in accessible to many other drugs, such as intracellular MED BOARDS;
abscesses and lung cavities? organisms and those sequestered in abscesses and TOPNOTCH MD
A. Isoniazid lung cavities. FROM UST)
B. Rifampicin
C. Ethambutol
D. Pyrazinamide
E. Streptomycin
81 1. Which of the following medications is NOT Medscape ANDREW TIU FINAL EXAM -
contraindicated in a patient taking RANOLAZINE? expect questions on never before heard drugs. (TOP 1 - AUG MARCH 2016
a. Losartan 2015 MED
b. Itraconazole BOARDS;
c. clarithromycin TOPNOTCH MD
d. metronidazole FROM CIM)
e. prednisone

82 2. Phoebe Marie, G2 P1 (1001), recently traveled to Halofantrine is not used for chemoprohylaxis ANDREW TIU FINAL EXAM -
Palawan and is taking Quinine for malaria. Which of because of its potential for quinidine like (TOP 1 - AUG MARCH 2016
the following medications can she take? cardiotoxicity (QT prolongation) and 2015 MED
a. clindamycin embryotoxicity. Likewise, doxycycline and BOARDS;
b. doxycycline primaquine are contraindicated during pregnancy. TOPNOTCH MD
c. primaquine Katzung Pharmacology Examination and Board FROM CIM)
d. Halofantrine Review 10th edition p.453
e. all of the above
83 3. Which of the following drugs is a reversible Memantine - low to moderate affinity uncompetitive ANDREW TIU FINAL EXAM -
acetylcholinesterase inhibitor that causes an increase NMDA receptor antagonist blocking receptor only (TOP 1 - AUG MARCH 2016
in concentrations of acetylcholine and is metabolized under conditions of excessive stimulation without 2015 MED
by cholinesterase and excreted mainly by the kidneys effect of normal neurotransmission BOARDS;
and used in Alzheimer's disease? Donepezil - acetylcholinesterase inhibitor but TOPNOTCH MD
a. Memantine metabolized by hepatic P - 450 enzymes FROM CIM)
b. Galantamine Galantamine - increases acetylcholine from
c. Rivastigmine surviving presynaptic nerve terminals by
d. Donepezil modulating nicotinic Ach receptor
e. none of the above Medscape
84 4. Which of the following is the mechanism why http://pedsinreview.aappublications.org/content/2 ANDREW TIU FINAL EXAM -
Cefuroxime is not used for meningitis? 9/8/264.full (TOP 1 - AUG MARCH 2016
a. poor penetration to CSF 2015 MED
b. levels achieved in CSF not adequate to sterilize it BOARDS;
c. potential for delayed sterilization and therapeutic TOPNOTCH MD
failures FROM CIM)
d. presence of efflux pumps
e. none of the above
85 5. Which of the following is the drug topically used for Goodman Gilman 12th edition. Dermatology section ANDREW TIU FINAL EXAM -
acne? (TOP 1 - AUG MARCH 2016
a. allylamine 2015 MED
b. clindamycin BOARDS;
c. mupirocin TOPNOTCH MD
d. bacitracin FROM CIM)
e. none of the above

86 6. Which of the following anti folate drugs is a blood Katzung Pharmacology Examination and Board ANDREW TIU FINAL EXAM -
schizonticide? Review 10th edition p.453 (TOP 1 - AUG MARCH 2016
a. Atovaquone 2015 MED
b. Artesunate BOARDS;
c. Halofantrine TOPNOTCH MD
d. Doxycycline FROM CIM)
e. Pyrimethamine

87 7. Which of the following is a long acting insulin? D and E - short acting ANDREW TIU FINAL EXAM -
a. Lantus A and B - intermediate acting (TOP 1 - AUG MARCH 2016
b. Lente Katzung Pharmacology Examination and Board 2015 MED
c. Glargine Review 10th edition p.363 BOARDS;
d. Detemir TOPNOTCH MD
e. Glulisine FROM CIM)

88 8. Which of the following is NOT true of Gentamicin? has greater efficacy when administered as a single ANDREW TIU FINAL EXAM -
a. killing action continues when plasma levels decline large dose than when given as multiple smaller (TOP 1 - AUG MARCH 2016
below measurable levels doses 2015 MED
b. lesser efficacy when administered as a single large Katzung Pharmacology Examination and Board BOARDS;
dose Review 10th edition p.399 TOPNOTCH MD
c. in vivo efficacy not directly related to time above FROM CIM)
MIC
d. as plasma level is increased above MIC, it kills an
increasing proportion of bacteria at a more rapid rate
e. toxicity depends on both critical plasma
TOPNOTCH MEDICAL BOARD PREP PHARMACOLOGY SUPEREXAM Page 11 of 83
For inquiries visit www.topnotchboardprep.com.ph or email us at topnotchmedicalboardprep@gmail.com
TOPNOTCH MEDICAL BOARD PREP PHARMACOLOGY SUPEREXAM
For inquiries visit www.topnotchboardprep.com.ph or email us at topnotchmedicalboardprep@gmail.com
Item QUESTION EXPLANATION AUTHOR TOPNOTCH
# EXAM
concentration and the time that such level is
exceeded

89 9. Which of the following chemotherapeutic drugs is Medscape ANDREW TIU FINAL EXAM -
used for bladder cancer (intravesical administration) expect questions on never before heard drugs. (TOP 1 - AUG MARCH 2016
and control of malignant pericardial/ pleural/ 2015 MED
peritoneal effusions (intracavitary)? BOARDS;
a. cyclophosphamide TOPNOTCH MD
b. thiotepa FROM CIM)
c. carmustine
d. lomustine
e. none of the above
90 10. Which of the following is a mixed FSH and LH B - FSH only ANDREW TIU FINAL EXAM -
agonist and is used to stimulate gonadal function C - LH only (TOP 1 - AUG MARCH 2016
including spermatogenesis and ovulation? D - GH agonist 2015 MED
a. menotropin E - GnRH agonist BOARDS;
b. urofollitropin Katzung Pharmacology Examination and Board TOPNOTCH MD
c. Lutropin Review 10th edition p.327 FROM CIM)
d. Somatropin
e. Gonadorelin
91 11. Which of the following is not likely used for D is used for CMV ANDREW TIU FINAL EXAM -
treatment of Herpes Simplex virus? Katzung Pharmacology Examination and Board (TOP 1 - AUG MARCH 2016
a. Acyclovir Review 10th edition p.429 2015 MED
b. Penciclovir BOARDS;
c. Famciclovir TOPNOTCH MD
d. Ganciclovir FROM CIM)
e. none of the above

92 12. A neonate in the NICU was observed by the ANDREW TIU FINAL EXAM -
Neonatologist fellow to have epicanthal folds, low (TOP 1 - AUG MARCH 2016
nasal bridge, short palpebral fissures, flat midface, 2015 MED
and indistinct philtrum. Which of the following BOARDS;
medications did the mother likely take during TOPNOTCH MD
pregnancy? FROM CIM)
a. phenytoin
b. carbamazepine
c. valproic acid
d. ethyl alcohol
e. warfarin
93 13. which of the following is the chemoprophylactic http://www.who.int/ith/ITH_chapter_7.pdf ANDREW TIU FINAL EXAM -
drug of choice for pregnant patients visiting (TOP 1 - AUG MARCH 2016
falciparum malaria endemic places? 2015 MED
a. chloroquine BOARDS;
b. mefloquine TOPNOTCH MD
c. doxycycline FROM CIM)
d. atovaquone - proguanil
e. artemether - lumefantrine
94 14. Which of the following is NOT true of phenytoin? metabolism is enhanced by presence of inducers of ANDREW TIU FINAL EXAM -
a. metabolism is nonlinear liver metabolism (phenobarbital, rifampin) and (TOP 1 - AUG MARCH 2016
b. drug binds extensively to proteins in plasma inhibited by other drugs (cimetidine, isoniazid) 2015 MED
c. fosphenytoin in the water soluble prodrug for Katzung Pharmacology Examination and Board BOARDS;
parenteral administration Review 10th edition p.217 TOPNOTCH MD
d. metabolism is induced by isoniazid FROM CIM)
e. drug of choice for grand mal seizures

95 15. A 54 year old male presents to the clinic with a acedapsone is the repository form that provides ANDREW TIU FINAL EXAM -
circular lesion on the arm and anesthesia on pin prick inhibitory plasma concentrations for several (TOP 1 - AUG MARCH 2016
of the center of the lesion. Patient was then brought months. It is the most active drug for leprosy. 2015 MED
to Cebu Skin Clinic for further evaluation. Dr. AM Mechanism of action is inhibition of folic acid BOARDS;
impression was leprosy tuberculoid type and decides synthesis. It is contraindicated to G6PD patients. TOPNOTCH MD
to start the patient with antimycobacterials. Which of Katzung Pharmacology Examination and Board FROM CIM)
the following is true of Dapsone? Review 10th edition p.415
a. fluorodapsone is the respository form
b. it is used as an alternative drug for treatment of
PCP pneumonia
c. it is not the most active drug for Leprosy
d. mechanism of action is to inhibit arabinosyl
transferase
e. can be given to G6PD patients
96 16. DC, a 69 year old female, presented with resting Katzung Pharmacology Examination and Board ANDREW TIU FINAL EXAM -
tremors and shuffling gait. Upon cleaning her Review 10th edition p.249 (TOP 1 - AUG MARCH 2016
bathroom, she accidentally damaged the bathroom 2015 MED
sink thus prompting consult. Having a good BOARDS;
understanding of the pathophysiology of Parkinson’s TOPNOTCH MD
disease, Dr. MC starts the patient with dopamine FROM CIM)
agonists. Which of the following can be given as
monotherapy for mild parkinsonism and has high
affinity for D3 receptor?
a. bromocriptine
b. pramipexole
c. ropinirole
d. apomorphine
TOPNOTCH MEDICAL BOARD PREP PHARMACOLOGY SUPEREXAM Page 12 of 83
For inquiries visit www.topnotchboardprep.com.ph or email us at topnotchmedicalboardprep@gmail.com
TOPNOTCH MEDICAL BOARD PREP PHARMACOLOGY SUPEREXAM
For inquiries visit www.topnotchboardprep.com.ph or email us at topnotchmedicalboardprep@gmail.com
Item QUESTION EXPLANATION AUTHOR TOPNOTCH
# EXAM
e. cabergoline

97 17. Which of the following is the regimen for a 29 A - hodgkin’s lymphoma ANDREW TIU FINAL EXAM -
year old male with a testicular mass? B - ovarian (TOP 1 - AUG MARCH 2016
a. ABVD regimen C - pancreatic 2015 MED
b. paclitaxel + carboplatin D - colorectal BOARDS;
c. gemcitabine + erlotinib Katzung Pharmacology Examination and Board TOPNOTCH MD
d. fluorouracil and leucovorin + oxaliplatin Review 10th edition p.468 FROM CIM)
e. PEB regimen

98 18. NT a 39 year old male with adult polycystic Cyclosporine binds to cyclophilin and tacrolimus ANDREW TIU FINAL EXAM -
kidney disease underwent kidney transplant. Which binds to FKBP. Both complexes inhibit calcineurin, a (TOP 1 - AUG MARCH 2016
of the following immunophilin inhibitors bind to FK - cytoplasmic phosphatase. Calcineurin regulates the 2015 MED
binding protein 12 and inhibit the mTOR pathway ability of the nuclear factor of activated T cells to BOARDS;
which inhibits the T - cell proliferation response to IL translocate to the nucleus and increase the TOPNOTCH MD
- 2? production of key cytokines such as IL 2, 3, and IFN FROM CIM)
a. sirolimus gamma. Cyclophilin and tacrolimus prevent the
b. tacrolimus increased production of cytokines that normally
c. cyclosporin occurs in response to T cell receptor activation.
d. cyclophiline Katzung Pharmacology Examination and Board
e. none of the above Review 10th edition p.482
99 19. A 23 year old female came in for unilateral Ergot alkaloids can produce marked and prolonged ANDREW TIU FINAL EXAM -
throbbing headache associated with nausea and alpha receptor mediated vasoconstriction. (TOP 1 - AUG MARCH 2016
photophobia. Which of the following ergot alkaloids Ergotamine is the prototype and has been a 2015 MED
has no effects on the dopamine receptor D2 but mainstay of treatment of acute attacks of migraine. BOARDS;
stimulates the uterine smooth muscle? Katzung Pharmacology Examination and Board TOPNOTCH MD
a. bromocriptine Review 10th edition p.160 FROM CIM)
b. ergonovine
c. ergotamine
d. LSD
e. methylergometrine
100 20. Which of the following prostaglandin derivatives Latanoprost increases outflow of aqueous humor ANDREW TIU FINAL EXAM -
increases outflow of aqueous humor? and reduces intraocular pressure in glaucoma. (TOP 1 - AUG MARCH 2016
a. misoprostol Katzung Pharmacology Examination and Board 2015 MED
b. alprostadil Review 10th edition p.176 BOARDS;
c. dinoprostone TOPNOTCH MD
d. epoprostenol FROM CIM)
e. none of the above

101 Thru what route is the usual portal of entry in Page 94 of Topnotch Handout. The usual setting of ANGELA DIAGNOSTIC
Inorganic Mercury poisoning?
either acute or chronic inorganic mercury poisoning PAULINE P. EXAM - AUG
A. Ingestion is thru Inhalation of inorganic mercury vapor. CALIMAG- 2015
B. Topical application LOYOLA (TOP 8 -
C. Inhalation FEB 2015 MED
D. Rectal BOARDS;
E. None of the above TOPNOTCH MD
FROM UST)
102 Using the Henderson-Hasselbach equation, in a pH Page 2 of Topnotch Handout. According to the ANGELA DIAGNOSTIC
above the dissociation constant, a weak acid is Henderson-Hasselbach equation Weak acid above PAULINE P. EXAM - AUG
present in the _____________ form.
the pKa is unprotonated and charged. CALIMAG- 2015
A. unprotonated, uncharged LOYOLA (TOP 8 -
B. protonated, charged FEB 2015 MED
C. protonated, uncharged BOARDS;
D. unprotonated, charged TOPNOTCH MD
E. unpredictable FROM UST)

103 The rectal route is said to have partial avoidance of Page 3 of Topnotch Handout. The superior rectal ANGELA DIAGNOSTIC
the first pass effect. Thru which of the following vein will drain to the Inferior mesenteric vein then PAULINE P. EXAM - AUG
venous drainage of the rectum will not bypass the to the portal vein, hence having a first pass effect. CALIMAG- 2015
first pass effect :
The middle rectal vein will drain thru the internal LOYOLA (TOP 8 -
A. Superior rectal vein iliac vein then to the inferior vena cava, while the FEB 2015 MED
B. Middle rectal vein inferior rectal vein will drain via the internal BOARDS;
C. Inferior rectal vein pudendal vein then thru the internal iliac vein then TOPNOTCH MD
D. Both B and C thru the inferior vena cava. FROM UST)
E. All of the above
104 15 units of Drug X was given to a patient. If this drug Page 4 of Topnotch Handout. In a drug undergoing ANGELA DIAGNOSTIC
undergoes first-order elimination how many half-lifes first-order elimination concentration decreases by PAULINE P. EXAM - AUG
will it take for the drug concentration to reach 1.88 50% for every half-life. SO if the drug was 15 units at CALIMAG- 2015
units:
the start, 1st half life- 7.5, 2nd half life 3.75, third LOYOLA (TOP 8 -
A. 1 half life 1.875. FEB 2015 MED
B. 2 BOARDS;
C. 3 TOPNOTCH MD
D. 4 FROM UST)
E. 5

TOPNOTCH MEDICAL BOARD PREP PHARMACOLOGY SUPEREXAM Page 13 of 83


For inquiries visit www.topnotchboardprep.com.ph or email us at topnotchmedicalboardprep@gmail.com
TOPNOTCH MEDICAL BOARD PREP PHARMACOLOGY SUPEREXAM
For inquiries visit www.topnotchboardprep.com.ph or email us at topnotchmedicalboardprep@gmail.com
Item QUESTION EXPLANATION AUTHOR TOPNOTCH
# EXAM
105 Which of the following statements is/are true Page 4 of Topnotch Handout. Binding affinity refers ANGELA DIAGNOSTIC
regarding the binding affinity of a drug:
to the fraction of receptors bound by a drug plotted PAULINE P. EXAM - AUG
A. Kd is the concentration required to bind 100% of against the log of the drug concentration. Kd is the CALIMAG- 2015
the drug receptors concentration required to bind 50% of the LOYOLA (TOP 8 -
B. The smaller the Kd the lesser the affinity of the receptors. The smaller the Kd, the greater the FEB 2015 MED
drug for its receptor affinity of a drug for its receptor, the more potent it BOARDS;
C. A smaller Kd means that the drug is less potent is. TOPNOTCH MD
D. Binding affinity refers to the fraction of receptors FROM UST)
bound by a drug plotted against the log of the drug
effect
E. None of the above
106 A drug is 90% cleared by the liver and 10% by the Page 7 of Topnotch Handout. Normal creatinine ANGELA DIAGNOSTIC
kidney. The normal dosage of the drug is 500mg/d. If clearance for healthy men is 97-137 mL/min. PAULINE P. EXAM - AUG
this drug is to be given for a patient with a creatinine Normal creatinine clearance for healthy women is CALIMAG- 2015
clearance of 30ml/min, what should be the corrected 88-128 mL/min. In this patient his crea clearance is LOYOLA (TOP 8 -
dose: A. 15 only 30ml/min. Hence to compute for the corrected FEB 2015 MED
B. 50 dose the formula is Corrected dose= Average dose BOARDS;
C. 450 (aaplied only to the part of the dose cleared by the TOPNOTCH MD
D. 465 kidney) x patient's crea clearance/ 100ml/min. In FROM UST)
E. 500 this case 500 x 0.10= 50; 30/100= 0.3; ).3 x 50= 15.
Corrected dose is 500 x 0.90=450+ 15=465.
107 Which of the following teratogenic drug and effect is Page 9 of Topnotch Handout. ANGELA DIAGNOSTIC
correctly paired?
Sulfonamides:kernicterus; Misoprostol: Mobius PAULINE P. EXAM - AUG
A. Sulfonamides: ototoxicity sequence; Phenytoin: Fetal hydantoin syndrome; CALIMAG- 2015
B. Penicillamine: cutis laxa Lithium: Ebstein's anomaly. LOYOLA (TOP 8 -
C. Misoprostol: Ebstein's anomaly FEB 2015 MED
D. Phenytoin: Mobius sequence BOARDS;
E. Lithium: kernicterus TOPNOTCH MD
FROM UST)
108 This drug inhibits the transport of Acetylcholine into Page 11 of Topnotch Handout. Storage of Ach into ANGELA DIAGNOSTIC
vesicles for storage by VAT:
vesicles is inhibited by vesamicol. PAULINE P. EXAM - AUG
A. Hemicholinium CALIMAG- 2015
B. Botulinum LOYOLA (TOP 8 -
C. Neostigmine FEB 2015 MED
D. Betanechol BOARDS;
E. Vesamicol TOPNOTCH MD
FROM UST)
109 A 55 y/o male presented at the ER with complaints of Page 12 of hand out. The patient is experiencing ANGELA DIAGNOSTIC
3 day diarrhea, frequent urination, increased symptoms of organophosphate poisoning. PAULINE P. EXAM - AUG
salivation and sweating. You noted his pupils to be Remember the DUMBBELSS mnemonic! Diarrhea, CALIMAG- 2015
constricted. He works in a factory manufacturing Urination, Miosis, Bronchospasm, Bradycardia, LOYOLA (TOP 8 -
pesticides for farms. Which of the following drugs will Excitation (skeletal muscle and CNS), Lacrimation, FEB 2015 MED
you give this patient?
Sweating and Salivation. The antidote to be given in BOARDS;
A. Pyridostigmine organophosphate poisoning is Atropine (Cholinergic TOPNOTCH MD
B. Edrophonium antagonist, muscarinic) and Pralidoxime FROM UST)
C. Pilocarpine (Cholinesterase regenerator)
D. Atropine
E. Varenicline
110 Your patient is in cardiogenic shock. Which of the Page 15 of Topnotch Handout. In cardiogenic shock ANGELA DIAGNOSTIC
following is the drug of choice to increase cardiac the DOC is Dobutamine. It is a selective Beta 1 PAULINE P. EXAM - AUG
contractility without the vasoconstricting effects?
Agonist hence it has no Alpha 1 vasoconstricting CALIMAG- 2015
A. Norepinephrine effects. LOYOLA (TOP 8 -
B. Dopamine FEB 2015 MED
C. Epinephrine BOARDS;
D. Phenylephrine TOPNOTCH MD
E. Dobutamine FROM UST)

111 Which of the following beta-blockers are less likely to Page 16 of Topnotch Handout. The two beta blockers ANGELA DIAGNOSTIC
cause bronchospasm in patients with asthma due to with intrinsic sympathomimetic activity are Pindolol PAULINE P. EXAM - AUG
its partial agonist activity?
and Acebutolol. CALIMAG- 2015
A. Carvedilol LOYOLA (TOP 8 -
B. Labetalol FEB 2015 MED
C. Pindolol BOARDS;
D. Propanolol TOPNOTCH MD
E. Nadolol FROM UST)

112 A 65 y/o male on Clonidine for a few months for his Page 17 of Topnotch Handout. For rebound ANGELA DIAGNOSTIC
hypertension abruptly discontinued his medication hypertension due to clonidine withdrawal the DOC PAULINE P. EXAM - AUG
because he felt he was feeling better. After 2 days, is Phentolamine. Clonidine is an alpha 2 agonist, CALIMAG- 2015
during which he had strong headaches. Three days sympathetic outflow blocker. Phentolamine Its LOYOLA (TOP 8 -
after withdrawal, the patient had home blood primary action is vasodilation due to α1 blockade, FEB 2015 MED
pressure levels of 140–150/95–105 mmHg. Ten days but can also lead to reflex tachycardia because of BOARDS;
after, the patient went to the emergency room with a hypotension and α2 inhibition, which increases TOPNOTCH MD
symptomatic hypertensive crisis at 220/130 mmHg. sympathetic tone. The primary application for FROM UST)
What is the drug of choice to treat the rebound phentolamine is for the control of hypertensive
hypertension?
emergencies.
A. Phentolamine
B. Hydrochlorthiazide
C. Propanolol
D. Hydralazine
E. Nicardipine

TOPNOTCH MEDICAL BOARD PREP PHARMACOLOGY SUPEREXAM Page 14 of 83


For inquiries visit www.topnotchboardprep.com.ph or email us at topnotchmedicalboardprep@gmail.com
TOPNOTCH MEDICAL BOARD PREP PHARMACOLOGY SUPEREXAM
For inquiries visit www.topnotchboardprep.com.ph or email us at topnotchmedicalboardprep@gmail.com
Item QUESTION EXPLANATION AUTHOR TOPNOTCH
# EXAM
113 By which mechanism does Angiotensin converting Page 19 of Topnotch Handout.ACE inhibitors ANGELA DIAGNOSTIC
enzyme inhibitors cause cough?
produce vasodilation by inhibiting the formation of PAULINE P. EXAM - AUG
A. By inhibiting ACE and formation of angiotensin II angiotensin II. ACE also breaks down bradykinin (a CALIMAG- 2015
B. By blocking the breakdown of bradykinin vasodilator substance). Therefore, ACE inhibitors, by LOYOLA (TOP 8 -
C. By facilitating the release of norepinephrine from blocking the breakdown of bradykinin, increase FEB 2015 MED
sympathetic adrenergic nerves bradykinin levels, which can contribute to the BOARDS;
D. By inhibiting norepinephrine reuptake vasodilator action of ACE inhibitors. The increase in TOPNOTCH MD
E. By blocking angiotensin II stimulation of bradykinin is also believed to be responsible for a FROM UST)
aldosterone secretion troublesome side effect of ACE inhibitors, namely, a
dry cough. Angiotensin II also facilitates the release
of norepinephrine from sympathetic adrenergic
nerves and inhibits norepinephrine reuptake by
these nerves. This effect of angiotensin II augments
sympathetic activity on the heart and blood vessels.
ACE inhibitors promote renal excretion of sodium
and water (natriuretic and diuretic effects) by
blocking the effects of angiotensin II in the kidney
and by blocking angiotensin II stimulation of
aldosterone secretion. This reduces blood volume,
venous pressure and arterial pressure.
114 This drug belongs to a class of antiarrhythmic drugs Page 24 of Topnotch Handout. Under the Singh- ANGELA DIAGNOSTIC
which prolongs AP duration, PR interval, QRS Vaughn Williams classification. The class being PAULINE P. EXAM - AUG
duration and QT interval: A. Procainamide described are Class 1A Antiarrhythmics. Classified CALIMAG- 2015
B. Lidocaine underwhich is Procainamide Quinidine and LOYOLA (TOP 8 -
C. Flecainide Disopyramide. FEB 2015 MED
D. Propanolol BOARDS;
E. Dofetilide TOPNOTCH MD
FROM UST)
115 A patient was diagnosed with familial combined Page 29 of Topnotch Handout. A synergistic ANGELA DIAGNOSTIC
hypercholesterolemia, which of the following combination for patients with familial combined PAULINE P. EXAM - AUG
synergistic drug combination will be most suitable for hypercholesterolemia (overproduction of VLDL) are CALIMAG- 2015
this patient:
niacin+resin and statin+fibrate. Fibrate nd resin LOYOLA (TOP 8 -
A. Niacin + Statin combinations are disadvantageous because they FEB 2015 MED
B. Statin + Ezetimibe confer an increased risk of cholelithiasis. while BOARDS;
C. Niacin + Resin Statin and resin combinations cause impaired statin TOPNOTCH MD
D. Statin + Resin absorption. Niacin and statin combinations are more FROM UST)
E. Fibrate + Resin for familial hypercholesterolemiawhere the defect is
in the LDL receptors hence increased LDL.
116 This prostaglandin F2 alpha analog is commonly Page 32 of Topnotch Handout. Latanoprost is ANGELA DIAGNOSTIC
used off label for its side effect which is lengthening commonly used off label for eyelash lengthening. It PAULINE P. EXAM - AUG
of eyelashes: A. Beraprost is a prostaglandin F2alpha analog that activates FP CALIMAG- 2015
B. Dinoprostone receptors, increases outflow of aquaeous humor and LOYOLA (TOP 8 -
C. Alprostail reduces intraocular pressure. Its other side effects FEB 2015 MED
D. Latanoprost are Blurred vision, eye irritation, or tearing BOARDS;
E. Gemeprost darkening of eyelid skin color, increase in brown TOPNOTCH MD
color in colored part of eye. FROM UST)
117 Which of the following statements is true for SIMILAR TO PREVIOUS BOARD EXAM ANGELA DIAGNOSTIC
cromolyn:
CONCEPT/PRINCIPLE. Page 33 of Topnotch PAULINE P. EXAM - AUG
A. It reduces synthesis of leukotrienes Handout. Cromolyn is a mast cell stabilizer by the CALIMAG- 2015
B. It has no bronchodilator action prevention of calcium influx, therefore preventing LOYOLA (TOP 8 -
C. It binds IgE antibodies on mast cells degranulation and release of histamine, leukotrienes FEB 2015 MED
D. It reduces the expression of cyclooxygenase and other mediators. A is moa of zileuton, C is for BOARDS;
E. It prevents vagal-stimulated bronchoconstriction omalizumab, D is for corticosteroids while E is for TOPNOTCH MD
Ipratropium 9muscarinic receptor antagonist. FROM UST)
118 A 23 y/o female previous RHD patient s/p valve Page 39 of Topnotch Handout. CYP 450 inhibitors ANGELA DIAGNOSTIC
replacement is being maintained on warfarin. 3 days prolong/increase warfarin's anticoagulant PAULINE P. EXAM - AUG
prior she had symptoms of PUD for which she self effectsince they inhibit drug clearance. Cimetidine is CALIMAG- 2015
medicated. She now presented at the ER due to black a CYP450 inhibitor. LOYOLA (TOP 8 -
tarry stools, epigastric pain, coffee ground emesis, FEB 2015 MED
gum bleeding and dizziness. Laboratory showed BOARDS;
prolonged PT. Which of the following drugs caused a TOPNOTCH MD
drug interaction with warfarin prolonging its FROM UST)
anticoagulant effect?
A. Cimetidine
B. Omeprazole
C. Bismuth
D. Sucralfate
E. Maalox
119 A 30 y/o male patient developed Page 52 of Topnotch Handout. For ANGELA DIAGNOSTIC
methemoglobinemia after receiving prilocaine during methemoglobinemia the antidote is methylene blue. PAULINE P. EXAM - AUG
a minor surgery. What should be administered to this CALIMAG- 2015
patient? LOYOLA (TOP 8 -
A. NAC FEB 2015 MED
B. Methylene blue BOARDS;
C. EDTA TOPNOTCH MD
D. Dimercaprol FROM UST)
E. Deferoxamine
120 Which nondepolarizing neuromuscular blocker Page 53 of Topnotch Handout. Atracurium is broken ANGELA DIAGNOSTIC
undergoes rapid breakdown by Hoffman elimination? down to inactive metabolites by (minor) ester PAULINE P. EXAM - AUG
A. Vecuronium hydrolysis and spontaneous Hoffman elimination CALIMAG- 2015
B. Mivacurium (major pathway) to Laudanosine. LOYOLA (TOP 8 -
C. Atracurium FEB 2015 MED
D. Tubcurarine BOARDS;
E. Rocuronium TOPNOTCH MD
FROM UST)

TOPNOTCH MEDICAL BOARD PREP PHARMACOLOGY SUPEREXAM Page 15 of 83


For inquiries visit www.topnotchboardprep.com.ph or email us at topnotchmedicalboardprep@gmail.com
TOPNOTCH MEDICAL BOARD PREP PHARMACOLOGY SUPEREXAM
For inquiries visit www.topnotchboardprep.com.ph or email us at topnotchmedicalboardprep@gmail.com
Item QUESTION EXPLANATION AUTHOR TOPNOTCH
# EXAM
121 These are agents or drugs that bind to a different Competitive/Reversible antagonist - bind to LYNN DARYL MIDTERM 1
receptor, producing an effect opposite to that receptors in a reversible way without activating the FELICIANO EXAM - AUG
produced by the drug it is antagonizing: effector system; Non-competitive/Irreversible VILLAMATER, 2015
A. Competitive antagonists antagonist - cause downward shift of the DRC; MD (TOP 5 - FEB
B. Irreversible antagonists Chemical antagonist - interact directly with the drug 2015 MED
C. Chemical antagonists being antagonized; PHYSIOLOGIC ANTAGONIST is BOARDS;
D. Physiologic antagonists the answer. Examples are histamine and TOPNOTCH MD
E. Partial agonists epinephrine, propanolol and thyroid hormone. FROM EAC)
SIMILAR TO PREVIOUS BOARD EXAM CONCEPT.
122 The patient is being maintained on oral anticoagulant. Phenobarbital, a barbiturate, is a Cytochrome P450 LYNN DARYL MIDTERM 1
Which of the following if taken by the patient will inducer which will increase clearance of other drugs, FELICIANO EXAM - AUG
decrease the effect of the drug he is presently taking? thus, decreasing their effects. SIMILAR TO VILLAMATER, 2015
A. Phenobarbital PREVIOUS BOARD EXAM CONCEPT. MD (TOP 5 - FEB
B. Sulfamethoxazole 2015 MED
C. Valproic acid BOARDS;
D. Ketoconazole TOPNOTCH MD
E. Vancomycin FROM EAC)

123 Which of the following is a mechanism of action of Hydralazine binds to and activates potassium LYNN DARYL MIDTERM 1
Hydralazine? channels on vascular smooth muscle resulting to FELICIANO EXAM - AUG
A. Alters intracellular calcium efflux of potassium and subsequent VILLAMATER, 2015
B. Opens potassium channel causing arteriolar and hyperpolarization of the cell. This prevents calcium- MD (TOP 5 - FEB
venular dilatation mediated activation and constriction of smooth 2015 MED
C. Blocks alpha-1 adrenergic receptor muscle, resulting in vasodilation. It dilates BOARDS;
D. Competitively bloocks Nn nicotinic receptor arterioles, but not veins. SIMILAR TO PREVIOUS TOPNOTCH MD
E. Blocks voltage-gated L-type calcium channels BOARD EXAM CONCEPT. FROM EAC)
124 The drug of choice for paroxysmal Supraventricular SIMILAR TO PREVIOUS BOARD EXAM CONCEPT. LYNN DARYL MIDTERM 1
tachycardia is: FELICIANO EXAM - AUG
A. Procainamide VILLAMATER, 2015
B. Verapamil MD (TOP 5 - FEB
C. Amiodarone 2015 MED
D. Lidocaine BOARDS;
E. Adenosine TOPNOTCH MD
FROM EAC)
125 The most common anti-arrythmic drug used for Lidocaine is the drug of choice for ventricular LYNN DARYL MIDTERM 1
cardiac arrest is: arrythmias post-MI, and digoxin-induced FELICIANO EXAM - AUG
A. Lidocaine arrythmias. It is the least cardiotoxic among VILLAMATER, 2015
B. Adenosine conventional anti-arrythmic. SIMILAR TO MD (TOP 5 - FEB
C. Flecainide PREVIOUS BOARD EXAM CONCEPT.. 2015 MED
D. Sotalol BOARDS;
E. Amiodarone TOPNOTCH MD
FROM EAC)
126 Which of the following diuretics may cause Amiloride (also Triamterene, Eplerenone, LYNN DARYL MIDTERM 1
hyperkalemia? Spironolactone) is a potassium sparing diuretics. All FELICIANO EXAM - AUG
A. Amiloride other options are potassium-wasting. SIMILAR TO VILLAMATER, 2015
B. Ethacrynic acid PREVIOUS BOARD EXAM CONCEPT. MD (TOP 5 - FEB
C. Indapamide 2015 MED
D. Mannitol BOARDS;
E. Acetozalamide TOPNOTCH MD
FROM EAC)
127 Which of the following anesthetic agent has analgesic Nitrous oxide , not nitric oxide, is used as an LYNN DARYL MIDTERM 1
property and used as an adjunct to other anesthetics? anesthetic agent with analgesic property. Fentanyl FELICIANO EXAM - AUG
A. Halothane is often used for anesthesia and analgesia. SIMILAR VILLAMATER, 2015
B. Nitric oxide TO PREVIOUS BOARD EXAM CONCEPT. MD (TOP 5 - FEB
C. Etomidate 2015 MED
D. Desflurane BOARDS;
E. Fentanyl TOPNOTCH MD
FROM EAC)

128 A 50-year old male with DM type II had been on This is a case of hypomagnesemia, which can be LYNN DARYL MIDTERM 1
metformin for nine years and presented with exacerbated by administration of a loop diuretic FELICIANO EXAM - AUG
diarrhea, paresthesia, and muscular weakness. On Furosemide is a loop diuretic that acts on the thick VILLAMATER, 2015
examination, he has hyperactive DTRs. Blood ascending limb of the loop of henle. This tubular MD (TOP 5 - FEB
electrolytes showed serum Na 138 mEq/L, Serum K= segment is responsible for a significant sodium 2015 MED
3.4 mEq/L, serum Ca=8.2 mg/dL and Mg 0.8 mEq/L. chloride reabsorption, as well as the site for calcium BOARDS;
Which of the following medications should you avoid and magnesium reabsorption. Side effects of TOPNOTCH MD
to prevent exacerbation of his symptoms? Furosemide : Hypokalemia, hypocalcemia, FROM EAC)
A. Spironolactone hypomagnesemia, hyperuricemia, dehydration,
B. Magnesium sulfate metabolic alkalosis, ototoxicity, sulfa allergy,
C. Furosemide nephritis
D. Indomethacin
E. Acetazolamide
129 Dicycloverine is used for: Dicycloverine is an antispasmodic, prescribed for LYNN DARYL MIDTERM 1
A. Paralytic ileus gastrointestinal tract spasm and irritable bowel FELICIANO EXAM - AUG
B. Biliary colic syndrome. It blocks the activity of acetylcholine on VILLAMATER, 2015
C. Intestinal spasm muscarinic receptors. It should not be used in MD (TOP 5 - FEB
D. Constipation patients with PARALYTIC ILEUS, myasthenia gravis, 2015 MED
E. Urinary retention narrow angle glaucoma, enlarged prostate, or BOARDS;
pyloric stenosis. Constipation is a side-effect of this TOPNOTCH MD
drug. SIMILAR TO PREVIOUS BOARD EXAM FROM EAC)
CONCEPT.

TOPNOTCH MEDICAL BOARD PREP PHARMACOLOGY SUPEREXAM Page 16 of 83


For inquiries visit www.topnotchboardprep.com.ph or email us at topnotchmedicalboardprep@gmail.com
TOPNOTCH MEDICAL BOARD PREP PHARMACOLOGY SUPEREXAM
For inquiries visit www.topnotchboardprep.com.ph or email us at topnotchmedicalboardprep@gmail.com
Item QUESTION EXPLANATION AUTHOR TOPNOTCH
# EXAM
130 Which of the following is NOT an effect of Naloxone? Naloxone is an opioid antagonist used for opioid LYNN DARYL MIDTERM 1
A. It will cause respiratory depression overdose. It competitively blocks opioid receptors FELICIANO EXAM - AUG
B. It may cause nausea and vomiting and rapidly reverses effects of opioid agonists. In VILLAMATER, 2015
C. It induces abstinence syndrome individuals who are acutely depressed by an MD (TOP 5 - FEB
D. It decreases constipation overdose of an opioid, the antagonist effectively 2015 MED
E. None of the above normalizes respiration, LOC, pupil size, bowel BOARDS;
activity, and awareness of pain. In dependent TOPNOTCH MD
subjects who appear normal while taking opioids, FROM EAC)
naloxone or naltrexone almost instantaneously
precipitates an abstinence syndrome. SIMILAR TO
PREVIOUS BOARD EXAM CONCEPT
131 The combination of metronidazole and alcohol will Metronidazole has a disulfiram-effect, so that nausea LYNN DARYL MIDTERM 1
most likely cause? and vomiting can occur if alcohol is ingested during FELICIANO EXAM - AUG
A. Ataxia therapy. (Katzung) SIMILAR TO PREVIOUS BOARD VILLAMATER, 2015
B. Blurring of vision EXAM CONCEPT. MD (TOP 5 - FEB
C. Dizziness 2015 MED
D. Nausea and vomiting BOARDS;
E. Pancreatitis TOPNOTCH MD
FROM EAC)
132 The mechanism of action of Nedocromil: Option A - Beta 2 agonist (Salbutamol, Terbutaline); LYNN DARYL MIDTERM 1
A. Activates beta-2 receptors in bronchial smooth Option C- leukotriene synthesis inhibitor (Zileuton); FELICIANO EXAM - AUG
muscle Option D - Muscarinic receptor antagonist VILLAMATER, 2015
B. Prevents calcium influx and stabilizes mast cells (Ipratropium); Option E - Leukotriene antagonist MD (TOP 5 - FEB
preventing release of histamine (Zafirlukast). Cromolyn, Nedocromil, and 2015 MED
C. Inhibits 5-lipoxygenase Lodoxamide are mast cell stabilizers, which acts by BOARDS;
D. Prevents vagal-stimulated bronchoconstriction preventing calcium influx and stabilizes mast cells, TOPNOTCH MD
E. Blocks cysteinyl leukotriene-1 receptor preventing degranulation and release of histamine, FROM EAC)
leukotrienes, and other mediators. SIMILAR TO
PREVIOUS BOARD EXAM CONCEPT.
133 The mechanism of action of Allopurinol : Option A and B - MOA of Colchicine; Option C and E - LYNN DARYL MIDTERM 1
A. Inhibits microtubule assembly MOA of Probenecid; MOA of Allopurinol: its active FELICIANO EXAM - AUG
B. Decreases macrophage migration and metabolite irreversibly inhibits xanthine oxidase VILLAMATER, 2015
phagocytosis and lowers production of uric acid. SIMILAR TO MD (TOP 5 - FEB
C. Compete with uric acid for reabsorption in the PREVIOUS BOARD EXAM CONCEPT. 2015 MED
proximal tubules BOARDS;
D. Irreversibly inhibits xanthine oxidase TOPNOTCH MD
E. Increases uric acid excretion FROM EAC)
134 The preferred antiseizure drug for pregnant women: Phenobarbital is the preferred antiseizure drug in LYNN DARYL MIDTERM 1
A. Valproic acid children and pregnant women. SIMILAR TO FELICIANO EXAM - AUG
B. Phenobarbital PREVIOUS BOARD EXAM CONCEPT. VILLAMATER, 2015
C. Phenytoin MD (TOP 5 - FEB
D. Carbamazepine 2015 MED
E. Topiramate BOARDS;
TOPNOTCH MD
FROM EAC)
135 Which of the following is an extraluminal Tissue amebicides act in the bowel wall and liver: LYNN DARYL MIDTERM 1
amoebicide? Metronidazole, emetine, chloroquine, tinidazole; FELICIANO EXAM - AUG
A. Metronidazole Luminal amebicide: Diloxanide furoate, Iodoquinol, VILLAMATER, 2015
B. Diloxanide furoate Paromomycin. SIMILAR TO PREVIOUS BOARD EXAM MD (TOP 5 - FEB
C. Lumefrantrine CONCEPT 2015 MED
D. Pentamidine BOARDS;
E. Nifurtimox TOPNOTCH MD
FROM EAC)
136 Sulfone in the management of leprosy causes? Drugs used in leprosy: Dapsone, Rifampicin, and LYNN DARYL MIDTERM 1
A. Inhibition of folic acid synthesis Clofazimine. Dapsone, a sulfone, is the most active FELICIANO EXAM - AUG
B. Inhibition of PABA drug used against M. leprae. It is bacteriostatic and VILLAMATER, 2015
C. Inhibition of DNA synthesis inhibits folic acid synthesis. Clofazimine acts by MD (TOP 5 - FEB
D. Inhibition of protein synthesis binding to guanine bases in bacterial DNA. 2015 MED
E. Inhibition of arabinosyl transferase BOARDS;
TOPNOTCH MD
FROM EAC)
137 Which of the following antiretroviral would cause Indinavir (Protease Inhibitor) - all Pis/'navirs' may LYNN DARYL MIDTERM 1
hyperlipidemia and hyperglycemia? cause hyperlipidemia, hyperglycemia, and insulin FELICIANO EXAM - AUG
A. Enfuvitide resistance as potential adverse events; Efuvirtide is VILLAMATER, 2015
B. Indinavir a fusion inhibitor, SE: Hypersensitivity, increased MD (TOP 5 - FEB
C. Zidovudine incidence of bacterial pneumonia, injection site 2015 MED
D. Delavirdine reaction; Zidovudine and Didanosine (NRTI) - all BOARDS;
E. Didanosine NRTI carry the risk of lactic acidosis with hepatic TOPNOTCH MD
steatosis FROM EAC)
138 Which of the following sulfonylurea is safest drug for Because of its short half-life, Tolbutamide is the LYNN DARYL MIDTERM 1
elderly diabetics? safest sulfonylurea for elderly diabetics. SIMILAR TO FELICIANO EXAM - AUG
A. Chlorpropramide PREVIOUS BOARD EXAM CONCEPT. VILLAMATER, 2015
B. Tolbutamide MD (TOP 5 - FEB
C. Tolazamide 2015 MED
D. Glyburide BOARDS;
E. Glimepiride TOPNOTCH MD
FROM EAC)
139 The danger of long term use of propylthiouracil is: The most dangerous complication of thioamides is LYNN DARYL MIDTERM 1
A. Cholestatic jaundice agranulocytosis, (granulocyte <500) an infrequent FELICIANO EXAM - AUG
B. Exfoliative dermatitis but potentially fatal adverse reaction. Hepatitis can VILLAMATER, 2015
C. Liver failure also be fatal. The most common adverse effect of MD (TOP 5 - FEB
D. Agranulocytosis PTU is maculopapular rash. (Katzung). SIMILAR TO 2015 MED
E. Arthralgia PREVIOUS BOARD EXAM CONCEPT. BOARDS;
TOPNOTCH MD
FROM EAC)

TOPNOTCH MEDICAL BOARD PREP PHARMACOLOGY SUPEREXAM Page 17 of 83


For inquiries visit www.topnotchboardprep.com.ph or email us at topnotchmedicalboardprep@gmail.com
TOPNOTCH MEDICAL BOARD PREP PHARMACOLOGY SUPEREXAM
For inquiries visit www.topnotchboardprep.com.ph or email us at topnotchmedicalboardprep@gmail.com
Item QUESTION EXPLANATION AUTHOR TOPNOTCH
# EXAM
140 This cell-cycle specific drug is commonly used for LYNN DARYL MIDTERM 1
testicular cancer. It does not cause myelosuppression, FELICIANO EXAM - AUG
however, dose modification is recommended in the VILLAMATER, 2015
setting of renal dysfunction. What is this MD (TOP 5 - FEB
chemotherapeutic drug? 2015 MED
A. Cyclophosphamide BOARDS;
B. Cisplatin TOPNOTCH MD
C. Busulfan FROM EAC)
D. Bleomycin
E. Etoposide
141 You are treating a patient with hyperthyroidism and Thioamides like methimazole and PTU inhibit the EDWARD HARRY MIDTERM 2
you prescribed methimazole, you know that this drug organification of iodine to tyrosine via the enzyme VALLAJERA, MD EXAM - AUG
inhibits which enzyme? thyroid peroxidase. (TOP 8 - FEB 2015
A. Sodium-Iodide co-transporter 2015 MED
B. Thyroid peroxidase BOARDS;
C. Proteases in the thyroid gland TOPNOTCH MD
D. All of the above FROM
E. None of the above PERPETUAL
BINAN)
142 Which of the following anti fungals will enhance the Ketoconazole is a potent cytochrome P450 inhibitor EDWARD HARRY MIDTERM 2
activity of cyclosporine? and therefore inhibits the metabolism of other drugs VALLAJERA, MD EXAM - AUG
A. Itraconazole and increases their effects, griseofulvin is a (TOP 8 - FEB 2015
B. Fluconazole cytochrome inducer and thus facilitates the 2015 MED
C. Ketoconazole metabolism of other drugs thus decreasing their BOARDS;
D. Griseofulvin effect. TOPNOTCH MD
E. Terbinafine FROM
PERPETUAL
BINAN)
143 Which among the following is the most potent opioid The opioid ohmefentanyl is the most potent opioid EDWARD HARRY MIDTERM 2
A. Ohmefentanyl VALLAJERA, MD EXAM - AUG
B. Morphine (TOP 8 - FEB 2015
C. Fentanyl 2015 MED
D. Pethidine BOARDS;
E. Remifentanil TOPNOTCH MD
FROM
PERPETUAL
BINAN)
144 What is the drug of choice in acute arrhythmias EDWARD HARRY MIDTERM 2
A. Metoprolol VALLAJERA, MD EXAM - AUG
B. Lidocaine (TOP 8 - FEB 2015
C. Quinine 2015 MED
D. Bisoprolol BOARDS;
E. None of the above TOPNOTCH MD
FROM
PERPETUAL
BINAN)
145 What is the drug of choice in preventing the EDWARD HARRY MIDTERM 2
recurrence of arryhthmia VALLAJERA, MD EXAM - AUG
A. Adenosine (TOP 8 - FEB 2015
B. Procainamide 2015 MED
C. Lidocaine BOARDS;
D. Carvedilol TOPNOTCH MD
E. None of the above FROM
PERPETUAL
BINAN)
146 The following antibiotics eliminate intestinal flora Erythromycin is not associated with EDWARD HARRY MIDTERM 2
thus causing pseudomembranous colitis except pseudomembranous colitis VALLAJERA, MD EXAM - AUG
A. 3rd gen cephalosporins (TOP 8 - FEB 2015
B. Aminoglycosides 2015 MED
C. Clindamycin BOARDS;
D. Erythromycin TOPNOTCH MD
E. None of the above FROM
PERPETUAL
BINAN)
147 Why is nitroglycerin preferred given sublingually? Sublingual route is preferred as there is less dose EDWARD HARRY MIDTERM 2
A. Giving it orally will require a higher dose needed and it is protected from hepatic first-pass VALLAJERA, MD EXAM - AUG
B. Giving it orally will produce a greater effect metabolism (TOP 8 - FEB 2015
C. It 2015 MED
D. Protect it from first-pass metabolism BOARDS;
E. None of the above TOPNOTCH MD
FROM
PERPETUAL
BINAN)
148 RH, a 50 year old bank executive was brought to the The patient has been poisoned with arsenic and the EDWARD HARRY MIDTERM 2
ER due to loss of consciousness, upon PE you noted drug of choice is dimercaprol VALLAJERA, MD EXAM - AUG
his breath smelling like sweet with garlicky odor, (TOP 8 - FEB 2015
what is the appropriate drug for this case 2015 MED
A. Dimercaprol BOARDS;
B. EDTA TOPNOTCH MD
C. Hydration FROM
D. Deferoxamine PERPETUAL
E. Flumazenil BINAN)

TOPNOTCH MEDICAL BOARD PREP PHARMACOLOGY SUPEREXAM Page 18 of 83


For inquiries visit www.topnotchboardprep.com.ph or email us at topnotchmedicalboardprep@gmail.com
TOPNOTCH MEDICAL BOARD PREP PHARMACOLOGY SUPEREXAM
For inquiries visit www.topnotchboardprep.com.ph or email us at topnotchmedicalboardprep@gmail.com
Item QUESTION EXPLANATION AUTHOR TOPNOTCH
# EXAM
149 RJLC, a 30 year old female was treated for Metronidazole eliminates the trophozoite forms of EDWARD HARRY MIDTERM 2
extraintestinal amebiasis, what is the drug of choice the parasite in the intestine or in the tissues. VALLAJERA, MD EXAM - AUG
to eliminate the extraintestinal parasites? (TOP 8 - FEB 2015
A. Paromomycin 2015 MED
B. Diloxanide furoate BOARDS;
C. Metronidazole TOPNOTCH MD
D. Pyrantel pamoate FROM
E. Praziquantel PERPETUAL
BINAN)
150 Allopurinol exerts its theraputic effect by: Allopurinol inhibits the enzyme xanthine oxidase EDWARD HARRY MIDTERM 2
A. Inhibiting excretion of uric acid at the PCT which metabolizes purines to form uric acid. VALLAJERA, MD EXAM - AUG
B. Increasing purine catabolism Therefore its activity inhibits the formation of uric (TOP 8 - FEB 2015
C. Competes with uric acid for excretion in the kidney acid 2015 MED
D. Inhibiting formation of uric acid BOARDS;
E. None of the above TOPNOTCH MD
FROM
PERPETUAL
BINAN)
151 This drug reduces the relapse rates for malaria Primaquine eradicates hypnozoites in the liver EDWARD HARRY MIDTERM 2
A. Quinine responsible for causing relapse VALLAJERA, MD EXAM - AUG
B. Arthemeter-lumefantrine (TOP 8 - FEB 2015
C. Mefloquine 2015 MED
D. Doxycycline BOARDS;
E. Primaquine TOPNOTCH MD
FROM
PERPETUAL
BINAN)
152 Chronic administration of phenobarbital causes Stimulation of the microsomal ethanol oxidizing EDWARD HARRY MIDTERM 2
metabolic tolerance via which mechanism? system is implicated in the development of tolerance VALLAJERA, MD EXAM - AUG
A. Stimulation of aldehyde dehydrogenase to phenobarbital aside from playing a minor role in (TOP 8 - FEB 2015
B. Stimulation of alcohol dehydrogenase individuals with chronic alcoholism. The rest have 2015 MED
C. Stimulation of MEOS no role in the development of tolerance to BOARDS;
D. Stimulation of Monoamine oxidase phenobarbital TOPNOTCH MD
E. None of the above FROM
PERPETUAL
BINAN)
153 Most common side effect of clomiphene citrate Hot flushes are the most common side effect of use EDWARD HARRY MIDTERM 2
A. Hot flushes of clomiphene citrate, the rest are occasional side VALLAJERA, MD EXAM - AUG
B. Constipation effects (TOP 8 - FEB 2015
C. Headache 2015 MED
D. Allergic skin reactions BOARDS;
E. None of the above TOPNOTCH MD
FROM
PERPETUAL
BINAN)
154 Which of the following antimalarial drugs may Quinine is precipitates hemolytic crisis in patients EDWARD HARRY MIDTERM 2
precipitate a hemolytic crisis in a patient with G6PD with malaria with G6PD deficiency. VALLAJERA, MD EXAM - AUG
A. Lumefantrine (TOP 8 - FEB 2015
B. Doxycycline 2015 MED
C. Artemether BOARDS;
D. Quinine TOPNOTCH MD
E. None of the above FROM
PERPETUAL
BINAN)
155 A man is being treated for intestinal strongyloidiasis, Ivermectin is the DOC in the treatment of EDWARD HARRY MIDTERM 2
you know that the drug of choice is: strongyloidiasis VALLAJERA, MD EXAM - AUG
A. Praziquantel (TOP 8 - FEB 2015
B. Ivermectin 2015 MED
C. Melarsoprol BOARDS;
D. Pyrimethamine-sulfa TOPNOTCH MD
E. Trimethoprim-sulfamethoxazole FROM
PERPETUAL
BINAN)
156 SMA, a 68 year old female with hypertension is taking Peptidyl dipeptidase is otherwise known as ACE EDWARD HARRY MIDTERM 2
captopril, you know that it is effective in hypertension which is the enzyme inhibited by ACE inhibitors like VALLAJERA, MD EXAM - AUG
because it inhibits: captopril. (TOP 8 - FEB 2015
A. Cathepsin C 2015 MED
B. Dipeptidyl dipeptidase-2 BOARDS;
C. Dipeptidyl dipeptidase-4 TOPNOTCH MD
D. Peptidyl dipeptidase FROM
E. Procarboxypeptidase PERPETUAL
BINAN)
157 Intake of potassium supplements is contraindicated Amiloride is a potassium sparing diuretic which EDWARD HARRY MIDTERM 2
in patients taking which of the following? therefore spares renal excretion of potassium VALLAJERA, MD EXAM - AUG
A. Ethacrynic acid raising the potassium levels and therefore is (TOP 8 - FEB 2015
B. Amiloride contraindicated in patients taking potassium 2015 MED
C. Hydrochlorothiazide supplements. BOARDS;
D. Acetazolamide TOPNOTCH MD
E. None of the above FROM
PERPETUAL
BINAN)

TOPNOTCH MEDICAL BOARD PREP PHARMACOLOGY SUPEREXAM Page 19 of 83


For inquiries visit www.topnotchboardprep.com.ph or email us at topnotchmedicalboardprep@gmail.com
TOPNOTCH MEDICAL BOARD PREP PHARMACOLOGY SUPEREXAM
For inquiries visit www.topnotchboardprep.com.ph or email us at topnotchmedicalboardprep@gmail.com
Item QUESTION EXPLANATION AUTHOR TOPNOTCH
# EXAM
158 Injection of a small dose of Ach will cause what? Ach causes depression of the SA and AV nodes thus EDWARD HARRY MIDTERM 2
A. Reflex bradycardia slowing the heart rate, hypotension through the VALLAJERA, MD EXAM - AUG
B. Hypotension relaxation of arterioles, splanchnic stimulation (TOP 8 - FEB 2015
C. Splanchnic stimulation through the stimulation of the muscarinic receptors 2015 MED
D. All of the above BOARDS;
E. None of the above TOPNOTCH MD
FROM
PERPETUAL
BINAN)
159 Though 1% is systemically absorbed when Bronchodilation is still the net effect of ipratropium, EDWARD HARRY MIDTERM 2
Ipratropium Bromide is given via nebulization, what the systemic effect is negligent as the drug is VALLAJERA, MD EXAM - AUG
is its net effect? administered to its site of action which is the lungs. (TOP 8 - FEB 2015
A. Reflex tachycardia 2015 MED
B. Bronchodilation BOARDS;
C. Bronchoconstriction TOPNOTCH MD
D. Laryngeal spasm FROM
E. None of the above PERPETUAL
BINAN)
160 KVM, a 29 year old female was brought to the ER due Pralidoxime regenerates acetylcholinesterase and is EDWARD HARRY MIDTERM 2
to ingestion of insecticide, you ordered pralidoxime only effective when the bond between the VALLAJERA, MD EXAM - AUG
to be given, what does it do? organophosphate and acetylcholinesterase hasn't (TOP 8 - FEB 2015
A. Induces the enzyme Acetylcholinesterase matured which takes 6-8 hours, afterwhich, 2015 MED
B. Regenerates acetylcholinesterase if given within 6- pralidoxime even if given is no longer effective. BOARDS;
8 hours after ingestion TOPNOTCH MD
C. Breaks the bond between organophosphate and FROM
acetylcholinesterase PERPETUAL
D. B and C only BINAN)
E. All of the above
161 A patient came in for consult due to infertility. It was SIMILAR TO PREVIOUS BOARD EXAM HAROLD JAY S. MIDTERM 3
found out that the patient is having ovulation CONCEPT/PRINCIPLE. Katzung 11th edition page BAYTEC, MD EXAM - AUG
disorder so you decided to give here clomiphene. It is 719.. "the most common adverse effects in patients (TOP 10 - FEB 2015
important to tell to the patient that the most common treated with this drug are hot flushes, which resemle 2015 MED
adverse effect of clomiphene is: those experienced by menopausal patients." BOARDS;
A. hot flushes TOPNOTCH MD
B. bleeding FROM FEU)
C. constipation
D. headache
E. nausea and vomiting
162 It is the antiarrythmic of choice for termination of SIMILAR TO PREVIOUS BOARD EXAM HAROLD JAY S. MIDTERM 3
ventricular tachycardia and prevention of ventricular CONCEPT/PRINCIPLE. Katzung 11th edition page BAYTEC, MD EXAM - AUG
fibrillation after cardioversion in the setting of acute 239. "Lidocaine is the agent of choice for termination (TOP 10 - FEB 2015
ischemia. of ventricular tachycardia and prevention of 2015 MED
A. adenosine ventricular fibrillation after cardioversion the BOARDS;
B. esmolol setting of acute ischemia. However, routine TOPNOTCH MD
C. lidocainne prophylactic use of lidocaine in this setting may FROM FEU)
D. procainamide actually increase total mortality, possibly by
E. Amiodarone increasing incidence of asystole, and is not the
standard of care. Most physicians administer IV
lidocaine only to patients with arrythmia"
163 Jan Deo is a newly diagnosed with essential Enalapril is an ACE inhibitor. One of the many HAROLD JAY S. MIDTERM 3
hypertension. He has no other co morbidities. Aside alternative names for ACE is peptidyl dipeptidase. BAYTEC, MD EXAM - AUG
from lifestyle modification, he was prescribed with SIMILAR TO PREVIOUS BOARD EXAM (TOP 10 - FEB 2015
Enalapril as his maintainance. Which of the following CONCEPT/PRINCIPLE 2015 MED
BEST describes the mechanism of action of the drug. BOARDS;
A. blocks Angiotensin receptor which will prevent TOPNOTCH MD
rise in BP FROM FEU)
B. inhibits peptidyl dipeptidase
C. direct vasodilator
D. reduces calcium uptake of smooth muscles
E. none of the above
164 A patient was given Nedocromil for asthma. Which of SIMILAR TO PREVIOUS BOARD EXAM HAROLD JAY S. MIDTERM 3
the following is the mechanism of action of the drug? CONCEPT/PRINCIPLE. Katzung 11th edition page BAYTEC, MD EXAM - AUG
A. Inhibits the release of histamine 349 (TOP 10 - FEB 2015
B. Direct relaxation of smooth muscle 2015 MED
C. Inhibits leukotriene pathway BOARDS;
D. mast cell stabilizer TOPNOTCH MD
E. none of the above FROM FEU)

165 Among the thioamide antithyroid drugs, which of the SIMILAR TO PREVIOUS BOARD EXAM HAROLD JAY S. MIDTERM 3
following is the one preferred for pregnant women CONCEPT/PRINCIPLE. Katzung 11th edition page BAYTEC, MD EXAM - AUG
because it crosses the placental barrier less readily 672. PTU is preferable because it is more strongly (TOP 10 - FEB 2015
giving lesser effect to the fetus protein-bound and therefore crosses the placcenta 2015 MED
A. potassium iodide less readily. BOARDS;
B. Ipodate TOPNOTCH MD
C. carbimazole FROM FEU)
D. methimazole
E. PTU
166 All of the following are mechanism of action of SIMILAR TO PREVIOUS BOARD EXAM HAROLD JAY S. MIDTERM 3
propylthiouracil EXCEPT: CONCEPT/PRINCIPLE. Katzung 11th edition page BAYTEC, MD EXAM - AUG
A. inhibits thyroid peroxidase-catalyzed reactions 672. All of the choices are mechanisms of action of (TOP 10 - FEB 2015
B. Blocks iodine organification PTU except for the blocking of uptake of iodide by 2015 MED
C. Blocks coupling of the iodotyrosinase the gland. Inhibition of thyroid peroxidase-catalyzed BOARDS;
D. Block uptake of iodide by the gland reaction and the blocking of iodine organification. TOPNOTCH MD
E. inhibits peripheral deiodination FROM FEU)

TOPNOTCH MEDICAL BOARD PREP PHARMACOLOGY SUPEREXAM Page 20 of 83


For inquiries visit www.topnotchboardprep.com.ph or email us at topnotchmedicalboardprep@gmail.com
TOPNOTCH MEDICAL BOARD PREP PHARMACOLOGY SUPEREXAM
For inquiries visit www.topnotchboardprep.com.ph or email us at topnotchmedicalboardprep@gmail.com
Item QUESTION EXPLANATION AUTHOR TOPNOTCH
# EXAM
167 Fluoxetine (prozac) is one of the most commonly use SIMILAR TO PREVIOUS BOARD EXAM HAROLD JAY S. MIDTERM 3
antidepressants worldwide. Which of the following is CONCEPT/PRINCIPLE. Katzung 11th edition page BAYTEC, MD EXAM - AUG
the mechanism of action of the drug? 513. SSRIs like Fluoxetine are agents that have their (TOP 10 - FEB 2015
A. Selectively inhibits serotonin reuptake primary action the inhibition of serotonin 2015 MED
B. Inhibits serotonin and norepinephrine reuptake transporter. BOARDS;
C. selectively inhibits reuptake of norepinephrine TOPNOTCH MD
D. directly antagonize serotonin FROM FEU)
E. directly antagonize epinephrine

168 Which among the following drugs deliver its action SIMILAR TO PREVIOUS BOARD EXAM HAROLD JAY S. MIDTERM 3
through selectively inhibiting reuptake of serotonin? CONCEPT/PRINCIPLE. Katzung 11th edition page BAYTEC, MD EXAM - AUG
A. venlafaxine 514. Fluoxetine is a SSRI. Venlafaxine is a SNRI. (TOP 10 - FEB 2015
B. imipramine Imipramine is a TCA. Selegiline is MAOi. Trazadone 2015 MED
C. selegiline is a serotonin antagonist. BOARDS;
D. fluoxetine TOPNOTCH MD
E. trazodone FROM FEU)

169 All of the following drugs are considered disease- ketoprofen is a non selective COX inhibitor HAROLD JAY S. MIDTERM 3
modifying antirheumatic drugs (DMARDs) except: BAYTEC, MD EXAM - AUG
A. Abatacept (TOP 10 - FEB 2015
B. Azathioprine 2015 MED
C. Chloroquine BOARDS;
D. Methotrexate TOPNOTCH MD
E. Ketoprofen FROM FEU)

170 Colchicine is a widely used drug in the treatment of B. are uricosuric agents like probenecid and HAROLD JAY S. MIDTERM 3
gout. Which among the following describes the sulfinpyrazone… C. allopurinol….D. Febuxostat BAYTEC, MD EXAM - AUG
mechanism of action of the drug? SIMILAR TO PREVIOUS BOARD EXAM (TOP 10 - FEB 2015
A. Binding to intracellular tubulin to excert anti CONCEPT/PRINCIPLE 2015 MED
inflammatory effect BOARDS;
B. It increases the excretion of uric acid through TOPNOTCH MD
urine FROM FEU)
C. inhibits xanthine oxidase catalized reactions
D. non purine xanthine oxidase inhibitor
E. none of the above
171 A patient came in at the ER having severe headache Take note, the patient has asthma… Katzung 11th HAROLD JAY S. MIDTERM 3
and heat intolerance. Examination revealed enlarged edition chapter 38 page 677. “If propanolol is BAYTEC, MD EXAM - AUG
thyroid, high blood pressure and severe tachycardia. contraindicated by the presence of severe heart (TOP 10 - FEB 2015
On history, his relative told you that he also has failure or asthma, hypertension and tachycardia may 2015 MED
frequent attacks of asthma. In your findings, you are be controlled with diltiazem, 90-120mg orally three BOARDS;
suspecting that he is having thyroid storm. Which of or four times daily or 5-10 mg/h by IV infusion.” TOPNOTCH MD
the following drugs can be given immediately to SIMILAR TO PREVIOUS BOARD EXAM FROM FEU)
relieve hypertension and tachycardia? CONCEPT/PRINCIPLE. Katzung 11th edition page
A. propanolol 677
B. esmolol
C. diltiazem
D. nifedipine
E. clonidine
172 A post stroke patient is taking in warfarin. Which Isoniazid in a P450 inhibitor which will delay the HAROLD JAY S. MIDTERM 3
among the following drugs will most likely increase excretion of warfarin when given. Other choices are BAYTEC, MD EXAM - AUG
the possibility of having warfarin toxicity when added P450 inducers (TOP 10 - FEB 2015
in his management? 2015 MED
A. Smoking BOARDS;
B. Barbiturates TOPNOTCH MD
C. Carbamazepine FROM FEU)
D. Isoniazid
E. Ethanol
173 A G7P7 (6107) came in for consult. She asks you that Rifampicin is a CYP450 inducer which hastens the HAROLD JAY S. MIDTERM 3
she wants to take oral contraceptive pills as family excretion of OCP BAYTEC, MD EXAM - AUG
planning. If given in conjunction with OCP, which (TOP 10 - FEB 2015
among the following will decrease the effectivity of 2015 MED
OCP? BOARDS;
A. Isoniazid TOPNOTCH MD
B. Cimetidine FROM FEU)
C. ketoconazole
D. erythromycin
E. Rifampicin
174 This drug inhibits intestinal absorption of phytosterol HAROLD JAY S. MIDTERM 3
and cholesterol which will eventually decrease the BAYTEC, MD EXAM - AUG
level of LDL. (TOP 10 - FEB 2015
A. rosuvastatin 2015 MED
B. ezetimibe BOARDS;
C. Niacin TOPNOTCH MD
D. Gemfibrozil FROM FEU)
E. Simvastatin

175 All of the following reactions are considered Phase 2 other choices are phase 1 reaction HAROLD JAY S. MIDTERM 3
reactions in drug metabolism except: BAYTEC, MD EXAM - AUG
A. glucoronidation (TOP 10 - FEB 2015
B. hydoxylation 2015 MED
C. Glutathione conjugation BOARDS;
D. sulfation TOPNOTCH MD
E. Methylation FROM FEU)

TOPNOTCH MEDICAL BOARD PREP PHARMACOLOGY SUPEREXAM Page 21 of 83


For inquiries visit www.topnotchboardprep.com.ph or email us at topnotchmedicalboardprep@gmail.com
TOPNOTCH MEDICAL BOARD PREP PHARMACOLOGY SUPEREXAM
For inquiries visit www.topnotchboardprep.com.ph or email us at topnotchmedicalboardprep@gmail.com
Item QUESTION EXPLANATION AUTHOR TOPNOTCH
# EXAM
176 Which of the following correctly describes FDA Drug A: category C------ B. Category A-------- C. Category D HAROLD JAY S. MIDTERM 3
category B? BAYTEC, MD EXAM - AUG
A. Either animal studies revealed adverse effects on (TOP 10 - FEB 2015
the fetus and there are no controlled studies in 2015 MED
women or studies in women and animals are not BOARDS;
available TOPNOTCH MD
B. Control studies in women fail to demonstrate a FROM FEU)
risk to the fetus in the first trimester and the
possibility of fetal harm is remote
C. there is positive evidence of human fetal risk but
the benefits from use in pregnant women may be
acceptable despite the risk
D. Animal reproduction studies have shown an
adverse effect that was not confirmed in controlled
studies in women in the first trimester
E. None of the above
177 Which of the following antineoplastic drugs can cause cisplatin are nephrotoxic and ototoxic. Vincristine HAROLD JAY S. MIDTERM 3
hemorrhagic cystitis? causes peripheral neuropathy. Methotrexate usually BAYTEC, MD EXAM - AUG
A. cyclophosphamide causes myelosuppression. Bleomycin is usually (TOP 10 - FEB 2015
B. cisplatin associated with pulmonary fibrosis 2015 MED
C. vincristine BOARDS;
D. methotrexate TOPNOTCH MD
E. Bleomycin FROM FEU)

178 A cancer patient is scheduled for methotrexate dexrazoxane is a rescue therapy for doxorubicin; HAROLD JAY S. MIDTERM 3
chemotherapy. Which of the following rescue agent/s mesna is for cyclophosphamide; amifostine is for BAYTEC, MD EXAM - AUG
is used to alleviate the toxic effect of the drug? cisplatin (TOP 10 - FEB 2015
A. Dexrazoxane 2015 MED
B. Mesna BOARDS;
C. Leucovorin TOPNOTCH MD
D. Amifostine FROM FEU)
E. None of the above

179 Which of the following antibiotics is a protein Ciprofloxacin is a fluoroquinolone which inhibits HAROLD JAY S. MIDTERM 3
synthesis inhibitor at 30S sub unit? topoisomerase. The other choices inhibits at 50s BAYTEC, MD EXAM - AUG
A. Ciprofloxacin subunit. (TOP 10 - FEB 2015
B. azithromycin 2015 MED
C. clindamycin BOARDS;
D. linezolid TOPNOTCH MD
E. Doxycycline FROM FEU)

180 Which of the following cephalosporins can be used to antipseudomonal cephalosporins are ceftazidime, HAROLD JAY S. MIDTERM 3
patients with Pseudomonas infection? cefepime, and cefoperazone BAYTEC, MD EXAM - AUG
A. cefuroxime (TOP 10 - FEB 2015
B. cefazolin 2015 MED
C. cefoxitin BOARDS;
D. Ceftazidime TOPNOTCH MD
E. Ceftriaxone FROM FEU)

181 What is the mechanism of action of metronidazole? A- fluoroquinolones. C- nitrofurantoin. JEAN PAOLO M. FINAL EXAM -
A. Inhibits DNA replication by binding to DNA DELFINO, MD AUG 2015
gyrase (TOP 10 - FEB
B. Reactive reduction by ferredoxin forming free 2015 MED
radicals BOARDS;
C. Forms multiple reactive intermediates when TOPNOTCH MD
acted upon by bacterial enzyme FROM FATIMA)
D. Unknown
E. None of the above
182 59 year old male, hypertensive for 22 years, came in my mnemonics: improves QUANTITY of life in CHF- JEAN PAOLO M. FINAL EXAM -
the ER due to difficulty of breathing. PE revealed ABS (ACEI/ARBS, Beta-Blockers, Spironolactone). DELFINO, MD AUG 2015
crackles on bilateral lower lobes of the lungs and Improves QUALITY of life- Digital Film (Digitalis, (TOP 10 - FEB
bipedal edema. Which of the drugs used in CHF can Furosemide). Only 3 beta blockers are found to be 2015 MED
prolong survival of patients? useful in CHF: Carvedilol, metoprolol and bisopolol. BOARDS;
A. carvedilol TOPNOTCH MD
B. atenolol FROM FATIMA)
C. digitalis
D. furosemide
E. A and B
183 Your patient requires a local anesthetic with long SIMILAR TO PREVIOUS BOARD EXAM JEAN PAOLO M. FINAL EXAM -
duration of action. What will you give? CONCEPT/PRINCIPLE.. my mnemonics for this: DELFINO, MD AUG 2015
A. procaine BeT-long. Local anesthetics with long duration of (TOP 10 - FEB
B. lidocaine action are Bupivacaine and Tetracaine. The longest 2015 MED
C. cocaine acting is Ropivacaine ("at the end of a long ROPe") BOARDS;
D. bupivacaine TOPNOTCH MD
E. None of the above FROM FATIMA)

184 What anti-retroviral agent causes peripheral SIMILAR TO PREVIOUS BOARD EXAM JEAN PAOLO M. FINAL EXAM -
neuropathy and pancreatitis? CONCEPT/PRINCIPLE.. Zidovudine causes bone DELFINO, MD AUG 2015
A. Zidovudine marrow suppression. Abacavir causes (TOP 10 - FEB
B. Zalcitabine hypersensitivity. Didanosine causesperipheral 2015 MED
C. Didanosine neuropathy and pancreatitis. Stavudine and BOARDS;
D. Abacavir Zalcitabine causes peripheral neuropathy. TOPNOTCH MD
E. Foscarnet FROM FATIMA)

TOPNOTCH MEDICAL BOARD PREP PHARMACOLOGY SUPEREXAM Page 22 of 83


For inquiries visit www.topnotchboardprep.com.ph or email us at topnotchmedicalboardprep@gmail.com
TOPNOTCH MEDICAL BOARD PREP PHARMACOLOGY SUPEREXAM
For inquiries visit www.topnotchboardprep.com.ph or email us at topnotchmedicalboardprep@gmail.com
Item QUESTION EXPLANATION AUTHOR TOPNOTCH
# EXAM
185 Which of the following anti-microbial agent is Bacteriostatic agents: All protein synthesis JEAN PAOLO M. FINAL EXAM -
bacteriostatic? inhibitors except aminoglycoside; ethambutol, DELFINO, MD AUG 2015
A. Co-trimoxazole nitrofurantoin; those with anti-metabolite (TOP 10 - FEB
B. Pyrazinamide mechanism of action- sulfonamide and trimethoprim 2015 MED
C. Metronidazole (but they become bactericidal when combined). BOARDS;
D. Aminoglycoside TOPNOTCH MD
E. Tetracycline FROM FATIMA)

186 Drug of choice for hydatid disease? Albendazole is the drug of choice for hydatid disease JEAN PAOLO M. FINAL EXAM -
A. Niclosamide and cysticercosis. DELFINO, MD AUG 2015
B. Praziquantel (TOP 10 - FEB
C. Albendazole 2015 MED
D. Mebendazole BOARDS;
E. Pyrantel pamoate TOPNOTCH MD
FROM FATIMA)

187 Which of these drugs will be the fastest to be JEAN PAOLO M. FINAL EXAM -
eliminated from the body? DELFINO, MD AUG 2015
A. 100mg drug with zero order kinetics; elimination (TOP 10 - FEB
rate is 25mg/2hrs 2015 MED
B. 100mg drug with first order kinetics; elimination BOARDS;
t 1/2 is 2 hours TOPNOTCH MD
C. 150mg drug with zero order kinetics; elimination FROM FATIMA)
rate is 25mg/hr
D. 150mg drug with first order kinetics; elimination
t 1/2 is 2 hours
E. All of these will be eliminated at the same time
188 Local anesthetics are not effective in inflamed Local anesthetics are weak bases, therefore, in an JEAN PAOLO M. FINAL EXAM -
infected tissues because? acidic environment like in infected tissues, the DELFINO, MD AUG 2015
A. The cationic form is increased protonated or cationic form of the drug (TOP 10 - FEB
B. Protonated form predominates predominates. This is water soluble so the drug 2015 MED
C. The drug is more water soluble undergoes rapid clearance rather than being able to BOARDS;
D. A and B cross biological membranes. TOPNOTCH MD
E. All of the above FROM FATIMA)

189 The following are adverse effects of amiodarone Adverse effects of amiodarone: bradycardia, heart JEAN PAOLO M. FINAL EXAM -
except? block, pulmonary fibrosis, elevated liver enzymes, DELFINO, MD AUG 2015
A. hyperthyroidism photodermatitis, corneal microdeposit, halos in (TOP 10 - FEB
B. hypothyroidism peripheral visual field, optic neuritis, 2015 MED
C. photodermatitis hypothyroidism (blocks the peripheral conversion of BOARDS;
D. pulmonary fibrosis T4 to T3) and hyperthyroidism (because it is also a TOPNOTCH MD
E. None of the above potential source of large amounts of inorganic FROM FATIMA)
iodine)
190 This opioid antagonist is used in opioid and alcohol Naltrexone reduces craving in alcohol dependence. JEAN PAOLO M. FINAL EXAM -
dependence Note: there are probably 4 questions about opiates DELFINO, MD AUG 2015
A. Nalmefene in our boards. (TOP 10 - FEB
B. Naltrexone 2015 MED
C. Naloxone BOARDS;
D. Nalbuphine TOPNOTCH MD
E. A and C FROM FATIMA)

191 What is the mechanism of action of PTU? There are 5 steps in thyroid hormone synthesis. 1. JEAN PAOLO M. FINAL EXAM -
A. Blocks iodination and organification iodide trapping; 2. iodination/organification; 3. DELFINO, MD AUG 2015
B. Inhibits iodide trapping coupling; 4. secretion; 5. peripheral conversion. (TOP 10 - FEB
C. Blocks coupling reaction Steps 2 to 5 are actually blocked by PTU but the 2015 MED
D. Blocks peripheral conversion of T4 to T3 main effect among these is the inhibition of BOARDS;
E. Inhibits secretion peripheral conversion. TOPNOTCH MD
FROM FATIMA)

192 This 3rd generation cephalosporin can cross the All 3rd generation cephalosporins can cross the JEAN PAOLO M. FINAL EXAM -
blood brain barrier blood brain barrier except: ceftibuten, cefpodoxime, DELFINO, MD AUG 2015
A. ceftizoxime cefixime, cefoperazone. (TOP 10 - FEB
B. cefpodoxime 2015 MED
C. ceftibuten BOARDS;
D. cefaclor TOPNOTCH MD
E. cefuroxime FROM FATIMA)

193 These drugs increase the outflow of aqueous humor Drugs that increase outflow of aqueous humor: JEAN PAOLO M. FINAL EXAM -
used in glaucoma except? pilocarpine, physostigmine, latanoprost, DELFINO, MD AUG 2015
A. timolol epinephrine. Drugs that decrease aqueous humor (TOP 10 - FEB
B. latanoprost secretion; mnemonics- TAMAD: Timolol, 2015 MED
C. epinephrine Acetazolamide, Mannitol, Apraclonidine, BOARDS;
D. pilocarpine Dorzolamide. TOPNOTCH MD
E. physostigmine FROM FATIMA)

194 Patient came in the clinic complaining of amenorrhea Risperidone causes hyperprolactinemia. JEAN PAOLO M. FINAL EXAM -
and galactorrhea. History revealed intake of an anti- DELFINO, MD AUG 2015
psychotic drug. What could have caused the (TOP 10 - FEB
symptoms of the patient? 2015 MED
A. Clozapine BOARDS;
B. Olanzapine TOPNOTCH MD
C. Risperidone FROM FATIMA)
D. Haloperidol
E. Chlorpromazine

TOPNOTCH MEDICAL BOARD PREP PHARMACOLOGY SUPEREXAM Page 23 of 83


For inquiries visit www.topnotchboardprep.com.ph or email us at topnotchmedicalboardprep@gmail.com
TOPNOTCH MEDICAL BOARD PREP PHARMACOLOGY SUPEREXAM
For inquiries visit www.topnotchboardprep.com.ph or email us at topnotchmedicalboardprep@gmail.com
Item QUESTION EXPLANATION AUTHOR TOPNOTCH
# EXAM
195 Which has the highest partition coefficient among the Methoxyflurane has the highest partition coefficient JEAN PAOLO M. FINAL EXAM -
inhalation anesthetics? which means that it has the slowest onset and DELFINO, MD AUG 2015
A. methoxyflurane recovery. (TOP 10 - FEB
B. Nitrous oxide 2015 MED
C. isoflurane BOARDS;
D. sevoflurane TOPNOTCH MD
E. desflurane FROM FATIMA)

196 Patient is diagnosed with Lennox-Gaustat Syndrome Topiramate is used as an adjunct in Lennox-Gaustat JEAN PAOLO M. FINAL EXAM -
and is taking maintenance drug. He later on Syndrome. Its adverse effects are unusual/weird for DELFINO, MD AUG 2015
developed urolithiasis. The patient is probably taking anti-seizure drugs (ex. Urolithiasis, myopia, (TOP 10 - FEB
what drug? glaucoma) 2015 MED
A. topiramate BOARDS;
B. lamotrigine TOPNOTCH MD
C. ethosuximide FROM FATIMA)
D. carbamazepine
E. levetiracetam
197 Escitalopram is an antidepressant agent that belongs Drugs that belong to SSRI: fluoxetine, sertraline, JEAN PAOLO M. FINAL EXAM -
to which drug class? paroxetine, fluvoxamine, citalopram, escitalopram DELFINO, MD AUG 2015
A. SSRI (TOP 10 - FEB
B. SNRI 2015 MED
C. TCA BOARDS;
D. MAOI TOPNOTCH MD
E. 5HT2 antagonist FROM FATIMA)

198 7 year old male was noted to have difficulty Methylphenidate is the drug of choice for ADHD in JEAN PAOLO M. FINAL EXAM -
sustaining attention in tasks or play activities in children 6 years old and above. Mechanism of action DELFINO, MD AUG 2015
school and at home. He often leaves his seat inside the is unknown but it mainly acts as a CNS stimulant (TOP 10 - FEB
classroom and runs about and climbs excessively. similar to amphetamines. 2015 MED
What is the treatment of choice for this case? BOARDS;
A. sibutramine TOPNOTCH MD
B. methylphenidate FROM FATIMA)
C. diazepam
D. behavioral therapy
E. magnesium pemoline
199 What drug acts on epithelial sodium channels in Triamterene and Amiloride are potassium sparing JEAN PAOLO M. FINAL EXAM -
cortical collecting ducts and is used to reduce diuretics that act on epithelial sodium channels DELFINO, MD AUG 2015
potassium excretion in cases of hypokalemia? (ENaC) in cortical collecting duct, causing decreased (TOP 10 - FEB
A. furosemide Na reabsorption and K excretion. Spironolactone 2015 MED
B. acetazolamide and eplerenone are also potassium sparing diuretics BOARDS;
C. spironolactone but they act on aldosterone receptors. TOPNOTCH MD
D. triamterene FROM FATIMA)
E. hydrochlorothiazide

200 Which of the following beta-blockers have local Beta-blockers with partial agonist effect on JEAN PAOLO M. FINAL EXAM -
anesthetic effect? adrenergic receptors have intrinsic sympathetic DELFINO, MD AUG 2015
A. Nadolol activity: LAPPCC (Labetalol, Acebutolol, Penbutolol, (TOP 10 - FEB
B. Penbutolol Pindolol, Carteolol, Celiprolol). Beta-blockers with 2015 MED
C. Pindolol membrane stabilizing activity have local anesthetic BOARDS;
D. Carteolol effect: LAMPP (Labetalol, Acebutolol, Metoprolol, TOPNOTCH MD
E. Atenolol Pindolol, Propranolol) FROM FATIMA)

201 In the stomach, aspirin will exist predominantly as Review the Henderson-Hasselbach equation. Aspirin GRACE ARVIOLA, DIAGNOSTIC
the ______________ form. (acetylsalicyclic acid) is acidic in nature. In an acidic MD (TOP 3 - AUG EXAM - FEB
A. Hydrophilic, ionized, polar environment like the stomach, it will exist 2014 MED 2015
B. Lipophilic, unionized, non-polar predominantly as the lipid-soluble, unionized, non- BOARDS;
C. Hydrophilic, unionized, non-polar polar form. TOPNOTCH MD)
D. Lipophilic, ionized, polar
E. Hydrophilic, unionized, polar

202 In the CHOP regimen for non-Hodgkin's lymphoma, Oncovin is vincristine. GRACE ARVIOLA, DIAGNOSTIC
which chemotherapeutic drug acts by inhibiting the MD (TOP 3 - AUG EXAM - FEB
mitotic spindle? 2014 MED 2015
A. Cyclophosphamide BOARDS;
B. Hydroxyurea TOPNOTCH MD)
C. Oncovin
D. Prednisone
E. Doxorubicin

203 A patient with leprosy is taking a drug that casts a red Rifampin causes red discoloration of body fluids but GRACE ARVIOLA, DIAGNOSTIC
color to the skin. He is probably taking: not the skin. MD (TOP 3 - AUG EXAM - FEB
A. Dapsone 2014 MED 2015
B. Rifampin BOARDS;
C. Clofazimine TOPNOTCH MD)
D. Ethambutol
E. Phenylbutazone

204 What part of the pharmacokinetics of penicillin is Probenecid raises plasma concentration of penicillin GRACE ARVIOLA, DIAGNOSTIC
affected by the drug probenecid? by inhibiting its tubular secretion in the renal MD (TOP 3 - AUG EXAM - FEB
A. Glomerular filtration tubules. 2014 MED 2015
B. Tubular reabsorption BOARDS;
C. Tubular secretion TOPNOTCH MD)
D. Hepatic uptake
E. Protein-binding with albumin

TOPNOTCH MEDICAL BOARD PREP PHARMACOLOGY SUPEREXAM Page 24 of 83


For inquiries visit www.topnotchboardprep.com.ph or email us at topnotchmedicalboardprep@gmail.com
TOPNOTCH MEDICAL BOARD PREP PHARMACOLOGY SUPEREXAM
For inquiries visit www.topnotchboardprep.com.ph or email us at topnotchmedicalboardprep@gmail.com
Item QUESTION EXPLANATION AUTHOR TOPNOTCH
# EXAM
205 Hoffman elimination is exhibited by: Hoofman elimination aka exhaustive methylation GRACE ARVIOLA, DIAGNOSTIC
A. Succinylcholine MD (TOP 3 - AUG EXAM - FEB
B. Diazepam 2014 MED 2015
C. Phenobarbital BOARDS;
D. Phenytoin TOPNOTCH MD)
E. Atracurium

206 In a patient suffering from organophosphate Because of its antimuscarinic effect on the heart, GRACE ARVIOLA, DIAGNOSTIC
poisoning with a cardiac rate of 45, what drug should atropine (the antidote for organophosphate MD (TOP 3 - AUG EXAM - FEB
be given? poisoning) has a positive chronotropic effect. 2014 MED 2015
A. Neostigmine BOARDS;
B. Atropine TOPNOTCH MD)
C. Epinephrine
D. Pancuronium
E. Edrophonium

207 Patients receiving cyclophosphamide should also be Leucovorin for methotrexate. Pyridoxine for GRACE ARVIOLA, DIAGNOSTIC
given: isoniazid. Thiamine for alcoholics. MD (TOP 3 - AUG EXAM - FEB
A. Mesna 2014 MED 2015
B. Leucovorin BOARDS;
C. Vitamin B6 TOPNOTCH MD)
D. Thiamine
E. Amifostine

208 Which statement is correct regarding the Aminoglycosides only work in arebic conditions. GRACE ARVIOLA, DIAGNOSTIC
aminoglycosides? They do not have cross-reactivity with the MD (TOP 3 - AUG EXAM - FEB
A. They work in anaerobic conditions. penicillins. Most drugs are nephrotoxic and ototoxic. 2014 MED 2015
B. They exhibit concentration-dependent killing. BOARDS;
C. Most drugs of this class are myelosuppresive. TOPNOTCH MD)
D. They cannot be given to patients allergic to
penicillins.
E. They are acidic in nature.
209 What is the second messenger system for the Beta-2 receptors are coupled to the cAMP second GRACE ARVIOLA, DIAGNOSTIC
receptors blocked by metoprolol? messenger system. MD (TOP 3 - AUG EXAM - FEB
A. cAMP 2014 MED 2015
B. IP3 BOARDS;
C. DAG TOPNOTCH MD)
D. cGMP
E. PLC

210 In the treatment of Parkinson's disease, what drug is Carbidopa peripherally inhibits the enzyme DOPA GRACE ARVIOLA, DIAGNOSTIC
given with levodopa to increase the drug's decarboxylase, the enzyme that converts DOPA to MD (TOP 3 - AUG EXAM - FEB
bioavailability? dopamine. Dopamine itself is unable to cross the 2014 MED 2015
A. Entacapone blood-brain barrier but DOPA can. Inhibition of the BOARDS;
B. Selegiline enzyme results in more DOPA available to enter the TOPNOTCH MD)
C. Carbidopa CNS. Within the CNS, DOPA is converted to
D. Fluoxetine dopamine by the same enzyme. However, it is not
E. Pyridoxine inhibited by carbidopa at this location.

211 Which drug used in the treatment of depression Imipramine is a tricyclic antidepressant. GRACE ARVIOLA, DIAGNOSTIC
possess a tricyclic ring? MD (TOP 3 - AUG EXAM - FEB
A. Imipramine 2014 MED 2015
B. Fluoxetine BOARDS;
C. Trazodone TOPNOTCH MD)
D. Venlafaxine
E. Bupropion

212 Which antiretroviral drug acts through the M2 ion Amantadine is also effective for influenza. GRACE ARVIOLA, DIAGNOSTIC
channel? MD (TOP 3 - AUG EXAM - FEB
A. Efavirenz 2014 MED 2015
B. Nevirapine BOARDS;
C. AZT TOPNOTCH MD)
D. Amantadine
E. Indinavir

213 Drug X has greater affinity for albumin than Drug Y. Only the free, unbound drug can interact with GRACE ARVIOLA, DIAGNOSTIC
Considering all other parameters are the same, what receptors, exert biologic activity, and be excreted by MD (TOP 3 - AUG EXAM - FEB
significant drug-drug interaction is expected to take the kidneys. 2014 MED 2015
place when both drugs are administered BOARDS;
simultaneously? TOPNOTCH MD)
A. Drug X will have a higher plasma concentration
than Drug Y.
B. Drug Y will not be filtered by the renal
glomerulus.
C. Drug Y will not be available to interact with its
receptors in the tissues.
D. The free plasma drug concentration of Drug Y will
increase.
E. Drug X can exert its full biologic activity.
214 Dissociative anesthesia is effectively achieved by the GRACE ARVIOLA, DIAGNOSTIC
combination of: MD (TOP 3 - AUG EXAM - FEB
A. Thiopental, diazepam, ketamine 2014 MED 2015
B. Ketamine, nitrous oxide, fentanyl BOARDS;
C. Succinylcholine, thiopental, diazepam TOPNOTCH MD)
D. Nitrous oxide, halothane, fentanyl
E. Succinylcholine, halothane, thiopental

TOPNOTCH MEDICAL BOARD PREP PHARMACOLOGY SUPEREXAM Page 25 of 83


For inquiries visit www.topnotchboardprep.com.ph or email us at topnotchmedicalboardprep@gmail.com
TOPNOTCH MEDICAL BOARD PREP PHARMACOLOGY SUPEREXAM
For inquiries visit www.topnotchboardprep.com.ph or email us at topnotchmedicalboardprep@gmail.com
Item QUESTION EXPLANATION AUTHOR TOPNOTCH
# EXAM
215 Among the inhaled anesthetics, nitrous oxide has the The MAC is a measure of an inhaled anesthetic's GRACE ARVIOLA, DIAGNOSTIC
_______ MAC and the ______ potency. potency. The relationship is inverse. MD (TOP 3 - AUG EXAM - FEB
A. Lowest, lowest 2014 MED 2015
B. Lowest, highest BOARDS;
C. Highest, lowest TOPNOTCH MD)
D. Highest, highest
E. Intermediate, intermediate

216 Which properties of digoxin make it an effective Digoxin also slows AV conduction (negative GRACE ARVIOLA, DIAGNOSTIC
treatment in cases of heart failure? dromotrope) MD (TOP 3 - AUG EXAM - FEB
A. Positive inotrope, positive chronotrope 2014 MED 2015
B. Positive inotrope, negative chronotrope BOARDS;
C. Negative inotrope, negative chronotrope TOPNOTCH MD)
D. Negative inotrope, positive chronotrope
E. None of the above

217 In anticoagulation, warfarin therapy is usually Warfarin acts by vitamin K anatagonism. The initial GRACE ARVIOLA, DIAGNOSTIC
overlapped with heparin for the first 1-2 days. Why is prothrombotic effect of warfarin is because the MD (TOP 3 - AUG EXAM - FEB
this so? synthesis of protein S, an anticoagulant, also 2014 MED 2015
A. To achieve supraoptimal anticoagulation during depends on vitamin K. Thus, protein S levels decline BOARDS;
critical periods of illness as warfarin and heparin initially and the effect of protein C (for which it TOPNOTCH MD)
have synergistic effects. serves as a co-factor) is also diminshed.
B. To prevent bleeding as heparin partially
counteracts warfarin's hemorrhagic property.
C. Warfarin is metabolized slowly thus leading to a
delay in anticoagulation if heparin is not also given.
D. To compensate for warfarin's initial
prothrombotic property.
E. Heparin decreases the clearance of warfarin thus
achieving greater plasma drug concentration of
warfarin.
218 A commercial product for colds contain the following: Phenylephrine constricts blood vessels through GRACE ARVIOLA, DIAGNOSTIC
Phenylephrine, chlorpheniramine, and paracetamol. stimulation of adrenergic receptors. This acts as a MD (TOP 3 - AUG EXAM - FEB
Which acts as the decongestant? decongestant. 2014 MED 2015
A. Paracetamol BOARDS;
B. Chlorpheniramine TOPNOTCH MD)
C. Phenylephrine
D. Both chlorpheniramine and phenylephrine
E. This product does not contain a decongestant.

219 Which antimalarial drug should not be given to a GRACE ARVIOLA, DIAGNOSTIC
patient with glucose-6-phosphate dehydrogenase MD (TOP 3 - AUG EXAM - FEB
deficiency? 2014 MED 2015
A. Chloroquine BOARDS;
B. Quinine TOPNOTCH MD)
C. Atovaquone-proquanil
D. Mefloquine
E. Primaquine

220 What drug should be given to an Ganciclovir is the drug of choice for CMV. Foscarnet GRACE ARVIOLA, DIAGNOSTIC
immunocompromised patient with CMV retinitis? is an alternative. MD (TOP 3 - AUG EXAM - FEB
A. Acyclovir 2014 MED 2015
B. Vidarabine BOARDS;
C. Indinavir TOPNOTCH MD)
D. Ganciclovir
E. AZT

221 What is the pharmacologic basis of using Timolol in MOA: letter A- alpha agonist and epinephrine; B- LEAN ANGELO MIDTERM
open angle glaucoma? Cholinomimetics ( physostigmine, pilocarpine) D- SILVERIO, MD EXAM 1 - FEB
A. Increaed outflow via dilatation of uveoscleral there is no such thing cAMP dependent decrease in (TOP 4 - AUG 2015
veins bicarbonate; however inhibition of Carbonic 2014 MED
B. It leads to opening of trabecular meshwork anhydrase leads to decrease humor production via BOARDS;
C. It suppresses the ciliary epithelium from decreased bicarbonate levels. TOPNOTCH MD),
producing aqueous humor MD
D. lt decrease production of aqueous humor by
decreasing levels of bicarbonate through cAMP
mediated pathway.
E. none of the above
222 A 7 yo male was brought to ER secondary to Botulinum toxin inhibits the release of vesicular LEAN ANGELO MIDTERM
symptoms suggestive of botulinum toxin ingestion, contents from all types of cholinergic nerve endings. SILVERIO, MD EXAM 1 - FEB
which of the following are likely included in the Since both ganglionic/autonomic and peripheral (TOP 4 - AUG 2015
symptoms of the patient? motor synapse are affected. Patient will present with 2014 MED
A. Muscle fasciculation hypotension and lack of muscular contraction. BOARDS;
B. Bronchoconstriction Blurred near vision is a result of paralysis of ciliary TOPNOTCH MD),
C. Increased bowel movement muscle aka cycloplegia MD
D. blurred near vision
E. hypertension
223 Which of the following will increase the hypokalemia, hypomagnesemia, hypercalcemia, and LEAN ANGELO MIDTERM
arrythmogenic side effect of digoxin? increase vagal tone increases the risk for digoxin SILVERIO, MD EXAM 1 - FEB
A. Decrease sympathetic discharge induced arrhytmia (TOP 4 - AUG 2015
B. Increase serum Potassium 2014 MED
C. Decrease serum Calcium BOARDS;
D. Increase Parasympathetic discharge TOPNOTCH MD),
E. None of the above MD

TOPNOTCH MEDICAL BOARD PREP PHARMACOLOGY SUPEREXAM Page 26 of 83


For inquiries visit www.topnotchboardprep.com.ph or email us at topnotchmedicalboardprep@gmail.com
TOPNOTCH MEDICAL BOARD PREP PHARMACOLOGY SUPEREXAM
For inquiries visit www.topnotchboardprep.com.ph or email us at topnotchmedicalboardprep@gmail.com
Item QUESTION EXPLANATION AUTHOR TOPNOTCH
# EXAM
224 Which of the following is correct regarding isosorbide ISMN is the oral form of nitroglycerin. Its mechanism LEAN ANGELO MIDTERM
mononitrate? is by activating guanylyl cyclase causing an increase SILVERIO, MD EXAM 1 - FEB
A. Can be use along with PDE inhibitors compared in cGMP eventually leading dephosphorylation of (TOP 4 - AUG 2015
to other forms of nitroglycerin MLC. Its antihypertensive effect is primarily via 2014 MED
B. It has a similar mechanism of action with venodilation. It has no direct inotropic or BOARDS;
Nitroprusside by phosphorylation of myosin light chronotropic effect to the heart. all forms of TOPNOTCH MD),
chain phosphate nitroglycerin should not concomitantly use with MD
C. It has no direct effect on cardiac muscle anyform of PDEI ( sildenafil) since it will cause
D. the primary mechanism for decrease in BP is due profound hypotension and hypoperfusion of critical
to decrease in afterload organs
E. B and C are correct
225 This drug when given IV and supraphysiologic dose Verapamil can also decrease AV node conduction LEAN ANGELO MIDTERM
will cause conduction block on AV node via increasing however via blocking L type Ca channels and not K SILVERIO, MD EXAM 1 - FEB
K efflux? channels. Fleicanide is a class IC drug that affects (TOP 4 - AUG 2015
A. Verapamil IV primarily Na channels present in both atria and 2014 MED
B. Adenosine ventricles. Amiodarone is the most efficacious of all BOARDS;
C. Fleicanide antiarrhythmic drug since it blocks Na, Ca, K and TOPNOTCH MD),
D. Amiodarone Beta receptors. the question is specific on its action MD
E. None of the above in reference to AV node. the best answer is
adenosine.
226 Which of the following drugs can cause prolonged QT antiarrhytmic drugs that affects the K channel at LEAN ANGELO MIDTERM
interval leading to Torsade de Pointes arrhytmia? phase 3 causes prolongation of Action Potential SILVERIO, MD EXAM 1 - FEB
A. Esmolol leading prolonged QT interval. These drugs are of (TOP 4 - AUG 2015
B. Propafenone the Class IA and III. Class IA drugs - procainamide, 2014 MED
C. Procainamide quinidine, disopyramide. Class III- Ibutilide, sotalol, BOARDS;
D. Verapamil dofetilide TOPNOTCH MD),
E. None of the above MD

227 Which in the following drugs is classified as 5HT1D Naratriptan and its prototype Sumatriptan are 5HT LEAN ANGELO MIDTERM
agonist? 1D agonist use for the treatment of acute migraine SILVERIO, MD EXAM 1 - FEB
A. Tegaserod and cluster headache. Tegaserod indicated for (TOP 4 - AUG 2015
B. Ketanserin constipation is a 5HT4 partial agonist.Ondansetron 2014 MED
C. Ondansetron is a 5-HT3 antagonist ise for antiemesis in post BOARDS;
D. Naratriptan operative and postchemotherapy patients. TOPNOTCH MD),
E. None of the above Ketanserin is a 5HT2 antagonist use for controlling MD
the systemic effects of carcinoid syndrome.
228 The primary indication of Epoprostenol? SIMILAR TO PREVIOUS BOARD EXAM LEAN ANGELO MIDTERM
A. Open angle glaucoma - increase outflow of CONCEPT/PRINCIPLE last aug 2014. Epoprostenol is SILVERIO, MD EXAM 1 - FEB
aqueous humor a Prostacyclin analog use for dialysis to prevent (TOP 4 - AUG 2015
B. Severe pulmonary hypertension platelet aggregation and also for pulmonary 2014 MED
C. Postpartum bleeding hypertension. Latanoprost or PGF2a analog is the BOARDS;
D. All of the above one use for glaucoma. Misoprostol or RU 486 (PGE1 TOPNOTCH MD),
E. None of the above analog) has an off label indication for induction of MD
labor. it primarily acts as an abortifacient.
229 What is the site of action of corticosteroids in terms of Corticosteroids exerts its anti inflammatory effect by LEAN ANGELO MIDTERM
its antiinflammatory effect? inhibiting the Phospholipase A2 enzyme which is SILVERIO, MD EXAM 1 - FEB
A. Cyclooxygenase 1 and 2 responsible for generation of arachidonic acid from (TOP 4 - AUG 2015
B. Phospholipase C membrane lipids. Phospholipase C is not connected 2014 MED
C. Phospholipase A2 with corticosteroids and its function is to generate BOARDS;
D. Thromboxane synthase IP3 and DAG. Cyclooxygenase and thromboxane TOPNOTCH MD),
E. All of the above synthase is primarily inhibit by Coxibs and NSAIDS. MD

230 the major systemic side effects of systemic Katzung Review of pharmacology 8th ed pp 171. . LEAN ANGELO MIDTERM
corticosteroids are much more likely to occur if at SILVERIO, MD EXAM 1 - FEB
least the duration of treatment is more than_____? (TOP 4 - AUG 2015
A. 14 days 2014 MED
B. 4 weeks BOARDS;
C. 3 weeks TOPNOTCH MD),
D. 20 days MD
E. 2 months

231 What is the most serious side effect of Methanol Methanol or wood alcohol is metabolized by alcohol LEAN ANGELO MIDTERM
ingestion? dehydrogenase to produce a product called SILVERIO, MD EXAM 1 - FEB
A. Retinal damage Formaldehyde and formic acid. This metabolite (TOP 4 - AUG 2015
B. Hepatotoxicity causes severe acidosis and prominent retinal 2014 MED
C. Blood dyscrasia damage. Nephrotoxicity is a major side effect of BOARDS;
D. Nephrotoxicity ethylene glycol poisoning via its oxalic acid TOPNOTCH MD),
E. Neurotoxicity- status epilepticus metabolite. treatment for both alcohol is by MD
administration of Fomepizole ( inhibitor alcohol
dehydrogenase) and ethanol ( competitive substrate
for alcohol dehydrogenase).
232 which of the following antiseizure drugs can inhibit Oxcarbazepine, carbamazepine, Barbiturates, LEAN ANGELO MIDTERM
cytochrome isoenzymes leading to increase plasma Phenytoin are all drug metabolizing inducers. Only SILVERIO, MD EXAM 1 - FEB
concentration of other drugs? Valproic acid in the choice is an inhibitor. (TOP 4 - AUG 2015
A. Phenobarbital Benzodiazepines do not affect the function of the 2014 MED
B. Lorazepam cytochromes. BOARDS;
C. Oxcarbazepine TOPNOTCH MD),
D. Valproic acid MD
E. None of the above

TOPNOTCH MEDICAL BOARD PREP PHARMACOLOGY SUPEREXAM Page 27 of 83


For inquiries visit www.topnotchboardprep.com.ph or email us at topnotchmedicalboardprep@gmail.com
TOPNOTCH MEDICAL BOARD PREP PHARMACOLOGY SUPEREXAM
For inquiries visit www.topnotchboardprep.com.ph or email us at topnotchmedicalboardprep@gmail.com
Item QUESTION EXPLANATION AUTHOR TOPNOTCH
# EXAM
233 Which of the following antiparkinsonian drugs can Katzung Review of pharmacology 8th ed pp 233. LEAN ANGELO MIDTERM
cause exacerbation of psychosis and produce ergot Both Levodopa and bromocriptine can cause SILVERIO, MD EXAM 1 - FEB
related effects such as pulmonary fibrosis and exacerbation of schizophrenia by increasing brain (TOP 4 - AUG 2015
erythromelalgia dopamine levels. However, only bromocriptine can 2014 MED
A. Amantadine cause ergot related side effects. BOARDS;
B. Bromocriptine TOPNOTCH MD),
C. Entacapone MD
D. Haloperidol
E. None of the above
234 Prolonged treatment with a PPAR alpha ligand in the PPAR alpha ligands is the MOA of fibric acid LEAN ANGELO MIDTERM
treatment of hyperlipidemia will increase the risk for derivatives ( Gemfibrozil and fenofibrate). It SILVERIO, MD EXAM 1 - FEB
the following condition? increases the supersaturation of bile leading (TOP 4 - AUG 2015
A. Cutaneous flushing cholelithiasis. Risk for hepatotoxicity only 2014 MED
B. Hyperuricemia leading to Gout significantly increase if gemfibrozil is combined with BOARDS;
C. Intestinal malabsorption HMG CoA inhibitor. Cutaneous flushing and TOPNOTCH MD),
D. Gallstones hyperuricemia are toxicity profile of Niacin. MD
E. hepatotoxicity Intestinal malabsorption is a primary side effect of
resins ( cholestyramine, colestipol, colsevelam)
235 Which of the following is not true in the Aminoglycoside exerts concentration dependent LEAN ANGELO MIDTERM
pharmacologic profile of Aminoglycosides? killing and post antibiotic effect. It also has curare SILVERIO, MD EXAM 1 - FEB
A. The primary mechanism of resistance is due to like action at high dose leading to respiratory (TOP 4 - AUG 2015
plasmid mediated group transferases paralysis. Aminoglycosides needs Oxygen dependent 2014 MED
B. Bactericidal action continuous even if the plasma transport to exerts inhibitory effect 30s ribosome. BOARDS;
drug concentration is below measuring levels therefore, it has no activity to anaerobes. acetylation TOPNOTCH MD),
C. It does not have an activity against anaerobic by plasmid mediated group transferase is the major MD
organisms mode of resistance.
D. high dose can cause respiratory paralysis
E. none of the above
236 Which of the following is a drug of choice for daily AIDS patient with a CD4 count of less than 50/UL are LEAN ANGELO MIDTERM
dose prophylaxis in AIDS patient with a CD4 count of prone to have Mycobacterium Avum Intracellulare SILVERIO, MD EXAM 1 - FEB
50/UL? infection. The only approved prophylactic treatment (TOP 4 - AUG 2015
A. Cotrimoxazole are daily dose of clarithromycin with or without 2014 MED
B. Kanamycin rifabutin and once weekly dose of azithromycin. BOARDS;
C. Azithromycin TOPNOTCH MD),
D. Erythromycin MD
E. Clarithromycin

237 Which in the following choices can be primarily use Sodium stibogluconate - treatment of all forms of LEAN ANGELO MIDTERM
for the treatment of hemolymphatic stage of leishmaniasis infection; Metronidazole is not SILVERIO, MD EXAM 1 - FEB
trypanosomiasis and for Pneumocystis jiroveci effective in the said infection, it is use for giardia and (TOP 4 - AUG 2015
infection trichomonas infection. Nifurtimox is for chagas 2014 MED
A. Sodium Stibogluconate diseae ( T cruzi infection). Cotrimoxazole is used for BOARDS;
B. Metronidazole P jiroveci but not effective in Trypanosomiasis TOPNOTCH MD),
C. Nifurtimox infection. MD
D. Pentamidine
E. Cotrimoxazole
238 What is the drug of choice for all forms of DEC is the drug of choice for loa loa infection, it is LEAN ANGELO MIDTERM
schistosomiasis and paragonimiasis? also an alternative drug for elephantiasis. Ivermectin SILVERIO, MD EXAM 1 - FEB
A. Diethylcarbamazine - DOC for onchocerciasis, cutaneous larva migransm (TOP 4 - AUG 2015
B. Praziquantel and strongyloides. Albendazole- ascariasis, 2014 MED
C. Ivermectin hookworm, pinworm, whipworm, cysticercosis and BOARDS;
D. Albendazole hydatid disease. Piperazine is an alternative drug for TOPNOTCH MD),
E. Piperazine ascariasis. MD

239 In what phase of the cell cycle does vinca alkaloid Vinca alkaloids ( vinblastine, vincristine, LEAN ANGELO MIDTERM
exerts its effect? vinorelbine) blocks the formation of mitotic spindle SILVERIO, MD EXAM 1 - FEB
A. G0 by preventing the assembly of tubin dimers into (TOP 4 - AUG 2015
B. G1 microtubules. Therefore it acts on the M phase. 2014 MED
C. S BOARDS;
D. G2 TOPNOTCH MD),
E. M MD

240 Which of the following drugs/drug regimen is B- testicular cancer, C- pancreatic cancer, D- LEAN ANGELO MIDTERM
primarily used for Hodgkin's Lymphoma choriocarcinoma. Aug 2014 SIMILAR TO PREVIOUS SILVERIO, MD EXAM 1 - FEB
A. ABVD- adriamycin, bleomycin, vincristine, BOARD EXAM CONCEPT/PRINCIPLE. (TOP 4 - AUG 2015
dacarbazine, prednisone 2014 MED
B. PEB - cisplatin, etoposide, bleomycin BOARDS;
C. Gemcitabine + 5 FU + radiation TOPNOTCH MD),
D. EMACO- etoposide, methotrexate, actinomycin D, MD
cyclophosphamide, vincristine
E. all of the above
241 25 year old female came in with chief complaint of SIMILAR TO PREVIOUS BOARD EXAM KEVIN BRYAN MIDTERM 2
cough of 3 weeks duration, this was accompanied by CONCEPT/PRINCIPLE.,Considered as the most LO, MD (TOP 7 - EXAM - FEB
night sweats, weight loss, body malaise. PE revealed active anti-TB drug, in fact the only drug given to AUG 2014 MED 2015
slightly pale palpebral conjunctiva, multiple bilateral treat latent tuberculosis BOARDS;
posterior cervical lymphadenopathies but with no TOPNOTCH MD)
adventitious breath sounds on bilateral lung fields.
The rest of the physical examination was
unremarkable, which of the following drugs is
considered to be the most active for her condition?
A. Rifampicin
B. Isoniazid
C. Pyrazinamide
D. Ethambutol
E. streptomycin

TOPNOTCH MEDICAL BOARD PREP PHARMACOLOGY SUPEREXAM Page 28 of 83


For inquiries visit www.topnotchboardprep.com.ph or email us at topnotchmedicalboardprep@gmail.com
TOPNOTCH MEDICAL BOARD PREP PHARMACOLOGY SUPEREXAM
For inquiries visit www.topnotchboardprep.com.ph or email us at topnotchmedicalboardprep@gmail.com
Item QUESTION EXPLANATION AUTHOR TOPNOTCH
# EXAM
242 A 56 year old patient diagnosed with rheumatic heart SIMILAR TO PREVIOUS BOARD EXAM KEVIN BRYAN MIDTERM 2
disease underwent mitral valve replacement and is CONCEPT/PRINCIPLE, which asked us directly LO, MD (TOP 7 - EXAM - FEB
placed on oral anticoagulant Warfarin, she comes to which of the following is a CYP inhibitor: cimetidine, AUG 2014 MED 2015
your clinic for a follow up check up and complaints of Warfarin is majorly metabolized by CYP 2C9, BOARDS;
heartburn and epigastric pain, which of the following cimetidine inhibits this enzyme among others which TOPNOTCH MD)
medications will you avoid prescribing? inhibits warfarin metabolism which increases its
A. Cimetidine toxicity
B. Aluminum Magnesium Hydroxide
C. lansoprazole
D. ranitidine
E. sucralfate
243 Which of the following medications is considered as SIMILAR TO PREVIOUS BOARD EXAM KEVIN BRYAN MIDTERM 2
an inhibitor of glucocorticoid synthesis which could CONCEPT/PRINCIPLE, I couldn’t believe this was LO, MD (TOP 7 - EXAM - FEB
be used in the treatment of cushing's syndrome and actually asked, aminoglutethimide inhibits AUG 2014 MED 2015
some types of cancers? desmolase blocking conversion of cholesterol to BOARDS;
A. Methylprednisolone pregnenolone reduces synthesis of all hormonally TOPNOTCH MD)
B. anastrozole active steroids, anastrozole inhibits only estrogen
C. Danazol synthesis, danazol is an androgen, flutamide is
D. Aminogluthetimide androgen antagonist
E. Flutamide
244 Which of the following drugs can cause cataract SIMILAR TO PREVIOUS BOARD EXAM KEVIN BRYAN MIDTERM 2
formation? CONCEPT/PRINCIPLE, one of the hardes pharma LO, MD (TOP 7 - EXAM - FEB
A. allopurinol questions we encountered, we did not know at that AUG 2014 MED 2015
B. pyrazinamide time that allopurinol was associated with cataracts :) BOARDS;
C. propanolol TOPNOTCH MD)
D. omeprazole
E. ciprofloxacin

245 Which of the following chemotherapeutic regimens SIMILAR TO PREVIOUS BOARD EXAM KEVIN BRYAN MIDTERM 2
could be used for the treatment of advanced CONCEPT/PRINCIPLE - MOPP nitrogen mustard, LO, MD (TOP 7 - EXAM - FEB
hodgkin's disease? oncovin, procarbazine, prednisone, take time to AUG 2014 MED 2015
A. FOLFOX know the common regimens for other cancers as BOARDS;
B. MOPP well, ex: folfox colorectal, CHOP nonhodgkins, BEP TOPNOTCH MD)
C. CHOP testicular, 7+3 AML
D. 7+3
E. BEP

246 Which of the following medications causes mydriasis SIMILAR TO PREVIOUS BOARD EXAM KEVIN BRYAN MIDTERM 2
but without cycloplegia? CONCEPT/PRINCIPLE also a difficult one, only LO, MD (TOP 7 - EXAM - FEB
A. tropicamide phenylephrine among the choices causes mydriasis AUG 2014 MED 2015
B. homatropine without affecting accomodation BOARDS;
C. cylopentolate TOPNOTCH MD)
D. phenylephrine
E. atropine

247 When a pregnant 18 year old female delivered her SIMILAR TO PREVIOUS BOARD EXAM KEVIN BRYAN MIDTERM 2
child, her baby was noted to have characteristic small CONCEPT/PRINCIPLE, know the common LO, MD (TOP 7 - EXAM - FEB
palpebral fissures, smooth philtrum and thin upper syndromes and teratogen, answer here is fetal AUG 2014 MED 2015
lip, which of the following substances when ingested alcohol syndrome BOARDS;
by the mother during pregnancy could potentially be TOPNOTCH MD)
the cause of such changes?
A. phenytoin
B. warfarin
C. ethanol
D. captopril
E. isotretinoin
248 What is the predominant reason why nitroglycerin is SIMILAR TO PREVIOUS BOARD EXAM KEVIN BRYAN MIDTERM 2
generally preferred not to be given via the oral route? CONCEPT/PRINCIPLE, high first pass effect is the LO, MD (TOP 7 - EXAM - FEB
A. could potentially cause dangerous systemic best answer, low bioavailability through the oral AUG 2014 MED 2015
hypotension route due to liver metabolism BOARDS;
B. is considerably less effective when given through TOPNOTCH MD)
this route
C. is immediately inactivated by exposure to low
gastric pH
D. there is high first pass effect
E. food in the stomach significantly retards
absorption
249 Which of the following is an inhalational anesthetic of SIMILAR TO PREVIOUS BOARD EXAM KEVIN BRYAN MIDTERM 2
choice for asthmatic patient because it causes the CONCEPT/PRINCIPLE, halothane generally is the LO, MD (TOP 7 - EXAM - FEB
least bronchospasm? inhaled agent of choice, sevoflurane is debatable, AUG 2014 MED 2015
A. halothane desflurane is pungent, the others are IV anesthetics BOARDS;
B. desflurane TOPNOTCH MD)
C. sevoflurane
D. thiopental
E. propofol

250 Which of the following drugs given contains an effect SIMILAR TO PREVIOUS BOARD EXAM KEVIN BRYAN MIDTERM 2
which suppresses nausea and vomiting ? CONCEPT/PRINCIPLE, difficult because LO, MD (TOP 7 - EXAM - FEB
A. propanolol chlorpromazine usually is used as a antipsychotic AUG 2014 MED 2015
B. procarbazine but has antiemetic effects belonging to the BOARDS;
C. chlorambucil phenothiazine class along with other drugs used to TOPNOTCH MD)
D. chlorpromazine control nausea vomiting like promethazine
E. chloramphenicol

TOPNOTCH MEDICAL BOARD PREP PHARMACOLOGY SUPEREXAM Page 29 of 83


For inquiries visit www.topnotchboardprep.com.ph or email us at topnotchmedicalboardprep@gmail.com
TOPNOTCH MEDICAL BOARD PREP PHARMACOLOGY SUPEREXAM
For inquiries visit www.topnotchboardprep.com.ph or email us at topnotchmedicalboardprep@gmail.com
Item QUESTION EXPLANATION AUTHOR TOPNOTCH
# EXAM
251 Which of the following medications could be given to SIMILAR TO PREVIOUS BOARD EXAM KEVIN BRYAN MIDTERM 2
high risk or immunocompromised patients with RSV CONCEPT/PRINCIPLE, acyclovir and ganciclovir for LO, MD (TOP 7 - EXAM - FEB
infections? herpes group, indinavir and fosamprenavir for hiv AUG 2014 MED 2015
A. Acyclovir BOARDS;
B. Fosamprenavir TOPNOTCH MD)
C. Indinavir
D. Ganciclovir
E. Ribavirin

252 Using drugs to block which of the following chemical SIMILAR TO PREVIOUS BOARD EXAM KEVIN BRYAN MIDTERM 2
mediators could potentially decrease leukocyte CONCEPT/PRINCIPLE, but leukotrienes could also LO, MD (TOP 7 - EXAM - FEB
chemotaxis? serve as chemotactic factors AUG 2014 MED 2015
A. prostaglandin H BOARDS;
B. cyclooxygenase TOPNOTCH MD)
C. bradykinin
D. leukotrienes
E. serotonin

253 A patient is taking fenofibrate for elevated SIMILAR TO PREVIOUS BOARD EXAM KEVIN BRYAN MIDTERM 2
trigylceride levels, which of the following laboratory CONCEPT/PRINCIPLE, fibrates can potentially LO, MD (TOP 7 - EXAM - FEB
examinations should the doctor request to monitor increase liver enzymes AUG 2014 MED 2015
for potential side effects upon follow up after 3 to 6 BOARDS;
months? TOPNOTCH MD)
A. CBC with differential count
B. urinalysis
C. AST, ALT
D. fasting blood glucose levels
E. serum total cholesterol and triglycerides
254 Which of the following medications can be used as SIMILAR TO PREVIOUS BOARD EXAM KEVIN BRYAN MIDTERM 2
part of a regimen for migraine prophylaxis? CONCEPT/PRINCIPLE, prophylaxis: pizotifen, TCA, LO, MD (TOP 7 - EXAM - FEB
A. ibuprofen anticonvulsants and beta blockers, methysergide, AUG 2014 MED 2015
B. acetaminophen flunarizine BOARDS;
C. sumatriptan TOPNOTCH MD)
D. propranolol
E. dihydroergotamine

255 Which of the following medications can decrease SIMILAR TO PREVIOUS BOARD EXAM KEVIN BRYAN MIDTERM 2
conduction through the AV node? CONCEPT/PRINCIPLE, verapamil is a non LO, MD (TOP 7 - EXAM - FEB
A. nifedipine dihydropyridine calcium channel blocker which AUG 2014 MED 2015
B. verapamil exerts its actions more on the heart decreasing AV BOARDS;
C. lithium nodal conduction TOPNOTCH MD)
D. prazosin
E. dypyridamole

256 A 24 year old female is taking oral contraceptives as SIMILAR TO PREVIOUS BOARD EXAM KEVIN BRYAN MIDTERM 2
family planning method. Which of the following drugs CONCEPT/PRINCIPLE, what came out as an enzyme LO, MD (TOP 7 - EXAM - FEB
is considered a CYP450 enzyme inducer which can inducer in ours was phenobarbital, griseofulvin is an AUG 2014 MED 2015
cause rapid metabolism of other drugs such as oral inducer all the others are inhibitors, memorize at BOARDS;
contraceptives which can potentially decrease their least the most common of them inducers and TOPNOTCH MD)
effectiveness? inhibitors, topnotch handouts are good reference
A. isoniazid
B. trimethoprim sulfamethoxazole
C. griseofulvin
D. Amiodarone
E. ketoconazole
257 Which of the following chemotherapeutic agents can SIMILAR TO PREVIOUS BOARD EXAM KEVIN BRYAN MIDTERM 2
produce pulmonary fibrosis as toxicity? CONCEPT/PRINCIPLE, know the characteristic LO, MD (TOP 7 - EXAM - FEB
A. bleomycin adverse effects and toxicities, the peculiar ones, the AUG 2014 MED 2015
B. doxorubicin ones that make the drugs famous, pulmo fibrosis = BOARDS;
C. irinotecan bleomycin TOPNOTCH MD)
D. etoposide
E. carmustine

258 Which among the following cardiac drugs can know the mechanism understand them, know and KEVIN BRYAN MIDTERM 2
decrease preload, afterload and contractility? master normal physiology, digoxin is a negative LO, MD (TOP 7 - EXAM - FEB
A. digoxin chronotropic but positive inotropic inc contractility, AUG 2014 MED 2015
B. captopril captopril inhibits ACE, acts on the RAAS decreases BOARDS;
C. amlodipine preload by decreasing aldosterone and sodium TOPNOTCH MD)
D. hydrochlorthiazide water retention hence decrease blood volume dec
E. propranolol venous return, amlodipine calcium antagonist
predominantly on the vessels decrease afterload by
dilating resistance vessels, hydrochlorthiazide
decreases preload by increasing secretion of both
sodium water, propranolol, non selective beta
blocker, decreases renin secretion hence RAAS
through B1 blockage this includes angiotensin 2 so
decreased both preload afterload, decreases
contractilitty through B1 blockade,
259 Which of the following antibiotics are highly protein SIMILAR TO PREVIOUS BOARD EXAM KEVIN BRYAN MIDTERM 2
bound which can act to displace other substances CONCEPT/PRINCIPLE, sulfa drugs are usually highly LO, MD (TOP 7 - EXAM - FEB
from albumin binding and increase free drug levels protein bound drugs which can displace other drugs AUG 2014 MED 2015
A. chloramphenicol making the free drug available to easily permeate BOARDS;
B. ciprofloxacin cell membranes or bind to receptors, it can also TOPNOTCH MD)
C. cotrimoxazole displace bilirubin from albumin raising bilirubin
D. tetracycline levels in neonates
E. gentamicin

TOPNOTCH MEDICAL BOARD PREP PHARMACOLOGY SUPEREXAM Page 30 of 83


For inquiries visit www.topnotchboardprep.com.ph or email us at topnotchmedicalboardprep@gmail.com
TOPNOTCH MEDICAL BOARD PREP PHARMACOLOGY SUPEREXAM
For inquiries visit www.topnotchboardprep.com.ph or email us at topnotchmedicalboardprep@gmail.com
Item QUESTION EXPLANATION AUTHOR TOPNOTCH
# EXAM
260 Which of the following drugs with mechanism of SIMILAR TO PREVIOUS BOARD EXAM KEVIN BRYAN MIDTERM 2
action that inhibits protein synthesis by inhibiting CONCEPT/PRINCIPLE, please know the mechanism LO, MD (TOP 7 - EXAM - FEB
binding of the tRNA to the mRNA ribosome complex of action of each drug class AUG 2014 MED 2015
A. tetracycline BOARDS;
B. clindamycin TOPNOTCH MD)
C. gentamycin
D. erythromycin
E. chloramphenicol

261 If a single dose of a known drug with first order Review first-order elimination. 50% is one half-life, RAYMUND MIDTERM 3
elimination is given intravenously, how long will it 75% is 2 X half-life and so on.. MARTIN LI, MD EXAM - FEB
take for 75% of the drug to be eliminated if the half- (TOP 1 - AUG 2015
life is 1.5 hours? 2014 MED
A. 1.5 hours BOARDS;
B. 2 hours TOPNOTCH MD)
C. 2.5 hours
D. 3 hours
E. 6 hours
262 Which of the following teratogenic effects is A is carbamazepine; C is Lithium, D is phenytoin RAYMUND MIDTERM 3
associated with valproic acid use in pregnancy? MARTIN LI, MD EXAM - FEB
A. Craniofacial anomalies (TOP 1 - AUG 2015
B. Neural tube defects 2014 MED
C. Ebstein anomaly BOARDS;
D. Fetal hydantoin syndome TOPNOTCH MD)
E. Microcephaly

263 You are assigned to go on duty as an intern of the Iron, lithium, cyanide, alcohol are poorly adsorbed RAYMUND MIDTERM 3
toxicology department of the hospital. You recall that by activated charcoal MARTIN LI, MD EXAM - FEB
activated charcoal will NOT be effective adsorbing (TOP 1 - AUG 2015
which of the following? 2014 MED
A. Iron BOARDS;
B. Theophylline TOPNOTCH MD)
C. Phenobarbital
D. Amitryptiline
E. Digoxin
264 Which of the following Cholinomimetic drugs are Donepezil, Rivastigmine, Tacrine used exclusively in RAYMUND MIDTERM 3
used exclusively in the treatment of Alzheimer's Alzheimer's MARTIN LI, MD EXAM - FEB
disease? (TOP 1 - AUG 2015
A. Donepezil 2014 MED
B. Physostigmine BOARDS;
C. Rivastigmine TOPNOTCH MD)
D. A and B
E. A and C

265 You prescribe a cancer patient who is undergoing Azathioprine/Mercaptopurine metabolism is RAYMUND MIDTERM 3
chemotherapy with allopurinol to address inhibited by allopurinol. MARTIN LI, MD EXAM - FEB
hyperuricemia. Upon reviewing his chart, you realize (TOP 1 - AUG 2015
caution is needed because of risk of toxicity. Which of 2014 MED
the following chemotherapeutic drus is most likely BOARDS;
being given to the patient? TOPNOTCH MD)
A. Azathioprine
B. Methotrexate
C. Fluorouracil
D. Cytarabine
E. All of the above
266 Adrenoreceptor blockers such as acebutolol and RAYMUND MIDTERM 3
pindolol are unique for having intrinsic sympathetic MARTIN LI, MD EXAM - FEB
activity. What is meant by this characteristic? (TOP 1 - AUG 2015
A. has a membrane-stabilizing activity 2014 MED
B. has partial agonist activity BOARDS;
C. has receptor selectivity TOPNOTCH MD)
D. A and B
E. All of the above

267 Which of the following drugs that used in gout has SIMILAR TO PREVIOUS BOARD EXAM RAYMUND MIDTERM 3
been associated with cataract formation? CONCEPT/PRINCIPLE MARTIN LI, MD EXAM - FEB
A. Colchicine (TOP 1 - AUG 2015
B. Allopurinol 2014 MED
C. Probenecid BOARDS;
D. Indomethacin TOPNOTCH MD)
E. Acetaminophen

268 You are treating a patient HIV-patient for an Cidofovir is phosphorylated by host kinases while RAYMUND MIDTERM 3
opportunistic viral infection affecting the eyes. If you Foscarnet doesn’t reqauire phosphorylation. Both MARTIN LI, MD EXAM - FEB
suspect viral strains that are thymidine-kinase are used in thymidine-deficient resistant strains of (TOP 1 - AUG 2015
deficient, you decide to choose an antiviral drug that CMV 2014 MED
is exclusively phosphorylated by host cell kinases but BOARDS;
which doesn't require viral kinase phosphorylation. TOPNOTCH MD)
Which of the following will you choose?
A. Acyclovir
B. Ganciclovir
C. Cidofovir
D. Foscarnet
E. Lamivudine

TOPNOTCH MEDICAL BOARD PREP PHARMACOLOGY SUPEREXAM Page 31 of 83


For inquiries visit www.topnotchboardprep.com.ph or email us at topnotchmedicalboardprep@gmail.com
TOPNOTCH MEDICAL BOARD PREP PHARMACOLOGY SUPEREXAM
For inquiries visit www.topnotchboardprep.com.ph or email us at topnotchmedicalboardprep@gmail.com
Item QUESTION EXPLANATION AUTHOR TOPNOTCH
# EXAM
269 A 63 year old male patient presenting with Nitroglcerin decreases afterload, thus, decreasing RAYMUND MIDTERM 3
intermittent complaints of chest pain upon walking end systolic volume. Compensatory sympathetic MARTIN LI, MD EXAM - FEB
several blocks or climbing several flights of stairs is activity increases heart rate. (TOP 1 - AUG 2015
prescribed with nitroglycerin to used on an as needed 2014 MED
basis. What will be the expected physiologic changes BOARDS;
as a result of this drug? TOPNOTCH MD)
A. Increased HR and and decreased left ventricular
end-systolic volume
B. Decreased HR and decreased left ventricular end-
systolic volume
C. Decreased HR and increased left ventricular end-
systolic volume
D. Increased HR and increased left ventricular end-
systolic volume
E. None of the above
270 Which of the following mechanism indicates high- E is INH high level resistance; A is INH low level RAYMUND MIDTERM 3
level resistance in a TB patient being treated with resistance; B is rifampicin resistance MARTIN LI, MD EXAM - FEB
Isoniazid? (TOP 1 - AUG 2015
A. deletions in inhA gene that encodes target enzyme 2014 MED
B. changes in drug sensitivity to RNA polymerase BOARDS;
C. expression of drug efflux system TOPNOTCH MD)
D. expression of inactivating enzymes
E. deletion in katG gene involved in the bioactivation
of the drug
271 A drug with diuretic activity is being studied based on Thiazide increases Na and K excretion and causes RAYMUND MIDTERM 3
its effects on electrolyte levels in the urine. It was metabolic alkalosis. It increases Ca reabsorption MARTIN LI, MD EXAM - FEB
found to moderately increase urine NaCl and urine K (TOP 1 - AUG 2015
while decreasing urine Ca with associated slight 2014 MED
increase in body pH and minimal change in urine BOARDS;
HCO3-. This drug has characteristics similar to that TOPNOTCH MD)
of?
A. Loop diuretic
B. Carbonic anhydrase inhibitor
C. K-sparing diuretic
D. Thiazide diuretic
E. Osmotic diuretic
272 Which of the following steroid antagonists inhibits Ketoconazole is a potent cytochrome p450 inhibitor. RAYMUND MIDTERM 3
cytochrome P450 enxymes necessary for all steroids? Aminoglutethimide inhibits conversion of MARTIN LI, MD EXAM - FEB
A. Ketoconazole cholesterol to pregnenolone. Metyrapone inhibits (TOP 1 - AUG 2015
B. Aminogluthetimide cortisol synthesis. Spinorolactone inhibits 2014 MED
C. Metyrapone aldosterone. Mifepristone inhibits progesterone BOARDS;
D. Spinorolactone TOPNOTCH MD)
E. Mifepristone

273 A patient with warfarin as a maintenance medication Erythromycin inhibits metabolism of warfarin RAYMUND MIDTERM 3
is treated for pneumonia. Prothrombin time causing an increase In INR MARTIN LI, MD EXAM - FEB
monitoring during treatment of the respiratory (TOP 1 - AUG 2015
infection reveals elevated INR but it decrease back to 2014 MED
baseline levels after treatment with the antibiotic is BOARDS;
completed. Which of the following is most likely TOPNOTCH MD)
given?
A. Amoxicillin
B. Ceftriaxone
C. Clindamycin
D. Erythromycin
E. TMP-SMX
274 A COPD patient with severe concomitant heart Ipratoropium is an anticholinergic used in COPD RAYMUND MIDTERM 3
condition is being prescribed a drug that will relieve because of less cardiac effects MARTIN LI, MD EXAM - FEB
his dyspnea and shortness of breath. Which of the (TOP 1 - AUG 2015
following bronchodilators will be the primary choice? 2014 MED
A. Salmeterol BOARDS;
B. Salbutamol TOPNOTCH MD)
C. Terbutaline
D. Atropine
E. Ipratropium
275 A physician prescribes a diabetic patient with an Thiozolidinediones - activates PPAR, a nuclear RAYMUND MIDTERM 3
antidiabetic agent that activates a nuclear receptor receptor to improve sensitivity to insulin MARTIN LI, MD EXAM - FEB
that increases transcription of GLUT-4 transporters in (TOP 1 - AUG 2015
adipose tissue. The drug prescribed is most likely: 2014 MED
A. Methotrexate BOARDS;
B. Insulin TOPNOTCH MD)
C. Pioglitazone
D. Glyburide
E. Sitagliptin

TOPNOTCH MEDICAL BOARD PREP PHARMACOLOGY SUPEREXAM Page 32 of 83


For inquiries visit www.topnotchboardprep.com.ph or email us at topnotchmedicalboardprep@gmail.com
TOPNOTCH MEDICAL BOARD PREP PHARMACOLOGY SUPEREXAM
For inquiries visit www.topnotchboardprep.com.ph or email us at topnotchmedicalboardprep@gmail.com
Item QUESTION EXPLANATION AUTHOR TOPNOTCH
# EXAM
276 A patient presented to your clinic with complaints of Methotrexate is known to cause pulmonary fibrosis RAYMUND MIDTERM 3
dyspnea and chronic cough. He states that he MARTIN LI, MD EXAM - FEB
frequently gets short of breath and is unable to take a (TOP 1 - AUG 2015
deep breath. History is significant for prolonged 2014 MED
treatment of rheumatoid arthritis. Chest X-ray BOARDS;
revealed fine reticulonodular densities. Which drug is TOPNOTCH MD)
most likely responsible?
A. Glucocorticoids
B. Methotrexate
C. Infliximab
D. Cyclosporine
E. Anakinra
277 If you need to give a skeletal muscle relaxant to a Succinylcholine is contraindicated in patient with RAYMUND MIDTERM 3
burn patient, you will AVOID prescribing which of the rhabdomyolysis, hyperkalemia, burn patients, MARTIN LI, MD EXAM - FEB
following drugs? malignant hyperthermia (TOP 1 - AUG 2015
A. Pancuronium 2014 MED
B. Baclofen BOARDS;
C. Dantrolene TOPNOTCH MD)
D. Vecuronium
E. Succinylcholine

278 Which of the following medications exert its Fondaparinux and LMW heparin combines with RAYMUND MIDTERM 3
anticoagulant effect through the inactivation of antithrombin and selectively inhibits factor X but MARTIN LI, MD EXAM - FEB
thrombin? NOT thrombin. Apixaban is a direct Xa inhibitor (TOP 1 - AUG 2015
A. Fondaparinux 2014 MED
B. Unfractionated heparin BOARDS;
C. Low molecular weight heparin TOPNOTCH MD)
D. Apixaban
E. All of the above
279 One of the following drugs is effective in reducing the RAYMUND MIDTERM 3
risk of ulcers in chronic NSAID users. Diarrhea is a MARTIN LI, MD EXAM - FEB
frequent side effect. Which is it? (TOP 1 - AUG 2015
A. Sucralfate 2014 MED
B. Ranitidine BOARDS;
C. Omeprazole TOPNOTCH MD)
D. Misoprostol
E. Cimetidine

280 Knowledge of potency and solubility is important in Low MAC - high potency; High MAC - low potency; RAYMUND MIDTERM 3
general anesthetic medications. Which of the Low blood:gas coefficient - low solubility; High MARTIN LI, MD EXAM - FEB
following characteristics refers to high potency? blood:gas coeeficient - high solubility (TOP 1 - AUG 2015
A. Low minimum alveolar concentration 2014 MED
B. High minimum alveolar concentration BOARDS;
C. Low blood:gas partition coefficient TOPNOTCH MD)
D. High blood:gas partition coefficient
E. None of the above

281 Which of the following anti hypertensive medications Intake of ACE inhibitors can produce teratogenic ERIC ROYD FINAL EXAM -
when taken during pregnancy can cause hypocalvaria effect in the fetus which includes renal dysgenesis TALAVERA, MD FEB 2015
in the fetus? and hypoplastic skulls (hypocalvaria) thus it is (TOP 1 - AUG
A. Carvedilol contraindicated during pregnancy 2014 MED
B. Amlodipine BOARDS;
C. Captopril TOPNOTCH MD)
D. HCTZ
E. Hydralazine

282 A 29 year old woman presented with amenorrhea, Bromocriptine is an effective dopamine agonist in ERIC ROYD FINAL EXAM -
galactorrhea and loss of sexual libido. On evaluation the CNS with the advantage of oral activity. The drug TALAVERA, MD FEB 2015
her serum prolactin was noted to be markedly inhibits prolactin secretion by activating pituitary (TOP 1 - AUG
elevated. Which of the following medications is most dopamine receptors 2014 MED
useful in the treatment of her condition? BOARDS;
A. Bromocriptine TOPNOTCH MD)
B. Cimetidine
C. Sumatriptan
D. Ergotamine
E. Ondansetron
283 A 47 year old obese male, who is a heavy alcoholic Metformin is associated with lactic acidosis. It ERIC ROYD FINAL EXAM -
beverage drinker, was recently diagnosed with should be avoided or used with extreme caution in TALAVERA, MD FEB 2015
diabetes mellitus. He was started on Metformin 500 patients who are heavy alcoholics since acute (TOP 1 - AUG
mg/tab TID. This patient is at risk of developing ethanol ingestion increaes the risk of lactic acidosis 2014 MED
which of the following complications? BOARDS;
A. Hypoglycemia TOPNOTCH MD)
B. Disulfiram like reaction
C. Congestive heart failure
D. Lactic acidosis
E. Diarrhea

TOPNOTCH MEDICAL BOARD PREP PHARMACOLOGY SUPEREXAM Page 33 of 83


For inquiries visit www.topnotchboardprep.com.ph or email us at topnotchmedicalboardprep@gmail.com
TOPNOTCH MEDICAL BOARD PREP PHARMACOLOGY SUPEREXAM
For inquiries visit www.topnotchboardprep.com.ph or email us at topnotchmedicalboardprep@gmail.com
Item QUESTION EXPLANATION AUTHOR TOPNOTCH
# EXAM
284 A 65 year old female presented to the ER due to This is a case of an acute decompensated heart ERIC ROYD FINAL EXAM -
progressive dyspnea. She is a known hypertesive but failure. Due to the marked systolic dysfunction as TALAVERA, MD FEB 2015
is poorly compliant with medications. On history, evident by a depressed ejection fraction, beta (TOP 1 - AUG
patient claims to experience orthopnea, paroxysmal blockers are contraindicated at this time as it may 2014 MED
nocturnal dyspnea and easy fatigability. On PE, her BP further lower the cardiac output. BOARDS;
is 80/50. There is prominent neck vein distention. S3 TOPNOTCH MD)
gallop, bibasal crackle and a grade 3 bipedal edema
were also appreciated. A 2d echo was done which
showed a depressed ejection fraction of 32%. Which
of the following medications should not be given at
this time for this patient ?
A. Furosemide
B. Digoxin
C. Metoprolol
D. Dobutamine
E. None of the above
285 A 75 year old male was brought to the ER due to Mannitol is used to decrease the ICP in patient's with ERIC ROYD FINAL EXAM -
sudden onset of right sided weakness accompanied hemorrhagic stroke. It acts as an osmotic diuretic TALAVERA, MD FEB 2015
by a progressive deterioration in sensorium. On PE, thus removes water from the intracellular (TOP 1 - AUG
BP was elevated at 220/100. Neurologic exam compartment. This rapid fluid shifting may cause 2014 MED
showed a GCS of 11, an MMT of 2/5 on the both right hyponatremia and pulmonary edema and as the BOARDS;
upper and lower extremities and a positive babinski water is excreted, hypernatremia may follow. TOPNOTCH MD)
on the right. A CT scan showed a massive
intracerebral hemorrhage involving the left basal
ganglia. A diuretic was started to decrease the
patient's intracranial pressure. Which of the following
is an associated adverse effect of this medication ?
A. Ototoxicity
B. Hyperlipidemia
C. Gynecomastia
D. Pulmonary Edema
E. Thrombocytosis
286 Which of the following is an orally active direct Apixaban and Rivaroxaban are both oral Factor Xa ERIC ROYD FINAL EXAM -
thrombin inhibitor? inhibitors. Enoxaparin and Fondaparinux are both TALAVERA, MD FEB 2015
A. Apixaban SC factor Xa inhibior. (TOP 1 - AUG
B. Rivaroxaban 2014 MED
C. Enoxaparin BOARDS;
D. Fondaparinux TOPNOTCH MD)
E. Dabigatran

287 A 48 year old female, a known case of Acute Valaciclovir is a prodrug, an esterified version of ERIC ROYD FINAL EXAM -
Myelogenous Leukemia (AML), who is on reinduction aciclovir that has greater oral bioavailability (about TALAVERA, MD FEB 2015
chemotherapy was started on anti viral prophylaxis 55%) than aciclovir (10–20%). It is phophorylated (TOP 1 - AUG
with Valacyclovir 500 mg/tab TID. What is the three times and this form acts as a competitive 2014 MED
mechanism of action of this anti viral medication? substrate for DNA polymerase and it leads to chain BOARDS;
A. Inhibition of Viral DNA polymerase termination TOPNOTCH MD)
B. Blockage of M2 proton channel
C. Inhibition of neuraminidase
D. Inhibition of viral reverse transcriptase
E. Prevents fusion of virus with the host cellular
membrane
288 Which of the following drugs of abuse is non Some drugs of abuse do not lead to addiction. This is ERIC ROYD FINAL EXAM -
addictive? the case for substances that alter perception without TALAVERA, MD FEB 2015
A. Amphetamine causing sensation of reward and euphoria. These (TOP 1 - AUG
B. Cocaine include LSD, Phencyclidine and ketamine 2014 MED
C. Morphine BOARDS;
D. LSD TOPNOTCH MD)
E. Heroin

289 A 70 year old male who is diagnosed with Colon Imatinib is a tyosine kinase inhibitor used in CML. ERIC ROYD FINAL EXAM -
Cancer Stage IV with metastasis to the liver who is Sorafenib inhibit multiple tyrosne kinase receptors TALAVERA, MD FEB 2015
currently on FOLFOX regimen was started on a and are primarily used in HCC. Trastuzumab inhibits (TOP 1 - AUG
monoclonal antibody which binds to the vascular cells that overexpress the Her-2/neu recepto in 2014 MED
endothelial growth factor (VEGF) receptor. What is Breast CA. Erlotinib inhibits epidermal growth factor BOARDS;
the medication that was given to the patient? receptor and is used for non small cell lung CA and TOPNOTCH MD)
A. Imatinib pancreatic cancer
B. Sorafenib
C. Trastuzumab
D. Bevacizumab
E. Erlotinib
290 Which of the following medications can be given once because of its long elimination half life (3-4 days), ERIC ROYD FINAL EXAM -
a week for prophylaxis against bactermia caused by weekly administration of azithromycin has proved TALAVERA, MD FEB 2015
M. avium complex (MAC) in AIDS patients? to be equivalent to daily administration of (TOP 1 - AUG
A. Azithromycin clarithromycin when used for prophylaxis against 2014 MED
B. Kanamycin MAC in AIDS BOARDS;
C. Ethambutol TOPNOTCH MD)
D. Rifampicin
E. Cycloserine

TOPNOTCH MEDICAL BOARD PREP PHARMACOLOGY SUPEREXAM Page 34 of 83


For inquiries visit www.topnotchboardprep.com.ph or email us at topnotchmedicalboardprep@gmail.com
TOPNOTCH MEDICAL BOARD PREP PHARMACOLOGY SUPEREXAM
For inquiries visit www.topnotchboardprep.com.ph or email us at topnotchmedicalboardprep@gmail.com
Item QUESTION EXPLANATION AUTHOR TOPNOTCH
# EXAM
291 A 37 year old female diagnosed with chronic Clozapine can cause agranulocytosis in a small but ERIC ROYD FINAL EXAM -
schizophrenia maintained on an antipsychotic significant number of patients (1-2%). This serious, TALAVERA, MD FEB 2015
medication was brought to the clinic due to a 3 day potentially fatal effect can develop rapidly usually (TOP 1 - AUG
history of fever. PE showed presence of tonsillar between the 6th to 18th week of treatment. It 2014 MED
exudates. A CBC was requested which showed a appears to be reversible upon discontinuation of BOARDS;
markedly decreased WBC count. The medication was treatment. Because of this risk, patient on clozapine TOPNOTCH MD)
immediately discontinued by the patient's physician. must have weekly blood counts for the first 6
What is the anti-psychotic medication that could have months of treatment and every 3 weeks thereafter
caused this problem?
A. Haloperidol
B. Quetiapine
C. Clozapine
D. Risperidone
E. Olanzapine
292 What is the major advantage of Etomidate over other Etomidate has no analgesic effects. Distribution is ERIC ROYD FINAL EXAM -
intravenous anesthetics? rapid from the brain to highly perfused tissue hence TALAVERA, MD FEB 2015
A. Has greater analgesic effects a relatively short duration of its anesthetic effect. It (TOP 1 - AUG
B. Causes minimal cardiovascular and respiratory has a high incidence of postoperative nausea and 2014 MED
depression vomiting, pain and myoclonic activity. Initial BOARDS;
C. Relatively longer duration of effect recovery is less rapid compared with propofol TOPNOTCH MD)
D. Has a lower incidence of postoperative nausea
and vomiting
E. Has a rapid onset of recovery when compared
with Propofol
293 A 24 year old female G1P0, 12 weeks AOG, sought Cephalosporins should be avoided in patients with ERIC ROYD FINAL EXAM -
consult due to purulent vaginal discharge and history of severe allergic reaction to penicillin. FQ TALAVERA, MD FEB 2015
dyspareunia. Gram stain of the vaginal discharge and doxycycline are avoided in pregnancy since it (TOP 1 - AUG
showed intracellular gram negative diplococci. The can cause cartilage and bone damage. 2014 MED
patient disclosed that she had a severe allergic Azithromycin, a macrolide antibiotic is the safest BOARDS;
reaction to amoxicillin a year ago. Which of the drug for this case. TOPNOTCH MD)
following is the safest antibiotic for this case ?
A. Doxycyline
B. Ciprofloxacin
C. Ceftriaxone
D. Gentamicin
E. Azithromycin
294 Which of the following is a major adverse effect SIMILAR TO PREVIOUS BOARD EXAM ERIC ROYD FINAL EXAM -
associated with use of fenofibrates? CONCEPT/PRINCIPLE. A major toxicity of TALAVERA, MD FEB 2015
A. Constipation fenofibrates is increased risk of gallstone which may (TOP 1 - AUG
B. Gallstones be due to enhanced biliary excretion 2014 MED
C. Hyperuricemia BOARDS;
D. Cardiac arrythmia TOPNOTCH MD)
E. Liver damage

295 A 45 year old male presented to the ER due to sudden Atropine is a mydriatic and cyloplegic and can ERIC ROYD FINAL EXAM -
onset of right eye pain accompanied by blurring of further increase IOP hence worsen angle closure TALAVERA, MD FEB 2015
vision. On examination there was note of ciliary glaucoma. (TOP 1 - AUG
injection over the right eye, IOP pressure was noted 2014 MED
to be elevated. A diagnosis of acute angle closure BOARDS;
glaucoma was made. Which of the following TOPNOTCH MD)
ophthalmic medications should be avoided ?
A. Brinzolamide
B. Timolol
C. Apraclonidine
D. Atropine
E. All of the above
296 Which of the following beta blockers possess intrinsic Acebutolol and Pindolol have partial agonist activity ERIC ROYD FINAL EXAM -
sympathomimetic activity? (ISA) TALAVERA, MD FEB 2015
A. Metoprolol (TOP 1 - AUG
B. Esmolol 2014 MED
C. Pindolol BOARDS;
D. Acebutolol TOPNOTCH MD)
E. Both C and D

297 Which of the following drugs is used to induce SIMILAR TO PREVIOUS BOARD EXAM ERIC ROYD FINAL EXAM -
ovulation in anovulatory women by selectively CONCEPT/PRINCIPLE TALAVERA, MD FEB 2015
blocking estrogen receptors in the pituitary, thus (TOP 1 - AUG
increasing FSH and LH output? 2014 MED
A. Raloxifene BOARDS;
B. Clomiphene TOPNOTCH MD)
C. Fulvestrant
D. Anastrozole
E. Progestins
298 A 39 year old female, known case of Graves' disease beta blockers are the most effective agents in acute ERIC ROYD FINAL EXAM -
but poorly compliant with medications, presented to thyrotixc arrythmias . Esmolo is a rapid parenteral TALAVERA, MD FEB 2015
the ER due to palpitations and shortness of breath. beta blocker (TOP 1 - AUG
She was hooked to a cardiac monitor which showed 2014 MED
supraventricular tachycardia. Which of the following BOARDS;
drugs would be most suitable for this case ? TOPNOTCH MD)
A. Esmolol
B. Quinidine
C. Flecainide
D. Disopyramide
E. Lidocaine

TOPNOTCH MEDICAL BOARD PREP PHARMACOLOGY SUPEREXAM Page 35 of 83


For inquiries visit www.topnotchboardprep.com.ph or email us at topnotchmedicalboardprep@gmail.com
TOPNOTCH MEDICAL BOARD PREP PHARMACOLOGY SUPEREXAM
For inquiries visit www.topnotchboardprep.com.ph or email us at topnotchmedicalboardprep@gmail.com
Item QUESTION EXPLANATION AUTHOR TOPNOTCH
# EXAM
299 A 67 year old woman, known diabetic for 20 years Metoclopramide is DOC for diabetic gastropathy ERIC ROYD FINAL EXAM -
and maintained on Gliclazide, complained of severe TALAVERA, MD FEB 2015
bloating and post prandial fullness. Evaluation was (TOP 1 - AUG
done and she was assessed to have diabetic 2014 MED
gastropathy. Which of the following medications BOARDS;
would be most appropriate for this case? TOPNOTCH MD)
A. Cimetidine
B. Metoclopramide
C. Alosetron
D. Loperamide
E. Bismuth subsalicylate
300 Which drug when taken together with Azathioprine Azathioprine is converted to mercaptopurine which ERIC ROYD FINAL EXAM -
has resulted in severe myelosupression by inhibiting is responsible for its immunosuppressant and TALAVERA, MD FEB 2015
the enzyme responsible for its metabolism ? hematotoxicity. Allopurinol inhibits xanthine (TOP 1 - AUG
A. Theophylline oxidase, the enzyme that metabolizes 2014 MED
B. Celecoxib mercaptopurine. BOARDS;
C. Allopurinol TOPNOTCH MD)
D. Digoxin
E. Rifampicin

301 During anaphylaxis, administration of IM epinephrine physiologic antagonist binds to a different receptor LEAN ANGELO BACK-UP
is essential because of its bronchodilatory effect in molecules producing an effect opposite to that SILVERIO, MD MIDTERM
the lungs. How does epinephrine acts to produce such produced by the drug it antagoniszes. It differs from (TOP 4 - AUG EXAM - FEB
an effect in this particular condition? a pharmacologic antagonist, which interacts with the 2014 MED 2015
A. Chemical antagonist same receptors as the drug it is inhibiting. the BOARDS;
B. physiologic antagonist antagonism of bronchoconstrictor effect of TOPNOTCH MD),
C. partial agonist histamine ( through histamine receptors) by MD
D. noncompetitive antagonist epinephrine bronchodilator action ( beta receptors).
E. none of the above chemical antagonist - reacts directly with the drug
being antagonized to remove it or to prevent it from
reaching its target. Katzung review 8th ed p 13.
302 which of the following drugs exhibit zero order zero order kinetics refers to a contant rate of LEAN ANGELO BACK-UP
kinetics ? elimination of a drug regardless of its concentration. SILVERIO, MD MIDTERM
A. Ethanol The following drugs exhibit zero order kinetics: (TOP 4 - AUG EXAM - FEB
B. tolbutamide aspirin, ethanol, warfarin,heparin, penytoin, 2014 MED 2015
C. warfarin tolbutamide, theophyline phenylbutazone, and BOARDS;
D. heparin salicylates. TOPNOTCH MD),
E. All of the above MD

303 The following drugs promote induction of liver CYP CYP inducers enhances drug metabolism. Ex. LEAN ANGELO BACK-UP
enzymes except ? Benzopyrene, carbamazepine, chlorcyclizine, SILVERIO, MD MIDTERM
A. Rifampin glutethimide, griseofulvin, phenobarbital, (TOP 4 - AUG EXAM - FEB
B. Carbamazepine phenylbutazone, phenytoim, rifampin, ritonavir ( 2014 MED 2015
C. Phenylbutazone chronic administration, acute -acts as inhibitor), st BOARDS;
D. Disulfiram johns wort. Katzung 11th ed p 64-65. TOPNOTCH MD),
E. Glutethimide MD

304 which of the following effects in parathion poisoning pralidoxime is a chemical antagonist acting as LEAN ANGELO BACK-UP
will not be antagonized by timely administration of cholinesterase regenerator. It cannot enter the BBB SILVERIO, MD MIDTERM
pralidoxime? because of its positively charged group and poor (TOP 4 - AUG EXAM - FEB
A. cyclospasm lipid solubility. Katzung review pp 69. 2014 MED 2015
B. Urinary incontinence BOARDS;
C. convulsion TOPNOTCH MD),
D. Negative dromotrophy MD
E. None of the above

305 Among the beta adrenergic blockers, which of the pindolol, acebutolol, carteolol, bopindolol, LEAN ANGELO BACK-UP
following will less likely to cause bradycardia and penbutolol,celiprolol are the beta blockers having SILVERIO, MD MIDTERM
increased VLDL concentration? also partial b agonist activity ( intrinsic (TOP 4 - AUG EXAM - FEB
A. metoprolol sympathomimetic activity). The advantage of this 2014 MED 2015
B. nebivolol property is that it is less likely to cause elevation of BOARDS;
C. carteolol plasma lipids without affecting its antihypertensive TOPNOTCH MD),
D. labetalol effect. Katzung 11th ed p 159. MD
E. None of the above

306 which of the following changes will promote digitalis any condition that can predisposed to hyperkalemia, LEAN ANGELO BACK-UP
induced cardiototoxicity? hypermagnesemia and hypocalcemia will inhibit the SILVERIO, MD MIDTERM
A. Acute tubular necrosis binding of digitalis to Na K ATPase resulting to (TOP 4 - AUG EXAM - FEB
B. hypermagnesemia decrease toxicity. Triamterene and ATN can induce 2014 MED 2015
C. hyperparathyroidism hyperkalemia. Katzung 11th ed p 216. BOARDS;
D. Triamterene administration TOPNOTCH MD),
E. All of the above MD

307 which of the following drugs is highly selective to phenytoin, lidocaine and mexiletine are classified as LEAN ANGELO BACK-UP
ischemic or prolonged depolarized purkinje fibers? class IB antiarrhythmics. This group affects SILVERIO, MD MIDTERM
A. Mexiletine primarily ischemic or depolarized ventricular tissue (TOP 4 - AUG EXAM - FEB
B. Procainamide and does not have any use in atrial arrhytmia. B- 2014 MED 2015
C. Propafenone procainamide is a class IA drug, C- propafenone is a BOARDS;
D. ibutilide Class IB agent, D- ibutilide is class III drugs. TOPNOTCH MD),
E. None of the above MD

TOPNOTCH MEDICAL BOARD PREP PHARMACOLOGY SUPEREXAM Page 36 of 83


For inquiries visit www.topnotchboardprep.com.ph or email us at topnotchmedicalboardprep@gmail.com
TOPNOTCH MEDICAL BOARD PREP PHARMACOLOGY SUPEREXAM
For inquiries visit www.topnotchboardprep.com.ph or email us at topnotchmedicalboardprep@gmail.com
Item QUESTION EXPLANATION AUTHOR TOPNOTCH
# EXAM
308 which of the following effects of acetazolamide is self the major effect of acetazolamide is bicarbonate LEAN ANGELO BACK-UP
limiting? diuresis resulting to metabolic acidosis. As increased SILVERIO, MD MIDTERM
A. Sodium reabsorption sodium is presented to the cortical collecting tubule, (TOP 4 - AUG EXAM - FEB
B. Tubular bicarbonate excretion some of the excess sodium is reabsorbed and 2014 MED 2015
C. CSF acidosis potassium is secreted resulting to significant BOARDS;
D. Aqueous humor production potassium wasting. as a result of bicarbonate TOPNOTCH MD),
E. None of the above depletion, sodium bicarbonate excretion slows even MD
with continued administration. Katzung review 8th
ed p 135
309 A 39 y/o male went for follow up consult secondary dermanent discoloration of the iris and eyelashes is LEAN ANGELO BACK-UP
to open angle glaucoma. He was compliant with all a well known side effect of latanoprost. Its SILVERIO, MD MIDTERM
the topical medications given to him. Indirect mechanism of action is to increase aqueous outflow. (TOP 4 - AUG EXAM - FEB
ophthalmoscopy revealed brownish discoloration in A- brinzolamide, dorsolamide, B- epinephrine. D- 2014 MED 2015
his iris. What is the mechanism of action of the drug timolol. Katzung review p 87. BOARDS;
that causes such change?. TOPNOTCH MD),
A. decrease aqueous secretion due to lack of HCO3 MD
B. increase outflow via uveoscleral veins
C. increase aqueous outflow
D. decrease aqueous secretion from the ciliary
epithelium
E. none of the above
310 which in the following antipsychotic medications has Olanzapine -weight gain, clozapine - agranulocytosis, LEAN ANGELO BACK-UP
the greatest risk of inducing Torsades de pointes haloperidol- extrapyramidal symptoms. SILVERIO, MD MIDTERM
arryhthmia? Thioridazine - retinal deposits. Among the atypical (TOP 4 - AUG EXAM - FEB
A. Fluphenazine antipsychotics, ziprasidone has the greatest risk of 2014 MED 2015
B. Clozapine inducing QT prolongation leading to ventricular BOARDS;
C. Thioridazine arrhythmias. Kazung review p 242. TOPNOTCH MD),
D. Ziprasidone MD
E. Haloperidol

311 Which of the following is true about Buspirone? Buspirone is a selective anxiolytic medication that LEAN ANGELO BACK-UP
A. Anxiolytic effect is secondary to direct does not cause, sedative, hypnotic, euphoric, and SILVERIO, MD MIDTERM
stimulation of GABAergic neurons acute withdrawal or rebound effect. It has no effect (TOP 4 - AUG EXAM - FEB
B. It is highly effect in acute panic attacks in psychomotor skills (driving skills). The 2014 MED 2015
C. Less psychomotor retardation mechanism of anxiolysis is secondary to partial BOARDS;
D. Abrupt discontinuation can lead acute withdrawal antagonism to 5HT1A receptors in the brain. it has TOPNOTCH MD),
syndrome no direct GABAergic activity. weeks are needed MD
E. all of the above before buspirone takes effect, therefore, it is not
effect in treating acute panic attacks. Katzung 11ed
p374.
312 what is the recommended treatment for a 29 y/o single dose administration of spectinomycin is LEAN ANGELO BACK-UP
G1PO female diagnosed with Gonnorrhea infection effective in treating Gonnorhea in a patient who has SILVERIO, MD MIDTERM
who has severe hypersensitivity to pennicilin? severe beta lactam hypersensitivity. It has lesser risk (TOP 4 - AUG EXAM - FEB
A. erythromycin of teratogenicity compared to conventional 2014 MED 2015
B. Spectinomycin aminglycosides. Clindamycin is active only on gram BOARDS;
C. Cetriaxone positive and anaerobic organisms. ceftriaxone is a TOPNOTCH MD),
D. Doxycycline 3rd generation cephalosporin effective against MD
E. Clindamycin Gonorrhea infection however it should be avoided
since the patient has beta lactam hypersensitivity.
Doxycycline should never be given for pregnant
patients. erythromycin is not effective treatment for
gonorrhea infection. Katzung 11th ed p 813.
313 the pungency of this inhaled anesthetics leading (SIMILAR TO PREVIOUS BOARD EXAM LEAN ANGELO BACK-UP
breath-holding limits it use in anesthesia induction? CONCEPT/PRINCIPLE) Katzung review 8th ed p 211 SILVERIO, MD MIDTERM
A. desflurane (TOP 4 - AUG EXAM - FEB
B. Isoflurane 2014 MED 2015
C. Sevoflurane BOARDS;
D. enflurane TOPNOTCH MD),
E. Halothane MD

314 what is the drug of choice for restless leg syndrome? the pathologic basis for restless leg syndrome is LEAN ANGELO BACK-UP
A. haloperidol dopamine blockade. Agonist to dopamine receptors SILVERIO, MD MIDTERM
B. aripriprazole such as pramipexole or ropinirole is very effective in (TOP 4 - AUG EXAM - FEB
C. Citalopram treating this condition. Katzung review 8th ed p 235 2014 MED 2015
D. Ropinirole BOARDS;
E. Propanolol TOPNOTCH MD),
MD

315 Prolonged treatment with a PPAR alpha ligand in the PPAR alpha ligands is the MOA of fibric acid LEAN ANGELO BACK-UP
treatment of hyperlipidemia will increase the risk for derivatives ( Gemfibrozil and fenofibrate). It SILVERIO, MD MIDTERM
the following condition? increases the supersaturation of bile leading (TOP 4 - AUG EXAM - FEB
A. Cutaneous flushing cholelithiasis. Risk for hepatotoxicity only 2014 MED 2015
B. Hyperuricemia leading to Gout significantly increase if gemfibrozil is combined with BOARDS;
C. Intestinal malabsorption HMG CoA inhibitor. Cutaneous flushing and TOPNOTCH MD),
D. Gallstones hyperuricemia are toxicity profile of Niacin. MD
E. hepatotoxicity Intestinal malabsorption is a primary side effect of
resins ( cholestyramine, colestipol, colsevelam)

TOPNOTCH MEDICAL BOARD PREP PHARMACOLOGY SUPEREXAM Page 37 of 83


For inquiries visit www.topnotchboardprep.com.ph or email us at topnotchmedicalboardprep@gmail.com
TOPNOTCH MEDICAL BOARD PREP PHARMACOLOGY SUPEREXAM
For inquiries visit www.topnotchboardprep.com.ph or email us at topnotchmedicalboardprep@gmail.com
Item QUESTION EXPLANATION AUTHOR TOPNOTCH
# EXAM
316 oral administration of this antibiotic will yield greater the pharmacokinetics of chlorampenicol is unusual LEAN ANGELO BACK-UP
blood levels compared when it is administered since blood levels are higher when the drug is given SILVERIO, MD MIDTERM
intravenously? at oral form rather than intravenous form. Katzung (TOP 4 - AUG EXAM - FEB
A. chlorampenicol 11th ed page 802 2014 MED 2015
B. Quinipristin -dalfopristin BOARDS;
C. Azithromycin TOPNOTCH MD),
D. Telithromycin MD
E. None of the above

317 what is the mechanism of action of nystatin? nystatin and ampothericin B are both polyene LEAN ANGELO BACK-UP
A. Formation of artificial pore in the fungal antifungal antibiotics. It binds to ergosterol and SILVERIO, MD MIDTERM
membrane creates an artificial pores in the fungal membrane (TOP 4 - AUG EXAM - FEB
B. Inhibit ergosterol synthesis leading to increase membrane permeability. Katzung 2014 MED 2015
C. Inhibit microtubular formation review 8th ed p 401 BOARDS;
D. Inhibits squalene epoxidase TOPNOTCH MD),
E. Inhibit the synthesis of B(1-2) glycan MD

318 the drug of choice for Chagas disease acts through the drug of choice for Chagas disease or american LEAN ANGELO BACK-UP
what mechanism of action? trypanosomiasis is nifurtimox. Its anitprotozoan SILVERIO, MD MIDTERM
A. Inhibit mitochondrial electron transport action is due to inhibition of trypanothiane (TOP 4 - AUG EXAM - FEB
B. Trypanothione reductase inhibition reductase. A- atovaquone MOA ( use for PCP and 2014 MED 2015
C. Suicide substrate of ornithine decarboxylase chloroquine resistant malaria) C- eflornithine ( use BOARDS;
D. Inhibits enzyyme sulfhydryl groups in cerebral stage of african trypanosomiasis). D- TOPNOTCH MD),
E. none of the above melarsoprol ( african sleeping sickness). SIMILAR MD
TO PREVIOUS BOARD EXAM CONCEPT/PRINCIPLE
Katzung review p 438-439.
319 what is the primary drug of treatment for ascariasis, B,C,D has no clinical effectivity in hydatid disease. LEAN ANGELO BACK-UP
pinworm, whipworm and hydatid disease? SILVERIO, MD MIDTERM
A. albendazole (TOP 4 - AUG EXAM - FEB
B. Mebendazole 2014 MED 2015
C. Pyrantel pamoate BOARDS;
D. Diethylcarbamazine TOPNOTCH MD),
E. All of the above MD

320 what is the dose limiting toxicity of vincristine ? vincristine does not cause serious myelosuppresive LEAN ANGELO BACK-UP
A. areflexia effect however it induces neurotoxicity such as SILVERIO, MD MIDTERM
B. Bone marrow suppression areflexia, ileus and peripheral neuritis. Katzung (TOP 4 - AUG EXAM - FEB
C. Acute tubular necrosis review 8th ed p 455. 2014 MED 2015
D. Hepatocyte degeneration BOARDS;
E. None of the above TOPNOTCH MD),
MD

321 Among insulin secretagogues, the risk of Biguanides are not insulin secretagogues. DEBBIE ROSE BACK-UP
hypoglycemia is least in? TANENGSY, MD MIDTERM
A. Meglitinides (TOP 5 - AUG EXAM - FEB
B. 1st generation sulfonylureas 2014 MED 2015
C. 2nd generation sulfonylureas BOARDS;
D. Biguanides TOPNOTCH MD)
322 What is the most important drug in pulmonary DEBBIE ROSE BACK-UP
tuberculosis? TANENGSY, MD MIDTERM
A. INH (TOP 5 - AUG EXAM - FEB
B. Pyrazinamide 2014 MED 2015
C. Rifampin BOARDS;
D. Ethambutol TOPNOTCH MD)
323 Pupillary construction is a characteristic effet of all DEBBIE ROSE BACK-UP
opioids, except: TANENGSY, MD MIDTERM
A. Oxycodone (TOP 5 - AUG EXAM - FEB
B. Meperidine 2014 MED 2015
C. Morphine BOARDS;
D. Fentanyl TOPNOTCH MD)
324 This antiplatelet inhibits aggregation by interfering Aspirin is a nonselective irreversible COX inhibitor. DEBBIE ROSE BACK-UP
with GpIIb/IIIa binding to fibrinogen and other Clopidogrel irreversibly inhibits platelet ADP TANENGSY, MD MIDTERM
ligands: receptor. Dipyridamole inhibits adenosine uptake (TOP 5 - AUG EXAM - FEB
A. Aspirin and inhibits phosphodiesterase enzymes that 2014 MED 2015
B. Clopidogrel degrade cyclic nucleotides. BOARDS;
C. Dipyridamole TOPNOTCH MD)
D. Tirofiban

325 Which tetracycline has the broadest spectrum of DEBBIE ROSE BACK-UP
microbial coverage? TANENGSY, MD MIDTERM
A. Tetracyline (TOP 5 - AUG EXAM - FEB
B. Doxycyline 2014 MED 2015
C. Minocycline BOARDS;
D. Tigecycline TOPNOTCH MD)
326 The following drugs' efficacy is directly related to What is described is the time-dependent killing DEBBIE ROSE BACK-UP
time above the minimal inhibitory concentration and property for which penicillins and cephalosporins TANENGSY, MD MIDTERM
becomes independent of concentration once the MIC are known. Aminoglycosides are known for their (TOP 5 - AUG EXAM - FEB
has been reached, except? concentration-dependent killing. 2014 MED 2015
A. Penicillins BOARDS;
B. Cephalosporins TOPNOTCH MD)
C. A & B
D. none of the above

TOPNOTCH MEDICAL BOARD PREP PHARMACOLOGY SUPEREXAM Page 38 of 83


For inquiries visit www.topnotchboardprep.com.ph or email us at topnotchmedicalboardprep@gmail.com
TOPNOTCH MEDICAL BOARD PREP PHARMACOLOGY SUPEREXAM
For inquiries visit www.topnotchboardprep.com.ph or email us at topnotchmedicalboardprep@gmail.com
Item QUESTION EXPLANATION AUTHOR TOPNOTCH
# EXAM
327 The following are bactericidal, except: All drugs stated are bactericidal. TMP and SXZ alone DEBBIE ROSE BACK-UP
A. TMP-SXZ are bacteriostatic but are bactericidal when in TANENGSY, MD MIDTERM
B. Vancomycin combination. (TOP 5 - AUG EXAM - FEB
C. Metronidazole 2014 MED 2015
D. Cefalexin BOARDS;
E. none of the above TOPNOTCH MD)

328 The following anti-tuberculosis drugs are correctly Rifampicin inhibits DNA dependent RNA polymerase DEBBIE ROSE BACK-UP
paired with their mechanisms of action, except: TANENGSY, MD MIDTERM
A. INH - inhibits mycolic acid synthesis (TOP 5 - AUG EXAM - FEB
B. Rifampin - inhibits RNA dependent DNA 2014 MED 2015
polymerase BOARDS;
C. Ethambutol - inhibits arabinosyl transferase TOPNOTCH MD)
D. none of the above
329 This drug requires phosphorylation by host cell Efavirenz, delaverdine, and nevirapine are non DEBBIE ROSE BACK-UP
kinases to be active: nucleoside reverse transcriptase inhibitors which do TANENGSY, MD MIDTERM
A. Efavirenz not require host cell kinase for activation, unlike (TOP 5 - AUG EXAM - FEB
B. Delavirdine tenofovir, a nucleoside reverse transcriptase 2014 MED 2015
C. Nevirapine inhibitor which does. BOARDS;
D. Tenofovir TOPNOTCH MD)
330 This antihistamine causes depolarization-induced Mebendazole selectively inhibits microtubule DEBBIE ROSE BACK-UP
paralysis in nematodes: synthesis and glucose uptake. Ivermectin interferes TANENGSY, MD MIDTERM
A. Mebendazole with GABA mediated neurotransmission. Piperazine (TOP 5 - AUG EXAM - FEB
B. Ivermectin is an agonist at GABA receptors. 2014 MED 2015
C. Pyrantel pamoate BOARDS;
D. Piperazine TOPNOTCH MD)
331 Which of the following is uricosuric? Allopurinol is a xanthine oxidase inhibitor. DEBBIE ROSE BACK-UP
A. Allopurinol Colchicine inhibits microtubule assembly. Celecoxib TANENGSY, MD MIDTERM
B. Colchicine is a COX-2 inhibitor. (TOP 5 - AUG EXAM - FEB
C. Sulfinpyrazone 2014 MED 2015
D. Celecoxib BOARDS;
TOPNOTCH MD)
332 Which drug is most effective in raising HDL? DEBBIE ROSE BACK-UP
A. Niacin TANENGSY, MD MIDTERM
B. Fibrates (TOP 5 - AUG EXAM - FEB
C. Statins 2014 MED 2015
D. Bile acid sequestrants BOARDS;
TOPNOTCH MD)
333 The metabolism of which drug is inhibited by Allopurinol inhibits the enzymatic inactivation of 6- DEBBIE ROSE BACK-UP
allopurinol and febuxostat? MP and its derivative, Azathioprine, by xanthine TANENGSY, MD MIDTERM
A. Cytarabine oxidase. Thus, when allopurinol is used concurrently (TOP 5 - AUG EXAM - FEB
B. Methotrexate with oral 6-MP/azathioprine, dosage of the 2014 MED 2015
C. Fluorouracil antineoplastic agent must be reduced to 1/2 to 1/3 BOARDS;
D. Mercaptopurine of usual dose. TOPNOTCH MD)
334 This regimen is utilized in the treatment of Hodgkin ABVD - adriamycin, bleomycin, vincristine, DEBBIE ROSE BACK-UP
lyphoma: dacarbazine, prednisone. PEB (cisplatin, etoposide, TANENGSY, MD MIDTERM
A. PEB regimen bleomycin) regimen is used in testicular CA. CHOP (TOP 5 - AUG EXAM - FEB
B. ABVD regimen (cyclophosphamide, doxorubicin, vincristine, 2014 MED 2015
C. CHOP regimen prednisone) regimen in non-Hodgkin lymphoma, BOARDS;
D. GnRH agonist and androgen receptor antagonist and GnRH agonist and androgen receptor antagonist TOPNOTCH MD)
in prostate CA treatment.
335 This drug is a monoclonal antibody that binds to Rituximab is a monoclonal antibody that binds to DEBBIE ROSE BACK-UP
VEGF and prevents it from interacting with VEGF surface protein in NHL inducing lysis, cytotoxicity TANENGSY, MD MIDTERM
receptors: and apoptosis. Trastuzumab is a monoclonal (TOP 5 - AUG EXAM - FEB
A. Bevacuzimab antibody that recognizes protein in breast CA cells 2014 MED 2015
B. Rituximab overexpressing HER-2neu receptor for EGF. Imatinib BOARDS;
C. Trastuzumab inhibits tyrosine kinase activity of the product of TOPNOTCH MD)
D. Imatinib bcr-abl oncogene expressed in CML.

336 This chemotherapeutic agent is known for its DEBBIE ROSE BACK-UP
association with heart toxicity in cumulative doses: TANENGSY, MD MIDTERM
A. 5-FU (TOP 5 - AUG EXAM - FEB
B. 6-MP 2014 MED 2015
C. Methotrexate BOARDS;
D. Doxorubicin TOPNOTCH MD)
337 The following laxative mechanisms are correctly Polyethylene glycol is an osmotic laxative. Examples DEBBIE ROSE BACK-UP
paired with its representative laxative drug, except: of stimulant laxatives include senna, bisacodyl, TANENGSY, MD MIDTERM
A. bulk forming - psyllium castor oil. (TOP 5 - AUG EXAM - FEB
B. stool softening - docusate 2014 MED 2015
C. osmotic - lactulose BOARDS;
D. stimulant - polyethylene glycol TOPNOTCH MD)
338 Ondansetron, highly effective in preventing DEBBIE ROSE BACK-UP
chemotherapy-induced nausea and vomiting, is an TANENGSY, MD MIDTERM
antagonist at which receptor? (TOP 5 - AUG EXAM - FEB
A. muscarinic 2014 MED 2015
B. B2 BOARDS;
C. 5-HT3 TOPNOTCH MD)
D. D2

TOPNOTCH MEDICAL BOARD PREP PHARMACOLOGY SUPEREXAM Page 39 of 83


For inquiries visit www.topnotchboardprep.com.ph or email us at topnotchmedicalboardprep@gmail.com
TOPNOTCH MEDICAL BOARD PREP PHARMACOLOGY SUPEREXAM
For inquiries visit www.topnotchboardprep.com.ph or email us at topnotchmedicalboardprep@gmail.com
Item QUESTION EXPLANATION AUTHOR TOPNOTCH
# EXAM
339 This drug stimulates platelet production and Filgrastim stimulates the function and production of DEBBIE ROSE BACK-UP
decreases the number of platelet transfusions neutrophils. TANENGSY, MD MIDTERM
required by patients undergoing bone marrow (TOP 5 - AUG EXAM - FEB
suppression in treatment for CA: 2014 MED 2015
A. Oprevelkin BOARDS;
B. Filgrastim TOPNOTCH MD)
C. Erythropoietin
D. Folic acid
340 Serum concentration of lithium in mEq/L favored for The serum concentration favored for long-term use DEBBIE ROSE BACK-UP
acutely manic patients is: in the prevention of reccurent manic-depressive TANENGSY, MD MIDTERM
A. 0.9-1.1 illness, on the other hand, is 0.6-0.75 mEq/L. (TOP 5 - AUG EXAM - FEB
B. 0.6-0.75 2014 MED 2015
C. 0.5-0.85 BOARDS;
D. 0.1-0.5 TOPNOTCH MD)
341 This direct acting cholinomimetic is used in the JESSICA MAE BACK-UP
treatment of postoperative ileum and neurogenic SANCHEZ, MD MIDTERM
bladder: (TOP 4 - AUG EXAM - FEB
A. Neostigmine 2014 MED 2015
B. Bethanechol BOARDS;
C. Pilocarpine TOPNOTCH MD)
D. Physostigmine
342 Use of aspirin as an anti-platelet inhibits primarily JESSICA MAE BACK-UP
which prostanoid? SANCHEZ, MD MIDTERM
A. Prostacyclin (TOP 4 - AUG EXAM - FEB
B. Thromboxane 2014 MED 2015
C. Prostaglandin BOARDS;
D. Epoprosterenol TOPNOTCH MD)
343 This is not a characteristic of phase I clinical trials in Determination of drug efficacy is done during phase JESSICA MAE BACK-UP
drug development: II of clinical trials. SANCHEZ, MD MIDTERM
A. Pharmacokinetic measurements are done (TOP 4 - AUG EXAM - FEB
B. Determination of drug’s efficacy 2014 MED 2015
C. Determination of maximum tolerated dose BOARDS;
D. Subjects are healthy volunteers TOPNOTCH MD)
344 The prime target of addictive drugs in the brain: Reference: Katzung. Basic and Clinical JESSICA MAE BACK-UP
A. Thalamocortical system Pharmacology, 11th ed. p. 554 SANCHEZ, MD MIDTERM
B. Nigrostriatal dopamine system (TOP 4 - AUG EXAM - FEB
C. Mesolimbic dopamine system 2014 MED 2015
D. Mesocortical dopamine system BOARDS;
TOPNOTCH MD)
345 A 30 year old female with bipolar disorder delivered JESSICA MAE BACK-UP
a baby with Ebstein anomaly. The drug that the SANCHEZ, MD MIDTERM
mother was taking during her pregnancy that caused (TOP 4 - AUG EXAM - FEB
this congenital anomaly is: 2014 MED 2015
A. Olanzapine BOARDS;
B. Lithium TOPNOTCH MD)
C. Carbamazepine
D. Risperidone
346 Ketamine exerts its anaesthetic effect by its inhibitory JESSICA MAE BACK-UP
action on these receptors: SANCHEZ, MD MIDTERM
A. NMDA (TOP 4 - AUG EXAM - FEB
B. GABA-A 2014 MED 2015
C. Glycine BOARDS;
D. Neuronal nictonic acetylcholine receptors TOPNOTCH MD)
347 A 20 year old male develops tachycardia, This is a case of malignant hyperthermia, which is JESSICA MAE BACK-UP
hypotension, muscle rigidity, and spiking fever associated with the combination of inhaled SANCHEZ, MD MIDTERM
immediately after start of surgery. Which of the anaesthetics and succinylcholine. (TOP 4 - AUG EXAM - FEB
following are commonly associated with this 2014 MED 2015
anaesthetic complication? BOARDS;
A. IV anesthetic and succinylcholine TOPNOTCH MD)
B. IV anesthetic and midazolam
C. Inhalational anesthetic and succinylcholine
D. Inhalational anesthetic and midazolam
348 Nitrates relieve angina by: JESSICA MAE BACK-UP
A. Decreasing venous return to the heart SANCHEZ, MD MIDTERM
B. Increasing intracardiac volume (TOP 4 - AUG EXAM - FEB
C. Increasing end diastolic left ventricular volume 2014 MED 2015
D. All of the above BOARDS;
TOPNOTCH MD)
349 The main indication of this agent is hypertension in JESSICA MAE BACK-UP
pregnancy: SANCHEZ, MD MIDTERM
A. Reserpine (TOP 4 - AUG EXAM - FEB
B. Prazosin 2014 MED 2015
C. Methyldopa BOARDS;
D. Clonidine TOPNOTCH MD)
350 This antihypertensive drug is absolutely JESSICA MAE BACK-UP
contraindicated in pregnancy because it can cause SANCHEZ, MD MIDTERM
renal damage in the fetus: (TOP 4 - AUG EXAM - FEB
A. Reserpine 2014 MED 2015
B. Hydralazine BOARDS;
C. Propranolol TOPNOTCH MD)
D. Captopril

TOPNOTCH MEDICAL BOARD PREP PHARMACOLOGY SUPEREXAM Page 40 of 83


For inquiries visit www.topnotchboardprep.com.ph or email us at topnotchmedicalboardprep@gmail.com
TOPNOTCH MEDICAL BOARD PREP PHARMACOLOGY SUPEREXAM
For inquiries visit www.topnotchboardprep.com.ph or email us at topnotchmedicalboardprep@gmail.com
Item QUESTION EXPLANATION AUTHOR TOPNOTCH
# EXAM
351 Which class of antibiotics can be safely given during JESSICA MAE BACK-UP
pregnancy? SANCHEZ, MD MIDTERM
A. Tetracycline (TOP 4 - AUG EXAM - FEB
B. Aminoglycoside 2014 MED 2015
C. Fluoroquinolones BOARDS;
D. Beta-lactams TOPNOTCH MD)
352 Vancomycin is administered orally for this condition: JESSICA MAE BACK-UP
A. Sepsis SANCHEZ, MD MIDTERM
B. Endocarditis (TOP 4 - AUG EXAM - FEB
C. Antibiotic-associated enterocolitis 2014 MED 2015
D. Meningitis BOARDS;
TOPNOTCH MD)
353 These anti-neoplastic agents are cell-cycle specific, JESSICA MAE BACK-UP
except: SANCHEZ, MD MIDTERM
A. 5-fluorouracil (TOP 4 - AUG EXAM - FEB
B. Cyclophosphamide 2014 MED 2015
C. Bleomycin BOARDS;
D. Vincristine TOPNOTCH MD)
354 The preferred anti-thyroid drug for thyroid storm: JESSICA MAE BACK-UP
A. Methimazole SANCHEZ, MD MIDTERM
B. Carbimazole (TOP 4 - AUG EXAM - FEB
C. Propylthiouracil 2014 MED 2015
D. Propranolol BOARDS;
TOPNOTCH MD)
355 Which of the following is a contraindication to the use Reference: Katzung. Basic and Clinical JESSICA MAE BACK-UP
of thiazolidinediones? Pharmacology, 11th ed. p. 743 SANCHEZ, MD MIDTERM
A. Alcohol ingestion (TOP 4 - AUG EXAM - FEB
B. Heart failure 2014 MED 2015
C. Cholelithiasis BOARDS;
D. Renal insufficiency TOPNOTCH MD)
356 Clomiphene acts to induce ovulation by: JESSICA MAE BACK-UP
A. Diminishing estrogen mediated negative feedback SANCHEZ, MD MIDTERM
at the pituitary (TOP 4 - AUG EXAM - FEB
B. Increasing the action of estrogen in the 2014 MED 2015
hypothalamus BOARDS;
C. Increasing the action of estrogen in the ovary TOPNOTCH MD)
D. Increasing the amount of estrogen receptors
357 Organophosphate toxicity is based on its ability to: JESSICA MAE BACK-UP
A. Compete with brain pyridoxal phosphate SANCHEZ, MD MIDTERM
B. Inhibit acetylcholinesterase (TOP 4 - AUG EXAM - FEB
C. Interrupt heme synthesis 2014 MED 2015
D. React with sulfhydryl groups binding to protein BOARDS;
and other enzymes TOPNOTCH MD)
358 A drug used for treatment of advanced prostatic Leuprolide combined with an androgen receptor JESSICA MAE BACK-UP
carcinoma by producing continuous gonadal antagonist such as flutamide is the primary medical SANCHEZ, MD MIDTERM
suppression: therapy for advanced prostate cancer and is as (TOP 4 - AUG EXAM - FEB
A. Ketoconazole effective as surgical castration. 2014 MED 2015
B. Testosterone undecanoate BOARDS;
C. Leuprolide Reference: Katzung. Basic and Clinical TOPNOTCH MD)
D. Cyproterone acetate Pharmacology, 11th ed. p. 654
359 The following drug/s is/are used to augment labor: JESSICA MAE BACK-UP
A. Oxytocin SANCHEZ, MD MIDTERM
B. Misoprostol (TOP 4 - AUG EXAM - FEB
C. Methyl ergonovine 2014 MED 2015
D. All of the above BOARDS;
TOPNOTCH MD)
360 Type II DM with insulin resistance will be best treated JESSICA MAE BACK-UP
with which of the following: SANCHEZ, MD MIDTERM
A. Sulfonylureas (TOP 4 - AUG EXAM - FEB
B. Gliptins 2014 MED 2015
C. Insulin glargine BOARDS;
D. TZDs TOPNOTCH MD)
361 A 26/M presented with RUQ pain and fever. Imaging For cases of amebic hepatic abscess you can give MAIRRE JAMES BACK-UP
showed an abscess and aspiration biopsy was done. either Metronidazole, 750 mg 3 times daily (or 500 GADDI, MD (TOP MIDTERM
The aspirate resembled anchovy paste in color and in mg IV every 6 hours) for 10 days or Tinidazole, 2 g 4 - AUG 2013 EXAM - FEB
consistency. What is/are the possible drug/s that you daily for 5 days PLUS a luminal agent. Alternatively MED BOARDS; 2015
can give for this patient? you can give Dehydroemetine2 or emetine,2 1 TOPNOTCH MD)
A. Metronidazole mg/kg SC or IM for 8–10 days, followed by (liver
B. Tinidazole abscess only) chloroquine, 500 mg twice daily for 2
C. Chloroquine days, then 500 mg daily for 21 days PLUS a luminal
D. A and B agent
E. All of the above Luminal agent (Diloxanide, iodoquinol,
paramomycin) Katzung 12th ed pg 928
362 Which antifungal drug inhibits β-glucan synthase Terbinafine - inhibits squalene oxidase; MAIRRE JAMES BACK-UP
causing a disruption in cell wall synthesis? Ketoconazole - inhibits synthesis of ergosterol; GADDI, MD (TOP MIDTERM
A. Terbinafine Fluconazole - inhibits thymidylate synthase; Ampho 4 - AUG 2013 EXAM - FEB
B. Ketoconazole B - polyene causes disruption of fungal cell walls MED BOARDS; 2015
C. Amphotericin B Katzung 12th ed pg 857 TOPNOTCH MD)
D. Fluconazole
E. Caspofungin

TOPNOTCH MEDICAL BOARD PREP PHARMACOLOGY SUPEREXAM Page 41 of 83


For inquiries visit www.topnotchboardprep.com.ph or email us at topnotchmedicalboardprep@gmail.com
TOPNOTCH MEDICAL BOARD PREP PHARMACOLOGY SUPEREXAM
For inquiries visit www.topnotchboardprep.com.ph or email us at topnotchmedicalboardprep@gmail.com
Item QUESTION EXPLANATION AUTHOR TOPNOTCH
# EXAM
363 A 65/M hypertensive, diabetic, smoker presents to Vancomycin and meropenem need to be adjusted MAIRRE JAMES BACK-UP
you with fever, productive cough with an infected non Katzung 12th ed pg 798 Metoprolol is metabolized GADDI, MD (TOP MIDTERM
healing wound in the foot. Labs showed anemia and in the liver and requires no adjustment pg 179 4 - AUG 2013 EXAM - FEB
very elevated creatinine.You schedule the patient for MED BOARDS; 2015
dialysis. As the prescribing physician, which of the TOPNOTCH MD)
following drug/s dose/s would you have to modify?
A. Vancomycin
B. Meropenem
C. Metoprolol
D. A and B
E. All of the above
364 A 34/F was diagnosed to have lepromatous leprosy. Clofazimine is a phenazine dye which causes red- MAIRRE JAMES BACK-UP
She was given the appropriate drugs which she took brown to nearly black skin discoloration. SE of GADDI, MD (TOP MIDTERM
religiously. One day she came back to you dapsone: methemoglobinemia; Rifampicin: red 4 - AUG 2013 EXAM - FEB
complaining that her skin changed to a red-brownish orange discoloration of body fluid Katzung 12th ed MED BOARDS; 2015
color. Which of the following drug/s most likely have pg 846 TOPNOTCH MD)
caused her predicament?
A. Dapsone
B. Rifampicin
C. Clofazimine
D. Rifabutin
E. Cycloserine
365 Which of the following drug/s can be used for Minocycline, 200 mg orally daily for 5 days, can MAIRRE JAMES BACK-UP
eliminating meningococcal carriage states? eradicate the meningococcal carrier state, but GADDI, MD (TOP MIDTERM
A. Minocycline because of side effects and resis- tance of many 4 - AUG 2013 EXAM - FEB
B. Pen G meningococcal strains, rifampin is preferred. MED BOARDS; 2015
C. Rifampin Katzung 12th ed pg 812 TOPNOTCH MD)
D. Demeclocycline
E. A and C

366 A 3/M was brought to the hospital for decreased Antidotes for ethylene glycol or methanol poisoning MAIRRE JAMES BACK-UP
sensorium. On further history, the child accidentally include ethanol and fomepizole. Although ethanol is GADDI, MD (TOP MIDTERM
ingested a mouthful of antifreeze. You then proceed effective, it is difficult to achieve safe and effective 4 - AUG 2013 EXAM - FEB
to give the child which antidote? blood levels that is why fomepizole is preferred. MED BOARDS; 2015
A. Disulfiram Katzung 12th ed pg 1037 TOPNOTCH MD)
B. Fomepizole
C. Famotidine
D. Methanol
E. There is no antidote
367 Which antihelminthic drugs rapidly kills adult worms MOA of mebendazole: Mebendazole probably acts by MAIRRE JAMES BACK-UP
presumably through inhibition of oxidative inhibiting microtubule synthesis; Praziquantel: GADDI, MD (TOP MIDTERM
phosphorylation or stimulation of ATPase activity? increase the permeability of trematode and cestode 4 - AUG 2013 EXAM - FEB
A. Mebendazole cell membranes to calcium, resulting in paralysis, MED BOARDS; 2015
B. Niclosamide dislodgement, and death; Pyrantel pamoate: TOPNOTCH MD)
C. Praziquantel neuromuscular blocking agent that causes release of
D. Pyrantel pamoate acetylcholine and inhibition of cholinesterase which
E. Ivermectin results in paralysis of worms followed by expulsion;
Ivermectin: paralyze nematodes and arthropods by
intensifying γ-aminobutyric acid (GABA)–mediated
transmission of signals in peripheral nerves.
Katzung 12th ed pg 941-945
368 A 60/M was diagnosed to have refractory follicular Cetuximab targets EGFR; Bevacizumab targets VEGF; MAIRRE JAMES BACK-UP
B-cell lymphoma. He was prescribed with an agent Omalizumab is an anti-IgE useful in asthma therapy; GADDI, MD (TOP MIDTERM
which binds CD20 on both normal and malignant B Natalizumab binds to α4-subunit of α4β1 and α4β7 4 - AUG 2013 EXAM - FEB
lymphocytes. Which of the following is this drug? integrins expressed on the surfaces of all leukocytes MED BOARDS; 2015
A. Rituximab except neutrophils. Used in MS and Crohn's disease. TOPNOTCH MD)
B. Cetuximab Katzung 12th ed pg 991-993
C. Bevacizumab
D. Omalizumab
E. Natalizumab
369 A 34/F asthmatic, scheduled to undergo The patient is most likely experiencing MAIRRE JAMES BACK-UP
cholelithiasis, was being induced by the bronchospasm and although all volatile anesthetics GADDI, MD (TOP MIDTERM
anesthesiologist. After starting the volatile anesthetic, possess varying degrees of bronchodilating 4 - AUG 2013 EXAM - FEB
the anesthesiologist noted increased peak air properties airway irritation induced by the MED BOARDS; 2015
pressure, prolonged expiration with the classic "shark pungency of isoflurane and desflurane may induce TOPNOTCH MD)
fin" capnograph. Which of the following most likely bronchospasm. These rarely occur with halothane,
have caused the above findings? sevoflurane and nitrous oxide since they are non
A. Halothane pungent. Katzung 12th ed pg 436
B. Sevoflurane
C. Desflurane
D. Nitrous oxide
E. None of the above
370 Which of the following regarding anticholesterol Fibrates stimulate PPAR alpha, most effective for MAIRRE JAMES BACK-UP
drugs is/are TRUE? decreasing elevated triglyceride levels. Katzung 12th GADDI, MD (TOP MIDTERM
A. Simvastatin, a reversible inhibitor of HMG-CoA ed pg 626, 628, 631 4 - AUG 2013 EXAM - FEB
reductase, is most effective in reducing LDL levels MED BOARDS; 2015
B. Niacin stimulates PPAR-α causing an increase in TOPNOTCH MD)
HDL and a decrese in triglycerides
C. There is increased risk of gallstone formation when
fibrates are combined with bile acid binding resins
D. A and C
E. All of the above

TOPNOTCH MEDICAL BOARD PREP PHARMACOLOGY SUPEREXAM Page 42 of 83


For inquiries visit www.topnotchboardprep.com.ph or email us at topnotchmedicalboardprep@gmail.com
TOPNOTCH MEDICAL BOARD PREP PHARMACOLOGY SUPEREXAM
For inquiries visit www.topnotchboardprep.com.ph or email us at topnotchmedicalboardprep@gmail.com
Item QUESTION EXPLANATION AUTHOR TOPNOTCH
# EXAM
371 A 50/M is about to be given 6-mercaptopurine for his Because allopurinol inhibits xanthine oxidase, MAIRRE JAMES BACK-UP
ulcerative colitis as an off-label use. The patient has simultaneous therapy with allopurinol and 6-MP GADDI, MD (TOP MIDTERM
multiple comorbidities which include hypertension, would result in increased levels of 6-MP, thereby 4 - AUG 2013 EXAM - FEB
diabetes, asthma and gout and takes losartan, leading to excessive toxicity. In this setting, the dose MED BOARDS; 2015
metformin, salmeterol, allopurinol and diclofenac for of mercaptopurine must be reduced by 50–75%. TOPNOTCH MD)
these conditions. Which of the above drugs would Katzung 12th ed pg 961-962
prompt you to rethink the dose of 6-MP that you are
about to administer?
A. Losartan
B. Metformin
C. Salmeterol
D. Allopurinol
E. Diclofenac
372 A 18/M previously diagnosed with epilepsy and Memorize inducers & inhibitors VERY HIGH YIELD MAIRRE JAMES BACK-UP
maintained on phenobarbital seizure free for the past GADDI, MD (TOP MIDTERM
2 years, presented to you with a 3 month history of 4 - AUG 2013 EXAM - FEB
cough, recurrent fever and weight loss. AFB was MED BOARDS; 2015
positive on two occasions and CXR show cavitations TOPNOTCH MD)
on the upper lung fields. You conclude that he has
PTB and you start him of HRZE. After a few days, you
were informed that you patient was in the ER and
their working impression is status epilepticus. Which
of the following drugs could have caused the
breakthrough seizures?
A. Isoniazid
B. Rifampicin
C. Pyrazinamide
D. Ethambutol
E. None of the above
373 A 50/M previously diagnosed with BPH develops Sympathomimetic agents may cause or worsen MAIRRE JAMES BACK-UP
mild fever, cough and rhinorrhea. Thinking that he urinary difficulty in patients with prostate GADDI, MD (TOP MIDTERM
just has the common cold, he self medicates with over enlargement due to smooth muscle contraction in 4 - AUG 2013 EXAM - FEB
the counter medication. After 12 hours he rushes to the bladder neck via stimulation of alpha-1 MED BOARDS; 2015
the ER and complains of inability to void. Which of adrenergic receptors. Katzung 12th ed pg 139 TOPNOTCH MD)
the following OTCs may have caused his condition?
A. Chlorphenamine
B. Phenylephrine
C. Acetaminophen
D. Ibuprofen
E. None of the above
374 Which of the following have been proven to cause Topnotch MAIRRE JAMES BACK-UP
pulmonary fibrosis? GADDI, MD (TOP MIDTERM
A. Methothrexate 4 - AUG 2013 EXAM - FEB
B. Bleomycin MED BOARDS; 2015
C. Busulfan TOPNOTCH MD)
D. Amiodarone
E. All of the above

375 A 21/M presented with recurrent fever, night sweats, ABVD (doxorubicin, bleomycin, vinblastine, and MAIRRE JAMES BACK-UP
enlarged abdomen and generalized dacarbazine) and MOPP (mechlorethamine, GADDI, MD (TOP MIDTERM
lymphadenopathy. Biopsy of a lymph node showed vincristine, procarbazine, and prednisone) have 4 - AUG 2013 EXAM - FEB
Reed-Sternberg cells. Which of the following been used to treat Hodgkin's lymphoma. ABVD has MED BOARDS; 2015
chemotherapeutic regimen/s can be given to the supplanted MOPP as the regimen of choice since it is TOPNOTCH MD)
patient? more effective and less toxic. CHOP and Rituximab
A. ABVD are used for NHL Katzung 12th ed pg 970
B. CHOP
C. MOPP
D. Rituximab
E. A and C
376 Drug X is currently undergoing clinical trials with it Phase 1- tested on small number of healthy MAIRRE JAMES BACK-UP
being tested on a small number of volunteers with the volunteers in order to find the maximum tolerated GADDI, MD (TOP MIDTERM
goal being to find the maximum tolerated dose. It is dose and prevent severe toxicity; Phase 2- studied 4 - AUG 2013 EXAM - FEB
now on what phase of clinical testing? on a modest number of patients with the target MED BOARDS; 2015
A. Phase 1 disease to test efficacy; Phase 3- evaluated on a large TOPNOTCH MD)
B. Phase 2 number of patients to further establish efficacy and
C. Phase 3 safety, usually a RCT with blinding and crossover;
D. Phase 4 Phase 4- post marketing surveillance Katzung 12th
E. Phase 5 ed pg 75
377 A 65/M was diagnosed to have CAP-HR. You plan to Clearance is involved in determining the MAIRRE JAMES BACK-UP
start Piperacillin-Tazobactam with a loading dose of maintenance dose Topnotch GADDI, MD (TOP MIDTERM
4.5g IV. Which among the following variables is not 4 - AUG 2013 EXAM - FEB
involved in the calculation of the loading dose? MED BOARDS; 2015
A. Volume of distribution TOPNOTCH MD)
B. Clearance
C. Bioavailability
D. Desired concentration
E. All are needed to calculate the LD
378 Which of the following inhibit/s cortisol synthesis? Aminoglutethimide blocks the conversion of MAIRRE JAMES BACK-UP
A. Aminoglutethimide cholesterol to pregnenolone and causes a reduction GADDI, MD (TOP MIDTERM
B. Ketoconazole in the synthesis of all hormonally active steroids. 4 - AUG 2013 EXAM - FEB
C. Mifepristone Ketoconazole is a potent nonselective inhibitor of MED BOARDS; 2015
D. A and B adrenal and gonadal steroid synthesis through TOPNOTCH MD)
E. All of the above inhibiton of P450 enzymes. Mifepristone is a
glucocorticoid receptor antagonist with strong
antiprogesterone activity. Katzung 12th ed pg 709-
711

TOPNOTCH MEDICAL BOARD PREP PHARMACOLOGY SUPEREXAM Page 43 of 83


For inquiries visit www.topnotchboardprep.com.ph or email us at topnotchmedicalboardprep@gmail.com
TOPNOTCH MEDICAL BOARD PREP PHARMACOLOGY SUPEREXAM
For inquiries visit www.topnotchboardprep.com.ph or email us at topnotchmedicalboardprep@gmail.com
Item QUESTION EXPLANATION AUTHOR TOPNOTCH
# EXAM
379 A 34/F came in to the ER complaining of severe eye All of the choices can decrease IOP and are used in MAIRRE JAMES BACK-UP
pain. On PE you notice that the cornea is steamy with the treatment of glaucoma. Beta blockers, osmotic GADDI, MD (TOP MIDTERM
accompanying conjunctival injection. You proceed to agents, alpha-2 agonists and carbonic anhydrase 4 - AUG 2013 EXAM - FEB
administer a drug that will cause an increase in the inhibitors decrease secretion of aqueous humor MED BOARDS; 2015
outflow of aqueous humor through the canal of from the ciliary epithelium. Cholinomimetics cause TOPNOTCH MD)
Schlemm resulting in a decrease in IOP. Which drug contraction of the ciliary muscle causing the
was administered? trabecular meshwork to open, increasing outflow.
A. Timolol Prostaglandins increase outflow through the canal of
B. Apraclonidine Schlemm. Nonselective alpha agonists increase
C. Latanoprost outflow via the uveoscleral veins. Topnotch
D. Pilocarpine
E. Epinephrine
380 Minoxidil causes vasodilation through: Choice B - calcium channel blockers; Choice C - MAIRRE JAMES BACK-UP
A. Hyperpolarization of smooth muscle membrane Nitroprusside, nitrates; Choice D - fenoldepam GADDI, MD (TOP MIDTERM
through opening of potassium channels 4 - AUG 2013 EXAM - FEB
B. Blockade of voltage gated calcium channels causing MED BOARDS; 2015
a reduction in calcium influx TOPNOTCH MD)
C. Release of nitric oxide from the drug itself
D. Activation of dopamine receptors
E. Blockade of beta receptors
381 Which of the following drugs is classified as Category The rest of the choices are category D drugs. SCOTT RILEY BACK-UP
X during pregnancy? ONG, MD (TOP 5 - MIDTERM
A. Valproate AUG 2014 MED EXAM - FEB
B. Statins BOARDS; 2015
C. Aspirin TOPNOTCH MD)
D. Captopril
E. Phenytoin

382 Which of the following is a balanced vasodilator-- Balanced vasodilators include ACEIs, ARBs, alpha- SCOTT RILEY BACK-UP
meaning, both an arterial and a venous vasodilator-- adrenergic blockers, alpha-central agonists, ONG, MD (TOP 5 - MIDTERM
used to treat both congestive heart failure and nitroprusside and nesiritide. AUG 2014 MED EXAM - FEB
coronary artery vasospasm? BOARDS; 2015
A. Verapamil TOPNOTCH MD)
B. Captopril
C. Nifedipine
D. Propranolol
E. Hydralazine
383 Which of the following diuretics do not act on the SIMILAR TO PREVIOUS BOARD EXAM SCOTT RILEY BACK-UP
luminal side of the renal tubules? CONCEPT/PRINCIPLE. Spironolactone is an ONG, MD (TOP 5 - MIDTERM
A. Mannitol aldosterone antagonist that blocks its binding to AUG 2014 MED EXAM - FEB
B. Furosemide mineralocorticoid receptors in the basolateral BOARDS; 2015
C. Spironolactone membrane of the cortical collecting tubules. The rest TOPNOTCH MD)
D. Acetazolamide of the choices act on the luminal side. Mannitol is an
E. Hydrochlorothiazide osmotic diuretic. Furosemide inhibits Na/K/Cl
transporters in the luminal side of the loop of Henle.
Acetazolamide inhibits carbonic anhydrase activity
at the luminal side of the PCT. Hydrochlorothiazide
inhibits Na/Cl transporters in the luminal side of the
DCT.
384 Which of the following vasodilators has also been SCOTT RILEY BACK-UP
used for the treatment of male pattern baldness due ONG, MD (TOP 5 - MIDTERM
to its side effect of inducing hair growth AUG 2014 MED EXAM - FEB
A. Verapamil BOARDS; 2015
B. Fenoldopam TOPNOTCH MD)
C. Nitroprusside
D. Minoxidil
E. Hydralazine

385 Which of the following is the drug of choice for tic Carbamazepine is also the drug of choice for SCOTT RILEY BACK-UP
disorders? trigeminal neuralgia. ONG, MD (TOP 5 - MIDTERM
A. Carbamazepine AUG 2014 MED EXAM - FEB
B. Lamotrigine BOARDS; 2015
C. Valproic acid TOPNOTCH MD)
D. Topiramate
E. Phenytoin

386 Which of the following drugs used in the treatment of SCOTT RILEY BACK-UP
peptic ulcer disease works by forming a protective ONG, MD (TOP 5 - MIDTERM
coating over ulcer beds and has known adverse AUG 2014 MED EXAM - FEB
effects of melena and darkening of the tongue? BOARDS; 2015
A. Sucralfate TOPNOTCH MD)
B. Bismuth salicylate
C. Psyllium
D. Senna
E. Kaolin
387 Which of the following analgesics would you avoid in SCOTT RILEY BACK-UP
a patient with history of epilepsy as it is known to ONG, MD (TOP 5 - MIDTERM
lower the seizure threshold? AUG 2014 MED EXAM - FEB
A. Ketorolac BOARDS; 2015
B. Aspirin TOPNOTCH MD)
C. Tramadol
D. Celecoxib
E. Indomethacin

TOPNOTCH MEDICAL BOARD PREP PHARMACOLOGY SUPEREXAM Page 44 of 83


For inquiries visit www.topnotchboardprep.com.ph or email us at topnotchmedicalboardprep@gmail.com
TOPNOTCH MEDICAL BOARD PREP PHARMACOLOGY SUPEREXAM
For inquiries visit www.topnotchboardprep.com.ph or email us at topnotchmedicalboardprep@gmail.com
Item QUESTION EXPLANATION AUTHOR TOPNOTCH
# EXAM
388 In which of the following drugs used for the SIMILAR TO PREVIOUS BOARD EXAM SCOTT RILEY BACK-UP
treatment of gout is cataract an important side effect? CONCEPT/PRINCIPLE. ONG, MD (TOP 5 - MIDTERM
A. Colchicine AUG 2014 MED EXAM - FEB
B. Probenicid BOARDS; 2015
C. Allopurinol TOPNOTCH MD)
D. Febuxostate
E. Indomethacin

389 Which of the following adrenergic receptors in the Activation of alpha-2 receptors decreases aqueous SCOTT RILEY BACK-UP
ciliary body of the eye causes a decrease in aqueous humor production, while activation of either beta-1 ONG, MD (TOP 5 - MIDTERM
humor production when activated? or beta-2 receptors increases aqeous humor AUG 2014 MED EXAM - FEB
A. alpha-1 production. Alpha-1 and beta-3 receptors have no BOARDS; 2015
B. alpha-2 known function in relation to aqueous humor TOPNOTCH MD)
C. beta-1 production.
D. beta-2
E. beta-3

390 Tocainide is an antiarrhythmic agent classified under SCOTT RILEY BACK-UP


which group? ONG, MD (TOP 5 - MIDTERM
A. Class IA AUG 2014 MED EXAM - FEB
B. Class IB BOARDS; 2015
C. Class IC TOPNOTCH MD)
D. Class II
E. Class III

391 Ondansetron is an antagonist of which serotonin 5HT3 receptor is also the only ligand-gated ion SCOTT RILEY BACK-UP
receptor? channel among the serotonin receptors. ONG, MD (TOP 5 - MIDTERM
A. 5HT1D AUG 2014 MED EXAM - FEB
B. 5HT1B BOARDS; 2015
C. 5HT2 TOPNOTCH MD)
D. 5HT3
E. 5HT4

392 Clomiphene is a selective estrogen-receptor C refers to leuprolide when administered in pulsatile SCOTT RILEY BACK-UP
modulator commonly used to induce ovulation in fashion. ONG, MD (TOP 5 - MIDTERM
patients undergoing assisted reproduction therapy. AUG 2014 MED EXAM - FEB
Which of the following best describes its mechanism BOARDS; 2015
of action? TOPNOTCH MD)
A. It potentiates the action of FSH and LH on the
ovaries.
B. It acts as a partial agonist in the pituitary to block
negative feedback from estradiol.
C. It acts as a GnRH analogue that increases the
release of FSH and LH from the pituitary.
D. It inhibits the release of inhibin from ovarian
follicles, thus facilitating FSH and LH secretion.
E. It is an LH analogue and mimics the physiologic LH
surge when given just before ovulation.
393 Which of the following anti-lipid drugs would you SCOTT RILEY BACK-UP
avoid giving to a diabetic patient due to its adverse ONG, MD (TOP 5 - MIDTERM
effect of inducing hyperglycemia? AUG 2014 MED EXAM - FEB
A. Cholestyramine BOARDS; 2015
B. Egetimibe TOPNOTCH MD)
C. Niacin
D. Gemfibrozil
E. Colestipol

394 Which of the following traditional antipsychotic Promethazine is histamine H1-antagonist. SCOTT RILEY BACK-UP
agents has now been more widely used for antiemetic Haloperidol and chlorpromazine are typical ONG, MD (TOP 5 - MIDTERM
treatment of nausea and vertigo? antipsychotics mainly used for psychotic disorders, AUG 2014 MED EXAM - FEB
A. Prochlorperazine not so much as antiemesis. Metoclopramide is a D2- BOARDS; 2015
B. Promethazine antagonist used as GI prokinetic agent. TOPNOTCH MD)
C. Haloperidol
D. Chlorpromazine
E. Metoclopramide

395 Which of the following is not used for the treatment Chlordiazepoxide is used for the treatment of SCOTT RILEY BACK-UP
of alcohol dependence? alcohol withdrawal, not dependence. Diazepam may ONG, MD (TOP 5 - MIDTERM
A. Chlordiazepoxide also be used for alcohol withdrawal. AUG 2014 MED EXAM - FEB
B. Disulfiram BOARDS; 2015
C. Naltrexone TOPNOTCH MD)
D. Acamprosate
E. None of the above

396 Which of the following antitumor antibiotics is cell- Bleomycin is specific for G2 phase of the cell cycle. SCOTT RILEY BACK-UP
cycle specific? The rest of the choices are non-cell-cycle specific. ONG, MD (TOP 5 - MIDTERM
A. Mitomycin AUG 2014 MED EXAM - FEB
B. Doxorubicin BOARDS; 2015
C. Bleomycin TOPNOTCH MD)
D. Actinomycin
E. None of the above

TOPNOTCH MEDICAL BOARD PREP PHARMACOLOGY SUPEREXAM Page 45 of 83


For inquiries visit www.topnotchboardprep.com.ph or email us at topnotchmedicalboardprep@gmail.com
TOPNOTCH MEDICAL BOARD PREP PHARMACOLOGY SUPEREXAM
For inquiries visit www.topnotchboardprep.com.ph or email us at topnotchmedicalboardprep@gmail.com
Item QUESTION EXPLANATION AUTHOR TOPNOTCH
# EXAM
397 Which of the following antimicrobial agent acts as a SCOTT RILEY BACK-UP
dihydrofolate reductase inhibitor? ONG, MD (TOP 5 - MIDTERM
A. Trimethoprim AUG 2014 MED EXAM - FEB
B. Sulfamethoxazole BOARDS; 2015
C. Metronidazole TOPNOTCH MD)
D. Nitrofurantoin
E. Rifampicin

398 In which of the following penicillins is interstitial SIMILAR TO PREVIOUS BOARD EXAM SCOTT RILEY BACK-UP
nephritis a known adverse effect, partly leading to a CONCEPT/PRINCIPLE. ONG, MD (TOP 5 - MIDTERM
decline in its popularity? AUG 2014 MED EXAM - FEB
A. Nafcillin BOARDS; 2015
B. Methicillin TOPNOTCH MD)
C. Carbenicillin
D. Bacitracin
E. Metampicillin

399 Which of the following antibiotics work by binding to SIMILAR TO PREVIOUS BOARD EXAM SCOTT RILEY BACK-UP
the D-Ala-D-Ala terminus of bacterial cell wall CONCEPT/PRINCIPLE. Piperacillin: inhibits ONG, MD (TOP 5 - MIDTERM
precursors, thus preventing peptidoglycan transpeptidase and binds to penicillin-binding AUG 2014 MED EXAM - FEB
polymerization? proteins in the cell membrane. Cycloserine: blocks BOARDS; 2015
A. Imipenem incorporation of D-Ala into the pentapeptide side TOPNOTCH MD)
B. Cycloserine chain of peptidoglycan. Bacitracin: interferes with a
C. Bacitracin late stage in cell wall synthesis.
D. Vancomycin
E. Piperacillin
400 Which of the following ophthalmic agents decreases Bimatoprost is a prostaglandin analogue that SCOTT RILEY BACK-UP
intraocular pressure by increasing outflow of increase outflow via the uveoscleral pathway. ONG, MD (TOP 5 - MIDTERM
aqueous humor through the uveoscleral or Apraclonidine is a alpha-2 receptor agonist that AUG 2014 MED EXAM - FEB
unconventional pathway? decreases aqueous humor production. Mannitol BOARDS; 2015
A. Homatropine works via osmosis. Timolol is a beta-receptor TOPNOTCH MD)
B. Bimatoprost antagonist that also decreases aqueous humor
C. Apraclonidine production. Homatropine has no effect on IOP.
D. Mannitol
E. Timolol
401 Which of the following chemotherapeutic drugs may SIMILAR TO PREVIOUS BOARD EXAM JOSE CARLO DIAGNOSTIC
be used for Lentigo Maligna? CONCEPT/PRINCIPLE MASANGKAY III, EXAM - AUG
A. Actinomycin MD (TOP 8 - FEB 2014
B. Bleomycin 2014 MED
C. Doxorubicin BOARDS;
D. Vinblastine TOPNOTCH MD)
E. Cisplatin

402 Which of the following chemotherapeutic drugs may SIMILAR TO PREVIOUS BOARD EXAM JOSE CARLO DIAGNOSTIC
be used for Lung Cancer? CONCEPT/PRINCIPLE MASANGKAY III, EXAM - AUG
A. Procarbazine MD (TOP 8 - FEB 2014
B. Cytarabine 2014 MED
C. Topotecan BOARDS;
D. Cisplatin TOPNOTCH MD)
E. Cyclophosphamide

403 Which of the following drugs is an effective SIMILAR TO PREVIOUS BOARD EXAM JOSE CARLO DIAGNOSTIC
prophylactic drug against migraine headache attacks? CONCEPT/PRINCIPLE MASANGKAY III, EXAM - AUG
A. Ondansetron MD (TOP 8 - FEB 2014
B. Sumatriptan 2014 MED
C. Ergotamine BOARDS;
D. Methylergonovine TOPNOTCH MD)
E. Propranolol

404 A newly formulated drug with a half life of 6 hours is 100% elimination will be reached in 4 half lives. (6 x JOSE CARLO DIAGNOSTIC
eliminated via 1st order kinetics, how many hours 4=24) MASANGKAY III, EXAM - AUG
will the drug be 100% eliminated from the body of a MD (TOP 8 - FEB 2014
normal person? 2014 MED
A. 6 hours BOARDS;
B. 12 hours TOPNOTCH MD)
C. 18 hours
D. 24 hours
E. 36 hours
405 In calculating the loading dose of a drug, which of the LD = (Vd x DPC) / Bioavailability JOSE CARLO DIAGNOSTIC
following parameters are not necessary? MASANGKAY III, EXAM - AUG
A. Volume of distribution MD (TOP 8 - FEB 2014
B. Desired Plasma Concentration 2014 MED
C. Bioavailability BOARDS;
D. Clearance TOPNOTCH MD)
E. None of the Above

406 Which of the following is a reversible inhibitor of Allopurinol is an IRREVERSIBLE Xanthine Oxidase JOSE CARLO DIAGNOSTIC
Xanthine Oxidase used in Chronic Gout? inhibitor, Febuxostat is a REVERSIBLE Xanthine MASANGKAY III, EXAM - AUG
A. Allopurinol Oxidase inhibitor, MD (TOP 8 - FEB 2014
B. Colchicine 2014 MED
C. Febuxostat BOARDS;
D. Mycophenolate Mofetil TOPNOTCH MD)
E. Probenecid

TOPNOTCH MEDICAL BOARD PREP PHARMACOLOGY SUPEREXAM Page 46 of 83


For inquiries visit www.topnotchboardprep.com.ph or email us at topnotchmedicalboardprep@gmail.com
TOPNOTCH MEDICAL BOARD PREP PHARMACOLOGY SUPEREXAM
For inquiries visit www.topnotchboardprep.com.ph or email us at topnotchmedicalboardprep@gmail.com
Item QUESTION EXPLANATION AUTHOR TOPNOTCH
# EXAM
407 A patient will be undergoing neurosurgery, the Ketamine is capable of Dissociative Anesthesia ( JOSE CARLO DIAGNOSTIC
neurosurgeon would like to maintain the patient's Analgesia, Amnesia and Catatonia with retained MASANGKAY III, EXAM - AUG
consciousness while the procedure is on going. What consciousness) MD (TOP 8 - FEB 2014
would be the preferred anesthetic of choice? 2014 MED
A. Ketamine BOARDS;
B. Etomidate TOPNOTCH MD)
C. Midazolam
D. Fentanyl
E. Propofol
408 A 14 year old child was diagnosed with JOSE CARLO DIAGNOSTIC
Schizophrenia, which of the following is the only MASANGKAY III, EXAM - AUG
antipsychotic approved for pediatric patients? MD (TOP 8 - FEB 2014
A. Clozapine 2014 MED
B. Risperidone BOARDS;
C. Quetiapine TOPNOTCH MD)
D. Olanzapine
E. Thioridazine

409 The following statements are true of JOSE CARLO DIAGNOSTIC


Aminoglycosides, EXCEPT: MASANGKAY III, EXAM - AUG
A. Act by Time-Dependent Killing Action MD (TOP 8 - FEB 2014
B. As the plasma level is increased above the MIC an 2014 MED
increasing proportion of bacteria are killed and at a BOARDS;
more rapid rate TOPNOTCH MD)
C. Has a post-antibiotic effect
D. Requires Oxygen for uptake
E. Amikacin has the widest coverage
410 Among the following Anti-arrhythmics, which of the K channel blockers like Sotalol can increase the QT JOSE CARLO DIAGNOSTIC
following has a highest potential for Torsade de interval thereby increasing the propensity to MASANGKAY III, EXAM - AUG
pointes? develop Torsade de Pointes. MD (TOP 8 - FEB 2014
A. Lidocaine 2014 MED
B. Propranolol BOARDS;
C. Sotalol TOPNOTCH MD)
D. Procainamide
E. Verapamil

411 A patient with CHF was rushed to the ED where you Spironolactone and Acetazolamide can aggravate the JOSE CARLO DIAGNOSTIC
are on-duty, ABG was done revealing Metabolic ongoing acidosis of the patient, the only ones left are MASANGKAY III, EXAM - AUG
Acidosis. His wife told you that he had a history of loop diuretics, but then Furosemide and Torsemide MD (TOP 8 - FEB 2014
severe allergic reaction to Sulfa drugs, but you need are both Sulfa containing hence cannot be given to 2014 MED
to give the patient a powerful diuretic, Unfortunately the patient. Ethacrynic acid is most suitable to this BOARDS;
the following drugs are the only ones available in patient. TOPNOTCH MD)
your Pharmacy, Among which of the following will
you give?
A. Furosemide
B. Spironolactone
C. Torsemide
D. Ethacrynic Acid
E. Acetazolamide
412 To improve the quality of life of patients with on- Flutamide an Androgen Antagonist is given in JOSE CARLO DIAGNOSTIC
going treatment for Prostate Adenocarcinoma with adjunct to Leuprolide to lessen the side effects of MASANGKAY III, EXAM - AUG
Leuprolide, which of the following medications Leuprolide like Gynecomastia, decreased libido, MD (TOP 8 - FEB 2014
should be added? Apoplexy and hot flushes.(SIMILAR TO PREVIOUS 2014 MED
A. Anastrazole BOARD EXAM CONCEPT/PRINCIPLE) BOARDS;
B. Examestane TOPNOTCH MD)
C. Flutamide
D. Prednisone
E. Morphine
413 A 9-year old Filipino child was brought to you in the Valproate is the DOC for Absence Seizures, most JOSE CARLO DIAGNOSTIC
clinics by her mother due to an observed blank-stare especially in our setting here in the Philippines, MASANGKAY III, EXAM - AUG
episodes noted by her mother which lasts from a few Ethosuximide is NOT commercially available. MD (TOP 8 - FEB 2014
seconds to a few minutes. Which of the following 2014 MED
medications is the most suitable for this patient? BOARDS;
A. Valproate TOPNOTCH MD)
B. Phenytoin
C. Phenobarbital
D. Carbamazepine
E. Lorazepam
414 Which of the following Beta Blockers has an Intrinsic Pindolol has ISA, which makes it a B blocker with a JOSE CARLO DIAGNOSTIC
Sympathomimetic Activity? partial Alpha agonist action. MASANGKAY III, EXAM - AUG
A. Carvedilol MD (TOP 8 - FEB 2014
B. Pindolol 2014 MED
C. Nadolol BOARDS;
D. Metoprolol TOPNOTCH MD)
E. Timolol

415 Early morning hypoglycemia is a challenge among JOSE CARLO DIAGNOSTIC


physicians in treating diabetic patients. When a MASANGKAY III, EXAM - AUG
diabetic patient monitors his Glucose at 3 AM MD (TOP 8 - FEB 2014
revealing Hypoglycemia with a Hyperglycemic 7 AM 2014 MED
CBG, This effect is termed as: BOARDS;
A. Somogyi TOPNOTCH MD)
B. Waning
C. Dawn
D. Dawn and Waning
E. Somogyi and Waning

TOPNOTCH MEDICAL BOARD PREP PHARMACOLOGY SUPEREXAM Page 47 of 83


For inquiries visit www.topnotchboardprep.com.ph or email us at topnotchmedicalboardprep@gmail.com
TOPNOTCH MEDICAL BOARD PREP PHARMACOLOGY SUPEREXAM
For inquiries visit www.topnotchboardprep.com.ph or email us at topnotchmedicalboardprep@gmail.com
Item QUESTION EXPLANATION AUTHOR TOPNOTCH
# EXAM
416 In a patient with Grave's Disease this drug is given to JOSE CARLO DIAGNOSTIC
decrease peripheral conversion of T4 to T3: MASANGKAY III, EXAM - AUG
A. Propranolol MD (TOP 8 - FEB 2014
B. I 131 2014 MED
C. Methimazole BOARDS;
D. PTU TOPNOTCH MD)
E. Lugol's Iodine

417 In a Diabetic patient with impaired renal function Before prescribing 2nd generation Sulfonylureases JOSE CARLO DIAGNOSTIC
which of the following drugs is contraindicated? liver and hepatic function must be checked first MASANGKAY III, EXAM - AUG
A. Nateglinide because they are contraindicated. MD (TOP 8 - FEB 2014
B. Glipizide 2014 MED
C. Metformin BOARDS;
D. Pioglitazone TOPNOTCH MD)
E. Acarbose

418 Which of the following Cephalosporins is synergistic 3rd gen Cephalosporins have syndergistic effects JOSE CARLO DIAGNOSTIC
with Aminoglycosides? with Aminoglycosides, 1st and 2nd gen MASANGKAY III, EXAM - AUG
A. Cefazolin cephalosporines can increase nephrotoxic effects of MD (TOP 8 - FEB 2014
B. Cefaclor aminoflycosides. 2014 MED
C. Cefixime BOARDS;
D. Cefipime TOPNOTCH MD)
E. Loracarbef

419 What class of anti-hypertensive drugs are also used in Alpha 1-A receptor is specifically seen in the Urinary JOSE CARLO DIAGNOSTIC
BPH? tract most especially the Prostate.Alpha 1A MASANGKAY III, EXAM - AUG
A. Alpha 1A Agonists antagonists/Quinazolines include Prazosin MD (TOP 8 - FEB 2014
B. Alpha 1B Agonists Terazosin, and Tamsulosin 2014 MED
C. Alpha 1A Antagonists BOARDS;
D. Alpha 1B Antagonists TOPNOTCH MD)
E. Alpha 2 Agonists

420 Cocaine has which of the following mechanism of JOSE CARLO DIAGNOSTIC
action? MASANGKAY III, EXAM - AUG
A. Promotes Adrenergic Synthesis MD (TOP 8 - FEB 2014
B. Inhibits Adrenergic Storage 2014 MED
C. Promotes Adrenergice Release BOARDS;
D. Inhibits Adrenergic Metabolism/Degradation TOPNOTCH MD)
E. Inhibits Adrenergic Reuptake

421 A condition where there is depletion of needed Idiosyncratic means unusual drug reponse, WEBSTER MIDTERM 1
substrates as a result of continuous activation by a infrequently observed in most patients; tolerance on ALINDOG, MD EXAM - AUG
drug and hence decrease in responsiveness, such as the other hand, refers to decrease in the intensity of (TOP 3 - FEB 2014
in chronic nitroglycerin use, is known as: response to drug as a consequence of continued 2014 MED
A. Tachyphylaxis administration. BOARDS;
B. Idiosyncratic response TOPNOTCH MD)
C. Tolerance
D. Resistance

422 Teratogenesis is one of the dreaded complications of WEBSTER MIDTERM 1


improper use of drugs during pregnancy. Of note, ALINDOG, MD EXAM - AUG
lithium, a drug primarily used to treat bipolar (TOP 3 - FEB 2014
disorders, can cause: 2014 MED
A. Ebstein's anomaly BOARDS;
B. Vaginal clear cell adenocarcinoma TOPNOTCH MD)
C. Phocomelia
D. Mobius sequence

423 Pralidoxime is an antidote to organophosphate WEBSTER MIDTERM 1


poisoning. Which of the following is not true about ALINDOG, MD EXAM - AUG
this drug? (TOP 3 - FEB 2014
A. It is a cholinesterase regenerator. 2014 MED
B. It requires to be given 6-8 hours before BOARDS;
organophosphate-cholinesterase binding occurs. TOPNOTCH MD)
C. It binds with the phosphorus of
organophosphates.
D. All of the above
E. None of the above
424 Methyldopa is a drug of choice in treating pre- Activation of alpha-2 adrenergic receptors by WEBSTER MIDTERM 1
eclampsia. Its ability to lower blood pressure is based methyldopa metabolites in the brainstem appears to ALINDOG, MD EXAM - AUG
on its action of: inhibit sympathetic nervous system output and (TOP 3 - FEB 2014
A. Activating alpha-1 adrenergic receptors lower blood pressure. This is also the mechanism of 2014 MED
B. Inhibiting alpha-1 adrenergic receptors action of clonidine. Activation of alpha-1 adrenergic BOARDS;
C. Activating alpha-2 adrenergic receptors receptors meanwhile, stimulates vasoconstriction TOPNOTCH MD)
D. Inhibiting alpha-2 adrenergic receptors and hence increase in BP.

425 Hydralazine lowers BP by causing arteriolar smooth WEBSTER MIDTERM 1


muscle relaxation. This effect is achieved by its ability ALINDOG, MD EXAM - AUG
to alter metabolism of which ion? (TOP 3 - FEB 2014
A. Sodium 2014 MED
B. Potassium BOARDS;
C. Magnesium TOPNOTCH MD)
D. Calcium

TOPNOTCH MEDICAL BOARD PREP PHARMACOLOGY SUPEREXAM Page 48 of 83


For inquiries visit www.topnotchboardprep.com.ph or email us at topnotchmedicalboardprep@gmail.com
TOPNOTCH MEDICAL BOARD PREP PHARMACOLOGY SUPEREXAM
For inquiries visit www.topnotchboardprep.com.ph or email us at topnotchmedicalboardprep@gmail.com
Item QUESTION EXPLANATION AUTHOR TOPNOTCH
# EXAM
426 Throbbing headaches in patients taking nitrates are WEBSTER MIDTERM 1
mainly attributed to: ALINDOG, MD EXAM - AUG
A. Chemical toxicity (TOP 3 - FEB 2014
B. Meningeal artery vasodilation 2014 MED
C. Reflex tachycardia BOARDS;
D. Abrupt lowering of BP TOPNOTCH MD)

427 Which diuretic is found to have long-term benefits WEBSTER MIDTERM 1


and can reduce mortality in chronic heart failure? ALINDOG, MD EXAM - AUG
A. Thiazide (TOP 3 - FEB 2014
B. Furosemide 2014 MED
C. Spironolactone BOARDS;
D. Ethacrynic acid TOPNOTCH MD)

428 Aside from its lipid-lowering action, statins are used WEBSTER MIDTERM 1
in coronary artery disease because it can also: ALINDOG, MD EXAM - AUG
A. Vasodilate stenotic segments of the coronary (TOP 3 - FEB 2014
B. Stabilize atherosclerotic plaques 2014 MED
C. Inhibits platelet aggregation BOARDS;
D. Decrease myocardial oxygen demand TOPNOTCH MD)

429 Which of the following is considered as a reliever of Terbutaline is a beta-2 receptor agonist used as a WEBSTER MIDTERM 1
asthma rather than a controller? fast-acting bronchodilator for asthma and is also ALINDOG, MD EXAM - AUG
A. Montelukast used in preventing premature labor (tocolysis). (TOP 3 - FEB 2014
B. Cromolyn 2014 MED
C. Omalizumab BOARDS;
D. Terbutaline TOPNOTCH MD)

430 Oral chelation treatment in hemochromatosis can be Therapeutic phlebotomy is the main line of WEBSTER MIDTERM 1
done by giving: treatment of hemochromatosis, especially the ALINDOG, MD EXAM - AUG
A. Deferoxamine primary type. Should chelation becomes warranted, (TOP 3 - FEB 2014
B. Deferasirox it can be done via IV (deferoxamine) or oral 2014 MED
C. EDTA (Deferasirox). BOARDS;
D. Penicillamine TOPNOTCH MD)

431 Aspirin toxicity results in increased respiratory drive Aspirin is not an inhibitor of electron-transport WEBSTER MIDTERM 1
that leads to hyperventilation and respiratory chain (ETC), but an uncoupler. Uncouplers of ALINDOG, MD EXAM - AUG
alkalosis. Which of these statements is not true about oxidative phosphorylation in mitochondria inhibit (TOP 3 - FEB 2014
aspirin intoxication? the coupling between the electron transport and 2014 MED
A. It is achieved when given at a dose of 150 mg/kg. phosphorylation reactions and thus inhibit ATP BOARDS;
B. It can manifest as a pure high anion gap metabolic synthesis without affecting the respiratory chain and TOPNOTCH MD)
acidosis in children. ATP synthase (H(+)-ATPase), thus heat is just
C. It is a known inhibitor of the electron transport produced without producing the needed ATPs.
chain.
D. All of the abbove
E. None of the above
432 Tranexamic acid is used to prevent and treat acute Plasminogen are converted to plasmin whose WEBSTER MIDTERM 1
bleeding episodes in patients who are at high risk, function is to lyse the formed fibrin. When it is ALINDOG, MD EXAM - AUG
especially those who have hemophilia and inhibited, hemostasis is promoted. (TOP 3 - FEB 2014
intracranial aneurysms. The mechanism of action of 2014 MED
this drug involves: BOARDS;
A. Inhibition of plasminogen activators TOPNOTCH MD)
B. Inhibition of plasmin-degrading factors
C. Potentiation of factor VIII activity
D. Delivery of Vitamin K to the clotting factor-
synthesizing cells
433 Treatment for inflammatory bowel disease: Sulfasalazine is poorly absorbed in the GIT and WEBSTER MIDTERM 1
A. Sulfadiazine therefore it is believed that it has its main action in ALINDOG, MD EXAM - AUG
B. Sulfasalazine lumen. It is used to treat IBDs by reducing the (TOP 3 - FEB 2014
C. Sulfamethoxazole synthesis of inflammatory mediators known as 2014 MED
D. Sulfatide eicosanoids and inflammatory cytokines. However, BOARDS;
unlike glucocorticoids, sulfasalazine is also a mild TOPNOTCH MD)
immunosuppressant.

434 Remarkable to these drugs is their ability to inhibit WEBSTER MIDTERM 1


cytochrome P450 causing significant interactions ALINDOG, MD EXAM - AUG
with other drugs, except: (TOP 3 - FEB 2014
A. Isoniazid 2014 MED
B. Cimetidine BOARDS;
C. Erythromycin TOPNOTCH MD)
D. Barbiturates

435 Which aminoglycoside has the narrowest therapeutic WEBSTER MIDTERM 1


window but with the widest coverage? ALINDOG, MD EXAM - AUG
A. Gentamicin (TOP 3 - FEB 2014
B. Streptomycin 2014 MED
C. Amikacin BOARDS;
D. Kanamycin TOPNOTCH MD)

TOPNOTCH MEDICAL BOARD PREP PHARMACOLOGY SUPEREXAM Page 49 of 83


For inquiries visit www.topnotchboardprep.com.ph or email us at topnotchmedicalboardprep@gmail.com
TOPNOTCH MEDICAL BOARD PREP PHARMACOLOGY SUPEREXAM
For inquiries visit www.topnotchboardprep.com.ph or email us at topnotchmedicalboardprep@gmail.com
Item QUESTION EXPLANATION AUTHOR TOPNOTCH
# EXAM
436 Antifolates sulfamethoxazole and trimethoprim are WEBSTER MIDTERM 1
bactericidal by virtue of their synergistic actions. ALINDOG, MD EXAM - AUG
Nonetheless, resistence can develop via plasmid- (TOP 3 - FEB 2014
mediated: 2014 MED
A. Decrease in sensitivity of dihydropteroate BOARDS;
synthase to sulfonamides TOPNOTCH MD)
B. Increase in PABA synthesis by the organism
C. Poor intracellular accumulation of the drugs
D. All of the above
E. None of the above
437 Isoniazid is the most important drug used in TB. It WEBSTER MIDTERM 1
inhibits mycolic acid synthesis and is therefore ALINDOG, MD EXAM - AUG
bactericidal. Possible adverse effects include neuritis, (TOP 3 - FEB 2014
insomnia and seizures. To prevent neurotoxicities, 2014 MED
patient should be supplemented with: BOARDS;
A. Folic acid TOPNOTCH MD)
B. Pyridoxine
C. Vitamin A
D. Riboflavin

438 What is the drug of choice for the cyst carrier state of Diloxanide furoate is a luminal amoebicide which is WEBSTER MIDTERM 1
Entamoeba histolytica? hydrolysed in the gut, thus releasing the free ALINDOG, MD EXAM - AUG
A. Diloxanide furoate diloxanide which acts as an amoebicide. It is given (TOP 3 - FEB 2014
B. Metronidazole alone in asymptomatic cyst passers. For patients 2014 MED
C. Tinidazole with active amoebic infections, it can be BOARDS;
D. Paramomycin administered along with metronidazole. TOPNOTCH MD)

439 A vinca alkaloid acting primarily in the M phase of WEBSTER MIDTERM 1


cancer cell cycle that is useful in the treatment of ALINDOG, MD EXAM - AUG
acute leukemias, lymphomas and neuroblastoma: (TOP 3 - FEB 2014
A. Paclitaxel 2014 MED
B. Vincristine BOARDS;
C. Doxorubicin TOPNOTCH MD)
D. Cytarabine

440 This is a mucosal protective drug used as an adjunct WEBSTER MIDTERM 1


treatment of PUD. It binds to injured tissues and ALINDOG, MD EXAM - AUG
forms a protective covering over ulcer beds, resulting (TOP 3 - FEB 2014
in accelerated healing: 2014 MED
A. Misoprostol BOARDS;
B. Bismuth salicylate TOPNOTCH MD)
C. Calcium carbonate
D. Sucralfate

441 Factor/s that influence/s passage of drugs through All are factors that influence passage of drugs JULIET KRISTINE MIDTERM 2
cell membranes: through cell membranes. Lipid solubility is the EVANGELISTA, EXAM - AUG
A. Presence of pores in the membrane most important limiting factor for permeation of MD (TOP 9 - FEB 2014
B. Amount of protein binding drugs because cells are covered by lipid membranes. 2014 MED
C. Lipid solubility BOARDS;
D. pH TOPNOTCH MD)
E. All of the above

442 In this clinical phase of drug development, the drug is Phase 1 trial involves evaluation of dose-response JULIET KRISTINE MIDTERM 2
studied in patients with the target disease to relationship and pharmacokinetics in normal EVANGELISTA, EXAM - AUG
determine its efficacy: volunteer patients with target disease. Phase 2 trial MD (TOP 9 - FEB 2014
A. Phase 0 involves determination whether the agent has the 2014 MED
B. Phase 1 desired efficacy at doses that are tolerated by sick BOARDS;
C. Phase 2 patients. Phase 3 trial is a large design including TOPNOTCH MD)
D. Phase 3 placebo and positive controls in a double-blind
E. Phase 4 crossover design. Phase 4 trial is a postmarketing
surveillance.
443 The most important pharmacologic action of digitalis The most important pharmacologic action of JULIET KRISTINE MIDTERM 2
in the treatment of heart failure is its ability to: digitalis in the treatment of heart failure is its ability EVANGELISTA, EXAM - AUG
A. Increase heart rate to increase myocardial contractile force by MD (TOP 9 - FEB 2014
B. Reduce venous pressure inhibiting Na/K ATPase and increaseing intracellular 2014 MED
C. Reduce arterial pressure calcium. BOARDS;
D. Produce diuresis in edematous patients TOPNOTCH MD)
E. Increase myocardial contractile force

444 Reserpine blocks this step of adrenergic Reserpine blocks vesicular transport in adrenergic JULIET KRISTINE MIDTERM 2
transmission: transmission. EVANGELISTA, EXAM - AUG
A. Synthesis of the neurotransmitter MD (TOP 9 - FEB 2014
B. Transport of neurotransmitter to vesicles 2014 MED
C. Release of the neurotransmitter BOARDS;
D. Reuptake into the presynaptic neuron TOPNOTCH MD)
E. Diffusion in the synaptic cleft

TOPNOTCH MEDICAL BOARD PREP PHARMACOLOGY SUPEREXAM Page 50 of 83


For inquiries visit www.topnotchboardprep.com.ph or email us at topnotchmedicalboardprep@gmail.com
TOPNOTCH MEDICAL BOARD PREP PHARMACOLOGY SUPEREXAM
For inquiries visit www.topnotchboardprep.com.ph or email us at topnotchmedicalboardprep@gmail.com
Item QUESTION EXPLANATION AUTHOR TOPNOTCH
# EXAM
445 A 25 year-old farmer was brought to the emergency Pralidoxime is a cholinesterase regenerator, an JULIET KRISTINE MIDTERM 2
room after having allegedly drank a poison. PE antidote for organophosphate poisoning. EVANGELISTA, EXAM - AUG
revealed that he had constricted pupils, excessive Pralidoxime is an example of chemical antagonism. MD (TOP 9 - FEB 2014
salivation and sweating, wheezing and had several Chemical antagonism is when an antagonist directly 2014 MED
bouts of vomiting and diarrhea. He was given interacts with the drug being antagonized to remove BOARDS;
antidotes Atropine and Pralidoxime. Pralidoxime it or prevent it from reaching its target. TOPNOTCH MD)
exert its effects by:
A. Destruction of the enzyme
B. Competitive antagonism
C. Noncompetitive antagonism
D. Physiologic antagonism
E. Chemical antagonism
446 It is the major second messenger of beta receptor cAMP is the major second messenger of beta JULIET KRISTINE MIDTERM 2
activation that participates in signal transduction: receptor activation that participates in signal EVANGELISTA, EXAM - AUG
A. Inositol triphosphate transduction. MD (TOP 9 - FEB 2014
B. cAMP 2014 MED
C. cGMP BOARDS;
D. Calcium TOPNOTCH MD)
E. Adenylyl cyclase

447 A neonate was given IV antibiotics for 3 days. Chloramphenicol is the drug that was given to JULIET KRISTINE MIDTERM 2
However, the baby developed hypothermia neonate causing gray baby syndrome. It inhibits EVANGELISTA, EXAM - AUG
associated with diarrhea and grayish color of the skin. transpeptidation at 50s subunit and it is MD (TOP 9 - FEB 2014
What is the mechanism of action of the drug that was bacteriostatic. Penicillin inhibits transpeptidation in 2014 MED
given to the baby? bacterial cell walls. Aminoglycoside inhibits protein BOARDS;
A. Inhibits transpeptidation in bacterial cell walls synthesis by binding to 30s subunit. Co-trimoxazole TOPNOTCH MD)
B. Inhibits protein synthesis by binding to 30s inhibits dihydropteroate synthase. Fluoroquinolone
subunit inhibits DNA replication by binding to DNA gyrase.
C. Inhibits transpeptidation at 50s subunit
D. Inhibits dihydropteroate synthase
E. Inhibits DNA replication by binding to DNA
gyrase
448 It is a drug that is used almost solely as an alternative Spectinomycin is an aminoglycoside that inhibits JULIET KRISTINE MIDTERM 2
treatment for gonorrhea in patients whose gonococci protein synthesis by binding to 30s subunit. It is the EVANGELISTA, EXAM - AUG
are resistant to other drugs is: drug of choice of drug-resistant gonorrhea and for MD (TOP 9 - FEB 2014
A. Docycycline gonorrhea in penicillin-allergic patients. 2014 MED
B. Spectinomycin BOARDS;
C. Ofloxacin TOPNOTCH MD)
D. Azithromycin
E. Tetracycline

449 Drugs like quinidine, procainamide and disopyramide Drugs like quinidine, procainamide and JULIET KRISTINE MIDTERM 2
are very useful in treatment of: disopyramide are very useful in treatment of EVANGELISTA, EXAM - AUG
A. Congestive heart failure arrhythmia. MD (TOP 9 - FEB 2014
B. Hypertension 2014 MED
C. Thyrotoxicosis BOARDS;
D. Status asthmaticus TOPNOTCH MD)
E. Arrhythmia

450 It is an anticonvulsant that is effective for neuropathic Gabapentin is an anticonvulsant drug, a GABA JULIET KRISTINE MIDTERM 2
pain: derivative which blocks calcium channels, increases EVANGELISTA, EXAM - AUG
A. Valproic acid GABA release, very useful in neuropathic pain also in MD (TOP 9 - FEB 2014
B. Gabapentin postherpetic neuralgia. 2014 MED
C. Phenytoin BOARDS;
D. Duloxetine TOPNOTCH MD)
E. Levetiracetam

451 A 43 year-old male was diagnosed with Pulmonary Pyrazinamide provoke avute gouty arthritis by JULIET KRISTINE MIDTERM 2
Tuberculosis. He was given Anti-TB medications. inhibiting urate excretion. EVANGELISTA, EXAM - AUG
However, he developed hyperuricemia. Which among MD (TOP 9 - FEB 2014
these drugs inhibits the excretion of urate causing the 2014 MED
above findings: BOARDS;
A. Ethionamide TOPNOTCH MD)
B. Streptomycin
C. Ethambutol
D. Pyrazinamide
E. Rifampicin
452 A 28 year-old woman wanted to get pregnant. A Clomiphene citrate enhances probability of JULIET KRISTINE MIDTERM 2
substance that enhances probability of ovulation by ovulation by blocking the inhibitory effect of EVANGELISTA, EXAM - AUG
blocking the inhibitory effect of estrogen and thus estrogen and thus stimulating the release of MD (TOP 9 - FEB 2014
stimulating the release of gonadotrophin from the gonadotrophin from the pituitary. 2014 MED
pituitary is: BOARDS;
A. Tamoxifen TOPNOTCH MD)
B. Danazol
C. Clomiphene
D. Progesterone
E. Oxytocin

TOPNOTCH MEDICAL BOARD PREP PHARMACOLOGY SUPEREXAM Page 51 of 83


For inquiries visit www.topnotchboardprep.com.ph or email us at topnotchmedicalboardprep@gmail.com
TOPNOTCH MEDICAL BOARD PREP PHARMACOLOGY SUPEREXAM
For inquiries visit www.topnotchboardprep.com.ph or email us at topnotchmedicalboardprep@gmail.com
Item QUESTION EXPLANATION AUTHOR TOPNOTCH
# EXAM
453 Upon taking Penicillin, a 20 year-old male suddenly Epinehrine is a sympathomimetic, nonselective and JULIET KRISTINE MIDTERM 2
experienced respiratory distress. A drug would be direct-acting. It is helpful in patients with EVANGELISTA, EXAM - AUG
most helpful to treat the respiratory symptoms was anaphylactic shock. Respiratory distress is treated MD (TOP 9 - FEB 2014
given in this patient. In comparison to by Epinephrine's action on beta 2 receptors 2014 MED
norepinephrine, epinephrine has more effects on: resulting to bronchodilation. BOARDS;
A. alpha 1 receptors TOPNOTCH MD)
B. alpha 2 receptors
C. beta 1 receptors
D. beta 2 receptors
E. dopamine receptors
454 Warfarin is prescribed to a 40 year-old male. The Bleeding with warfarin can be reversed by JULIET KRISTINE MIDTERM 2
excessive anticoagulant effect results to bleeding in administration of Vitamin K, FFP and by EVANGELISTA, EXAM - AUG
this patient can be reversed by: discontinuance of the drug. MD (TOP 9 - FEB 2014
A. Administration of vitamin K 2014 MED
B. Administration of fresh frozen plasma BOARDS;
C. Discontinuance of the drug TOPNOTCH MD)
D. A and C
E. All of the above

455 An anticancer agent that is prescribed for melanoma: Carmustine is an alkylating agent, cell cycle non- JULIET KRISTINE MIDTERM 2
A. Carmustine specific that is given for melanoma. EVANGELISTA, EXAM - AUG
B. Cytarabine MD (TOP 9 - FEB 2014
C. Vincristine 2014 MED
D. Vinblastine BOARDS;
E. Cisplatin TOPNOTCH MD)

456 An antihelminthic agent for strongyloidiasis: Ivermectin is the drug of choice for strongyloidiasis. JULIET KRISTINE MIDTERM 2
A. Nifurtimox It intensifies GABA-mediated neurotransmission in EVANGELISTA, EXAM - AUG
B. Praziquantel nematodes and immobilizes parasites. MD (TOP 9 - FEB 2014
C. Ivermectin 2014 MED
D. Diethylcarbamazine BOARDS;
E. Niclosamide TOPNOTCH MD)

457 A 60 year-old female patient was diagnosed with Acrolein is the metabolite of Cyclophosphamide that JULIET KRISTINE MIDTERM 2
breast cancer. She was given an alkylating agent, causes hemorrhagic cystitis. EVANGELISTA, EXAM - AUG
Cyclophosphamide. Later in the treatment, she MD (TOP 9 - FEB 2014
developed hemorrhagic cystitis due to this toxic 2014 MED
metabolite: BOARDS;
A. Amifostine TOPNOTCH MD)
B. Acrolein
C. Mercaptopurine
D. Lomustine
E. Mesna
458 Chlorpromazine may be used not only in treating Chlorpromazine may be used not only in treating JULIET KRISTINE MIDTERM 2
schzophrenia but is also effective in: schzophrenia but is also effective in reducing nausea EVANGELISTA, EXAM - AUG
A. Reducing nausea and vomiting and vomiting. MD (TOP 9 - FEB 2014
B. Allergies 2014 MED
C. Hypertension BOARDS;
D. Treating bipolar disorders TOPNOTCH MD)
E. Sleep disorders

459 The best treatment option for an obese patient with Metformin is a biguanide which is the first-line JULIET KRISTINE MIDTERM 2
Type 2 Diabetes Mellitus is: treatment for Type 2 DM, also a drug of choice for EVANGELISTA, EXAM - AUG
A. Glibenclamide obese diabetics. MD (TOP 9 - FEB 2014
B. Metformin 2014 MED
C. Repaglinide BOARDS;
D. Acarbose TOPNOTCH MD)
E. Insulin

460 A 10 year-old child with asthma was prescribed with Montelukast blocks leukotriene receptor for JULIET KRISTINE MIDTERM 2
a drug which inhibits binding of leukotriene D4 to its leukotrienes C4, D4, E4 preventing airway EVANGELISTA, EXAM - AUG
receptor on target tissues. The drug prescribed is: inflammation and bronchoconstriction in asthmatic MD (TOP 9 - FEB 2014
A. Zileuton patients. 2014 MED
B. Ipratropium BOARDS;
C. Montelukast TOPNOTCH MD)
D. Omalizumab
E. Theophylline

461 The most dangerous hallucinogenic agent. LUISA BACK-UP


A. LSD SARANILLO, MD MIDTERM
B. phencyclidine (TOP 6 - FEB EXAM AUG
C. MDMA 2014 MED 2014 - FOR
D. cocaine BOARDS; INCLUSION IN
E. Marijuana TOPNOTCH MD) THE SAMPLEX

462 A 50 year old smoker, hypertensive patient LUISA BACK-UP


complained of 2 month cough and dyspnea. He was SARANILLO, MD MIDTERM
on Carvedilol for his hypertension. What is the side (TOP 6 - FEB EXAM AUG
effect of this drug that aggravates the condition. 2014 MED 2014 - FOR
A. bronchospasm BOARDS; INCLUSION IN
B. bronchoconstriction TOPNOTCH MD) THE SAMPLEX
C. tachycardia
D. tremor
E. all of the choices

TOPNOTCH MEDICAL BOARD PREP PHARMACOLOGY SUPEREXAM Page 52 of 83


For inquiries visit www.topnotchboardprep.com.ph or email us at topnotchmedicalboardprep@gmail.com
TOPNOTCH MEDICAL BOARD PREP PHARMACOLOGY SUPEREXAM
For inquiries visit www.topnotchboardprep.com.ph or email us at topnotchmedicalboardprep@gmail.com
Item QUESTION EXPLANATION AUTHOR TOPNOTCH
# EXAM
463 Mr. X was maintaining an unrecalled drug for the past LUISA BACK-UP
few months. He is now complaining of visual changes SARANILLO, MD MIDTERM
or "yellow halo vision". What is the drug that is (TOP 6 - FEB EXAM AUG
known to cause such effect? 2014 MED 2014 - FOR
A. amiodarone BOARDS; INCLUSION IN
B. ethambutol TOPNOTCH MD) THE SAMPLEX
C. digoxin
D. furosemide
E. Procainamide
464 A 45 year old male complained of chest pain upon drugs used in angina pectoris are the following: LUISA BACK-UP
exertion but relieved by rest. The doctor prescribed those that increase the oxygen supply are: nitrates SARANILLO, MD MIDTERM
him a beta blocker. What is the effect of beta blocker? and calcium blockers; those that decrease the (TOP 6 - FEB EXAM AUG
A. Increase oxygen supply oxygen demand are: beta blockers and still calcium 2014 MED 2014 - FOR
B. Decrease oxygen demand blockers BOARDS; INCLUSION IN
C. Increase oxygen demand TOPNOTCH MD) THE SAMPLEX
D. decrease oxygen supply
E. A and B

465 Monday disease is caused by occupational exposure LUISA BACK-UP


to: SARANILLO, MD MIDTERM
A. lead (TOP 6 - FEB EXAM AUG
B. dust 2014 MED 2014 - FOR
C. arsenic BOARDS; INCLUSION IN
D. nitrates TOPNOTCH MD) THE SAMPLEX
E. Gun powder

466 What is the predominant form of a weakly acidic drug The physiologic pH is at 7.35-7.45 which is above LUISA BACK-UP
with pKa of 5 at physiologic pH? the pKa of this drug 5. At pH above pKa, weakly SARANILLO, MD MIDTERM
A. Unprotonated charged acidic drug is predominantly unprotonated charged, (TOP 6 - FEB EXAM AUG
B. Protonated charged if it is below pKa, the predominant form is 2014 MED 2014 - FOR
C. Unprotonated uncharged protonated uncharged. BOARDS; INCLUSION IN
D. Protonated uncharged TOPNOTCH MD) THE SAMPLEX
E. None of the choices

467 It is the flow or transfer of a drug to the bloodstream. Absorption is the transfer of a drug to the LUISA BACK-UP
A. distribution bloodstream. Distribution is the entry of a drug to SARANILLO, MD MIDTERM
B. bioequivalence the target organ. Elimination is the termination of (TOP 6 - FEB EXAM AUG
C. absorption drug action. Bioavailability is the fraction of a drug 2014 MED 2014 - FOR
D. bioavailability that reaches the systemic circulation. BOARDS; INCLUSION IN
E. Elimination Bioequivalence is the comparability of 2 related TOPNOTCH MD) THE SAMPLEX
drugsnto achieve peak blood concetration.

468 What type of muscarinic receptors is used by the M1 - nerve endings, M2-heart, M3 - smooth muscle, LUISA BACK-UP
heart? glands, endothelium SARANILLO, MD MIDTERM
A. M1 (TOP 6 - FEB EXAM AUG
B. M2 2014 MED 2014 - FOR
C. M3 BOARDS; INCLUSION IN
D. N1 TOPNOTCH MD) THE SAMPLEX
E. N2

469 Rivastigmine is used for: Rivastigmine is a cholinomimetic drug used for LUISA BACK-UP
A. Motion sickness Alzheimer's disease. SARANILLO, MD MIDTERM
B. Myasthenia gravis (TOP 6 - FEB EXAM AUG
C. Schizophrenia 2014 MED 2014 - FOR
D. Parkinson's disease BOARDS; INCLUSION IN
E. Alzheimer's disease TOPNOTCH MD) THE SAMPLEX

470 The drug that is used for cardiogenic shock as a last LUISA BACK-UP
resort. SARANILLO, MD MIDTERM
A. epinephrine (TOP 6 - FEB EXAM AUG
B. dopamine 2014 MED 2014 - FOR
C. dobutamine BOARDS; INCLUSION IN
D. norepinephrine TOPNOTCH MD) THE SAMPLEX
E. Vasopressin

471 A 58 year old male hypertensive was diagnosed with Alpha1 blockers are antihypertensive that is also LUISA BACK-UP
benign prostatic hyperplasia. What is the used for BPH. Prazosin, tamsulosin, and terazosin SARANILLO, MD MIDTERM
antihypertensive drug that is the most selective for are alpha1 blockers, but the most selective for (TOP 6 - FEB EXAM AUG
prostatic smooth muscle? prostatic smooth muscle is tamsulosin. 2014 MED 2014 - FOR
A. prazosin BOARDS; INCLUSION IN
B. tamsulosin TOPNOTCH MD) THE SAMPLEX
C. losartan
D. metoprolol
E. Terazosin
472 A 75 year old male diabetic, hypertensive and a Spironolactone, triamterene, and amiloride are all LUISA BACK-UP
diagnosed case of CHF came in due to edema and potassium sparing diuretic. Triamterene and SARANILLO, MD MIDTERM
dyspnea. If the patient had hypokalemia, a potassium amiloride inhibit the ENaC of collecting duct, while (TOP 6 - FEB EXAM AUG
sparing diuretic is indicated. This potassium sparing spironolactone inhibits aldosterone receptor in 2014 MED 2014 - FOR
diuretic inhibits ENaC - epithelial sodium channel in collecting ducts. BOARDS; INCLUSION IN
collecting duct. TOPNOTCH MD) THE SAMPLEX
A. Amiloride
B. triamterene
C. spironolactone
D. all of the above
E. A and B only

TOPNOTCH MEDICAL BOARD PREP PHARMACOLOGY SUPEREXAM Page 53 of 83


For inquiries visit www.topnotchboardprep.com.ph or email us at topnotchmedicalboardprep@gmail.com
TOPNOTCH MEDICAL BOARD PREP PHARMACOLOGY SUPEREXAM
For inquiries visit www.topnotchboardprep.com.ph or email us at topnotchmedicalboardprep@gmail.com
Item QUESTION EXPLANATION AUTHOR TOPNOTCH
# EXAM
473 A 100kg male patient had elevated LDL and VLDL. LUISA BACK-UP
What vitamin is also used as antihyperlipidemic SARANILLO, MD MIDTERM
drug? (TOP 6 - FEB EXAM AUG
A. Vitamin A 2014 MED 2014 - FOR
B. pyridoxine BOARDS; INCLUSION IN
C. niacin TOPNOTCH MD) THE SAMPLEX
D. resin
E. Cholestyramine

474 What is the drug of choice for paroxysmal Esmolol, sotalol, verapamil are all used for the LUISA BACK-UP
supraventricular tachycardia? treatment of supraventricular tachycardia. For SARANILLO, MD MIDTERM
A. Esmolol paroxysmal supraventricular tachycardia, the drug (TOP 6 - FEB EXAM AUG
B. adenosine of choice is adenosine. 2014 MED 2014 - FOR
C. verapamil BOARDS; INCLUSION IN
D. Sotalol TOPNOTCH MD) THE SAMPLEX
E. Any of the choices

475 A 10 year old child, known asthmatic had episodes of Salbutamol is the druig of choice for acute asthma LUISA BACK-UP
nocturnal asthma attacks approximately 2-3x/week. attack, while aminophylline is used for prophylaxis SARANILLO, MD MIDTERM
In order to prevent subsequent attack, what drug is against nocturnal attacks. (TOP 6 - FEB EXAM AUG
used for prophylaxis against nocturnal attack? 2014 MED 2014 - FOR
A. Salbutamol + Ipratropium BOARDS; INCLUSION IN
B. Salbutamol TOPNOTCH MD) THE SAMPLEX
C. salmeterol
D. cromolyn
E. Aminophylline

476 A 4 year old child, febrile at 38deg. C was given Paracetamol or acetaminophen selectively inhibits LUISA BACK-UP
Paracetamol at 15mkd. What is the mechanism of COX-3. COX-2 selective inhibitors are coxib such as SARANILLO, MD MIDTERM
action of this drug? celecoxib. Irreversible COX-1 and 2 inhibitor is (TOP 6 - FEB EXAM AUG
A. COX-2 selective inhibitor aspirin. TNF - α inhibitors are infliximab, 2014 MED 2014 - FOR
B. IL-1 and IL-6 inhibitors adalimumab and etanercept. BOARDS; INCLUSION IN
C. COX-3 selective inhibitor TOPNOTCH MD) THE SAMPLEX
D. Irreversible COX-1 and COX-2 inhibitor
E. TNF - α inhibitor

477 One of the choices does not belong to the third All of the choices except cefoxitin are all third LUISA BACK-UP
generation cephalosporins. generation cephalosporin. Cefoxitin is a second SARANILLO, MD MIDTERM
A. cefotaxime generation. (TOP 6 - FEB EXAM AUG
B. ceftazidime 2014 MED 2014 - FOR
C. ceftriaxone BOARDS; INCLUSION IN
D. cefoxitin TOPNOTCH MD) THE SAMPLEX
E. Ceftizoxime

478 A 32 year old G2P1 mother delivered a live term baby The physical description of the neonate is consistent LUISA BACK-UP
Boy with an upturned nose, mild midfacial with fetal hydantoin syndrome which is the side SARANILLO, MD MIDTERM
hypoplasia, long upper lip, and lower distal digital effect of phenytoin (TOP 6 - FEB EXAM AUG
hypoplasia. Upon review of maternal history, the 2014 MED 2014 - FOR
mother was taking an anti-epileptic drug. The drug BOARDS; INCLUSION IN
that is most likely used was: TOPNOTCH MD) THE SAMPLEX
A. phenytoin
B. valproic acid
C. diazepam
D. carbamazepine
E. lamotrigine
479 A 50 year old obese patient was diagnosed to have Metformin is the first line treatment for type 2 DM LUISA BACK-UP
DM type 2. There were no other comorbidities. What and is the drug of choice for obese patients. It is SARANILLO, MD MIDTERM
hypoglycemic drug should you prescribed with this contraindicated in patients with renal disease, (TOP 6 - FEB EXAM AUG
patient? hepatic disease, and alcoholism 2014 MED 2014 - FOR
A. Insulin BOARDS; INCLUSION IN
B. Gliclazide TOPNOTCH MD) THE SAMPLEX
C. Metformin
D. Pioglitazone
E. Sitagliptin
480 Penicillin is the drug of choice for syphilis. It acts Penicillin inhibits transpeptidase enzyme needed for LUISA BACK-UP
through: the bacterial cell wall synthesis. Vancomycin inhibits SARANILLO, MD MIDTERM
A. Inhibition of transpeptidase D-ala D ala synthase. Caspofungin inhibits glucan (TOP 6 - FEB EXAM AUG
B. Inhibition of topoisomerase II synthase. Flouroquinolone inhibits topoisomerase II 2014 MED 2014 - FOR
C. Inhibition of glucan synthase and IV. BOARDS; INCLUSION IN
D. Inhibition of D-ala-D-ala terminus TOPNOTCH MD) THE SAMPLEX
E. None of the choices

481 Marc, an asthmatic, is on theophylline. He has a Theophylline has a narrow therapeutic window and ANGELIS FINAL EXAM -
number of other medical conditions. Which of the drug interactions can lead to severe morbidity. ANDREA COCOS, AUG 2014
following medications may be safely administered Erythromycin decreases the clearance of MD (TOP 1 - FEB
with theophylline? theophylline and increases its serum levels. 2014 MED
A. Penicillin Phenytoin and rifampicin have the oppositve effect. BOARDS;
B. Erythromycin Advice: Memorize the inducers and inhibitors of the TOPNOTCH MD)
C. Phenytoin CYP450 system!
D. Rifampicin
482 What is the maximum dose of Vincristine? SIMILAR TO PREVIOUS BOARD EXAM ANGELIS FINAL EXAM -
A. 100 micrograms CONCEPT/PRINCIPLE ANDREA COCOS, AUG 2014
B. 200 micrograms MD (TOP 1 - FEB
C. 1 milligram 2014 MED
D. 2 milligrams BOARDS;
TOPNOTCH MD)

TOPNOTCH MEDICAL BOARD PREP PHARMACOLOGY SUPEREXAM Page 54 of 83


For inquiries visit www.topnotchboardprep.com.ph or email us at topnotchmedicalboardprep@gmail.com
TOPNOTCH MEDICAL BOARD PREP PHARMACOLOGY SUPEREXAM
For inquiries visit www.topnotchboardprep.com.ph or email us at topnotchmedicalboardprep@gmail.com
Item QUESTION EXPLANATION AUTHOR TOPNOTCH
# EXAM
483 A pregnant patient complains of multiple external The most common treatments for genital warts in ANGELIS FINAL EXAM -
genital warts. Which of the following agents can be pregnancy are cryotherapy, laser removal, and ANDREA COCOS, AUG 2014
used in this patient? trichloroacetic acid. The rest of the choices are MD (TOP 1 - FEB
A. Podofilox solution under Pregnancy Category C. 2014 MED
B. Imiquimod cream BOARDS;
C. Trichloroacetic acid TOPNOTCH MD)
D. Sinecatechins ointment

484 The drug of choice for leptospirosis is: SIMILAR TO PREVIOUS BOARD EXAM ANGELIS FINAL EXAM -
A. Doxycycline CONCEPT/PRINCIPLE The rest of the choices are ANDREA COCOS, AUG 2014
B. Penicillin alternate drug choices for leptospirosis. MD (TOP 1 - FEB
C. Ceftriaxone 2014 MED
D. Erythromycin BOARDS;
TOPNOTCH MD)
485 A 23-year-old presents with abdominal pain, Ciprofloxacin is the recommended empiric ANGELIS FINAL EXAM -
tenesmus, and mucoid diarrhea with frank blood. treatment for patients with acute dysenteriae. ANDREA COCOS, AUG 2014
Which is the best empiric treatment? SIMILAR TO PREVIOUS BOARD EXAM MD (TOP 1 - FEB
A. Ciprofloxacin CONCEPT/PRINCIPLE 2014 MED
B. Metronidazole BOARDS;
C. Amoxicillin TOPNOTCH MD)
D. Cotrimoxazole

486 The most powerful hallucinogen known to man is: SIMILAR TO PREVIOUS BOARD EXAM ANGELIS FINAL EXAM -
A. LSD CONCEPT/PRINCIPLE ANDREA COCOS, AUG 2014
B. marijuana MD (TOP 1 - FEB
C. methamphetamine 2014 MED
D. phencyclidine BOARDS;
TOPNOTCH MD)
487 It is the only licensed chemotherapy drug for SIMILAR TO PREVIOUS BOARD EXAM ANGELIS FINAL EXAM -
advanced melanoma: CONCEPT/PRINCIPLE There was quite a number of ANDREA COCOS, AUG 2014
A. adriamycin questions on oncology drugs. MD (TOP 1 - FEB
B. bleomycin 2014 MED
C. cisplatin BOARDS;
D. Dacarbazine TOPNOTCH MD)
488 Plasma magnesium levels should be maintained at ANGELIS FINAL EXAM -
this therapeutic range to prevent eclamptic ANDREA COCOS, AUG 2014
convulsions: MD (TOP 1 - FEB
A. 2-4 meq/L 2014 MED
B. 4-7 meq/L BOARDS;
C. 8-10 meq/L TOPNOTCH MD)
D. 10-12 meq/L

489 This is the only ligand-gated serotonin receptor: ANGELIS FINAL EXAM -
A. 5HT1 ANDREA COCOS, AUG 2014
B. 5HT2 MD (TOP 1 - FEB
C. 5HT3 2014 MED
D. 5HT4 BOARDS;
TOPNOTCH MD)
490 This autonomic drug should be given with caution in Reserpine which crosses the blood brain barrier is ANGELIS FINAL EXAM -
patients with depressive disorders: notorious for causing depression and increasing ANDREA COCOS, AUG 2014
A. guanethidine suicidal tendencies. MD (TOP 1 - FEB
B. metyrosine 2014 MED
C. vesamicol BOARDS;
D. Reserpine TOPNOTCH MD)
491 If a patient has COPD, which of the following beta- Atenolol, metoprolol, esmolol, betaxolol, and ANGELIS FINAL EXAM -
blockers would be more preferrable to use, acebutolol (A-BEAM) are more beta1-selective ANDREA COCOS, AUG 2014
theoretically? hence are advantageous in patients with comorbid MD (TOP 1 - FEB
A. propranolol pulmonary disease. 2014 MED
B. carvedilol BOARDS;
C. labetalol TOPNOTCH MD)
D. atenolol

492 An 8-year-old being treated with a combination of Major toxicities of methotrexate include ANGELIS FINAL EXAM -
chemotherapeutic agents had difficulty eating and gastrointestinal mucositis, bone marrow ANDREA COCOS, AUG 2014
drinking due to red, inflamed sores in her mouth and suppression, skin erythema and hepatic dysfunction. MD (TOP 1 - FEB
esophagus. Which of the following antineoplastic SIMILAR TO PREVIOUS BOARD EXAM 2014 MED
agents is the most likely etiology? CONCEPT/PRINCIPLE BOARDS;
A. Methotrexate TOPNOTCH MD)
B. Vinblastine
C. Doxorubicin
D. Prednisone
493 A young boy with G6PD deficiency would note tea- ANGELIS FINAL EXAM -
colored urine after the intake of which drug/s: ANDREA COCOS, AUG 2014
A. primaquine MD (TOP 1 - FEB
B. cotrimoxazole 2014 MED
C. nitrofurantoin BOARDS;
D. All of the above TOPNOTCH MD)
494 This drug activates peroxisome proliferator-activated Do not confuse with the mechanism of action of ANGELIS FINAL EXAM -
receptors, a group of nuclear receptors, with greatest fibrates which is activation of the nuclear ANDREA COCOS, AUG 2014
specificity to PPAR-γ receptors: transcription factor PPAR-α. MD (TOP 1 - FEB
A. pioglitazone 2014 MED
B. gemfibrozil BOARDS;
C. sitagliptin TOPNOTCH MD)
D. Exenatide

TOPNOTCH MEDICAL BOARD PREP PHARMACOLOGY SUPEREXAM Page 55 of 83


For inquiries visit www.topnotchboardprep.com.ph or email us at topnotchmedicalboardprep@gmail.com
TOPNOTCH MEDICAL BOARD PREP PHARMACOLOGY SUPEREXAM
For inquiries visit www.topnotchboardprep.com.ph or email us at topnotchmedicalboardprep@gmail.com
Item QUESTION EXPLANATION AUTHOR TOPNOTCH
# EXAM
495 Constipation is one of the major side effects of this Aluminum hydroxide causes constipation (ALang ANGELIS FINAL EXAM -
drug: tae). Magnesium hydroxide causes diarrhea ANDREA COCOS, AUG 2014
A. Magnesium hydroxide (MaGtatae). Colchicine also causes diarrhea if taken MD (TOP 1 - FEB
B. Aluminum hydroxide in excess. 2014 MED
C. Omeprazole BOARDS;
D. Colchicine TOPNOTCH MD)
496 A patient presents with petit mal seizures. The drug Petit mal seizures, also known as absence seizures, ANGELIS FINAL EXAM -
of choice for this type of seizure is: involves a brief, sudden lapse of consciousness. The ANDREA COCOS, AUG 2014
A. phenytoin DOC is ethosuximide. MD (TOP 1 - FEB
B. topiramate 2014 MED
C. ethosuximide BOARDS;
D. lamotrigine TOPNOTCH MD)
497 A psychotic patient on haloperidol presents with Tardive dyskinesia is also one of the toxicities of ANGELIS FINAL EXAM -
rigidity, hyperpyrexia and autonomic instability. This antipsychotics aside from NMS. It is a condition ANDREA COCOS, AUG 2014
condition is known as: where the patient presents with stereotypic oral- MD (TOP 1 - FEB
A. Serotonin syndrome facial movements. 2014 MED
B. Neuroleptic malignant syndrome BOARDS;
C. Tardive dyskinesia TOPNOTCH MD)
D. Malignant hyperthermia

498 This diuretic can also be used in the treatment of Technically, mannitol can also be used but ANGELIS FINAL EXAM -
glaucoma: acetazolamide is a better answer. SIMILAR TO ANDREA COCOS, AUG 2014
A. acetazolamide PREVIOUS BOARD EXAM CONCEPT/PRINCIPLE MD (TOP 1 - FEB
B. hydrochlorothiazide 2014 MED
C. mannitol BOARDS;
D. amiloride TOPNOTCH MD)
499 It is a monoclonal anti-IgE antibody used in the Trastuzumab aka herceptin is a monoclonal ANGELIS FINAL EXAM -
treatment of asthma resistant to inhaled steroids and antibody that interferes with the HER2/neu ANDREA COCOS, AUG 2014
long-acting B2-agonists: receptor, rituximab is against the protein CD20 on B MD (TOP 1 - FEB
A. trastuzumab cells, and infliximab is against TNF-alpha. 2014 MED
B. rituximab BOARDS;
C. infliximab TOPNOTCH MD)
D. omalizumab

500 This drug inhibits platelet aggregation by irreversibly Cilostazol is a phosphodiesterase III inhibitor, ANGELIS FINAL EXAM -
blocking ADP receptors: tirofiban binds to GP IIb/IIIa receptor, and alteplase ANDREA COCOS, AUG 2014
A. cilostazol is a thrombolytic (not an anti-platelet) which MD (TOP 1 - FEB
B. tirofiban converts plasminogen to plasmin. 2014 MED
C. alteplase BOARDS;
D. Ticagrelor TOPNOTCH MD)
501 These drugs act by preventing the binding of the Katzung 10th ed, p. 11 - Pharmacologic anatgonists JAN CHARMAINE BACK-UP
agonist molecule to the receptor but do not activate bind to the receptor preventing agonist binding PALOMAR, MD MIDTERM
generation of a signal: thereby blocking the agonist's biologic actions. (TOP 9 - FEB EXAM AUG
A. Pharmacologic antagonists 2014 MED 2014
B. Inverse agonists BOARDS;
C. Competitive agonists TOPNOTCH MD)
D. Partial agonists
E. Orphan drug

502 This route of drug administration is the most Katzung 10th ed, p. 41 JAN CHARMAINE BACK-UP
convenient, however bioavailability may be less than PALOMAR, MD MIDTERM
100% because of incomplete absorption and first- (TOP 9 - FEB EXAM AUG
pass elimination. 2014 MED 2014
A. Intramuscular BOARDS;
B. Subcutaneous TOPNOTCH MD)
C. Oral
D. Transdermal
E. All of the above
503 Loca is a 23 year old G4P4 (4004) who has been on Rifampin revs up the cytochrome P450 enzyme JAN CHARMAINE BACK-UP
oral contraceptives for 8 months now. What is the system thereby enhancing the metabolism and PALOMAR, MD MIDTERM
expected drug-drug interaction If she was to start on hence the elimination of the oral contraceptive the (TOP 9 - FEB EXAM AUG
an anti-tuberculosis drug that acts by inhibitting RNA patient is taking resulting to its significantly lower 2014 MED 2014
synthesis and which imparts orange color to urine serum level. Katzung 10th ed, p.774. BOARDS;
and sweat? TOPNOTCH MD)
A. Enhanced efficacy of the oral contraceptive she is
taking
B. Enhanced potency of the oral contraceptive she is
taking
C. reduced activity of the cytochrome P450 isoforms
D. Increased elimination of the anti-tuberculosis
drug
E. Significantly lower serum level of the oral
contraceptive
504 It is during this phase of drug development and Phase 1 trials determine the probable limits of the JAN CHARMAINE BACK-UP
testing that the effects of the drug as a function of safe clinical dosage range in a small number of PALOMAR, MD MIDTERM
dosage are established in a small number of healthy healthy volunteers. In Phase 2, the drug is studied in (TOP 9 - FEB EXAM AUG
volunteers. patients with the target disease to determine its 2014 MED 2014
A. Phase 1 efficacy. In phase 3, the drug is evaluated in larger BOARDS;
B. Phase 2 numbers of patients with the target disease to TOPNOTCH MD)
C. Phase 3 further establish safety and efficacy. Phase 4
D. Phase 4 constitutes monitoring the safety of the new drug
E. Phase 5 under actual conditions of use in large number of
patients. Katzung 10th ed., 70-71.

TOPNOTCH MEDICAL BOARD PREP PHARMACOLOGY SUPEREXAM Page 56 of 83


For inquiries visit www.topnotchboardprep.com.ph or email us at topnotchmedicalboardprep@gmail.com
TOPNOTCH MEDICAL BOARD PREP PHARMACOLOGY SUPEREXAM
For inquiries visit www.topnotchboardprep.com.ph or email us at topnotchmedicalboardprep@gmail.com
Item QUESTION EXPLANATION AUTHOR TOPNOTCH
# EXAM
505 Which of the following drugs lower blood pressure by Metyrosine inhibits tyrosine hydroxylase which is JAN CHARMAINE BACK-UP
preventing normal physiologic release of responsible for the conversion of Tyrosine to Dopa, PALOMAR, MD MIDTERM
norepinephrine from postganglionic sympathetic the rate-limiting step in catecholamine synthesis. (TOP 9 - FEB EXAM AUG
neurons? Cocaine blocks re-uptake of NE from the synaptic 2014 MED 2014
A. Metyrosine cleft thereby prolonging its effect. Reserpine blocks BOARDS;
B. Cocaine dopamine transport into the vesicle. Botulinum TOPNOTCH MD)
C. Reserpine blocks Acetylcholine release from the nerve terminal
D. Botulinum toxin into the junctional cleft. Study figures 6-3 and 6-4 of
E. Guanethidine Katzung 10th ed, p. 79 and 81.
506 A tricyclic antidepresssant drug long used to reduce Katzung 115: Ipratropium-is an anti-muscarinic JAN CHARMAINE BACK-UP
incontinence in institutionalized elderly patients and drug used as an inhalational drug for asthma. PALOMAR, MD MIDTERM
in children with bedwetting problems because of its Scopolamine- is an antimuscarinic drug used to (TOP 9 - FEB EXAM AUG
strong anti-muscarinic actions: remedy seasickness. Oxybutinin- is an 2014 MED 2014
A. Ipratropium antimuscarinic selective for M3 receptors used to BOARDS;
B. Imipramine relieve bladder spasm after urologic surgery and is TOPNOTCH MD)
C. Scopolamine also valuable in reducing involuntary voiding in
D. Oxybutinin patients with neurologic disease. Fluoxetine is an
E. Fluoxetine SSRI.
507 Which of the following statements regarding diuretics The collecting tubule and not the distal tubule is the JAN CHARMAINE BACK-UP
is not true? final site NaCl reabsorption. It is responsible for PALOMAR, MD MIDTERM
A. Diuretics lower blood pressure primarily by tight regulation of body fluid volume and for (TOP 9 - FEB EXAM AUG
depleting body sodium stores. determining the final sodium concentration of the 2014 MED 2014
B. The most common adverse effect of thiazide urine. Katzung 10th ed., 239. BOARDS;
diuretics is potassium depletion. TOPNOTCH MD)
C. The intercalated cells in the collecting tubule are
the primary sites of H+ secretion.
D. The distal convoluted tubule is the final site of NaCl
reabsorption and is responsible for the final sodium
concentration of the urine.
E. The thick ascending limb of the loop of Henle is the
"diluting segment".
508 Which of the following drug acts by selectively Katzung 10th ed., 169. Metoprolol, Esmolol, JAN CHARMAINE BACK-UP
inhibitting stimulation of B1 adrenoceptor and may Atenolol, Acebutolol, Betaxolol, Bisoprolol are PALOMAR, MD MIDTERM
be advantageous in treating hypertensive patients selective B1-blockers. Propranolol and Nadolol are (TOP 9 - FEB EXAM AUG
who also suffer from asthma, diabetes, or peripheral non-selective B-blockers. Labetalol and Carvedilol 2014 MED 2014
vascular disease? have combined alpha and b-blocking activity. BOARDS;
A. Nadolol TOPNOTCH MD)
B. Metoprolol
C. Propranolol
D. Labetalol
E. Carvedilol
509 It is a very efficacious orally active arteriolar Katzung 10th ed, 173. JAN CHARMAINE BACK-UP
vasodilator that is also available in topical PALOMAR, MD MIDTERM
preparations used to stimulate hair growth for (TOP 9 - FEB EXAM AUG
correction of baldness. 2014 MED 2014
A. Minoxidil BOARDS;
B. Sodium Nitroprusside TOPNOTCH MD)
C. Hydralazine
D. Diazoxide
E. Fenoldopam
510 A Class I-B anti-arrhythmic drug which has a high Katzung 10th ed., 225 Quinidine- Class IA, associated JAN CHARMAINE BACK-UP
degree of effectiveness in arrhythmias associated with cinchonism; Flecainide- Class IC, very effective PALOMAR, MD MIDTERM
with acute myocardial infarction. in suppressing premature vebtricular contractions; (TOP 9 - FEB EXAM AUG
A. Quinidine Propranolol-Class 2, Amiodarone-broad spectrum of 2014 MED 2014
B. Flecainide actions BOARDS;
C. Propranolol TOPNOTCH MD)
D. Lidocaine
E. Amiodarone

511 A potent antiandrogen that is usually coadministered Katzung 10th ed., 679. this is SIMILAR TO PREVIOUS JAN CHARMAINE BACK-UP
with leuprolide that has been used in the treatment of BOARD EXAM CONCEPT/PRINCIPLE PALOMAR, MD MIDTERM
prostatic carcinoma that frequently causes (TOP 9 - FEB EXAM AUG
gynecomastia and occassionally mild reversible 2014 MED 2014
hepatic toxicity: BOARDS;
A. Goserelin TOPNOTCH MD)
B. Spironolactone
C. Cyproterone
D. Ketoconazole
E. Flutamide
512 Disulfiram causes extreme discomfort in patients who Katzung 10th ed., 371. this was SIMILAR TO JAN CHARMAINE BACK-UP
drink alcoholic beverages by inhibitting which PREVIOUS BOARD EXAM CONCEPT/PRINCIPLE PALOMAR, MD MIDTERM
enzyme resulting to accumulation of acetaldehyde (TOP 9 - FEB EXAM AUG
causing flushing, throbbing headache, nausea, 2014 MED 2014
vomiting, sweating, hypotension and confusion BOARDS;
within few hours of drinking alcohol: TOPNOTCH MD)
A. alcohol dehydrogenase
B. aldehyde dehydrogenase
C. alcohol decarboxylase
D. aldehyde decarboxylase
E. none of the above

TOPNOTCH MEDICAL BOARD PREP PHARMACOLOGY SUPEREXAM Page 57 of 83


For inquiries visit www.topnotchboardprep.com.ph or email us at topnotchmedicalboardprep@gmail.com
TOPNOTCH MEDICAL BOARD PREP PHARMACOLOGY SUPEREXAM
For inquiries visit www.topnotchboardprep.com.ph or email us at topnotchmedicalboardprep@gmail.com
Item QUESTION EXPLANATION AUTHOR TOPNOTCH
# EXAM
513 The following helminths are matched with their The drug of choice in strongylodiasis and JAN CHARMAINE BACK-UP
respective drug of choice except: onchocerciasis that acts by intensifying GABA- PALOMAR, MD MIDTERM
A. Fasciola hepatica - Bithionol mediated transmission of signals in peripheral (TOP 9 - FEB EXAM AUG
B. Echinococcus granulosus - Albendazole nerves resulting to paralysis of the worms is 2014 MED 2014
C. Stongyloides stercoralis - Thiabendazole Ivermectin. Thiabendazole is an alternative drug BOARDS;
D. Schistosoma mansoni - Praziquantel only. Katzung 10th ed., 870; See also p.868 for the TOPNOTCH MD)
E. Dracunculus medinensis - Metronidazole table of drug of choice for other helminthic
infections
514 What is the drug of choice for the eradication of Primaquine is the only available agent active against JAN CHARMAINE BACK-UP
dormant liver forms of Plasmodium vivax and the dormant hypnozoite stages of p. vivax and p. PALOMAR, MD MIDTERM
Plasmodium ovale? ovale. (radical cure) Katzung 10th ed., 852 (TOP 9 - FEB EXAM AUG
A. Quinidine 2014 MED 2014
B. Primaquine BOARDS;
C. Mefloqione TOPNOTCH MD)
D. Chloroquine
E. Lumefantrine

515 This halogen in a 1:20,000 solution is bactericidal in 1 Katzung 10th ed., 822. Phenolic disinfectants are the JAN CHARMAINE BACK-UP
minute and kills spores in 15 minutes. It is the most oldest of the surgical antiseptics. They are used for PALOMAR, MD MIDTERM
active antiseptic for intact skin: hard surface decontamination in hospitals and labs. (TOP 9 - FEB EXAM AUG
A. Phenol They are no longer used as a disinfectant because of 2014 MED 2014
B. Hypochlorus acid its corrosive effect on tissues, its toxicity when BOARDS;
C. Sodium hypochlorite absorbed (hyperbilirubinemia in newborns) and its TOPNOTCH MD)
D. Iodine carcinogenic effect. Sodium hypochlorite (household
E. Hexachlorophene bleach) in 1:10 dilution which provides 5000 ppm of
available chlorine is recommended by CDC for
disinfection of blood spills and to kill spores.
516 This antiviral agent against Influenza A acts by katzung 10th ed., 815. JAN CHARMAINE BACK-UP
blocking the M2 proton ion channel of the virus PALOMAR, MD MIDTERM
particle and by inhibitting uncoating of the viral RNA (TOP 9 - FEB EXAM AUG
within infected host cells, thus preventing its 2014 MED 2014
replication. BOARDS;
A. Amantadine TOPNOTCH MD)
B. Ribavirin
C. Oseltamivir
D. Entecavir
E. None of the abpve
517 This drug is a derivative of rifamycin which is Katzung 10th ed., 777. Rifabutin is a less potent JAN CHARMAINE BACK-UP
indicated in place of rifampin for treatment of inducer of the cytochrome P450 enzyme (compared PALOMAR, MD MIDTERM
tuberculosis in HIV-infected patients who are to Rifampin) resulting to slower elimination and (TOP 9 - FEB EXAM AUG
receiving concurrent protease inhibitors: longer halflife of protease inhibitors used in treating 2014 MED 2014
A. Rifapentine HIV patients is cases of HIV with concurrent BOARDS;
B. Rifabutin tuberculosis. Rifapentine like Rifampin is a potent TOPNOTCH MD)
C. Ritonavir inducer of cytochrome p450 enzymes and should
D. Rifadin not be used to treat HIV-infected patients because of
E. Rimactane an unaccepptably high relapse rate with rifampin-
resistant organisms. Rtonavir is a protease inhibitor.
Rifadin and Rimactane are brand names of Rifampin.
518 A 35 year old woman came to your clinic because of Quinolones act by blocking bacterial DNA synthesis JAN CHARMAINE BACK-UP
dysuria. She has been experiencing burning on by inhibitting bacterial topoisomerase II and IV. PALOMAR, MD MIDTERM
urination with increased frequency and urgency to Moxifloxacin is the only non-renally cleared (TOP 9 - FEB EXAM AUG
urinate for 5 days now. Urinalysis revealed WBC of fluoroqionolone and thus achieves low urinary level 2014 MED 2014
20-30/hpf, bacteria=moderate, nitrite=positive. You making it ineffective for cases of UTI. Katzung 10th BOARDS;
may prescribe the following drugs to her except for: ed., 768. TOPNOTCH MD)
A. Ciprofloxacin
B. Levofloxacin
C. Ofloxacin
D. Moxifloxacin
E. None of the above
519 The following are true of sulfonamides except: It is interesting that rickettsiae are not inhibitted by JAN CHARMAINE BACK-UP
A. Sulfonamides are structural analogs of p- sulfonamides but are actually stimulated in their PALOMAR, MD MIDTERM
aminobenzoic acid and act by inhibitting growth. Katzung 10th ed., 763. (TOP 9 - FEB EXAM AUG
dihydropteroate synthase and folate production. 2014 MED 2014
B. Sulfonamides are very effective against BOARDS;
rickettsiae. TOPNOTCH MD)
C. Combination of a sulfonamide with an inhibitor of
dihydrofolate reductase provides synergistic activity
because of sequential inhibition of folate synthesis.
D. trimethoprim-sulfamethoxazole is the drug of
choice for infections such as Pneumocystis jiroveci
pneumonia.
E. Sulfadiazine with pyrimethamine is first-line
therapy for treatment of acute toxoplasmosis.
520 A 45 year old soldier has been maintaining on This is a case of serotonin syndrome which is a JAN CHARMAINE BACK-UP
fluoxetine for his major depressive disorder. Which of condition precipitated when MAO inhibitors are PALOMAR, MD MIDTERM
the following drugs can precipitate a potentially fatal given with serotonin agaonists, especially (TOP 9 - FEB EXAM AUG
syndrome of hypertension, hyperreflexia, tremor, antidepressants of the SSRI class. Amytriptyline and 2014 MED 2014
clonus, hyperhtermia, diarrhea, mydriasis and Imipramine are TCAs. Fluoxetine, Sertraline BOARDS;
agitation within hours when taken consurrently with and Citalopram are SSRIs. Phenelzine is a MAO TOPNOTCH MD)
fluoxetine? inhibitor. Katzung 10th ed., 267.
A. Amytriptyline
B. Imipramine
C. Sertraline
D. Citalopram
E. Phenelzine

TOPNOTCH MEDICAL BOARD PREP PHARMACOLOGY SUPEREXAM Page 58 of 83


For inquiries visit www.topnotchboardprep.com.ph or email us at topnotchmedicalboardprep@gmail.com
TOPNOTCH MEDICAL BOARD PREP PHARMACOLOGY SUPEREXAM
For inquiries visit www.topnotchboardprep.com.ph or email us at topnotchmedicalboardprep@gmail.com
Item QUESTION EXPLANATION AUTHOR TOPNOTCH
# EXAM
521 A 37 year old man comes to the physician because of MIGUEL RAFAEL MIDTERM 1
a 1 week history of pain with swallowing. He received RAMOS, MD (TOP EXAM - FEB
the diagnosis of AIDS 4 years ago, but has not been 3 - FEB 2012 2013
able to tolerate highly active antiretroviral therapy. MED BOARDS;
He currently takes TMP-SMX. Vital signs are within TOPNOTCH MD)
normal limits. Examination shows a few white
plaques over the pharynx. CD4+ count is 50/mm3
and plasma HIV viral load is 50000 copies/mL. Which
of the following is the most appropriate
pharmacotherapy?
A) Acyclovir
B) Amphotericin B
C) Fluconazole
D) Foscarnet
522 A 72 year old man is brought to the physician by his MIGUEL RAFAEL MIDTERM 1
daughter because of a 7 month history of difficulty RAMOS, MD (TOP EXAM - FEB
with memory. He frequently loses his keys and misses 3 - FEB 2012 2013
appointments and regular family dinners. He lives MED BOARDS;
alone, but recently family members had to drive him TOPNOTCH MD)
on errands and help him with daily tasks at home. He
has not had difficulty sleeping and does not use illicit
drugs. Mental status examination shows an irritable
mood and labile affect. There is no evidence of
depressed mood, anxiety, or hallucinations. Which of
the following is the most appropriate
pharmacotherapy for this patient?
A) Alprazolam
B) Donepezil
C) Dextroamphetamine
D) imipramine
523 A 47 year old woman comes to the physician for a MIGUEL RAFAEL MIDTERM 1
routine health maintenance examination. She has a 2 RAMOS, MD (TOP EXAM - FEB
year history of venous insufficiency and noted daily 3 - FEB 2012 2013
swelling of her ankles that worsens in the evening. MED BOARDS;
She takes no medications. Her pulse is 80 bpm, and TOPNOTCH MD)
blood pressure is 160/100 mmHg. Cardiopulmonary
examination shows no abnormalities. Examination of
the lower extremities shows 2+ edema and increased
pigmentation .Which of the following
antihypertensive medications would most likely
exacerbate this patient’s swelling?
A) Atenolol
B) Lisinopril
C) Losartan
D) Nifedipine
524 A 55-year-old man has had crushing substernal chest MIGUEL RAFAEL MIDTERM 1
pain on exertion over the past 6 weeks. He had a RAMOS, MD (TOP EXAM - FEB
myocardial infarction 2 months ago. He takes 3 - FEB 2012 2013
nitroglycerin as needed and one aspirin daily. He has MED BOARDS;
smoked two packs of cigarettes daily for 30 years. TOPNOTCH MD)
Examination shows normal heart sounds and no
carotid or femoral bruits. Treatment with a β-
adrenergic blocking agent is most likely to improve
his symptoms due to which of the following
mechanisms?
A) Decreasing diastolic relaxation
B) Decreasing myocardial contractility
C) Dilating the coronary arteries
D) Peripheral vasodilation
525 A 72-year-old man comes for a routine follow-up COPD and Peripheral arterial disease >> MIGUEL RAFAEL MIDTERM 1
examination. He has chronic obstructive pulmonary contraindications for Beta-Blockers RAMOS, MD (TOP EXAM - FEB
disease treated with β-adrenergic agonists and 3 - FEB 2012 2013
ipratropium by metered-dose inhaler and mild MED BOARDS;
arterial insufficiency of the lower extremities treated TOPNOTCH MD)
with aspirin. His blood pressure is 160/60 mm Hg,
pulse is 70/min, and respirations are 12/min.
Funduscopic examination shows arteriovenous
nicking. Pedal pulses are decreased bilaterally. Which
of the following antihypertensive drugs is most likely
to cause adverse effects in this patient?
A) α2-Adrenergic agonist
B) α-Adrenergic blocking agent
C) β-Adrenergic blocking agent
D) Angiotensin-converting enzyme (ACE) inhibitor

TOPNOTCH MEDICAL BOARD PREP PHARMACOLOGY SUPEREXAM Page 59 of 83


For inquiries visit www.topnotchboardprep.com.ph or email us at topnotchmedicalboardprep@gmail.com
TOPNOTCH MEDICAL BOARD PREP PHARMACOLOGY SUPEREXAM
For inquiries visit www.topnotchboardprep.com.ph or email us at topnotchmedicalboardprep@gmail.com
Item QUESTION EXPLANATION AUTHOR TOPNOTCH
# EXAM
526 A 17-year-old boy is brought to the emergency MIGUEL RAFAEL MIDTERM 1
department by his parents because of bizarre RAMOS, MD (TOP EXAM - FEB
behavior for 6 hours. Last night he was out with 3 - FEB 2012 2013
friends, and since returning, he has been confused MED BOARDS;
and has "trashed" his room. His blood pressure is TOPNOTCH MD)
165/95 mm Hg. He is hypervigilant, has little
spontaneous speech, and is disoriented to place and
time. He appears catatonic but abruptly becomes
assaultive two times and needs to be restrained.
Which of the following is the most likely substance
taken?
A) Cocaine
B) Ecstasy
C) LSD
D) PCP
527 A 72-year-old man comes to the physician because of MIGUEL RAFAEL MIDTERM 1
a 2-month history of urination twice nightly and RAMOS, MD (TOP EXAM - FEB
occasional urinary frequency and urgency. He has a 3 - FEB 2012 2013
15-year history of type 2 diabetes mellitus now MED BOARDS;
moderately well controlled with glyburide. His father TOPNOTCH MD)
was diagnosed with prostate cancer at the age of 70
years, and his sister died of complications from
systemic lupus erythematosus. His blood pressure is
135/86 mm Hg. Cardiopulmonary examination shows
no abnormalities. Abdominal examination shows no
suprapubic fullness or tenderness. There is mild
enlargement of the prostate with no palpable
nodules. His postvoid residual volume is 10 mL.
Serum studies show a urea nitrogen (BUN) level of 45
mg/dL and creatinine level of 3.8 mg/dL. Urine
dipstick shows 3+ protein. Which of the following is
most likely to have prevented progression of this
patient's renal disease?
A) Oral finasteride therapy
B) Oral terazosin therapy
C) Oral cyclophosphamide and prednisone therapy
D) Oral enalapril therapy
528 A 47-year-old man is admitted to the hospital after MIGUEL RAFAEL MIDTERM 1
threatening to harm a radio announcer he believed RAMOS, MD (TOP EXAM - FEB
was broadcasting his thoughts. Over the past 20 3 - FEB 2012 2013
years, he has had multiple psychiatric MED BOARDS;
hospitalizations for threatening people who he TOPNOTCH MD)
believed were plotting against him, trying to control
his mind, or causing him to hear voices by implanting
devices in his head. Past symptoms improved with
neuroleptic therapy; after discharge, he discontinued
the medication and his symptoms worsened. Which
of the following is the most appropriate
pharmacotherapy to decrease this patient's risk for
future hospitalization?
A) Clozapine
B) Fluphenazine hydrochloride
C) Haloperidol decanoate
D) Risperidone
529 A 47-year-old man is admitted to the hospital after MIGUEL RAFAEL MIDTERM 1
threatening to harm a radio announcer he believed RAMOS, MD (TOP EXAM - FEB
was broadcasting his thoughts. Over the past 20 3 - FEB 2012 2013
years, he has had multiple psychiatric MED BOARDS;
hospitalizations for threatening people who he TOPNOTCH MD)
believed were plotting against him, trying to control
his mind, or causing him to hear voices by implanting
devices in his head. Past symptoms improved with
neuroleptic therapy; after discharge, he discontinued
the medication and his symptoms worsened. Which
of the following is the most appropriate
pharmacotherapy to decrease this patient's risk for
future hospitalization?
A) Clozapine
B) Fluphenazine hydrochloride
C) Haloperidol decanoate
D) Risperidone
530 A 32-year-old woman comes to the physician because MIGUEL RAFAEL MIDTERM 1
of a 3-month history of increasing pain and stiffness RAMOS, MD (TOP EXAM - FEB
in her wrists, hands, and ankles. During this period, 3 - FEB 2012 2013
she also has had progressive fatigue and morning MED BOARDS;
stiffness lasting 2 hours. She has a 1-year history of TOPNOTCH MD)
rheumatoid arthritis treated with naproxen.
Examination shows redness, swelling, and warmth
over the wrist, hand, and ankle joints bilaterally.
There are nontender subcutaneous nodules over the
extensor surfaces of both elbows. X-ray films of the
hands show diffuse osteopenia and erosions over
several of the distal metacarpal bones. Which of the
following is the most appropriate pharmacotherapy?
A) Add oral cyclophosphamide
B) Add oral gold

TOPNOTCH MEDICAL BOARD PREP PHARMACOLOGY SUPEREXAM Page 60 of 83


For inquiries visit www.topnotchboardprep.com.ph or email us at topnotchmedicalboardprep@gmail.com
TOPNOTCH MEDICAL BOARD PREP PHARMACOLOGY SUPEREXAM
For inquiries visit www.topnotchboardprep.com.ph or email us at topnotchmedicalboardprep@gmail.com
Item QUESTION EXPLANATION AUTHOR TOPNOTCH
# EXAM
C) Add oral methotrexate
D) Add oral penicillamine

531 An otherwise healthy 19-year-old woman comes to Oral isotretinoin is never the first option for acne, MIGUEL RAFAEL MIDTERM 1
the physician because of a 3-year history of only used when topical treatments have failed RAMOS, MD (TOP EXAM - FEB
intermittent facial blemishes. She drinks wine 3 - FEB 2012 2013
occasionally on weekends. She takes no medications. MED BOARDS;
Examination shows multiple 1- to 2-mm red and TOPNOTCH MD)
white papules and on the forehead and cheeks. Which
of the following is the most appropriate initial
pharmacotherapy?
A) Topical benzoyl peroxide
B) Systemic corticosteroids
C) Oral isotretinoin
D) Topical corticosteroids
532 A 57-year-old man has been hospitalized for 2 days Heparin-induced thrombocytopenia >>> the MIGUEL RAFAEL MIDTERM 1
for treatment of unstable angina pectoris. He is Heparin-Platelet4 complex are attacked by IgG in the RAMOS, MD (TOP EXAM - FEB
currently receiving intravenous heparin and human bodies that essentially activate the platelet 3 - FEB 2012 2013
undergoing evaluation for coronary artery bypass and thus form clots. These clots are the reason free MED BOARDS;
grafting. His blood pressure is 160/90 mm Hg, pulse platelets are low. TOPNOTCH MD)
is 88/min, and respirations are 16/min. Laboratory
studies show:

Platelet count 90,000/mm3
Prothrombin time 12 sec (INR=1.1)
Partial thromboplastin time 35 sec

Which of the following is the most likely cause of
these findings?
A) Excessive platelet destruction
B) Factor VIII deficiency
C) Inadequate platelet production
D) Uncontrolled activation of coagulation and
fibrinolytic cascades
533 A 42-year-old woman, gravida 2, para 2, comes to the MIGUEL RAFAEL MIDTERM 1
physician because of a 3-month history of swelling of RAMOS, MD (TOP EXAM - FEB
her legs and mild abdominal pain and bloating. 3 - FEB 2012 2013
Abdominal examination shows no abnormalities. MED BOARDS;
Rectovaginal examination shows fullness in the right TOPNOTCH MD)
adnexa. Transvaginal ultrasonography shows an
irregular mass in the right ovary with some
solid components to a predominantly cystic lesion.
Her serum CA 125 level is 120 U/mL (N<35).
Treatment with which of the following is most likely
to have prevented this patient's symptoms?
A) Antiestrogens
B) Antiprogestationals
C) Medroxyprogesterone
D) Oral contraceptives
534 A 67-year-old man is hospitalized for treatment of insulin and glucose >> fastest way to lower down MIGUEL RAFAEL MIDTERM 1
renal insufficiency. Three days after admission, his serum K RAMOS, MD (TOP EXAM - FEB
pulse is 40/min. An ECG shows tall, tented T waves. 3 - FEB 2012 2013
Serum studies show a sodium level of 134 mEq/L, MED BOARDS;
potassium level of 6.9 mEq/L, and glucose level of 85 TOPNOTCH MD)
mg/dL. The most appropriate next step in
management is intravenous administration of which
of the following?
A) Calcium, furosemide, and 3% saline
B) Calcium, insulin, and digitalis
C) Calcium, insulin, and glucose
D) Glucose, furosemide, and phosphate
535 A 37 year old man comes to the physician for a Start HAART with CD4 below 350 and Pneumocystis MIGUEL RAFAEL MIDTERM 1
follow-up examination. He was diagnosed with HIV prophylaxis with CD4 below 200 RAMOS, MD (TOP EXAM - FEB
infection 3 weeks ago. He is asymptomatic and takes 3 - FEB 2012 2013
no edications. His temperature is 37.2 C, pulse is 100 MED BOARDS;
bpm, and blood presure is 100/60 mmHg. TOPNOTCH MD)
Examination shows no abnormalities except for
erythematous scaling at the hairline and nasolabial
folds. His CD4+ T-lymphocyte count is 160/mm3 and
plasma HIV viral load is 25,000 copies/mL. Which of
the following is the most appropriate
pharmacotherapy?
A) Pneumocystis jiroveci prophylaxis only
B) Three-drug antiretroviral therapy only
C) Three-drug antiretroviral therapy and P. jiroveci
TOPNOTCH MEDICAL BOARD PREP PHARMACOLOGY SUPEREXAM Page 61 of 83
For inquiries visit www.topnotchboardprep.com.ph or email us at topnotchmedicalboardprep@gmail.com
TOPNOTCH MEDICAL BOARD PREP PHARMACOLOGY SUPEREXAM
For inquiries visit www.topnotchboardprep.com.ph or email us at topnotchmedicalboardprep@gmail.com
Item QUESTION EXPLANATION AUTHOR TOPNOTCH
# EXAM
prophylaxis
D) Two-drug antiretroviral therapy and P. jiroveci
prophylaxis

536 A 55-year-old woman was recently diagnosed with an Phenoxybenzamine >> nonselective, IRREVERSIBLE MIGUEL RAFAEL MIDTERM 1
adrenal mass due to symptoms of sweating, elevated alpha blocker..Phentolamine>> reversible alpha RAMOS, MD (TOP EXAM - FEB
blood pressure, and severe headaches. Which of the blocker. 3 - FEB 2012 2013
following nonselective, irreversible alpha blocker is MED BOARDS;
the drug of choice as a preoperative agent used in this TOPNOTCH MD)
disease?
A) Phentolamine
B) Phenoxybenzamine
C) Prazosin
D) Yohimbine
537 A 72-year-old man was diagnosed to have glaucoma. Pilocarpine >> ciliary muscle contraction >> opening MIGUEL RAFAEL MIDTERM 1
Which of the following drugs induces ciliary muscle trabecular meshwork and increasing the outflow RAMOS, MD (TOP EXAM - FEB
contraction thereby opening the trabecular 3 - FEB 2012 2013
meshwork and increasing the outflow? MED BOARDS;
A) Timolol TOPNOTCH MD)
B) Mannitol
C) Epinephrine
D) Pilocarpine
538 A 58-year-old woman with newly diagnosed type 2 tolbutamide >> disulfuram reaction MIGUEL RAFAEL MIDTERM 1
diabetes mellitus presents to the emergency RAMOS, MD (TOP EXAM - FEB
department complaining of vomiting, severe 3 - FEB 2012 2013
headache, dizziness, blurry vision, and dyspnea. She MED BOARDS;
says that she had been at a party when the symptoms TOPNOTCH MD)
began. Her skin is notably flushed on physical
examination. Which of the following medications is
responsible for this reaction?
A) Tolbutamide
B) Glipizide
C) Glyburide
D) Metformin
539 A 25-year-old woman with AIDS comes to the MIGUEL RAFAEL MIDTERM 1
physician because of a 10-day history of fever, RAMOS, MD (TOP EXAM - FEB
shortness of breath, night sweats, fatigue, and a non- 3 - FEB 2012 2013
productive cough. She had been treated with MED BOARDS;
antiretroviral agents for 2 years but stopped taking TOPNOTCH MD)
her medications 6 weeks ago. She has no known
allergies. Her temperature is 38.6 C, pulse is 110/min,
respiration are 20/min, and blood pressure is 110/60
mm Hg. Examination shows no cyanosis. Diffuse fine
crackles are heard bilaterally. An x-ray of the chest
shows bilateral interstitial infiltrates. A silver stain of
sputum is positive for cysts and organisms. Which of
the following is the most appropriate
pharmacotherapy?
A) Ceftriaxone
B) Erythromycin
C) Cotrimazole
D) Co-trimoxazole
540 A 27-year-old nulligravid woman comes to the MIGUEL RAFAEL MIDTERM 1
physician for preconceptional counseling. She has a RAMOS, MD (TOP EXAM - FEB
mechanical mitral heart valve and chronic 3 - FEB 2012 2013
rheumatoid arthritis. Her cardiac status is New York MED BOARDS;
Heart Association Class II. She feels well. Current TOPNOTCH MD)
daily medications include warfarin, prednisone, and
acetaminophen with codeine. Examination shows
no abnormalities except for audible clicking from the
heart valve. Which of the following is the most
appropriate advice for this patient?
A) Chemical dependency counseling before
pregnancy
B) Discontinuation of anticoagulant therapy during
pregnancy
C) Discontinuation of prednisone during pregnancy
D) Switching from warfarin to heparin before
pregnancy

TOPNOTCH MEDICAL BOARD PREP PHARMACOLOGY SUPEREXAM Page 62 of 83


For inquiries visit www.topnotchboardprep.com.ph or email us at topnotchmedicalboardprep@gmail.com
TOPNOTCH MEDICAL BOARD PREP PHARMACOLOGY SUPEREXAM
For inquiries visit www.topnotchboardprep.com.ph or email us at topnotchmedicalboardprep@gmail.com
Item QUESTION EXPLANATION AUTHOR TOPNOTCH
# EXAM
541 What is the half-life of alcohol? Ethanol, over most of its plasma concentration ABDELSIMAR FINAL EXAM -
A. 2 hours range, exhibits zero-order elimination. Thus, there is OMAR II, MD FEB 2014
B. 4 hours no constant half life of elimination as in drugs (TOP 2 - AUG
C. 8 hours exhibiting first order elimination. (Katzung Board 2013 MED
D. 12 hours Review 9e, p.6) BOARDS;
E. None of the above TOPNOTCH MD -
200 QUESTIONS)
AND MARC
DENVER
TIONGSON, MD
(40 QUESTIONS)
542 A 48/M farmer with a history of alcohol abuse, is A chemical antagonist interacts directly with the ABDELSIMAR FINAL EXAM -
rushed to the emergency room after the patient has drug being antagonized to remove it or prevent it OMAR II, MD FEB 2014
deliberately ingested 300 mL of an unknown from binding to its target. Pralidoxime which (TOP 2 - AUG
pesticide while drunk. On examination, you note that combines avidly with the phosphorus in 2013 MED
the patient has pinpoint pupils and frothy secretions organosphosphate cholinesterase inhibitors is an BOARDS;
at the mouth. After you determine that the patient has example. (KBR9e, p. 14) TOPNOTCH MD -
ingested malathion; you then immediately start 200 QUESTIONS)
treatment with pralidoxime. Pralidoxime and AND MARC
malathion are examples of: DENVER
A. Competitive pharmacologic antagonists TIONGSON, MD
B. Irreversible pharmacologic antagonists (40 QUESTIONS)
C. Physiologic antagonists
D. Structural antagonists
E. Chemical antagonists
543 A 48-year old Caucasian male is referred to you for 50% of white and African american persons are slow ABDELSIMAR FINAL EXAM -
chronic cough, night sweats and weight loss. Sputum acetylators of INH, hydralazine and procainamide. OMAR II, MD FEB 2014
AFB was positive on three occasions; findings of chest KBR9e, p33 (TOP 2 - AUG
X ray done was also consistent with pulmonary 2013 MED
tuberculosis. Before you start drug therapy, you BOARDS;
remember that a much larger percentage of TOPNOTCH MD -
Caucasians are slow acetylators of one of the anti-TB 200 QUESTIONS)
drugs. The dosage of which of the following may need AND MARC
to be modified. DENVER
A. Isoniazid TIONGSON, MD
B. Rifampicin (40 QUESTIONS)
C. Streptomycin
D. Pyrazinamide
E. Ethambutol
544 A recently-diagnosed hypertensive was started on a ABDELSIMAR FINAL EXAM -
BP lowering agent. Due to financial constraints, the OMAR II, MD FEB 2014
patient abruptly stopped taking the prescribed (TOP 2 - AUG
medications after stocks were consumed. A day later, 2013 MED
the patient was brought to the clinic for light- BOARDS;
headedness; on admission, patient's BP was 180/100. TOPNOTCH MD -
Which of the following anti-hypertensives, known for 200 QUESTIONS)
rebound hypertension, was most likely taken by the AND MARC
patient? DENVER
A. Enalapril TIONGSON, MD
B. Furosemide (40 QUESTIONS)
C. Methyldopa
D. Clonidine
E. Metoprolol
545 A 24/G3P2(2002) who came in with a BP of 200/140 A describes the MOA of verapamil; D describes ABDELSIMAR FINAL EXAM -
was given Hydralazine. What is the mechanism of minoxidil; E describes fenoldopam. All are direct OMAR II, MD FEB 2014
action of hydralazine? vasodilators. (TOP 2 - AUG
A. Reduction of Ca influx via L-type channels 2013 MED
B. Antagonism of alpha-adrenergic receptors in blood BOARDS;
vessels TOPNOTCH MD -
C. Release of NO 200 QUESTIONS)
D. Hyperpolarization of smooth muscles due to AND MARC
opening of K channels DENVER
E. Activation of dopamine D1 receptors TIONGSON, MD
(40 QUESTIONS)
546 Which of the following diuretics is incorrectly paired Thiazides increase reabsorption of Ca and prevent ABDELSIMAR FINAL EXAM -
with its associated toxicity? Ca stone formation. Hepatic encephalopathy can OMAR II, MD FEB 2014
A. Thiazide - renal stone ensue with acetazolamide because it causes (TOP 2 - AUG
B. Acetazolamide - hepatic encephalopathy alkalinization of urine due to decreased bicarbonate 2013 MED
C. Bumetanide - hypokalemic metabolic alkalosis reabsorption. This prevents ammonia from turning BOARDS;
D. Eplerenone - hyperkalemic metabolic acidosis to ammonium. Ammonia is more easily reabsorbed. TOPNOTCH MD -
E. Mannitol - pulmonary edema 200 QUESTIONS)
AND MARC
DENVER
TIONGSON, MD
(40 QUESTIONS)
547 Leukotrienes are among the most potent Zileuton is a lipooxygenase inhibitor. ABDELSIMAR FINAL EXAM -
bronchoconstrictors implicated in the pathogenesis of OMAR II, MD FEB 2014
bronchial asthma. Which of the following is a (TOP 2 - AUG
leukotriene receptor inhibitor that has found clinical 2013 MED
application in asthma prophylaxis? BOARDS;
A. Zileuton TOPNOTCH MD -
B. Zafirlukast 200 QUESTIONS)
C. Ipratropium AND MARC
D. Budesonide DENVER
E. Bosentan TIONGSON, MD
(40 QUESTIONS)
TOPNOTCH MEDICAL BOARD PREP PHARMACOLOGY SUPEREXAM Page 63 of 83
For inquiries visit www.topnotchboardprep.com.ph or email us at topnotchmedicalboardprep@gmail.com
TOPNOTCH MEDICAL BOARD PREP PHARMACOLOGY SUPEREXAM
For inquiries visit www.topnotchboardprep.com.ph or email us at topnotchmedicalboardprep@gmail.com
Item QUESTION EXPLANATION AUTHOR TOPNOTCH
# EXAM
548 A 28/M car mechanic was brought to the emergency Ethylene glycol and methanol poisoning may be ABDELSIMAR FINAL EXAM -
room after ingesting automobile antifreeze. ABG treated with fomepizole, an alcohol dehydrogenase OMAR II, MD FEB 2014
reveals high anion gap metabolic acidosis; while inhibitor, which prvents conversion of (TOP 2 - AUG
urinalysis reveals calcium oxalate crystals. You methanol/ehtylene glycol to toxic metabolites. 2013 MED
suspect ethylene glycol poisoning. Which of the BOARDS;
following drugs is recommended? TOPNOTCH MD -
A. Naltrexone 200 QUESTIONS)
B. Diazepam AND MARC
C. Disulfiram DENVER
D. Acamprosate TIONGSON, MD
E. Fomepizole (40 QUESTIONS)
549 A patient with neuropathic pain was prescribed Although gabapentin is a structural analog of GABA, ABDELSIMAR FINAL EXAM -
gabapentin. What is the primary mechanism of action it does NOT activate GABA receptors directly. OMAR II, MD FEB 2014
of gabapentin? (TOP 2 - AUG
A. Direct activation of GABA reeptors 2013 MED
B. Inhibition of T type Ca2+ channels BOARDS;
C. Blockade of voltage-gated Na channels TOPNOTCH MD -
D. Inhibition of GABA transporter (GAT-1) prolonging 200 QUESTIONS)
the action of the neurotransmitter AND MARC
E. Enhancement of K+ channel permeability causing DENVER
neuronal hyperpolarization TIONGSON, MD
(40 QUESTIONS)
550 A 6/F is brought to the clinic due to frequent episodes Ethosuzimide and valproic acid are the preferred ABDELSIMAR FINAL EXAM -
of sudden activity arrest with staring and minimal drugs for absence seizures because they cause OMAR II, MD FEB 2014
eyelid flutter occurring for 10 - 20 seconds, 5 - 10 minimal sedation. (TOP 2 - AUG
times per day. During such episodes, patient is 2013 MED
unresponsive to voice or tactile stimulation. She is BOARDS;
diagnosed with absence seizure. Which of the TOPNOTCH MD -
following antiseizure drugs is most appropriate? 200 QUESTIONS)
A. Carbamazepine AND MARC
B. Valproic acid DENVER
C. Phenytoin TIONGSON, MD
D. Phenobarbital (40 QUESTIONS)
E. Levetiracetam
551 Administration of most inhaled anesthetics cause a ABDELSIMAR FINAL EXAM -
decrease in the following except: OMAR II, MD FEB 2014
A. Arterial blood pressure (TOP 2 - AUG
B. Brain metabolic rate 2013 MED
C. Myocardial function BOARDS;
D. Tidal volume TOPNOTCH MD -
E. None of the above 200 QUESTIONS)
AND MARC
DENVER
TIONGSON, MD
(40 QUESTIONS)
552 A 53/M, admitted at the Medical ICU, after having a Lidocaine and other Class IB drugs act primarily in ABDELSIMAR FINAL EXAM -
myocardial infarction, develops ventricular ischemic tissues and are best for arrhythmias OMAR II, MD FEB 2014
tachycardia. Which of the following anti-arrhythmics following MI. (TOP 2 - AUG
is most appropriate? 2013 MED
A. Quinidine BOARDS;
B. Mexiletine TOPNOTCH MD -
C. Adenosine 200 QUESTIONS)
D. Lidocaine AND MARC
E. Flecainide DENVER
TIONGSON, MD
(40 QUESTIONS)
553 Flumazenil may be effective in patients who Most S-H drugs facilitate the actions of GABA, a ABDELSIMAR FINAL EXAM -
overdosed on the following sedative-hypnotics major inhibitory transmitter. Benzodiazepines OMAR II, MD FEB 2014
except: potentiate GABA by increasing frequency of Cl ion (TOP 2 - AUG
A. Alprazolam channel opening; its action is blocked by flumazenil, 2013 MED
B. Zolpidem a BZ receptor antagonist. Flumazenil, on the other BOARDS;
C. Thiopental hand, does NOT block the effects of barbiturates. TOPNOTCH MD -
D. Midazolam 200 QUESTIONS)
E. Lorazepam AND MARC
DENVER
TIONGSON, MD
(40 QUESTIONS)
554 Which of the following IV anesthetics is the only one ABDELSIMAR FINAL EXAM -
that causes CV stimulation? Its use in neurosurgical OMAR II, MD FEB 2014
operations is limited due to its propensity to increase (TOP 2 - AUG
ICP. 2013 MED
A. Midazolam BOARDS;
B. Propofol TOPNOTCH MD -
C. Fentanyl 200 QUESTIONS)
D. Ketamine AND MARC
E. Thiopental DENVER
TIONGSON, MD
(40 QUESTIONS)

TOPNOTCH MEDICAL BOARD PREP PHARMACOLOGY SUPEREXAM Page 64 of 83


For inquiries visit www.topnotchboardprep.com.ph or email us at topnotchmedicalboardprep@gmail.com
TOPNOTCH MEDICAL BOARD PREP PHARMACOLOGY SUPEREXAM
For inquiries visit www.topnotchboardprep.com.ph or email us at topnotchmedicalboardprep@gmail.com
Item QUESTION EXPLANATION AUTHOR TOPNOTCH
# EXAM
555 Sisa, 28/F, recently-diagnosed schizophrenic was Neuroleptic malignant syndrome is characterized by ABDELSIMAR FINAL EXAM -
admited to the Psychiatry ward and started on FEVER, RIGIDITY, and AUTONOMIC INSTABILITY. OMAR II, MD FEB 2014
Risperidone. Three days later, the patient developed NMS is linked to intake with several atypical (TOP 2 - AUG
fever (39.9C) and rigidity. Patient was notably antipsychotics. 2013 MED
confused. Vital signs were as follows: BP 170/120, HR BOARDS;
118, RR 22. What could explain Sisa's symptoms? TOPNOTCH MD -
A. Extrapyramidal symptoms 200 QUESTIONS)
B. Tardive dyskinesia AND MARC
C. Serotonin syndrome DENVER
D. Neuroleptic malignant syndrome TIONGSON, MD
E. Malignant hyperthermia (40 QUESTIONS)
556 Formation of methyltransferases that alter drug Question could be rephrased into: which acts on the ABDELSIMAR FINAL EXAM -
binding sites on the 50S ribosomal subunit is the 50s subunit? Macrolides such as azithromycin do. OMAR II, MD FEB 2014
primary mechanism of resistance to which antibiotic: (TOP 2 - AUG
A. Streptomycin 2013 MED
B. Tetracycline BOARDS;
C. Amikacin TOPNOTCH MD -
D. Azithromycin 200 QUESTIONS)
E. Levofloxacin AND MARC
DENVER
TIONGSON, MD
(40 QUESTIONS)
557 A 41/M, presenting with high fever and chills, is Aztreonam is active only against Gram-(-) organisms ABDELSIMAR FINAL EXAM -
admitted at the hospital. He reports having had and has NO cross-reactivity with penicillins. OMAR II, MD FEB 2014
serious allergic reactions to amoxicillin one year ago. (TOP 2 - AUG
Blood GS/CS reveals growth of Gram negative 2013 MED
bacteremia. Which of the following antibiotics is BOARDS;
MOST appropriatE? TOPNOTCH MD -
A. Cefazolin 200 QUESTIONS)
B. Ampicillin-sulbactam AND MARC
C. Piperacillin-tazobactam DENVER
D. Aztreonam TIONGSON, MD
E. Vancomycin (40 QUESTIONS)
558 Ceftriaxone is commonly given as an empiric Organisms NOT covered by cephalosporins are ABDELSIMAR FINAL EXAM -
antibiotic in the treatment of bacterial meningitis. L.A.M.E.: Listeria monocytogenes, Atypicals OMAR II, MD FEB 2014
However, it must be noted that this antibiotic will (chlamydia, mycoplasma), MRSA, and Enterococci. (TOP 2 - AUG
NOT cover the following etiologic agent in meningitis: 2013 MED
A. Neisseria meningitidis BOARDS;
B. Streptococcus pneumoniae TOPNOTCH MD -
C. Listeria monocytogenes 200 QUESTIONS)
D. Group B streptococcus AND MARC
E. C and D DENVER
TIONGSON, MD
(40 QUESTIONS)
559 A 10/F, diagnosed case of HSV encephalitis, was given Acyclovir, foscarnet and ganciclovir are DNA ABDELSIMAR FINAL EXAM -
IV acyclovir. Acyclovir is an efffective antiviral polymerase inhibitors. OMAR II, MD FEB 2014
because it inhibits: (TOP 2 - AUG
A. DNA polymerase 2013 MED
B. RNA polymerase BOARDS;
C. Aspartate protease TOPNOTCH MD -
D. Neuraminidase 200 QUESTIONS)
E. Hemagluttinin AND MARC
DENVER
TIONGSON, MD
(40 QUESTIONS)
560 A 55/M comes in due to sudden headache described Nimodipine given for 21 days should be started on ABDELSIMAR FINAL EXAM -
as the "worst headache of his life." He is a diagnosed admission for vasospasm prophylaxis. OMAR II, MD FEB 2014
hypertensive, with poor compliance to medications. (TOP 2 - AUG
PE reveals nuchal tenderness (positive Kernig's sign), 2013 MED
and a left cranial nerve palsy. Brain CT reveals diffuse BOARDS;
subarachnoid hemorrhage in the basal cisterns. TOPNOTCH MD -
Which of the following drugs has a proven role in the 200 QUESTIONS)
management of his condition? AND MARC
A. Nimodipine DENVER
B. Propanolol TIONGSON, MD
C. Captopril (40 QUESTIONS)
D. Nifedipine
E. Aspirin
561 The drug of choice for clostridium tetani infection The use of penicillin (10–12 million units IV, given BLAKE WARREN MIDTERM 2
because of the drug's excellent antimicrobial activity daily for 10 days) has been recommended, but ANG, MD (TOP 1 - EXAM - FEB
and the absence of the GABA-antagonistic activity metronidazole (500 mg every 6 h or 1 g every 12 h) AUG 2013 MED 2014
a. Penicillin G is preferred by some experts on the basis of this BOARDS;
b. Metrobidazole drug's excellent antimicrobial activity and the TOPNOTCH MD)
c. Vancomycin absence of the GABA-antagonistic activity seen with
d. Chloramphenicol penicillin.

562 a second generation Cephalosporin with intrinsic Cefoxitin has anaerobe coverage apart from its BLAKE WARREN MIDTERM 2
activity against anaerobes: activity against gram positive and some gram neg ANG, MD (TOP 1 - EXAM - FEB
a. cefoxitin infections AUG 2013 MED 2014
b. cefipime BOARDS;
c. cefaclor TOPNOTCH MD)
d. ceftriaxone
0

TOPNOTCH MEDICAL BOARD PREP PHARMACOLOGY SUPEREXAM Page 65 of 83


For inquiries visit www.topnotchboardprep.com.ph or email us at topnotchmedicalboardprep@gmail.com
TOPNOTCH MEDICAL BOARD PREP PHARMACOLOGY SUPEREXAM
For inquiries visit www.topnotchboardprep.com.ph or email us at topnotchmedicalboardprep@gmail.com
Item QUESTION EXPLANATION AUTHOR TOPNOTCH
# EXAM
563 this oral iron compound has the highest elemental Ferrous sulfate (FEOSOL, others) is the hydrated BLAKE WARREN MIDTERM 2
iron content: salt, FeSO4×7H2O, which contains 20% iron. Dried ANG, MD (TOP 1 - EXAM - FEB
a. ferrous sulfate ferrous sulfate (32% elemental iron) is also AUG 2013 MED 2014
b. ferrous gluconate available. Ferrous fumarate (FEOSTAT, others) BOARDS;
c. ferrous fumarate contains 33% iron and is moderately soluble in TOPNOTCH MD)
d. iron dextran water, stable, and almost tasteless. Ferrous
gluconate (FERGON, others) also has been
successfully used in the therapy of iron-deficiency
anemia. The gluconate contains 12% iron.
Polysaccharide-iron complex (NIFEREX, others), a
compound of ferrihydrite and carbohydrate, is
another preparation with comparable absorption.
The effective dose of all of these preparations is
based on iron content.
564 which anong the following drugs should be avoided in Phenobarbitals should be avoided in patients BLAKE WARREN MIDTERM 2
patients with porphyria? diagbosed with porphyria ANG, MD (TOP 1 - EXAM - FEB
a. phenytoin AUG 2013 MED 2014
b. secobarbital BOARDS;
c. midazolam TOPNOTCH MD)
d. sodium valproate

565 Xenobiotic metabolizing enzymes have historically Phase 1 reaction does little to the water solubility of BLAKE WARREN MIDTERM 2
been grouped into the phase 1 reactions and the a drug metabolized ANG, MD (TOP 1 - EXAM - FEB
phase 2 reactions. Which is NOT TRUE regarding the AUG 2013 MED 2014
former? BOARDS;
a. enzymes carry out oxidation, reduction, or TOPNOTCH MD)
hydrolytic reactions
b. enzymes lead to the introduction of what are called
functional groups resulting in a modification of the
drug
c. the reaction process adds an -OH, -COOH, -SH, -O-
or NH2 group.
d. the addition of functional groups significantly
increase the water solubility of the drug
566 NOT True of Phase 2 enzymes in Xenobiotic Because of conjugation of these substances, they BLAKE WARREN MIDTERM 2
metabolism: have a higher molecular weight ANG, MD (TOP 1 - EXAM - FEB
a. they facilitate the elimination of drugs AUG 2013 MED 2014
b. inactivation of electrophilic and potentially toxic BOARDS;
metabolites produced by oxidation. TOPNOTCH MD)
c. phase 2 reactions produce a metabolite with
improved water solubility
d. products have decreased molecular weight
567 majority of phase2 reactions involving Glucoronic conjugating enzymes, notably the UGTs, are all BLAKE WARREN MIDTERM 2
acid conjugation are found in what cell compartment? located in the endoplasmic reticulum of the cell ANG, MD (TOP 1 - EXAM - FEB
a. ER AUG 2013 MED 2014
b. cytosol BOARDS;
c. Mitochondrial matrix TOPNOTCH MD)
d. Golgi bodies

568 Notable to these drugs are its capability of inducing All except Cimetidine induce CYP450 enzyme BLAKE WARREN MIDTERM 2
CP450 enzyme system in the liver except: complex ANG, MD (TOP 1 - EXAM - FEB
a. Phenytoin AUG 2013 MED 2014
b. Cimetidine BOARDS;
c. Carbamazepine TOPNOTCH MD)
d. Griseofulvin

569 Flushing is a unique side effect of Nicotinamide used The cutaneous effects include flushing and pruritus BLAKE WARREN MIDTERM 2
in hyperlipidemia. The drug that may be used to of the face and upper trunk, skin rashes, and ANG, MD (TOP 1 - EXAM - FEB
counter this effect is: acanthosis nigricans. Flushing and associated AUG 2013 MED 2014
a. Acetaminophen pruritus are prostaglandin-mediated. Taking an BOARDS;
b. Aspirin aspirin each day alleviates the flushing in many TOPNOTCH MD)
c. Chlorphenamine patients.
d. Arginine

570 Local anesthetics block nerve conduction through Local anesthetics block conduction by decreasing or BLAKE WARREN MIDTERM 2
inhibition of what electrolyte? preventing the large transient increase in the ANG, MD (TOP 1 - EXAM - FEB
a. Potassium permeability of excitable membranes to Na+ that AUG 2013 MED 2014
b. Sodium normally is produced by a slight depolarization of BOARDS;
c. Calcium the membrane TOPNOTCH MD)
d. All of the above

571 a dreadful side effect, although not common, of this Clozapine is notorious for its side effect of BLAKE WARREN MIDTERM 2
atypical antipsychotic is agranulocytosis. agranulocytosis, although uncommon ANG, MD (TOP 1 - EXAM - FEB
a. Quetiapine AUG 2013 MED 2014
b. Clozapine BOARDS;
c. Risperidone TOPNOTCH MD)
d. Aripiprazole

TOPNOTCH MEDICAL BOARD PREP PHARMACOLOGY SUPEREXAM Page 66 of 83


For inquiries visit www.topnotchboardprep.com.ph or email us at topnotchmedicalboardprep@gmail.com
TOPNOTCH MEDICAL BOARD PREP PHARMACOLOGY SUPEREXAM
For inquiries visit www.topnotchboardprep.com.ph or email us at topnotchmedicalboardprep@gmail.com
Item QUESTION EXPLANATION AUTHOR TOPNOTCH
# EXAM
572 True of Acetaminophen: Observational studies suggest that acetaminophen, BLAKE WARREN MIDTERM 2
a. It covalently binds to COX inhibiting prostaglandin which is a , very weak antiinflammatory agent at the ANG, MD (TOP 1 - EXAM - FEB
synthesis typical daily dose of 1000 mg is associated with a AUG 2013 MED 2014
b. very weak antiinflammatory agent at the typical reduced incidence of gastrointestinal adverse effects BOARDS;
daily dose of 1000 mg compared to tNSAIDs TOPNOTCH MD)
c. incidence of Gastrointestinal adverse effects
comparable to Ibuprofen
d. its inhibition of thromboxane A2 synthesis
discourages its use in dengue cases
573 A condition characterized by the acute onset of Due to the association with Reye's syndrome, aspirin BLAKE WARREN MIDTERM 2
encephalopathy, liver dysfunction, and fatty and other salicylates are contraindicated in children ANG, MD (TOP 1 - EXAM - FEB
infiltration of the liver and other viscera known as and young adults less than 20 years old with fever AUG 2013 MED 2014
Reye's syndrome is associated with what drug? associated with viral illness. Reye's syndrome is BOARDS;
a. Aminosalicylate characterized by the acute onset of encephalopathy, TOPNOTCH MD)
b. Long chain fatty acids liver dysfunction, and fatty infiltration of the liver
c. Indomethacin and other viscera
d. nitric oxide

574 incidence of Reye syndrome among patients less than Less than 0.1% of children who took aspirin BLAKE WARREN MIDTERM 2
20 who took Aspirin concurrent with influenza A/B developed Reye syndrome, but more than 80% of ANG, MD (TOP 1 - EXAM - FEB
infection: patients diagnosed with Reye syndrome had taken AUG 2013 MED 2014
a. 0.1% aspirin in the past 3 weeks BOARDS;
b. 1% TOPNOTCH MD)
c. 5%
d. 10%

575 . A good drug combination along with thiazides by Urinary K+ loss can be a problem with thiazides. BLAKE WARREN MIDTERM 2
virtue of its effect on potassium retention: Angiotensin converting enzyme (ACE) inhibitors and ANG, MD (TOP 1 - EXAM - FEB
a. losartan angiotensin receptor antagonists will attenuate AUG 2013 MED 2014
b. amlodipine diuretic-induced loss of potassium to some degree, BOARDS;
c. verapamil and this is a consideration if a second drug is TOPNOTCH MD)
d. metoprolol required to achieve further blood pressure
reduction beyond that attained with the diuretic
alone.
576 a highly lipophilic derivative of obiquinone that has Atorvaquone is promising synthetic derivative with BLAKE WARREN MIDTERM 2
activity against plasmodium species and potent activity against Plasmodium species and the ANG, MD (TOP 1 - EXAM - FEB
Pneumocystis pneumonia among patients intolerant opportunistic pathogens Pneumocystis carinii and AUG 2013 MED 2014
to Co-trimoxazole: Toxoplasma gondii. After limited clinical trials, the BOARDS;
a. Atorvaquone FDA approved this compound in 1992 for treatment TOPNOTCH MD)
b. Quinine of mild-to-moderate P. carinii pneumonia in patients
c. Proguanil intolerant to trimethoprim-sulfamethoxazol
d. artemisinin

577 The least nephrotoxic among the aminoglycosides is: The nephrotoxic potential varies among individual BLAKE WARREN MIDTERM 2
a. Tobramycin aminoglycosides. The relative toxicity correlates ANG, MD (TOP 1 - EXAM - FEB
b. Kanamycin with the concentration of drug found in the renal AUG 2013 MED 2014
c. Amikacin cortex in experimental animals. Neomycin, which BOARDS;
d. Streptomycin concentrates to the greatest degree, is highly TOPNOTCH MD)
nephrotoxic in human beings and should not be
administered systemically. Streptomycin does not
concentrate in the renal cortex and is the least
nephrotoxic
578 A fusion inhibitor used in the treatment of HIV BLAKE WARREN MIDTERM 2
infection ANG, MD (TOP 1 - EXAM - FEB
a. Efavirenz AUG 2013 MED 2014
b. Lamivudine BOARDS;
c. Enfuvirtide TOPNOTCH MD)
d. Atazanavir

579 In dermatologic pharmacology, approximately how An amount of topical medication sufficient to cover BLAKE WARREN MIDTERM 2
many grams of topical preparation is needed to cover affected body surfaces in repeated applications must ANG, MD (TOP 1 - EXAM - FEB
the body surface? be dispensed to the patient. A general rule is that AUG 2013 MED 2014
a. 10g approximately 30 g is required to cover the body BOARDS;
b. 20g surface. TOPNOTCH MD)
c. 30g
d. 40g

580 A topical preparation that are concentric spherical Newer vehicles include liposomes and microgel BLAKE WARREN MIDTERM 2
shells of phospholipids in an aqueous medium that formulations. Liposomes are concentric spherical ANG, MD (TOP 1 - EXAM - FEB
may enhance percutaneous absorption. shells of phospholipids in an aqueous medium that AUG 2013 MED 2014
a. Liposome may enhance percutaneous absorption. Variations in BOARDS;
b. Microgrel size, charge, and lipid content can influence TOPNOTCH MD)
c. Transfersome liposome function substantially. Liposomes
d. Micelle penetrate compromised epidermal barriers more
efficiently (Korting et al., 1991). Microgels are
polymers that may enhance solubilization of certain
drugs, thereby enhancing penetration and
diminishing irritancy.

TOPNOTCH MEDICAL BOARD PREP PHARMACOLOGY SUPEREXAM Page 67 of 83


For inquiries visit www.topnotchboardprep.com.ph or email us at topnotchmedicalboardprep@gmail.com
TOPNOTCH MEDICAL BOARD PREP PHARMACOLOGY SUPEREXAM
For inquiries visit www.topnotchboardprep.com.ph or email us at topnotchmedicalboardprep@gmail.com
Item QUESTION EXPLANATION AUTHOR TOPNOTCH
# EXAM
581 Theophylline has a pKa of 8.7, what is the Above pKa, the unprotonated and uncharged form of TIMOTHY TANG MIDTERM 1
predominant form at pH 11.3? a weak base predominates. LEE SAY, MD EXAM - FEB
A. Ionized and protonated (TOP 4 - AUG 2014
B. Non-ionized and protonated 2013 MED
C. Charged and unprotonated BOARDS;
D. Uncharged and unprotonated TOPNOTCH MD)
E. None of the above

582 Some drugs like magnesium sulfate needs a loading Loadign dose = (Vd) (desired plasma TIMOTHY TANG MIDTERM 1
dose in order to reach steady state at a reduced concentration)/(bioavailabitlity) LEE SAY, MD EXAM - FEB
amount of time. The following are used to compute the minimum effective dose is usually the desired (TOP 4 - AUG 2014
for loading dose EXCEPT? plasma concentration. Clearance is used to calculate 2013 MED
A. Clearance the maintenance dose BOARDS;
B. Volume of Distribution TOPNOTCH MD)
C. Minimum effective dose
D. Bioavailabiloity
E. No exception
583 A 5 year old known case of seizure disorder Phenytoin is an inducer, which will lower TIMOTHY TANG MIDTERM 1
maintained on phenytoin was rushed to the ER due to theophylline levels. Asthma may not be adequately LEE SAY, MD EXAM - FEB
an asthma attack. IV theophylline was given to treated with this monotherapy. (TOP 4 - AUG 2014
control the asthma using usual dose computations. 2013 MED
What are the consequences of the drug-drug BOARDS;
interactions? TOPNOTCH MD)
A. The patient will be more prone to the toxic side-
effects of theophylline.
B. The patient may be at risk for a seizure attack.
C. The patient is not being treated adequately for his
asthma.
D. There will be an increased in plasma levels of
phenytoin.
E. No drug interactions exist beween theophylline
and phenytoin
584 Which of the following substances will increase the Cocaine and TCA prevents reuptake of TIMOTHY TANG MIDTERM 1
available adrenergic neurotransmitter in the synaptic norepinephrine. Review the inhibitors for each step LEE SAY, MD EXAM - FEB
cleft by inibiting reuptake? in the process of neurotransmitter release for both (TOP 4 - AUG 2014
A. Botulinum the sympathetic and parasympathetic system. 2013 MED
B. MAOIs BOARDS;
C. Cocaine TOPNOTCH MD)
D. Guanethidine
E. Reserpine

585 Which of the following drugs for glaucoma increased Beta blockers, CAIs and osmotic agents (mannitol) TIMOTHY TANG MIDTERM 1
aqueous humor outflow through the non- decreases aqueous humor production. Alpha LEE SAY, MD EXAM - FEB
conventional pathway without decreaing the adrenergic agonist increases outflow and also (TOP 4 - AUG 2014
secretion of aqueous humor by the ciliary body? decreases the secretion of the ciliary body. 2013 MED
A. β-blockers Prostaglandins increased outflow but has no effect BOARDS;
B. Prostaglandin analogues on production. TOPNOTCH MD)
C. α-adrenergic agonist
D. Carbonic anhydrase inhibitors
E. Mannitol
586 Which of the following beta-blockers demonstrates Because of its relatively β1 selectivity, betaxolol has TIMOTHY TANG MIDTERM 1
the relative selectivity in the manner described? fewer side effects. Timolol and levobunolol are non- LEE SAY, MD EXAM - FEB
A. Betaxolol: relatively selective for β2-receptors selective β-blockers (TOP 4 - AUG 2014
B. Timolol: relatively selective for β1-receptors 2013 MED
C. Levobunolol: relatively selective for β2-receptors BOARDS;
D. Betaxolol: relatively selective for β1-receptors TOPNOTCH MD)
E. Levobunolol: relatively selective for β1-receptors

587 What hypertensive drug commonly used for the Methyldopa is a selective alpha 2 agonist. TIMOTHY TANG MIDTERM 1
outpatient treatment of preeclampsia is prominent LEE SAY, MD EXAM - FEB
for its side-effect of hemolytic anemia with a positive (TOP 4 - AUG 2014
Coomb's test? 2013 MED
A. α1 adrenergic agonist BOARDS;
B. α2 adrenergic antagonist TOPNOTCH MD)
C. α1 adrenergic agonist
D. α2 adrenergic antagonist
E. non-selective α antagonist
588 Which of the following antiarrhythmics primarily Class 2 anti arrhythmics or B blockers exert its effort TIMOTHY TANG MIDTERM 1
exerts its action on phase 4 of the cardiac action on phase 4 and prolongs the PR interval LEE SAY, MD EXAM - FEB
potential, prolonging PR interval? (TOP 4 - AUG 2014
A. Procainamide 2013 MED
B. Lidocaine BOARDS;
C. Propanolol TOPNOTCH MD)
D. Amiodarone
E. Verapamil

589 Which of the following loop diuretics is not prone to Ethacrynic acid does not contain a sulfa group. TIMOTHY TANG MIDTERM 1
induce an allergic response? LEE SAY, MD EXAM - FEB
A. Furosemide (TOP 4 - AUG 2014
B. Bumetanide 2013 MED
C. Brinzolamide BOARDS;
D. Torsemide TOPNOTCH MD)
E. Ethacrynic acid

TOPNOTCH MEDICAL BOARD PREP PHARMACOLOGY SUPEREXAM Page 68 of 83


For inquiries visit www.topnotchboardprep.com.ph or email us at topnotchmedicalboardprep@gmail.com
TOPNOTCH MEDICAL BOARD PREP PHARMACOLOGY SUPEREXAM
For inquiries visit www.topnotchboardprep.com.ph or email us at topnotchmedicalboardprep@gmail.com
Item QUESTION EXPLANATION AUTHOR TOPNOTCH
# EXAM
590 Which of the following is a major toxicity associalted Gemfibrozil is associated with cholelithiasis. TIMOTHY TANG MIDTERM 1
with gemfibrozil therapy? Myopathy is only expected in combination with LEE SAY, MD EXAM - FEB
A. Bloating and constipation statins. (TOP 4 - AUG 2014
B. Cholelithiasis 2013 MED
C. Hyperuricemia BOARDS;
D. Myopathy TOPNOTCH MD)
E. Severe cardiac arryhtmia

591 Which of these anesthetics is known to cause acute Barbiturates such as thiopental can cause acute TIMOTHY TANG MIDTERM 1
porphyria? porphyria. LEE SAY, MD EXAM - FEB
A. Nitrous oxide (TOP 4 - AUG 2014
B. Halothane 2013 MED
C. Ketamine BOARDS;
D. Thiopental TOPNOTCH MD)
E. Propofol

592 Which of the following is the drug of choice for partial Carbamazepine, phenytoin and lamotrigine are drug TIMOTHY TANG MIDTERM 1
seizures? of choices for partial seizures LEE SAY, MD EXAM - FEB
A. Valproic acid (TOP 4 - AUG 2014
B. Phenobarbital 2013 MED
C. Lamotrigine BOARDS;
D. Ethosuximide TOPNOTCH MD)
E. Topiramate

593 Which drugs reduces the risk of suicide? Clozapine is the anit-psychotic that reduces the risk TIMOTHY TANG MIDTERM 1
A. Clozapine of suicide. LEE SAY, MD EXAM - FEB
B. Clonidine (TOP 4 - AUG 2014
C. Clonazepam 2013 MED
D. Clofazimine BOARDS;
E. Clofibrate TOPNOTCH MD)

594 In the acute settings the following drugs are useful for Propylthiouracil, iodides and propanolol are the first TIMOTHY TANG MIDTERM 1
the emergent management of thyroid storm EXCEPT? line drugs for thyroid storm. Hydrocortisone may be LEE SAY, MD EXAM - FEB
A. Propanolol added for realtive adrenal insufficiency in the acute (TOP 4 - AUG 2014
B. KI attack. Methimazole may be used, but is not usually 2013 MED
C. Propylthiouracil given in the acute therapy of thyroid storm and PTU BOARDS;
D. Methimazole is preferred. TOPNOTCH MD)
E. Hydrocortisone

595 Which of the following insulin preparations mimics The long acting insulin mimics the normal secretion TIMOTHY TANG MIDTERM 1
the secretion pattern of insulin by the normal of the pancreas in the non-fed state. LEE SAY, MD EXAM - FEB
pancreas in the fasting state? (TOP 4 - AUG 2014
A. Lispro 2013 MED
B. Regular BOARDS;
C. NPH TOPNOTCH MD)
D. Glulisine
E. Detemir

596 Which of the following antibiotics does NOT follow The broad class of Beta lactams including the class of TIMOTHY TANG MIDTERM 1
time-dependent killing action? monobactams and aztreonams exhibit time- LEE SAY, MD EXAM - FEB
A. Amoxicillin dependent killing action. Bacitracin and vancomycin (TOP 4 - AUG 2014
B. Aztreonam that also act on the cell wall exhibit time dependent 2013 MED
C. Metronidazole killing. Metronidazole, FQ and AG are concentration- BOARDS;
D. Bacitracin dependent. TOPNOTCH MD)
E. Vancomycin

597 Which of the following anti-malarial is The fetus is G6PD deficient, so primaquine cannot be TIMOTHY TANG MIDTERM 1
contraindicated for pregnant women? used. To achieve radical cure for pregnant women, LEE SAY, MD EXAM - FEB
A. Chloroquine primaquine is given after delivery of the baby. (TOP 4 - AUG 2014
B. Quinine 2013 MED
C. Quinidine BOARDS;
D. Primaquine TOPNOTCH MD)
E. Mefloquine

598 Which of the following anti-neoplastic drug is highly Carmustine and Lomustine are highly lipophilic, TIMOTHY TANG MIDTERM 1
lipophilic, use for the treatment of cancers that are used to treat brain cancer. LEE SAY, MD EXAM - FEB
not readily permeable through vascular channels? (TOP 4 - AUG 2014
A. Bleomycin 2013 MED
B. Busulfan BOARDS;
C. Carmustine TOPNOTCH MD)
D. Cyclophosphamide
E. Carboplatin

599 Which novel and innovative drug is used to treat Abciximab - Glycoprotein Iib/IIIa (anti-platelet) TIMOTHY TANG MIDTERM 1
rheumatoid arthritis through its action as an antibody Omalizumab - IgE (asthma) LEE SAY, MD EXAM - FEB
against the protein CD20, found in the surface of B- Infliximab - TNF (inflammatory bowel disease) (TOP 4 - AUG 2014
cells? Trastuzumba - Her2neu (Breast cancer) 2013 MED
A. Abciximab BOARDS;
B. Rituximab TOPNOTCH MD)
C. Omalizumab
D. Infliximab
E. Trastuzumab

TOPNOTCH MEDICAL BOARD PREP PHARMACOLOGY SUPEREXAM Page 69 of 83


For inquiries visit www.topnotchboardprep.com.ph or email us at topnotchmedicalboardprep@gmail.com
TOPNOTCH MEDICAL BOARD PREP PHARMACOLOGY SUPEREXAM
For inquiries visit www.topnotchboardprep.com.ph or email us at topnotchmedicalboardprep@gmail.com
Item QUESTION EXPLANATION AUTHOR TOPNOTCH
# EXAM
600 Which of the following substances can be adsorbed by Remember that activated charcoal is effective only TIMOTHY TANG MIDTERM 1
activated charcoal, effectively decontaminating the to organic substances, so if it is inorganic like ions or LEE SAY, MD EXAM - FEB
gastrointestinal tract? acids/bases, it is not effective. Alcohol, though (TOP 4 - AUG 2014
A. Barbiturates organic does not enhance its elimination with 2013 MED
B. HCl activated charcoal. BOARDS;
C. Lithium TOPNOTCH MD)
D. Cyanide
E. Lye

601 Salmeterol has a slower onset of action as compared The quickier onset of action of formoterol can be RACHELLE FINAL EXAM -
with formoterol in terms of producing explained by diffusion microkinetic model. This MENDOZA, MD FEB 2013
bronchodilation. This is because of: model suggests that formoterol and salmeterol are (TOP 9 - AUG
A. Increased lipophilicity of salmeterol both retained in the lipid bilayer adjacent to the 2012 MED
B. Faster degradation of salmeterol beta2-adrenergic receptor. Because of salmeterol's BOARDS;
C. More efficient absorption of formoterol lipophilicity, it associates more with the lipid bilayer TOPNOTCH MD)
D. Higher affinity of formoterol to beta adrenergic than with the receptor, accounting for its slower
receptors onset of action.
E. A and D
602 If the plasma concentration of a drug declines with First Order Kinetics : elimination of the Drug is RACHELLE FINAL EXAM -
“first order kinetics,” this means that: directly proportional to its plasma concerntration. MENDOZA, MD FEB 2013
A. The halflife is constant regardless of plasma its dependent on its half life, which remains (TOP 9 - AUG
concentration constant. First order implies that no matter how 2012 MED
B. The drug is largely metabolized in the liver after much concerntration of the Drug u give it will be BOARDS;
oral administration and has low bioavailability eliminated 50% by its first half- life TOPNOTCH MD)
C. The rate of elimination is proportionate to the rate
of administration at all times
D. The drug has a high volume of distribution
E. The drug is eliminated with a constant
concentration regardlesss of the plasma
concentration
603 A 35 year old female underwent treatment for Danazol is a derivative of the synthetic steroid RACHELLE FINAL EXAM -
endometriosis for 12 months. She then developed ethisterone, a modified testosterone, also known as MENDOZA, MD FEB 2013
male-pattern hair growth, deepening of the voice and 17-alpha-ethinyl testosterone. Danazol inhibits (TOP 9 - AUG
acne. The drug given is most likely: ovarian steroidogenesis resulting in decreased 2012 MED
A. GnRH agonist secretion of estradiol and may increase androgens. BOARDS;
B. Danazol This drug causes androgenic side effects, such as TOPNOTCH MD)
C. Combined oral contraceptive pills hirsutism, deepening of the voice, acne and oily skin.
D. Medroxyprogesterone acetate
E. GnRH antagonist
604 In the case above, which of the following patient's All the other side effects of danazol is reversible, RACHELLE FINAL EXAM -
manifestations may be IRREVERSIBLE? except for deepening of the voice. MENDOZA, MD FEB 2013
A. Acne (TOP 9 - AUG
B. Male-pattern hair growth 2012 MED
C. Deepening of the voice BOARDS;
D. All of the above TOPNOTCH MD)
E. B and C

605 Which of the following anti-hypertensives has been Reserpine is an indole alkaloid antipsychotic and RACHELLE FINAL EXAM -
recalled from the market due to its side effect of antihypertensive drug that has been used for the MENDOZA, MD FEB 2013
producing rebound suicide? control of high blood pressure and for the relief of (TOP 9 - AUG
A. Guanethidine psychotic symptoms. Reserpine mediated depletion 2012 MED
B. Phenoxybenzamine of monoamine neurotransmitters in the synapses is BOARDS;
C. Reserpine often cited as evidence to the theory that depletion TOPNOTCH MD)
D. Trimetaphan of the neurotransmitters causes subsequent
depression in humans
606 Organs involved in xenobiotic metabolism are as Xenobiotic metabolizing enzymes are found in most RACHELLE FINAL EXAM -
follows, EXCEPT: tissues in the body with the highest levels located in MENDOZA, MD FEB 2013
A. Liver the tissues of the gastrointestinal tract (liver, small (TOP 9 - AUG
B. Small intestines and large intestines). Other organs that contain 2012 MED
C. Lung significant xenobiotic-metabolizing enzymes include BOARDS;
D. Nasal mucosa the tissues of the nasal mucosa and lung, which play TOPNOTCH MD)
E. None of the above important roles in the first-pass metabolism of drugs
that are administered through aerosol sprays.
607 A pure opioid antagonist with a greater affinity for μ Naloxone is a pure opioid anatagonist and is used to RACHELLE FINAL EXAM -
receptors and used for acute opioid overdose: counter the effects of opiate overdose, for example MENDOZA, MD FEB 2013
A. Morphine heroin or morphine overdose. The other drugs are (TOP 9 - AUG
B. Naloxone opioid agonists. 2012 MED
C. Codeine BOARDS;
D. Dextrometorphan TOPNOTCH MD)
E. Diphenoxylate

608 Given a hypothetical species of bacteria that is Chloramphenicol inhibits peptidyl transferase. RACHELLE FINAL EXAM -
resistant to antibacterial agents that prevent Clindamycin and erythromycin inhibits MENDOZA, MD FEB 2013
translocation during translation, which of the translocation. (TOP 9 - AUG
following can be used effectively for this bacterial 2012 MED
infection? BOARDS;
A. Chloramphenicol TOPNOTCH MD)
B. Clindamycin
C. Erythromycin
D. A and B
E. B and C

TOPNOTCH MEDICAL BOARD PREP PHARMACOLOGY SUPEREXAM Page 70 of 83


For inquiries visit www.topnotchboardprep.com.ph or email us at topnotchmedicalboardprep@gmail.com
TOPNOTCH MEDICAL BOARD PREP PHARMACOLOGY SUPEREXAM
For inquiries visit www.topnotchboardprep.com.ph or email us at topnotchmedicalboardprep@gmail.com
Item QUESTION EXPLANATION AUTHOR TOPNOTCH
# EXAM
609 The drug of choice for the treatment of bladder and Bethanechol is a muscarinic agonist that is RACHELLE FINAL EXAM -
bowel atony is: administered orally to promote GI and bladder MENDOZA, MD FEB 2013
A. Neostigmine motility, usually used in post-op ileus and diabetic (TOP 9 - AUG
B. Metoclopramide neurogenic bladder. 2012 MED
C. Pilocarpine BOARDS;
D. Bethanecol TOPNOTCH MD)
E. None of the above

610 Beta-lactamase inhibitors are usually given with beta- Synergism is giving two active drugs and the RACHELLE FINAL EXAM -
lactam containing antibiotics, such as penicillins. This combined effect is greater than the sum of their MENDOZA, MD FEB 2013
is done to produce a favorable drug interaction, effects (1+1=3). Potentiation is the addition of an (TOP 9 - AUG
specifically: inactive drug to an active drug, leading to increased 2012 MED
A. Synergism effect of the active drug (1+0=3). BOARDS;
B. Potentiation TOPNOTCH MD)
C. Additive
D. Antagonism
E. None of the above
611 A 67-year old male, on warfarin maintenance after Cimetidine inhibits metabolism of warfarin, thereby RACHELLE FINAL EXAM -
undergoing valve replacement surgery, was brought prolonging its effect. MENDOZA, MD FEB 2013
to the ER due to unresolving epistaxis and easy (TOP 9 - AUG
bruising noted for the past week. In asking the 2012 MED
history, the following substances are also being taken BOARDS;
by the patient. Which of them contributed to the TOPNOTCH MD)
patient's condition?
A. Cimetidine
B. Digoxin
C. Griseofulvin
D. Procainamide
612 Clopidogrel exerts its antithrombotic effect through Clopidogrel works by irreversibly inhibiting a RACHELLE FINAL EXAM -
which of the following mechanisms: receptor called P2Y12, an adenosine diphosphate MENDOZA, MD FEB 2013
A. Irreversible inhibition of ADP receptor (ADP) chemoreceptor on platelet cell membranes. (TOP 9 - AUG
B. Inhibition of thromboxane synthesis 2012 MED
C. Reversible blockade of glycoprotein IIb/IIIa BOARDS;
D. Conversion of plasminogen to plasmin TOPNOTCH MD)
E. Posttranslational modification of vitamin K-
dependent clotting factors
613 Sympathomimetic drug that causes mydriasis without Atropine and tropicamide cause mydriasis and RACHELLE FINAL EXAM -
cycloplegia: cycloplegia. Timolol has no mydriatic effect. MENDOZA, MD FEB 2013
A. Atropine (TOP 9 - AUG
B. Tropicamide 2012 MED
C. Phenyephrine BOARDS;
D. Pilocarpine TOPNOTCH MD)
E. Timolol

614 A 33-year old farmer, after 1 hour of spraying The patient had an insecticide poisoning. RACHELLE FINAL EXAM -
insecticides, started to note generalized body Insecticides usually contain organophosphates or MENDOZA, MD FEB 2013
weakness, drowsiness, shortness of breath, numbness carbamates. Before 6 hours, pralidoxime and (TOP 9 - AUG
and severe epigastric pain. After 10 hours, he was atropine should be both administered. In this 2012 MED
brought to the ER due to loss of consciousness. He patient, 10 hours has already elapsed, hence BOARDS;
was soaking in sweat, drooling with saliva and pralidoxime can no longer be effective. TOPNOTCH MD)
diarrheic. What should be administered to this
patient?
A. Atropine
B. Pralidoxime
C. Neostigmine
D. A and B
E. B and C
615 Lactulose is the most frequently administered Lactulose is an osmotic agent. Bulk forming agents RACHELLE FINAL EXAM -
laxative in post-MI patients. It is classified as: include insoluble fibers (psyllium). Stool softeners MENDOZA, MD FEB 2013
A. Bulk-forming laxative include docusate Na. Emollients include mineral oil. (TOP 9 - AUG
B. Osmotic laxative 2012 MED
C. Stool softener BOARDS;
D. Emollient TOPNOTCH MD)
E. None of the above

616 A hypertensive patient was found to have elevated Fibrates are indicated for hypertriglyceridemia. RACHELLE FINAL EXAM -
triglycerides, normal LDL and decreased HDL levels. Fibrates typically lower triglycerides by 20% to MENDOZA, MD FEB 2013
Which of the following is most efficacious for this 50%. Level of the good cholesterol HDL is also (TOP 9 - AUG
patient? increased. Fibrates may decrease LDL, though 2012 MED
A. Simvastatin generally to a lesser degree than statins. BOARDS;
B. Ezetimibe TOPNOTCH MD)
C. Colestipol
D. Fenofibrate
E. Orlistat
617 Ceftazidime, 3rd gen cephalosporin, is considered to RACHELLE FINAL EXAM -
Which of the following cephalosporins is highly be most efficacious agaianst pseudo monas among MENDOZA, MD FEB 2013
effective against pseudomonas? cephalosporins. (TOP 9 - AUG
A. Cefazolin 2012 MED
B. Cefuroxime BOARDS;
C. Ceftazidime TOPNOTCH MD)
D. Cefaclor
E. Ceftriaxone

TOPNOTCH MEDICAL BOARD PREP PHARMACOLOGY SUPEREXAM Page 71 of 83


For inquiries visit www.topnotchboardprep.com.ph or email us at topnotchmedicalboardprep@gmail.com
TOPNOTCH MEDICAL BOARD PREP PHARMACOLOGY SUPEREXAM
For inquiries visit www.topnotchboardprep.com.ph or email us at topnotchmedicalboardprep@gmail.com
Item QUESTION EXPLANATION AUTHOR TOPNOTCH
# EXAM
618 Which of the following antivirals is a reverse Lamivudine is used in hepatitis B as well as in HIV RACHELLE FINAL EXAM -
transcriptase inhibitor that is used in the treatment of infection. MENDOZA, MD FEB 2013
Hepatitis B infection? (TOP 9 - AUG
A. Amantadine 2012 MED
B. Ganciclovir BOARDS;
C. Lamivudine TOPNOTCH MD)
D. Interferon alpha
E. Acyclovir

619 Which of the following is NOT true about insulin The insulin receptor is a transmembrane receptor RACHELLE FINAL EXAM -
action? that is activated by insulin, IGF-I, IGF-II and belongs MENDOZA, MD FEB 2013
A. It stimulates glycogen synthesis in muscle fiber to the large class of tyrosine kinase receptors. (TOP 9 - AUG
B. It inhibits lipolysis in the adipocyte 2012 MED
C. It stimulates fatty acid synthesis in the hepatocytes BOARDS;
D. It binds to G receptor to increase intracelluar cAMP TOPNOTCH MD)
620 Disseminated intravascular coagulation (DIC) is a DIC clinical findings: ↓ Fibrinogen, ↑ fibrin split RACHELLE FINAL EXAM -
condition characterized by widespread stimulation of products, ↓ platelets, ↑ PT, ↑ PTT, fragmented red MENDOZA, MD FEB 2013
thrombosis. Which of the following is NOT consistent blood cells on peripheral smear (TOP 9 - AUG
with the clinical findings in DIC? 2012 MED
A. Increased PTT BOARDS;
B. Increased PT TOPNOTCH MD)
C. Decreased fibrinogen
D. Increased fibrin split products
E. None of the above
621 A 53 year old woman with newly diagnosed type 2 This is a disulfiram like reaction after drinking VON ANDRE DIAGNOSTIC
diabetes presents to mergency department alcohol at a cocktail party. TOLBUTAMIDE is MEDINA, MD EXAM - AUG
complaining of vomiting, severe headache, dizziness, associated with this kind of reaction (TOP 4 - FEB 2012
blurry vision, and DOB. She says that she had been at 2012 MED
a cocktail party when the symptoms began. her skin is BOARDS;
notably flushed on physical examination. Which of the TOPNOTCH MD)
following medications is responsible for this
reaction?
A. Acarbose
B. Glipizide
C. Glyburide
D. Metformin
E. Tolbutamide
622 A 65 year old man comes to the physician because he BPH. Finasteride is a 5 alpha reductase inhibitor that VON ANDRE DIAGNOSTIC
awakens to urintae several times per night and has inhibits conversion of testosterone to DHT. MEDINA, MD EXAM - AUG
developed problems starting and stopping his stream (TOP 4 - FEB 2012
of urine. A biopsy of the prostate shows enlargement 2012 MED
and dilation of the prostate glands but no dysplasia. BOARDS;
Which of the following is the most appropriate TOPNOTCH MD)
pharmacological treatment for this patient?
A. Finasteride
B. Flutamide
C. Ketoconazole
D. Spirinolactone
E. Yohimbine
623 A 45 year old man who has received long treatment this is TARDIVE DYSKINESIA, which is a VON ANDRE DIAGNOSTIC
for schizophrenia has recently been noted to display complication of long term antipsychotic therapy MEDINA, MD EXAM - AUG
involuntary movements that include lateral with the older agents like, FLUPHENAZINE. (TOP 4 - FEB 2012
deviations of the jaw and "fly catching" motions in his 2012 MED
tongue. Which of the following agents is the most BOARDS;
likely cause of his involuntary movements? TOPNOTCH MD)
A. Clozapine
B. Fluphenazine
C. Lithium
D. Risperidone
E. selegiline
624 Which of the following drugs can be used to reduce VON ANDRE DIAGNOSTIC
intraocular pressure in the treatment of glaucoma? MEDINA, MD EXAM - AUG
A. Pilocarpine (TOP 4 - FEB 2012
B. Acetazolamide 2012 MED
C. Neostigmine BOARDS;
D. Timolol TOPNOTCH MD)
E. All of the above

625 Dantrolene is the drug of choice to treat malignant VON ANDRE DIAGNOSTIC
hyperthermia caused by succinylcholine because? MEDINA, MD EXAM - AUG
A. It blocks Ca release from sarcoplasmic reticulum (TOP 4 - FEB 2012
B. It induces contraction of skeletal muscle 2012 MED
C. It increases the rate of succinylcholine BOARDS;
metabolism TOPNOTCH MD)
D. Succinylcholine binding to nicotinic receptors is
antagonized by dantrolene
E. Dantrolene acts centrally to reduce fever.
626 In the kidney, which one of the following is inhibited Thiazide diuretics inhibit sodium chloride VON ANDRE DIAGNOSTIC
by thiazide diuretics? reabsorption in DCT. MEDINA, MD EXAM - AUG
A. Sodium chloride reabsorption in the early part of (TOP 4 - FEB 2012
the distal convuluted tubule 2012 MED
B. Water removal from intracellular space by osmosis BOARDS;
C. Reabsorption of Calcium TOPNOTCH MD)
D. Aldosterone action on the nephron
E. Excretion of chloride

TOPNOTCH MEDICAL BOARD PREP PHARMACOLOGY SUPEREXAM Page 72 of 83


For inquiries visit www.topnotchboardprep.com.ph or email us at topnotchmedicalboardprep@gmail.com
TOPNOTCH MEDICAL BOARD PREP PHARMACOLOGY SUPEREXAM
For inquiries visit www.topnotchboardprep.com.ph or email us at topnotchmedicalboardprep@gmail.com
Item QUESTION EXPLANATION AUTHOR TOPNOTCH
# EXAM
627 Which of the following agents will increase Dobutamine increases myocardial contractility, but VON ANDRE DIAGNOSTIC
myocardial contractility with the LEAST effect on it has little effect on TPR. Beta 1 selective. MEDINA, MD EXAM - AUG
total peripheral resistance? (TOP 4 - FEB 2012
A. Epinephrine 2012 MED
B. Phenylephrine BOARDS;
C. Terbutaline TOPNOTCH MD)
D. Dobutamine
E. Carbachol

628 Which of the following antihypertive agents Hydralazine is CI in ptients with myocardial VON ANDRE DIAGNOSTIC
commonly used in hypertension in pregnancy is ischemia. MEDINA, MD EXAM - AUG
contraindicated to patients having myocardial (TOP 4 - FEB 2012
ischemia? 2012 MED
A. Methyldopa BOARDS;
B. Methergine TOPNOTCH MD)
C. Clonidine
D. Hydralazine
E. Magnesium sulfate
629 Which of the following beta blocker drugs has/have Pindolol and Acebutolol have intrinsic VON ANDRE DIAGNOSTIC
intrinsic sympathomimetic activity? sympathomimetic activity. MEDINA, MD EXAM - AUG
A. Timolol (TOP 4 - FEB 2012
B. Acebutolol 2012 MED
C. Pindolol BOARDS;
D. Betaxolol TOPNOTCH MD)
E. B and C

630 Which of the following nonselective , irreversible DOC for pheochromocytoma is Phenoxybenzamine, VON ANDRE DIAGNOSTIC
alpha blocker is the drug of choice as a preoperative it is a nonselective, IRREVERSIBLE alpha blocker.. MEDINA, MD EXAM - AUG
agent used in Pheochromocytoma? Phentolamine is a reversible alpha blocker. (TOP 4 - FEB 2012
A. Phentolamine 2012 MED
B. Phenoxybenzamine BOARDS;
C. Prazosin TOPNOTCH MD)
D. Yohimbine
E. Clonidine

631 Succor Mendiola was diagnosed to have glaucoma. Pilocarpine induces ciliary muscle contraction VON ANDRE DIAGNOSTIC
Which of the following drugs induces ciliary muscle thereby opening the trabecular meshwork and MEDINA, MD EXAM - AUG
contraction thereby opening the trabecular increasing the outflow (TOP 4 - FEB 2012
meshwork and increasing the outflow? 2012 MED
A. Timolol BOARDS;
B. Mannitol TOPNOTCH MD)
C. Epinephrine
D. Latanoprost
E. Pilocarpine
632 Which of the following vasodilators has the common Nitroprusside is a parenteral vasodilator known to VON ANDRE DIAGNOSTIC
side effect of headache, hypotension and CYANIDE cause CYANIDE toxicity. MEDINA, MD EXAM - AUG
toxicity? (TOP 4 - FEB 2012
A. Fenoldopam 2012 MED
B. Nitroprusside BOARDS;
C. Nifedipine TOPNOTCH MD)
D. Dopamine
E. Minoxidil

633 Which of the following drug/s is/are notorius in VON ANDRE DIAGNOSTIC
causing gingival hyperplasia? MEDINA, MD EXAM - AUG
A. Nifedipine (TOP 4 - FEB 2012
B. Cyclosporine 2012 MED
C. Phenytoin BOARDS;
D. Verapamil TOPNOTCH MD)
E. All of the above

634 Which of the following statements concerning VON ANDRE DIAGNOSTIC


barbiturates is true? MEDINA, MD EXAM - AUG
A. Barbiturates can increase bleeding time when (TOP 4 - FEB 2012
administered to patients taking anticoagulants 2012 MED
B. Barbiturates are contraindicated in patients with BOARDS;
acute intermittent porphyria TOPNOTCH MD)
C. Patients tolerant to the therapeutic actions of
barbiturates are also tolerant to analgesic effect of
morphine
D. Barbiturates are used to prevent withdrawal
symptoms associated with heroin dependence
E. All of the above
635 Which of the following has the lowest incidence of Clozapine is a new drug with least extrapyramidal VON ANDRE DIAGNOSTIC
extrapyramidal symptoms but is known to cause effect but is known to cause agranulocytosis. MEDINA, MD EXAM - AUG
agranulocytosis? (TOP 4 - FEB 2012
A. Clozapine 2012 MED
B. Fluphanazine BOARDS;
C. Chlorpromazine TOPNOTCH MD)
D. Pimozide
E. Haloperidol

TOPNOTCH MEDICAL BOARD PREP PHARMACOLOGY SUPEREXAM Page 73 of 83


For inquiries visit www.topnotchboardprep.com.ph or email us at topnotchmedicalboardprep@gmail.com
TOPNOTCH MEDICAL BOARD PREP PHARMACOLOGY SUPEREXAM
For inquiries visit www.topnotchboardprep.com.ph or email us at topnotchmedicalboardprep@gmail.com
Item QUESTION EXPLANATION AUTHOR TOPNOTCH
# EXAM
636 Which of the following statements about the VON ANDRE DIAGNOSTIC
mechanism of action of ipratropium? MEDINA, MD EXAM - AUG
A. It acts cebtrally to decrease vagal Ach release (TOP 4 - FEB 2012
B. It inhibits pulmonary Ach receptors 2012 MED
C. It decreases mast cell release of histamine BOARDS;
D. It blocks the action of histamine at H1 receptors TOPNOTCH MD)
E. None of the above

637 Which of the following anti TB drugs is known to VON ANDRE DIAGNOSTIC
cause hyperuricemia and is noted to be the most MEDINA, MD EXAM - AUG
hepatotoxic? (TOP 4 - FEB 2012
A. Rifampicin 2012 MED
B. Ethambutol BOARDS;
C. Pyrazinamide TOPNOTCH MD)
D. Streptomycin
E. Isoniazid

638 A 5 year old boy with Trisomy 21 was diagnosed to Asparaginase, depletes serum asparagine. It is the VON ANDRE DIAGNOSTIC
have acute lymphoblastic leukemia. What is the most main and the most impt drug in the management od MEDINA, MD EXAM - AUG
important drug in the therapeutic regimen of this ALL. (TOP 4 - FEB 2012
disease entity? 2012 MED
A. Rituximab BOARDS;
B. Cytarabine TOPNOTCH MD)
C. Idarubucin
D. Asparaginase
E. Prednisone
639 Which of the following ergot alkaloids is known to VON ANDRE DIAGNOSTIC
cause retroperitoneal fibrosis? MEDINA, MD EXAM - AUG
A. Methylsergide (TOP 4 - FEB 2012
B. Dihydroergotamine 2012 MED
C. Methylergonovine BOARDS;
D. Sumatriptan TOPNOTCH MD)
E. None of the above

640 Which of the following medication-rescue therapy VON ANDRE DIAGNOSTIC


pair is INCORRECT? MEDINA, MD EXAM - AUG
A. Methotrexate : Leucovorin (TOP 4 - FEB 2012
B. Cyclophosphamide : MESNA 2012 MED
C. Doxorubicin : Dexrazoxane BOARDS;
D. Cisplatin : Amifostine TOPNOTCH MD)
E. None of the above

641 Allen Dimo, is a 38 y/o G8P8(6208), known Captopril is an ACE inhibitor and its use during LITO JAY DIAGNOSTIC
hypertensive and diabetic, turned out to be pregnant gestation may result in IUGR, acute fetal renal MACARAIG, MD EXAM - AUG
for 5 weeks already. What anti-hypertensive failure, PDA, and even fetal death. (TOP 8 - FEB 2013
medication should you AVOID to give to the patient? 2013 MED
A. Propranolol BOARDS;
B. Captopril TOPNOTCH MD)
C. Hydralazine
D. Methyldopa
E. None of the above
642 Assuming that the patient above is NOT pregnant. ACE inhibitors block the formation of Angiotensin II LITO JAY DIAGNOSTIC
What is the best medication for her? which is a potent vasoconstrictor of the afferent and MACARAIG, MD EXAM - AUG
A. Propranolol efferent Renal blood vessels. When administered to (TOP 8 - FEB 2013
B. Captopril the patient, it prevents diabetic nephropathy by 2013 MED
C. Hydralazine increasing blood flow to the kidneys. BOARDS;
D. Methyldopa TOPNOTCH MD)
E. None of the above

643 Excessive use of Nitroprusside may result in Methylene blue LITO JAY DIAGNOSTIC
Methemoglobinemia. The antidote for this condition MACARAIG, MD EXAM - AUG
is? (TOP 8 - FEB 2013
A. Methylene blue 2013 MED
B. Silver Nitrate BOARDS;
C. 100% oxygen TOPNOTCH MD)
D. Dialysis
E. Blood transfusion
644 This is the anti-Asthma drug which is only of worth if LITO JAY DIAGNOSTIC
given as prophylaxis. MACARAIG, MD EXAM - AUG
A. Salbutamol (TOP 8 - FEB 2013
B. Budesonide 2013 MED
C. Salmeterol BOARDS;
D. Cromolyn TOPNOTCH MD)
E. Hydrocortisone

645 One of the following drugs causes an increase in Rifampicin is an inducer. All other choices are LITO JAY DIAGNOSTIC
metabolism of other drugs when administered inhibitors. MACARAIG, MD EXAM - AUG
simultaneously. (TOP 8 - FEB 2013
A. Valproic Acid 2013 MED
B. Rifampicin BOARDS;
C. Venlafaxine TOPNOTCH MD)
D. Ritonavir
E. Ketoconazole

TOPNOTCH MEDICAL BOARD PREP PHARMACOLOGY SUPEREXAM Page 74 of 83


For inquiries visit www.topnotchboardprep.com.ph or email us at topnotchmedicalboardprep@gmail.com
TOPNOTCH MEDICAL BOARD PREP PHARMACOLOGY SUPEREXAM
For inquiries visit www.topnotchboardprep.com.ph or email us at topnotchmedicalboardprep@gmail.com
Item QUESTION EXPLANATION AUTHOR TOPNOTCH
# EXAM
646 Before intubation, prior to surgery, a 66 year-old Succinylcholine is being metabolized by the enzyme LITO JAY DIAGNOSTIC
male Asthmatic was injected with Succinylcholine. He pseudocholinesterase, hence deficiency or lack of MACARAIG, MD EXAM - AUG
was noted to remain apneic and paralyzed for an this enzyme will lead to extended drug effect of (TOP 8 - FEB 2013
extended period of time. The patient most likely Succinyhlcholine. 2013 MED
A. Received an excessive dose of the drug BOARDS;
B. has been recieving aminoglycosides TOPNOTCH MD)
C. has impaired renal function
D. had an allergic reaction
E. has pseudocholinesterase deficiency
647 What is the drug of choice for trigeminal neuralgia? SIMILAR TO PREVIOUS BOARD EXAM LITO JAY DIAGNOSTIC
A. Lamotrigine CONCEPT/PRINCIPLE MACARAIG, MD EXAM - AUG
B. Gabapentin (TOP 8 - FEB 2013
C. Pregabalin 2013 MED
D. Carbamazepine BOARDS;
E. B or C TOPNOTCH MD)

648 Which among the lipid-lowering medicatins causes LITO JAY DIAGNOSTIC
flushing? MACARAIG, MD EXAM - AUG
A. Statins (TOP 8 - FEB 2013
B. Niacin 2013 MED
C. Cholestyramine BOARDS;
D. A and B TOPNOTCH MD)
E. B and C

649 Which anti-parasitic drug exerts its actions on the SIMILAR TO PREVIOUS BOARD EXAM LITO JAY DIAGNOSTIC
microtubules of helminths? CONCEPT/PRINCIPLE MACARAIG, MD EXAM - AUG
A. Mebendazole (TOP 8 - FEB 2013
B. Stibogluconate 2013 MED
C. Ivermectin BOARDS;
D. Diloxanide Furoate TOPNOTCH MD)
E. Metronidazole

650 Zidovudine is a very remarkable drug for HIV. SIMILAR TO PREVIOUS BOARD EXAM LITO JAY DIAGNOSTIC
Howeve,r precaution for its use should be observed CONCEPT/PRINCIPLE MACARAIG, MD EXAM - AUG
because of this side effect. (TOP 8 - FEB 2013
A. pancreatitis 2013 MED
B. diarrhea BOARDS;
C. myelosuppression TOPNOTCH MD)
D. Pulmonary fibrosis
E. Optic neuritis

651 A 50 year old female consulted due to palpitations. RAI may induce production of cancer cells but ALL LITO JAY DIAGNOSTIC
This was accompanied by frequent bowel movements patients administered with RAI will suffer from MACARAIG, MD EXAM - AUG
and heat intolerance. You gave her Radioactive iodine permanent hypothyroidism and will require (TOP 8 - FEB 2013
and you know that its most common side effect is? exogenous thyroxine as supplement 2013 MED
A. Papillary Thyroid CA BOARDS;
B. Medullary Thyroid CA TOPNOTCH MD)
C. Rebound hyperthyroidism
D. Permanent hypothyroidism
E. Rebound tachycardia

652 Which among the following diuretics can also be used Carbonic anhydrase inhibitors LITO JAY DIAGNOSTIC
for mountain sickness? MACARAIG, MD EXAM - AUG
A. Acetazolamide (TOP 8 - FEB 2013
B. Furosemide 2013 MED
C. Thiazide BOARDS;
D. Mannitol TOPNOTCH MD)
E. None

653 Cimetidine is used as anti-PUD drug and it acts as Cimetidine is a H2 blocker LITO JAY DIAGNOSTIC
a/an? MACARAIG, MD EXAM - AUG
A. mucuprotectant (TOP 8 - FEB 2013
B. H1 blocker 2013 MED
C. pH neutralizer BOARDS;
D. A and B TOPNOTCH MD)
E. None

654 Which among the following is/are opioid agonist? Loperamide is an opioid that acts on Kappa LITO JAY DIAGNOSTIC
A. Hyoscine receptors to increase gastric emptying time. MACARAIG, MD EXAM - AUG
B. Metoclopramide (TOP 8 - FEB 2013
C. Loperamide 2013 MED
D. Attapulgite BOARDS;
E. AlMg(OH) TOPNOTCH MD)

TOPNOTCH MEDICAL BOARD PREP PHARMACOLOGY SUPEREXAM Page 75 of 83


For inquiries visit www.topnotchboardprep.com.ph or email us at topnotchmedicalboardprep@gmail.com
TOPNOTCH MEDICAL BOARD PREP PHARMACOLOGY SUPEREXAM
For inquiries visit www.topnotchboardprep.com.ph or email us at topnotchmedicalboardprep@gmail.com
Item QUESTION EXPLANATION AUTHOR TOPNOTCH
# EXAM
655 After the oath-taking, you decided to establish a clinic The antidote for organophosphate poisoning is LITO JAY DIAGNOSTIC
on your province. A 63 year-old farmer was brought Pralidoxime. BUT the golden period of 6 hours had MACARAIG, MD EXAM - AUG
to you, soaked in sweat and salivating. When you already passed. And the best medication to give in (TOP 8 - FEB 2013
asked his son about the history, you were told that he this case is ATROPINE. 2013 MED
was found in that condition at the middle of the field, BOARDS;
which is an 8-hour travel from your clinic, with a TOPNOTCH MD)
bottle of Malathion on his side. Assuming you have all
the following medications in hand, what is the most
appropriate to give?
A. Pralidoxime
B. Physostigmine
C. Atropine
D. Scopolamine
E. Atracurium
656 On your clinic, an 18 month old infant was brought This is absence seizure and DOC is Ethosuximide LITO JAY DIAGNOSTIC
for consult due to "blank stares" with a duration of 5- MACARAIG, MD EXAM - AUG
10 seconds. The best medication to be given for this (TOP 8 - FEB 2013
case is 2013 MED
A. Phenobarbital BOARDS;
B. Valproic acid TOPNOTCH MD)
C. Ethosuximide
D. Levepiracetam
E. Carbamazepine
657 You encountered a 2 year old patient suffering from Salbutamol is a short-acting beta agonist and may LITO JAY DIAGNOSTIC
an acute asthmatic attack. You immediately nebulized stimulate beta 1 receptors of the heart, leading to MACARAIG, MD EXAM - AUG
him with Salbutamol. Knowing its mode of action, you transient tachycardia. (TOP 8 - FEB 2013
would expect that the patient may have 2013 MED
A. Seizures BOARDS;
B. Dry mouth TOPNOTCH MD)
C. Tachycardia
D. Somnolence
E. Excessive salivation
658 Furosemide is a loop diuretic that is known for its Kanamycin is an aminoglycoside is very ototoxic LITO JAY DIAGNOSTIC
ototoxicity. Which among the following drugs should MACARAIG, MD EXAM - AUG
you AVOID administering it simultaneously? (TOP 8 - FEB 2013
A. Penicillin 2013 MED
B. Ceftriaxone BOARDS;
C. Clindamycin TOPNOTCH MD)
D. Kanamycin
E. Erythromycin

659 Which among the following drugs is commonly used LITO JAY DIAGNOSTIC
in controlling the manic phase of a bipolar patient? MACARAIG, MD EXAM - AUG
A. Lithium (TOP 8 - FEB 2013
B. Selegiline 2013 MED
C. Reserpine BOARDS;
D. Haloperidol TOPNOTCH MD)
E. Biperiden

660 Being an IM resident, you are assigned to watch over Ondansetron is a 5TH-3 anatagonist and is the drug LITO JAY DIAGNOSTIC
a patient who is undergoing chemotherapy. Which of choice for post-chemo vomiting. MACARAIG, MD EXAM - AUG
drug is best to be given to prevent post-chemo (TOP 8 - FEB 2013
vomiting? 2013 MED
A. Metoclopramide BOARDS;
B. Hyoscine N-Butyl-bromide TOPNOTCH MD)
C. Ondansteron
D. Methylergonovine
E. Methyrgine
661 This is time of drug solubility and absorption and polar, ionized - less soluble in lipids, excreted; non - HAZEL KAREN MIDTERM 2 -
clearance? polar, unionized - more soluble, absorbed RAZ, MD (TOP 6 - AUG 2013
A. Ionized drugs are better absorbed in the GIT FEB 2013 MED
B. Polar drugs have decreased clearance BOARDS;
C. Non-ionized drugs are better absorbed in the GIT TOPNOTCH MD)
D. Non-polar drugs have increased clearance
E. None of the above

662 Route of administration with instantaneous Compared with other routes of administration, the HAZEL KAREN MIDTERM 2 -
absorption, bypasses first pass effect with I00% intravenous route is the fastest way to deliver fluids RAZ, MD (TOP 6 - AUG 2013
bioavailabilty? and medications throughout the body. FEB 2013 MED
A. IV BOARDS;
B. SL TOPNOTCH MD)
C. Oral
D. Rectal
E. None of the above

663 Drugs which undergo zero order kinetics, except? WHATPET - Warfarin, Heparin, ASA, Tolbutamide, HAZEL KAREN MIDTERM 2 -
A. Warfarin Phenytoin, Ethanol, Theophylline RAZ, MD (TOP 6 - AUG 2013
B. Heparin FEB 2013 MED
C. Hydralazine BOARDS;
D. Aspirin TOPNOTCH MD)
E. None of the above

TOPNOTCH MEDICAL BOARD PREP PHARMACOLOGY SUPEREXAM Page 76 of 83


For inquiries visit www.topnotchboardprep.com.ph or email us at topnotchmedicalboardprep@gmail.com
TOPNOTCH MEDICAL BOARD PREP PHARMACOLOGY SUPEREXAM
For inquiries visit www.topnotchboardprep.com.ph or email us at topnotchmedicalboardprep@gmail.com
Item QUESTION EXPLANATION AUTHOR TOPNOTCH
# EXAM
664 In this phase of clinical trials, the drig is evaluated in a I - normal volunteers, pharmacokinteics and dose HAZEL KAREN MIDTERM 2 -
moderate number of target patients to test dose sensitive acute effects RAZ, MD (TOP 6 - AUG 2013
efficacy of the drug? II - moderate number of PATIENTS, dose efficacy FEB 2013 MED
A. I III - large # of patients, toxicities BOARDS;
B. II IV - post - marketing, idiosyncratic reactions TOPNOTCH MD)
C. III
D. IV
E. None of the above

665 Which of the following drugs is a direct acting Bethanecol for bowel and bladder atony HAZEL KAREN MIDTERM 2 -
cholinoninetic used for treatment of glaucoma, RAZ, MD (TOP 6 - AUG 2013
sjoqren and sicca syndrome? FEB 2013 MED
A. Betanechol BOARDS;
B. Pilocarpine TOPNOTCH MD)
C. Physostigmine
D. Neostigmine
E. None of the above

666 Muscarinic antagonists used for the treatment of Triamterene - potassium sparing diuretic HAZEL KAREN MIDTERM 2 -
parkinsonism, except? RAZ, MD (TOP 6 - AUG 2013
A. Trinexyphenidyl FEB 2013 MED
B. Triamterene BOARDS;
C. Benztropine TOPNOTCH MD)
D. Biperiden
E. None of the above

667 Drug of choice for uncomplicated essential Thiazide diuretics are first line of treatment for HAZEL KAREN MIDTERM 2 -
hypertension? uncomplicated hypertension. RAZ, MD (TOP 6 - AUG 2013
A. Furosemide FEB 2013 MED
B. Metoprolol BOARDS;
C. HCTZ TOPNOTCH MD)
D. Captopril
E. None of the above

668 B - Blocker with intrinsic sympathomimetic activity? Acebutolol and Pindolo has ISA. HAZEL KAREN MIDTERM 2 -
A. Acebutolol RAZ, MD (TOP 6 - AUG 2013
B. Atenolol FEB 2013 MED
C. esmolol BOARDS;
D. metroprolol TOPNOTCH MD)
E. None of the above

669 What is the MOST-SLECTIVE alpha 1 selective HAZEL KAREN MIDTERM 2 -


antagonist used in the treatment of BPH? However, all of the mentioned drugs are alpha-1 RAZ, MD (TOP 6 - AUG 2013
A. Prazosin selective antagonists used in the treatment of BPH FEB 2013 MED
B. tamsulosin (Prazosin, Tamsulosin, Terazosin, Doxazosin). BOARDS;
C. Terazosin Tamsulosin is the most selective alpha-1 selective TOPNOTCH MD)
D. Doxazosin antagonist.
E. all of the above . Tamsulosin is a selective α1 receptor antagonist
that has preferential selectivity for the α1A receptor
in the prostate versus the α1B receptor in the blood
vessels.
When alpha 1 receptors in the bladder neck and the
prostate are blocked, this causes a relaxation in
smooth muscle and therefore less resistance to
urinary flow. Due to this the pain associated with
BPH can be reduced.
670 Mechanism of action of cyanide poisoning? Complex I - amytal, rotenone; Complex II - malonate; HAZEL KAREN MIDTERM 2 -
A. Inhibits complex III of ETC Complex III - antimycin A, Dimercaprol; Complex IV - RAZ, MD (TOP 6 - AUG 2013
B. Uncoupler CN, CO, Na azide; Complex V - Oligomycin
FEB 2013 MED
C. Inhibits oxidative bursts Uncoupler - ASA, 2,4 - DNP BOARDS;
D. Inhibits complex IV of ETC TOPNOTCH MD)
E. None of the above

671 An anticoagulant which has a more selective effect on It is an anticoagulant used to prevent and treat deep HAZEL KAREN MIDTERM 2 -
factor Xa? vein thrombosis or pulmonary embolism, and is RAZ, MD (TOP 6 - AUG 2013
A. hepanin given as a subcutaneous injection (by a health care FEB 2013 MED
B. Lepirodin provider or the patient). BOARDS;
C. enoxaparin TOPNOTCH MD)
D. warfarin
E. None of the above

672 Antihyperlipidemic agent which is contraindicated in Fibrates and cholestipol increases risk of forming HAZEL KAREN MIDTERM 2 -
patients prone to GB stone formation? gallstones. RAZ, MD (TOP 6 - AUG 2013
A. Genifibrozil FEB 2013 MED
B. Niacin BOARDS;
C. Ezetimibe TOPNOTCH MD)
D. Simvastatin
E. None of the above

TOPNOTCH MEDICAL BOARD PREP PHARMACOLOGY SUPEREXAM Page 77 of 83


For inquiries visit www.topnotchboardprep.com.ph or email us at topnotchmedicalboardprep@gmail.com
TOPNOTCH MEDICAL BOARD PREP PHARMACOLOGY SUPEREXAM
For inquiries visit www.topnotchboardprep.com.ph or email us at topnotchmedicalboardprep@gmail.com
Item QUESTION EXPLANATION AUTHOR TOPNOTCH
# EXAM
673 Mechanism of action of chloroquine except:? Chloroquine suppress T - lymphocyte response HAZEL KAREN MIDTERM 2 -
A. Suppression of B-Lymphocyte response to RAZ, MD (TOP 6 - AUG 2013
mitogens FEB 2013 MED
B. Free-Radical Trapping BOARDS;
C. Lysosomal enzyme stabilization TOPNOTCH MD)
D. Decreased leukocyte chemotaxis
E. None of the above

674 Mechanism of action of diazepam? Increases frequency of channel opening HAZEL KAREN MIDTERM 2 -
A. Increases duration of Cl- channel opening RAZ, MD (TOP 6 - AUG 2013
B. Increases frequency of Cl- channel opening FEB 2013 MED
C. Agonist at 5HT1a and D2 receptors BOARDS;
D. Blocks voltage gated Na+ channels TOPNOTCH MD)
E. None of the above

675 Drug of choice for partial seizures? Diazepam - s. epilepticus HAZEL KAREN MIDTERM 2 -
A. Diazepam Phenobarbital - s. epilepticus RAZ, MD (TOP 6 - AUG 2013
B. Phenobarbital Valproic Acid - absence, myoclonic FEB 2013 MED
C. Valproic Acid Carbamazepine - partial seizures BOARDS;
D. Carbamazepine TOPNOTCH MD)
E. None of the above

676 Causes analgesia, amnesia and catatonia with Dissociative anesthesia HAZEL KAREN MIDTERM 2 -
retained conciousness? RAZ, MD (TOP 6 - AUG 2013
A. Midazolam FEB 2013 MED
B. Ketamine BOARDS;
C. Thiopental TOPNOTCH MD)
D. Fentanyl
E. None of the above

677 Early morning hyperglycemia characterized by Waning of insulin dose - low dose insulin HAZEL KAREN MIDTERM 2 -
development of hypoglycemia around 3AM and pre- Dawn Phenomenon - wrong type of insulin RAZ, MD (TOP 6 - AUG 2013
breakfast hyperglycemia:? Somogyi Effect - high dose of insulin FEB 2013 MED
A. Waning of insulin dose BOARDS;
B. Dawn Phenomenon TOPNOTCH MD)
C. Somogyi Effect
D. Combined Waning + Dawn
E. None of the above

678 Antibiotic which inhibits transpeptidation by binding Chloramphenicol is a bacteriostatic drug that stops HAZEL KAREN MIDTERM 2 -
to 50s subunit, used in the treatment of meningitis bacterial growth by inhibiting protein synthesis. RAZ, MD (TOP 6 - AUG 2013
secondary to strep, H.inf, Neisseria neningitidis, Chloramphenicol prevents protein chain elongation FEB 2013 MED
causes aplastic anemia as its most common by inhibiting the peptidyl transferase activity of the BOARDS;
idiosyncratic effect? bacterial ribosome. TOPNOTCH MD)
A. Chloramphenicol
B. Erythromycin
C. Linezolid
D. Clindamycin
E. None of the above
679 Mechanism of action of mebendazole? Mebendazole causes degenerative alterations in the HAZEL KAREN MIDTERM 2 -
A. Inhibits microtubule assembly tegument and intestinal cells of the worm by binding RAZ, MD (TOP 6 - AUG 2013
B. Inhibits glucose uptake in rematudes to the colchicine-sensitive site of tubulin, thus FEB 2013 MED
C. Causes dipolarization induced paralysis inhibiting its polymerization or assembly into BOARDS;
D. Increase permeability of membrane to Ca2+ microtubules. The loss of the cytoplasmic TOPNOTCH MD)
causing muscle paralysis microtubules leads to impaired uptake of glucose by
E. None of the above the larval and adult stages of the susceptible
parasites, and depletes their glycogen stores.
680 Drug of choice for prostatic carcinoma? Leuprolide acts as an agonist at pituitary GnRH HAZEL KAREN MIDTERM 2 -
A. gemcitabine receptors. By interrupting the normal pulsatile RAZ, MD (TOP 6 - AUG 2013
B. Paditaxel stimulation of, and thus desensitizing, the GnRH FEB 2013 MED
C. Cisplatin receptors, it indirectly down regulates the secretion BOARDS;
D. Leuprolide of gonadotropins luteinizing hormone (LH) and TOPNOTCH MD)
E. None of the above follicle-stimulating hormone (FSH), leading to
hypogonadism and thus a dramatic reduction in
estradiol and testosterone levels in both sexes.
681 Doxylamine is an ethanolamine H1 antagonist drug Answer: C. Vitamin B6 (pp. 389 [.pdf file], Katzung MICHELLE JAY MIDTERM 1 -
previously utilized as a possible treatment of nausea Pharmacology, 9th edition) FRANCISCO, MD AUG 2013
and vomiting of pregnancy. Now, this drug is still Notes: Several H1 antagonist drugs have been (TOP 9 - FEB
available over-the-counter as sleep aid. What B- studied for possible use in treating "morning 2013 MED
vitamin is a component of Doxylamine? sickness." The piperazine derivatives were BOARDS;
A. Vitamin B2 withdrawn from such use when it was demonstrated TOPNOTCH MD)
B. Vitamin B3 that they have teratogenic effects in rodents.
C. Vitamin B6 Doxylamine, an ethanolamine H1 antagonist, was
D. Vitamin B12 promoted for this application as a component of
Bendectin, a prescription medication that also
contained pyridoxine.
* SIMILAR TO PREVIOUS BOARD EXAM
CONCEPT/PRINCIPLE

TOPNOTCH MEDICAL BOARD PREP PHARMACOLOGY SUPEREXAM Page 78 of 83


For inquiries visit www.topnotchboardprep.com.ph or email us at topnotchmedicalboardprep@gmail.com
TOPNOTCH MEDICAL BOARD PREP PHARMACOLOGY SUPEREXAM
For inquiries visit www.topnotchboardprep.com.ph or email us at topnotchmedicalboardprep@gmail.com
Item QUESTION EXPLANATION AUTHOR TOPNOTCH
# EXAM
682 A few substances that are almost completely inert in Answer: B. Antiflatulent (pp. 1520 [.pdf file], MICHELLE JAY MIDTERM 1 -
the chemical sense nevertheless have significant Katzung Pharmacology, 9th edition) FRANCISCO, MD AUG 2013
pharmacologic effects. An example of these Notes: Some antacid preparations include (TOP 9 - FEB
substances is Simethicone, which is included in many simethicone, an antiflatulent to relieve symptoms of 2013 MED
antacid preparations in order to act as bloating and pressure. BOARDS;
A. Antispasmodic *SIMILAR TO PREVIOUS BOARD EXAM TOPNOTCH MD)
B. Antiflatulent CONCEPT/PRINCIPLE
C. Antiemetic
D. Antiminth
683 3. Future clinicians may screen every patient for a Answer: C. Pharmacogenetics (pp. 3 and 82 [.pdf MICHELLE JAY MIDTERM 1 -
variety of individual differences in drug response. file] Katzung Pharmacology, 9th edition) FRANCISCO, MD AUG 2013
Differences in the rate of acetylation of isoniazid (TOP 9 - FEB
among groups of individuals exemplifies 2013 MED
A. Pharmacodynamics BOARDS;
B. Posology TOPNOTCH MD)
C. Pharmacogenetics
D. Biochemorphology
684 A college friend consults you regarding the suitability Answer: A. A drug which inhibits the release of MICHELLE JAY MIDTERM 1 -
of the medication given by his doctor for severe norepinephrine from sympathetic nerve FRANCISCO, MD AUG 2013
hypertension. He complains of postural and exercise endings. The adverse effects described pertains to (TOP 9 - FEB
hypotension (“dizziness”), some diarrhea, and Guanethidine (pp. 237, 241, 244 and 251 [.pdf file], 2013 MED
problems with ejaculation during sexual activity. Katzung Pharmacology, 9th edition). BOARDS;
Given those adverse effects stated, which of the Notes: TOPNOTCH MD)
following mechanisms of action is most likely related • A drug which decreases blood pressure as a result
to the drug described by your friend? of a decrease in cardiac output due to its
A. A drug which inhibits the release of nonselective β-blockade – Propranolol
norepinephrine from sympathetic nerve endings. • An oral vasodilator which relaxes smooth muscles
B. A drug which decreases blood pressure as a result of arterioles, thereby decreasing the systemic
of a decrease in cardiac output due to its nonselective vascular resistance – Hydralazine
β-blockade. • A drug which inhibits the converting enzyme
C. An oral vasodilator which relaxes smooth muscles peptidyl dipeptidase that hydrolyzes angiotensin I to
of arterioles, thereby decreasing the systemic angiotensin II and also inactivates bradykinin –
vascular resistance. Captopril
D. A drug which inhibits the converting enzyme
peptidyl dipeptidase that hydrolyzes angiotensin I to
angiotensin II and also inactivates bradykinin.
685 The Board of Medicine decided to change the Answer: C. Loperamide (pp. 713 [.pdf file], Katzung MICHELLE JAY MIDTERM 1 -
program of examination for the August 2013 Pharmacology, 9th edition) – must know, MPL 1.0 FRANCISCO, MD AUG 2013
Physician Licensure Examination (PLE) changing the (TOP 9 - FEB
first subject to Pharmacology. Because of this, a non- 2013 MED
prepared Non-Topnotch examinee suddenly got an BOARDS;
attack of diarrhea while on his way to the testing site. TOPNOTCH MD)
If he will stop at a nearby drugstore to buy an over-
the-counter opioid with an antidiarrheal action, he
will be asking for
A. Codeine
B. Dextromethorphan
C. Loperamide
D. Diphenoxylate
686 Sources of drugs can either be natural or synthetic. Answer: D. Amphotericin B (pp. 1104 [.pdf file], MICHELLE JAY MIDTERM 1 -
These might be derived from plant sources, animal or Katzung Pharmacology, 9th edition.) FRANCISCO, MD AUG 2013
marine products, minerals, microorganisms and Notes: incomplete list (TOP 9 - FEB
others from laboratory sources. Many antibiotics Streptomycin à Strept. griseus 2013 MED
were derived from Genus Streptomyces, Penicillium, Neomycin à Strept. fradiae BOARDS;
Micromonospora, etc. Which of the following drugs is Paromomycin à Strept. rimosus TOPNOTCH MD)
a polyene antibiotic derived from Streptomyces Kanamycin (der. Amikacin) à Strept.kanamyceticus
nodosus? Tobramycin à Strept. tenebrarius
A. Netilmicin Ivermectin à Strept. avermitilis
B. Amikacin Mitomycin à Strept. caespitosus
C. Streptomycin Erythromycin à Strept. erythreus
D. Amphotericin B Lincomycin (der. Clindamycin) à Strept. lincolnensis
Rifamycin (der. Rifampicin) à Strept. mediterranei
Gentamicin à Micromonospora purpurea
Sisomicin (der. Netilmicin) à Micromonospora
inyoensis

687 Mrs. T. Anga, 25 yo, G1P1 was misdiagnosed to have Answer: A. Methyldopa (pp. 235 [.pdf file], Katzung MICHELLE JAY MIDTERM 1 -
preeclampsia because of the inaccuracy of her history Pharmacology, 9th edition). FRANCISCO, MD AUG 2013
and was given an anti-hypertensive to control her Notes: The symptoms experienced by the patient are (TOP 9 - FEB
blood pressure. Fortunately, patient had no consistent with Autoimmune Hemolytic Anemia 2013 MED
postpartum complications and BP was controlled. She which is a rare adverse effect of methyldopa. BOARDS;
continued taking the previously given Methyldopa is a drug which lowers blood pressure TOPNOTCH MD)
antihypertensive but was lost to follow-up. After chiefly by reducing the peripheral vascular
maintaining the drug for more than 12 months, resistance and variably reduces heart rate and
patient developed intermittent episodes of headache, cardiac output. Ten to twenty percent of patients
myalgias, jaundice and passage of dark brown urine. undergoing therapy for longer than 12 months from
She finally went back to her physician for consult and methyldopa will have positive Coomb’s Test (or
laboratory testing revealed a positive Antiglobulin Antiglobulin Test).
Test. If the condition was drug-induced, what is the
most likely offending agent?
A. Methyldopa
B. Hydralazine
C. Clonidine
D. Magnesium Sulfate

TOPNOTCH MEDICAL BOARD PREP PHARMACOLOGY SUPEREXAM Page 79 of 83


For inquiries visit www.topnotchboardprep.com.ph or email us at topnotchmedicalboardprep@gmail.com
TOPNOTCH MEDICAL BOARD PREP PHARMACOLOGY SUPEREXAM
For inquiries visit www.topnotchboardprep.com.ph or email us at topnotchmedicalboardprep@gmail.com
Item QUESTION EXPLANATION AUTHOR TOPNOTCH
# EXAM
688 Baby B. Ulate, suffers from infestation of the “unholy Answer: B. Selectively inhibits microtubule MICHELLE JAY MIDTERM 1 -
trinity” of roundworms, namely Ascaris lumbricoides, synthesis and glucose uptake in nematodes is the FRANCISCO, MD AUG 2013
Trichuris trichiura and Ancylostoma duodenale. What mechanism of action of Mebendazole, the drug of (TOP 9 - FEB
is the mechanism of action of the drug suitable for the choice for Ascariasis (pp. 40 of part2, 2013 MED
patient’s condition? Pharmacology, Topnotch Handouts). BOARDS;
A. Increases permeability to calcium causing muscle Notes: TOPNOTCH MD)
paralysis, vacuolization and death. • Increases permeability to calcium causing muscle
B. Selectively inhibits microtubule synthesis and paralysis, vacuolization and death – Praziquantel
glucose uptake in nematodes. (DOC for Trematodes and Cestodes)
C. Immobilizes the parasite and alters their surface • Immobilizes the parasite and alters their surface
structure displacing them from the tissues and structure displacing them and making them more
making them more susceptible to destruction by host susceptible to destruction by host defense
defense mechanisms. mechanisms – Diethylcarbamazine (DOC for
D. Intensifies GABA-mediated neurotransmission in Filariasis)
nematodes immobilizing the parasite. • Intensifies GABA-mediated neurotransmission in
nematodes immobilizing the parasite – Ivermectin
(DOC for Strongyloidiasis)
• “Unholy trinity” of roundworms – Ascaris,
Whipworm and Hookworms
689 The following drugs are included in the ABVD Answer: A. Vinblastine (pp 41 [part 1], MICHELLE JAY MIDTERM 1 -
regimen for Hodgkin’s Lymphoma, except: Pharmacology, Topnotch Handouts). FRANCISCO, MD AUG 2013
A. Vinblastine Notes: (TOP 9 - FEB
B. Doxorubicin A – Adriamycin (Doxorubicin) 2013 MED
C. Dacarbazine B – Bleomycin BOARDS;
D. Oncovin V – Vincristine (Oncovin) TOPNOTCH MD)
E. Bleomycin D – Dacarbazine

690 The following drugs accelerate substrate metabolism Answer: D. Cimetidine (pp. 8 [part 1], MICHELLE JAY MIDTERM 1 -
causing a decrease in the pharmacologic action of the Pharmacology, Topnotch Handouts; pp. 77 [.pdf file], FRANCISCO, MD AUG 2013
co-administered drug, except; Katzung Pharmacology, 9th edition). The question (TOP 9 - FEB
A. Griseofulvin defines a CYP450 Inducer. The only CYP450 2013 MED
B. Rifampicin inhibitor among the choices is Cimetidine. BOARDS;
C. Ethanol TOPNOTCH MD)
D. Cimetidine

691 Mr. A. Lipin, a farmer from a distant hacienda was Answer: B. Atropine (pp. 141, 149 and 162, Katzung MICHELLE JAY MIDTERM 1 -
brought to the ED and admitted due to Pharmacology, 9th edition) FRANCISCO, MD AUG 2013
unconsciousness, profuse salivation and shallow Notes: Atropine is the prototype antimuscarinic (TOP 9 - FEB
breathing. It took 7 hours of travel time from the drug to be given initially in this patient suffering 2013 MED
hacienda to the nearest Provincial Hospital. BP from organophosphate poisoning. It is best to give BOARDS;
140/90 mmHg, PR 52 bpm, temp 30OC and with both Atropine and Pralidoxime if the golden period TOPNOTCH MD)
constricted pupils. As an ED physician, what of 6 hours was not violated. The likelihood of aging
medication will you initially give that will most likely of the phosphate enzyme complex occurs after 6
benefit the patient? hours; however, recent reports suggest that
A. Pralidoxime administration of multiple doses of pralidoxime over
B. Atropine several days may be useful in severe poisoning.
C. Pralidoxime + Atropine
D. Physostigmine
692 A patient in PACU is in no apparent distress. The V/S Answer: D MICHELLE JAY MIDTERM 1 -
are stable except for HR of 128 bpm that is irregular Patient has atrial fibrillation w/ rapid ventricular FRANCISCO, MD AUG 2013
with no p wave. Which of the following treatment response. B-blocker, Ca Channel blocker or Digoxin (TOP 9 - FEB
option would not be appropriate as initial therapy? may be used. Supraventricular tachycardia, not a.fib 2013 MED
A. Metoprolol responds to adenosine. BOARDS;
B. Diltiazem TOPNOTCH MD)
C. Digoxin
D. Adenosine

693 This factor indicates the spread of local anesthetic in Answer: B MICHELLE JAY MIDTERM 1 -
CSF? Density of local anesthetic in relation to density of FRANCISCO, MD AUG 2013
A. Addition of narcotic to local anesthetic. CSF at normal body temp.is termed as baricity. (TOP 9 - FEB
B. Density of local anesthetic Degree of spread is determnined primarily by 2013 MED
C. Patient’s body surface area density of anesthetic & patient’s position. BOARDS;
D. Dose administered TOPNOTCH MD)
694 A young, male patient presents with mental Answer: B MICHELLE JAY MIDTERM 1 -
deterioration and tremors. He has brown Wilson’s disease: excess copper deposition (cornea = FRANCISCO, MD AUG 2013
pigmentation in a ring surrounding the periphery of Kayser-Fleischer ring) (TOP 9 - FEB
his cornea and altered liver function tests. Which 2013 MED
treatment modality should he receive? BOARDS;
A. Dimercaprol TOPNOTCH MD)
B. Penicillamine
C. EDTA
D. Deferoxamine
695 In the treatment of CHF, this drug is characterized as Answer: B MICHELLE JAY MIDTERM 1 -
having a selective beta-1 antagonistic activity and Dobutamine is a relatively b-1 selective FRANCISCO, MD AUG 2013
devoid of dopaminergic activity: adrenoreceptor agonist that causes peripheral (TOP 9 - FEB
A. Fenoldopam vasodilation in some vascular beds useful for many 2013 MED
B. Dobutamine cases of acute heart failure. BOARDS;
C. Dopamine TOPNOTCH MD)
D. Isoproterenol

TOPNOTCH MEDICAL BOARD PREP PHARMACOLOGY SUPEREXAM Page 80 of 83


For inquiries visit www.topnotchboardprep.com.ph or email us at topnotchmedicalboardprep@gmail.com
TOPNOTCH MEDICAL BOARD PREP PHARMACOLOGY SUPEREXAM
For inquiries visit www.topnotchboardprep.com.ph or email us at topnotchmedicalboardprep@gmail.com
Item QUESTION EXPLANATION AUTHOR TOPNOTCH
# EXAM
696 A 7 year-old male child with nephritic syndrome on Answer: B MICHELLE JAY MIDTERM 1 -
steroid therapy was exposed to a sister with measles. After susceptible person is exposed to measles, FRANCISCO, MD AUG 2013
You would administer: either measles vaccine given within 72 hrs or (TOP 9 - FEB
A. Measles vaccine immune globulin given within 6 days can prevent or 2013 MED
B. Human immunoglobulin modify disease. However, px has nephritic syndrome BOARDS;
C. Antisera & is immunocompromised so an active measles TOPNOTCH MD)
D. Vitamin A vaccine may even be harmful.

697 At the E.R., you saw a 24-yr old woman complaining Answer: C MICHELLE JAY MIDTERM 1 -
of severe, acute, generalized headache. PE revealed T Lab picture: N.meningitides. Tx = Pen G 4 million FRANCISCO, MD AUG 2013
400C with (+) nuchal rigidity. Her CSF showed gram- units IV q4h (TOP 9 - FEB
neg diplococci & 200 leukocytes/mm3 (98% PMNs). 2013 MED
Administration of which of these is the next BOARDS;
important management: TOPNOTCH MD)
A. IV sulphonamides
B. Intrathecal penicillin
C. IV penicillin
D. Intratechal chloramphenicol
698 An asthmatic patient whose symptoms occur less Answer: B MICHELLE JAY MIDTERM 1 -
than once a week, lasts a few hours to a few days and Px’s asthma is classified as intermittent. *Review FRANCISCO, MD AUG 2013
has nighttime symptoms twice a month is best GINA guidelines (TOP 9 - FEB
managed with an: 2013 MED
A. Inhaled long-acting B2 agonist at H.S. BOARDS;
B. Inlahed short-acting B2 agonist PRN TOPNOTCH MD)
C. Inhaled steroid OD
D. Oral B2 agonist TID
699 The absorption of most orally administered drug, Answer: A MICHELLE JAY MIDTERM 1 -
which occur in the small intestine, may be enhanced Increasing gastric emptying time allow drugs to FRANCISCO, MD AUG 2013
by reach the small intestine sooner, hence increasing (TOP 9 - FEB
A. Taking agents that accelerate gastric emptying time the rate of drug absorption. 2013 MED
B. The presence of food BOARDS;
C. Stimulation of hepatic microsomal CYP3A4 TOPNOTCH MD)
enzymes
D. Increasing intestinal motility
700 Adverse events associated with the use of quinolones Answer: E MICHELLE JAY MIDTERM 1 -
include all of the following except: Quinolones inhibit topoisomerase II (DNA gyrase) & FRANCISCO, MD AUG 2013
A. Tendinitis and possible tendon rupture IV impairing DNA synthesis. Adverse effects include (TOP 9 - FEB
B. Seizures GI intolerance, antibiotic-asso.colitis, cutaneous 2013 MED
C. Arthropathy in children rxns, hepatotoxicity, prolongation of QT interval and BOARDS;
D. Clostridium difficile colitis Achilles tendon rupture. TOPNOTCH MD)
E. Narrowing of QT interval

TOPNOTCH MEDICAL BOARD PREP PHARMACOLOGY SUPEREXAM Page 81 of 83


For inquiries visit www.topnotchboardprep.com.ph or email us at topnotchmedicalboardprep@gmail.com
TOPNOTCH MEDICAL BOARD PREP PHARMACOLOGY SUPEREXAM
For inquiries visit www.topnotchboardprep.com.ph or email us at topnotchmedicalboardprep@gmail.com

Item # ANSWER 87 C 175 B 263 A 351 D


1 B 88 B 176 D 264 E 352 C
2 D 89 B 177 A 265 A 353 B
3 C 90 A 178 C 266 B 354 C
4 D 91 D 179 E 267 B 355 B
5 E 92 D 180 D 268 C 356 A
6 B 93 B 181 B 269 A 357 B
7 D 94 D 182 A 270 E 358 C
8 A 95 B 183 D 271 D 359 A
9 BONUS - no 96 B 184 C 272 A 360 D
correct answer 97 E 185 E 273 D 361 E
10 D 98 A 186 C 274 E 362 E
11 E 99 C 187 C 275 C 363 D
12 A 100 E 188 E 276 B 364 C
13 E 101 C 189 E 277 E 365 E
14 B 102 D 190 B 278 B 366 B
15 B 103 A 191 D 279 D 367 B
16 E 104 C 192 A 280 A 368 A
17 E 105 E 193 A 281 C 369 C
18 E 106 D 194 C 282 A 370 D
19 B 107 B 195 A 283 D 371 D
20 B 108 E 196 A 284 C 372 B
21 D 109 D 197 A 285 D 373 B
22 A 110 E 198 B 286 E 374 E
23 B 111 C 199 D 287 A 375 A
24 B 112 A 200 C 288 D 376 A
25 B 113 B 201 B 289 D 377 B
26 C 114 A 202 C 290 A 378 D
27 E 115 C 203 C 291 C 379 C
28 C 116 D 204 C 292 B 380 A
29 A 117 B 205 E 293 E 381 B
30 D 118 A 206 B 294 B 382 B
31 C 119 B 207 A 295 D 383 C
32 C 120 C 208 B 296 E 384 D
33 C 121 D 209 A 297 B 385 A
34 E 122 A 210 C 298 A 386 B
35 B 123 A 211 A 299 B 387 C
36 B 124 E 212 D 300 C 388 C
37 D 125 A 213 D 301 B 389 B
38 B 126 A 214 B 302 E 390 B
39 D 127 E 215 C 303 D 391 D
40 B 128 C 216 B 304 C 392 B
41 E 129 C 217 D 305 C 393 C
42 E 130 A 218 C 306 C 394 A
43 C 131 D 219 E 307 A 395 A
44 A 132 B 220 D 308 B 396 C
45 B 133 D 221 C 309 C 397 A
46 B 134 B 222 D 310 D 398 B
47 D 135 A 223 D 311 C 399 D
48 B 136 A 224 C 312 B 400 B
49 D 137 B 225 B 313 D 401 A
50 A 138 B 226 C 314 D 402 D
51 B 139 D 227 D 315 D 403 E
52 B 140 D 228 B 316 A 404 D
53 B 141 B 229 C 317 A 405 D
54 A 142 C 230 A 318 B 406 C
55 A 143 A 231 A 319 A 407 A
56 A 144 B 232 D 320 A 408 B
57 B 145 A 233 B 321 A 409 A
58 C 146 D 234 D 322 A 410 C
59 B 147 D 235 E 323 B 411 D
60 B 148 A 236 E 324 D 412 C
61 B 149 C 237 D 325 D 413 A
62 C 150 D 238 B 326 D 414 B
63 A, D 151 E 239 E 327 E 415 A
64 B 152 C 240 A 328 B 416 A
65 D 153 A 241 B 329 D 417 B
66 C 154 D 242 A 330 C 418 C
67 A 155 B 243 D 331 C 419 C
68 B 156 D 244 A 332 A 420 E
69 E 157 B 245 B 333 D 421 C
70 C 158 D 246 D 334 B 422 A
71 B 159 B 247 C 335 A 423 E
72 A 160 D 248 D 336 D 424 C
73 D 161 A 249 A 337 D 425 D
74 B 162 C 250 D 338 C 426 B
75 B 163 B 251 E 339 A 427 C
76 E 164 D 252 D 340 A 428 B
77 D 165 E 253 C 341 B 429 D
78 C 166 D 254 D 342 B 430 B
79 B 167 A 255 B 343 B 431 C
80 B 168 D 256 C 344 C 432 A
81 A 169 E 257 A 345 B 433 B
82 A 170 A 258 E 346 A 434 D
83 C 171 C 259 C 347 C 435 C
84 C 172 D 260 A 348 A 436 D
85 B 173 E 261 D 349 C 437 B
86 E 174 B 262 B 350 D 438 A
TOPNOTCH MEDICAL BOARD PREP PHARMACOLOGY SUPEREXAM Page 82 of 83
For inquiries visit www.topnotchboardprep.com.ph or email us at topnotchmedicalboardprep@gmail.com
TOPNOTCH MEDICAL BOARD PREP PHARMACOLOGY SUPEREXAM
For inquiries visit www.topnotchboardprep.com.ph or email us at topnotchmedicalboardprep@gmail.com
439 B 528 C 617 C
440 D 529 C 618 C
441 E 530 C 619 D
442 C 531 A 620 E
443 E 532 D 621 E
444 B 533 A 622 A
445 E 534 C 623 B
446 B 535 C 624 E
447 C 536 B 625 A
448 B 537 D 626 A
449 E 538 A 627 D
450 B 539 D 628 D
451 D 540 C 629 E
452 C 541 E 630 B
453 D 542 E 631 E
454 E 543 A 632 B
455 A 544 D 633 E
456 C 545 C 634 B
457 B 546 A 635 A
458 A 547 B 636 B
459 B 548 E 637 C
460 C 549 B 638 D
461 B 550 B 639 A
462 A 551 E 640 E
463 C 552 D 641 B
464 B 553 C 642 B
465 D 554 D 643 A
466 A 555 D 644 D
467 C 556 D 645 B
468 B 557 D 646 E
469 E 558 C 647 D
470 D 559 A 648 B
471 B 560 A 649 A
472 E 561 B 650 C
473 C 562 A 651 D
474 B 563 C 652 A
475 E 564 B 653 E
476 C 565 D 654 C
477 D 566 D 655 C
478 A 567 A 656 C
479 C 568 B 657 C
480 A 569 B 658 D
481 A 570 B 659 A
482 D 571 B 660 C
483 C 572 B 661 C
484 B 573 A 662 A
485 A 574 A 663 C
486 A 575 A 664 B
487 D 576 A 665 B
488 B 577 D 666 B
489 C 578 C 667 C
490 D 579 C 668 A
491 D 580 A 669 B
492 A 581 D 670 D
493 D 582 A 671 C
494 A 583 C 672 A
495 B 584 C 673 A
496 C 585 B 674 B
497 B 586 D 675 D
498 A 587 C 676 B
499 D 588 C 677 C
500 D 589 E 678 A
501 A 590 B 679 A
502 C 591 D 680 D
503 E 592 C 681 C
504 A 593 A 682 B
505 E 594 D 683 C
506 B 595 E 684 A
507 D 596 C 685 C
508 B 597 D 686 D
509 A 598 C 687 A
510 D 599 B 688 B
511 E 600 A 689 A
512 B 601 A 690 D
513 C 602 A 691 B
514 B 603 B 692 D
515 D 604 C 693 B
516 A 605 C 694 B
517 B 606 E 695 B
518 D 607 B 696 B
519 B 608 A 697 C
520 E 609 D 698 B
521 C 610 B 699 A
522 B 611 A 700 E
523 D 612 A
524 B 613 C
525 C 614 A
526 D 615 B
527 D 616 D
TOPNOTCH MEDICAL BOARD PREP PHARMACOLOGY SUPEREXAM Page 83 of 83
For inquiries visit www.topnotchboardprep.com.ph or email us at topnotchmedicalboardprep@gmail.com

You might also like